0% found this document useful (0 votes)
1K views408 pages

N. Antonov, M. Vygodsky, V. Nikitin, A. Sankin-Problems in Elementary Mathematics For Home Study-MIR Publishers

Livro Russo

Uploaded by

Airton321
Copyright
© © All Rights Reserved
We take content rights seriously. If you suspect this is your content, claim it here.
Available Formats
Download as PDF, TXT or read online on Scribd
0% found this document useful (0 votes)
1K views408 pages

N. Antonov, M. Vygodsky, V. Nikitin, A. Sankin-Problems in Elementary Mathematics For Home Study-MIR Publishers

Livro Russo

Uploaded by

Airton321
Copyright
© © All Rights Reserved
We take content rights seriously. If you suspect this is your content, claim it here.
Available Formats
Download as PDF, TXT or read online on Scribd
You are on page 1/ 408

N.

ANTONOV
M. VYGODSKY, V. NIKITIN
A. SANKIN

PROBLEMS
IN ELEMENTARY
MATHEMATICS
for Home Study
Translaled from ihe fíussian
by
L E O N ID LE VA N T

MIR PUBLISHERS
MOSCOW
H. O. AH TOHOB
M. H. B b ir O ^ C K íin
B. B. HHKHTHH
A . II. CAHKHH

CBOPHMK 3 A R A H
1103JíEMEHTAPHOH MATEMATIiKE

H3AATEJ1LCTBO
«H A Y K A »
MUCKBA
Jla amAUücnoMr x .>mk ;
CONTENTS

Formulas for Reference ........................................ 6

PART ONE
ARITHMETIC AND ALGEBRA
Problems Answer
and
Solution
Chapter 1. Arithmetic C a lc u ia t io n s ................ 11 89
Chapter II. Algebraic Transform ations........ 14 9i»
Chapter III. Algebraic E qu ation s.................... 22 112
Chapter IV. Logarithmic and Exponential Equa-
t i o n s ............................................... 29 142
Chapter V. Progressions ........................................ 32 159
Chapter VI. Combinatorics and Nc\vton’ s Binomial
T h e o r e m ........................................ 30 109
Chapter VII. Algebraic and Arithmetic Problems . 39 177

PART TWO
GEOMETRY AND TRIGONOMETRY
Chapter V III. Plane G eom etry........................ 55 208
Chapter IX . Polyhedrons.................................... 62 240
Chapter X . Solids of Revolution ............................ 76 327
Chapter X I. Trigonometric Transformations . . . 81 363
Chapter X II. Trigonometric Equations ................ 84 372
Chapter X III. Inverso Trigonometric Functions . . 87 395
FORMULAS FOR REFEREN CB

I. ARITHMETIC AND ALGEBRA

Proportions
a c
I. In the proportion — a and d are the extremes, b and c are the
mcans. The principal property of the propor
proportion:
a -d — b-c
2. Interchanging the terras:
b lA, d
(») “ f (c) < d) -C =
rf

3. Derived proportions: if , then the following proportions hoU


b ~ d

a±6 c+ d a -h c a c
(a )
• (h) tr r7 t"T -T
/ drol ution
1. lhat is an>bn•e" -----(a •h-c),i

3. D'"-a»=.am+'i
- ( t )"- *(*>"
4. a1" : a'< - am~ 5. i ; a’* o0 : «"
(K {rt"1)'1 nm;i

Frolulíon *

I. 7) a-b- c <f-"j b- c, that is r,J

: , that is -
b
y

The roots are supposed to be arithmelic, cí. p bu.


Formulas for Heferenre 7

n n
3. a m = ™ a '1, that is ™ a'‘ = a m

Quadratic Equations

1. Tlie equation of Uh* form x2 p x ~ q — 0 is sotved hy using tlie for­


mula

2. The equation of Iho form ax2-i-bx-\-c~0 is solved hy using the for­


mula
—6 —4crc

3. The equatiou of tlie íorm ax1 - f 2A-x -f c — 0 is solved hy using lhe for
mula
-k ± '\ / k -i-a c
a
4. If xj and x2 are the roots of the equation x2 - f px -f- q =-- 0, then x, -j-
- f x2 — ~ f) and xjx2 - <j
5. x2 -j- px - f q - (x — x,) (x — x2), where xx and x2 are the roots of the
equation x2 - f px -|- q - O
6. ax2 - f bx - f c •- a (x — x,) (x — x2), where X| and x2 are the roots of
the equation «x2 + bx -}- c -- <)

Progressions (sce page 32)

Logarithms*
1. Symholically, log(JA' = x is equivaleu! to a1' = ;V> hence we have the
identity a a ~ A
2. Iogaa = l ; 3. log(1l —0; 4. loga (Ar..l/) = logaA '-f !ogaAf

5. lüga ~ = lo-4nN-k,íiaM- ti: Ioga (W™)—mlogfliV

7. loga y x - íogaA'
8. For the modulus whieh makcs possiblc convcrsion froin a system of loga­
rithms to the base b to that of logarithms to the base a see page 142.

* The numbers a (the logarithmic base) and N are assumed to be positive,


a heing not equal to unity.
8 Formulas for Reference

Combinatorics
1. A ^ — m (m — 1) (m— 2) . . . (m ~ -n -f-l); 2. Pm—1-2-3 . . . m—m\
0 „„ A& m(m — 1) ( to— 2) . . . ( to— n - f l )
; 4. c g . - c i r 71
m~ Pn 1 -2 .3 ... n
Newton's Binomial
1. (x-[-a),n~ ^ m + Cííla ^ “ í-}-C ^ a 2xm-24_ . ..
2. General term of expansion:
n + ^ C ^ x m -k

3. i + C - I„ + CJl + . . . : + C g ‘- 2+ C S - 1+ l = 2”>
4. l — Ck + Cfc, — C & + . . . ± i = 0

II. GEOMETRY AND TRIGONOMETRY


The Circumferenee of a Circle and the Length of Its Arc
C —2nR\ l — -T5^== P a (a is the degree measure of the arc and a , its
loU
radian^measure)
Areas

Tríangle: S —^ (a is the base and the altitude); S—


= V / J( p - fl) (P -Í> )(P — c) (P is the semiperimeter and a, b and c, the sides);
c a6sinC ‘
S == ----- 2~
2 * l/í
For an equilateral tríangle S — — . (a is the side)
Parallelograrn: S = bh (b is the base and k , the altitude)
Rhombus: S — ~ % (dj and d2 are the diagonais)

Trapezoid: 5 = (a and b are the bases and hy tho altitude); S ~ m h


(m is the median).
Pa
Regular polygon; 5 = (P is the perimeter and at the apothem)
Circle: S ~ tiR2

Circular sector: o — - y — —^ - = (a is the degree measure of the sec-


tor arc; a , its radian measure and l , the length ol the arc)

Surfaces
Prism: =* Pl (P is the perimeter of a right section and l, the lateral edge)
Pa
Regular pyramid: S*aí — - y (P is the perimeter of the base and a, the slant
height)
Formulas for Reference 9

p tp
Fruslum of a regular pyrarnid: S[at — —~ — - a (Pt and P2 are the perime-
ters of the bases and a, the slant height)
Cylinder: S[ot — 2nRIf
Cone: 5 ;aí ~= nRl (l is the generator)
Frustum of a cone: = Ji (Rj -j- R2) l
Sphere: S = AnR-

Volumes
Prism: V = SH (S is the area of the base and If. the altitude)
n
Pyrarnid: 1 SIt

Frustum of a pyrarnid: 1' = - —• (5 j -j~ S» -f- *l/ó’^'2)


Cylinder: V - nR2Il
nR~H
Cone: F —
3

Frustum of a cone: F = ^ - y (Rj 4" + ^ 1^ 2)


A
Sphere: Vss-^-ítR^

Conversion of the Degree Measure of an Angle to its Radian Measure and F<re
Versa

a= ■■; a '~ a (a is the radian measure of the angle and a, its


180 Jt
degree measure)

Addition Formulas
1. sin (a ± p) —sin a eos p ;£ cos a sin P
2. cos (a ± P) —cos a cos p + sin a sin p
o x , .o , tan a ± tan p
3. tan (a ± *p )“- 7-rrr-------
i -+■ tan a ;----
tan p5

Double-Angle and Ifalf-Angle Formulas


1. sin 2a = 2 sin ci cos a; 2. cos2a = cos* a-sm - a

3. tan 2a =
2 tan a
1— tan2 a '
4. sin
2 =/^
t-f-cosa 1f 1— cos a
- a
5. cos -—= ± ] / “ l- ; 6. tan
2 ~- V 14-cos a
a sin a
8. tan -
i — cos a
7. tan
2 i -f cos a ' sin a
40 Formulas for Reference

Reducing Trigonometric Expressions to Forms Convenient for Taking Logarithms


1. sin a - f sin P ~ 2 sin —y — cos a ^ —
„ . . Q „ a -fP . a — P
2. sina — sin p = 2 cos—~ s i n — ■—

3. c m a + cos P==2 cos^— ^ c o s ^ - ~


0 . a + p . a —p
cos a — cos t — 2 s m —£-^sin—^-2-
sin (a ± p)
5. tan a ± tan p •

6. i-f-cosa—-2cos2 - 7. i — cosa — 2 sin2 -

Some /mportant Relatlons


cos (a — p) — cos (a -f- P)
1. sin a sin ft:
2
cos (a — p} cos (a-f- p)
2. cos a cos {4 =
2
sin (a-f-P)-f-sin {a — p)
3. sin a cos p
2

Basic Relations Between the Elemenis of a Right-Angled Triangle


(a and b are the legs; c, the hypotenuse; A and B, the acutc angles; C, the
right angle)
1. a c sin A --- c cos B\ 2. b — c sin B — c cos A
3. a --- b tan A - b cot B\ 4. b ~ a tan B -•= a cot A

Basic Relations Between lhe Elemenis of an Arbitrarij Triangle


(<i, b and c are the sidos; B and C\ the angles)
, a b c , , .
1. —— — ------—- (law of smes)
sin A sm B sin C
2. tJ2 ----- b- -r- c- — 2òc cos /I (law of cosines)
, A+ B
. tan — jr—
a->-b 2 .
(law of tangenls)
A - lí
tan

Relations Between the l'alues of / nverse Trigonometric Functions


(arcsin .r, arccos .r and arctan .r are the principal values of the corresponding
inverso trigonometric lunctinns)
1. Arcsin .r ■•■= nk r (— 1)* arcsin .r
2. Arccos .i 2nk arccos .r
3. Arctan j ■- nA -r arctan j ; A- is any integer (positive (»r negativo).
PROBLEMS

P A R T ONE

ARITUMETIC AND ALGEBRA

C IU P T E 1 I 1
ARITHMET1C CALCULATIONS

1. ------------- Ei 4--------------i í-
9 3 t
2 1 5 ^ -2 0 8 ^ + i
0.012 0.041U4\
3. ---- L — 1 ; í . ( —j — : —s-7— j •4:>()U -
: 0.005 ’ V 5 0.4 /

( 853 ü ~ 83! 8 j ■ f. „ i U n3 Õ - m T 2 j: , í , Í
5. ’ n nrv)

(95è - 934 )'2T + (>-373 . „ (44 - 44 ) ' 2¥ + i, (;


7. 0.2 ’ 0.2

( 1 2 ‘ . _ « ^ _ 5 | ) . í3 .5 + 0 .H , _ ^ ( 4 + 4 4 Q 4 + 2- 13
9. 0 .0 2 ’
‘ S i w
: * H «

W
1

0 " 14
11.
i - 4 . 4 ”
12. -
,

(21 — 1.25): 2.5


l3è + 4-375) M9T

13.
0.134 -f 0.05
; 14. (58B - 5oá ) :u-8+ 4 -° -223
íft t H 2 ;1 6
______
18 ü 14 15 ~ 7
1 .
15. 0.04
(2.1-1.9G 5):(i.2.0-045) 1 : 0.25
16. 0.00325:0.013
Problems

17.
[(40ro-38á ) :10-9+(¥ -|»)-1n ]'4-2

( 2 .4 + 1 4.375 (2 .7 5 — 1
. 67
18.
2___ l_ : 200
3 6 8 2ü - ° - /l5

( 6- 4 ) : 0.03 ( 0 .3 -i ) .4
19.
K3^-2-05)-4- ( 1.88 + 2 . + . g0 _
' 3 : (0.2 — 0.1) , (34.06 — 33.Sl)-4 1
20 . 20 :
: 1.2.5-40.8 + 1.2) ^ 0.84 : (28.57 — 25.15) J + T : 2Í

3 + — 0.09 : ( 0.1 5 :2 4 - )
21.
0.32-6 + 0.03 — (5.3 - 3.88) 1-0.07

22- 1 ~ : 2.7 4 2.7 :1.35 + (o.4 : 2 . ( 4 .2 - 1 1 )

23. ( 1 0 : 2 | + 7 . 5 : 1 0 ) . ( | - ^ . 0 . 2 5 4 . 1 )
/0.216 : 2 /190
24. - + \ + 0.G95 :1
l 0.15 3 ■ ■ h h 225
244J
1 ,3 .7
25.
1 7 . ( 4 ■5 ' 1 3) +
5 “ (° - 5 + T “ é ,

0.228: [(1 .5 2 0 1- Ji+S:-°.305):0.12]


8.8077
■ u 20— (28.2 : (13.333-0.3-r 0.000t)j-2.004
:0.31 - -jj-0 .í))-0 .2 -j-0 .15 j :0.02
28.

^ n -0-22:,M) 4
t+
29. 2 Ü.25
, i : 3 - ü-» :-3 - 4ü
1 + 2 .2 -1 0
Chapter / . Ârithmetic Calculations 13

75: -1.75-1 4-
30. (*• 8 I ' 12
: (0.79: 0.7 f 0.3)
^ i Z _ o .0325 j : 400

4.5 : (4 7 .3 7 5 - (26 18-0.75) -2.4 : 0.88^


31.
17.81 : 1 . 3 7 - 2 3 + 1 -|
3 D
32. Find the number, 3.6 per cent of which araount to
3 .4-4.2: 0.1
( l :0 .3 - 2 Í ) .0 .3 l2 5

33. Compute

(«B=
34, Compute
23
12 + 4 1 ^ :2 6 0
35' 14
-800: 12
0 28\ 0.8.' .5-1.3
31/ 0.5-2.7-1.92

ÍU7\
j w : ( 0 - 3 + " + - " + > ; llfl'| (0.6 4 5 :0 3 - 1 )
30 t + 3 -9 r
x (4 :6 .2 5 — Í. + J .1.96 )
35. Compute

[ ( 4 - 6B -n ): (8T - T - 4 ) + á : s lx
• X (1 -0 .7 5 )
36. Compute
2.045.0.033+10.518395-0.464774 :0.0562
0.003092:0.0001— 5.188
37. Compute
( H - 2 Í| ) : ( 2 + + l f ) - ( + - i ) ( + + )

38. Compute
( « | - 4° f f i ) { [ 4 - 3 T ( 2 7 - , T )]« 0 .1 6 }

39. Compute
..1 0 //2 5
45< 3 ~ 4484
: 31
( 4- 4: :4
14 Problems

40. Compute
0.8: (4 -1 .2 5 ) (l,08 —4 ) : 4 .
' 5 ' ' ' 1 + (1 .2 -0 .5 ): 4-
0.G4- ,£
25 ( 49- “ 4 ) ' 2*Í7
V”
41. Compute

[ ^ i - ( l 4 - 4 ) ( ‘0 T - 7 l) ] :2 2 Í

42. Compute

( 5 4 ~ ) : 0.003 (0 .S -jg )-» T


: 62 20 -17.81:0.0137
I(4 “ 2-,,5)/*]4 (*• 88-4-2 j:

43. Compute x, if
4 . r _ ., o . o / 0

44. Compute x. if

í (4•(i25-H •I ) : x+<2-5:1-25!: u-75] : 1iíi


27
-0.375 } : 0.125-j- - 1 ) : (0.555--1.4790:13.7)
(t - (1 -
I-iud x , if
(2-7--0.S).21
(1.6-j-154.ü(i : 70.3) : 1.9
3 ' '■ « 1 2.625
(5.2 -- i . 4 , 4 3) : 4.3
( 2| -> -

C HAPT F. R II
ALGEBRA1C TRAN SFORMATIONS
SimpUfy lhe jollowing expressions:
a-4-fe — c
46. (a- — 6- — c 1-|-2?7c): a -}- 6 -j- c

Evaluate the result al a = 8.6; íi — ] /3 ; c = 3

47. — 1
Chapter / / . Alyebraic Transformations 15

_ x _____________ 2
48. ax — 2a2 x2 x — 2ax — 2a i +
3x-t- x2
3 -f a-
3x — f>\
)
______ _ j _________________ _ X-
a2— 4x2 ' 2j 24-Üj: — ax — 3a
- A / 2a+ 10 . 130— a 30 Q 3a3 8a2— 3a
5Ü* V 3 z n r r + H n r 3 r + 7 “ ,í

j i<(-r — y)2
51.
•>*+ »* •**— » '
53.

«~ 1 2(o - l) i(a + DL . ______e_____l x


54. a2— 2« -j- 1 a2-f- a - 2 a2—3a+ 2 J X
36a3— 144a —3üa24 -144
a3-l-27
r 3 <, *-f2) r 2j 2— x — 10 *1 ,
55. L 2 (x 3 + x2-f x - f 1) ' 2 {.c3 — x2 -f- x — 1) J

• _________ i _ i
L x2 + l 1 2 (x - f l ) 2 (x — 1)J
/ x—y *'- + » 2+ i í - 2'\ _ 4.r*-f-4x2«/-j-y2— 4
56. x“i — xy — 2i/2 ,f *2+ i/-}-*y-í-z
\ 2y - z
az -\-a~~2 r (o + 2)2—«2 3 I
57. n 1— 3a'1 | 4a2—4 «2 - 0 J

2q2( H - o)2" 2 _ 2aib + c ) « - i


an2-—a-i — 2a2— a ' a2c — a (nc — r)
_______J______ , ______ ‘________;________‘______
a (a—b) (a — c) b {b— a) (b—c) ' c (c — a)(c — b)
an l + íf l + j )- ! f , |_< 02+ z2)
’ i —(<a+ ^)-x' 2ax J
Evalnate the result al x —a
——-1
r.
f)fln+2 ^ 1
61. l 2 ± Ç ^ - ( i b ( W - a -) - ' ] : 1 2an6 4- 3a"*! —
2a - b }

62*.
H t ) >
( y ~ a ~ .y b )2-S-2Vab

Prior to solving subscquent prohlems reati ihe notes ou pp. 90 to 92.


10 Problems

!.3-0»
63. —t-- p'\ai
„ r ___
2ab +, .ò2) (a2- b-) (a + b) ■—
, ____
a -b y ' - '' 7' f {a + 6)2

64. v/ "8 i(7 + 4 V r3 ) v ^ 2 1 / 6 Í - 4 / 2 Í

65. (a + l)(g - — 1)(1 ^ 2 a f ga)-( ) ’

66 V |l+ ‘ )>; | -f * l / -.______________ y$


v 3a ' 9 + i8a-i+9a~2

67. ah ’y a1- nb-n- j/" ( a — b f 1


15
68 .
1/6 + 1 : i/B —2 3-
69.
l/a - l/a -6 1 1 Va+b ) : ( ‘ + / £ b)

70. 1 . 1
fc+ l / ã / ‘ - a-ifc-2

I—a
1-f a
71.
VI i T+í
72. Evalunte the cxpression
-r;/- y ^ r r y ^ r r i
*»- > v --! y p r r i

:<l * — y ( a y ) •y = j(b - y )
73. Evaluale the expression
\/a -{.■bx -j- 1/a— bx
\^a-f- />x— "j/ a — òx

al■'■-rarU'
S im plijy the follotcinx e.rpressions:
]_ ^
* (m - x)_ -- (w — x>"

{/7i -7- x)- — {m - x)“


Chapter I I . Algebraic Transformations 17

if x = J ~ | and m > 0, 0 < n < l .

75. - ( l —x2)2
j
+i
r+r
j ' L
P ~ * 2)
2 - r

if x = 2k2 (1 +fc)_1 and * > 1.

2a2 (x2a~2—4-j-4a2*“2)
___ _2
77. ( 2 / i * - a ¥ -
V i - a 2x-z -
2ax (x2—a2) 2

V i ± V i \ - ' , t / y s + y t \->
26 y S / T V 2a y t /
( a-\-~\/ab \~l /fe + V a fo \ -l
\ 2aò ) + \ 2ab j

79 t v °+ y * y«+ * \~2 / y ° —y » y ° + * \ -2
\ "\/a + x "^/a -{" ^ ' "V"0 i~ 1 ^ /a — ~\/x >

f+
V * 2+ a
80. j ^y V -t-« -
*|/a24-o ' ^ x-f-"j/x2 + a
X
H-
y^=T
81. 2;c + l / í 2- l ( l + ^ r i i+ y * * -i
82. Compute

83. Evaluate the expression


( o + l ) - l + (5 + i r l

at a = (2 + l / 3 ) '‘ and 6 = ( 2 - V 3 ) _1.


2-01338
18 Problems

Simplify the following expressions:


z + V x ^— ^x x — y x2— 4x
84.
x — ~]/x2— 4x í-f y ^ _ 4 Í
n+ 2— ~\/n2—4
85. ' -{~
7 + 2 1 „ + 2 + V Í2 3 4

-»/ * ~y,g
. / V g -—
~ ~]/x—
V x — a2
g2 ~\/x
00* K ly s + y s r :;;* ' " j / x — "J/z

87. 88. ( 2* + 2V ) : [ ( + '+ 3 / ]


X + X2 1

89. Prove the identity

L + A ,
r+ 4 ^ i^ 3
a*— a

90. Compute
3 3
a2 + t2 ^ i
2 ' a y a— b y b
{a2— aò)3

at a = 1.2 and b * 5 *

Simplify the following expressions:


1 i i t i 1 1 1 |i
91. [(a2 + í>2) (a2 + 5ò2) — (a2 + 2bz) (a2— 262) ] ; (2a + 3a2fe2)
Evaluate the result at a = 54 and 6 = 6.

[ («+<>) 2+ (a-i>)~Sl + [(a + t)~ a- ( a - f c ) s] *


92.
‘ ' ‘ _ 1 -i
|(a+& f 2+ (a -& ) 2] -[(.+ & )' - ( a — 6) 2]

1 (i — a2)2 + a2 ( l — a2) 2
93. a2(l — a2) 2-
1, , ' 1— a2
i - f [ a ( i — a2)
Chapter I I . A Igebraic Transformaiions 19

2 x2 V ( i - f x2)-*— V l - f x 2
94 l + x2
(* + l) (*2+ * )
t _ 1
i 1
(fí2— X2)2-}- x2(fl2-—X2) 2
95.

96. (p2+ 32)- 2 (p~l + ? " 1) 4 — I — — (p 2 + 9 2)


___ __ (í>2+ « 2)3
97 r (<,+ y <,2*):(*+i'ai2)—1 i i8
L \/~ã— j -r Vz ^

a— "\/ax
( 1/ i + l ) 2- 4a— 9a~l t a — 4-f-3a~
y i-v *
98. 99. ~I IT ~r I ~
( l / a - j - l )3— a ~\/a-\-2 _2a2—3a 2 a2—a 2

100. [ ( « - * ) / { ± Í - + « - 6] [ ( « - 6) ( / 1 ± | ~ 1) ]

>,/Õ6—

m .

103. r . . . ..; ....- + ... 2+ 8x ; 16

104. ^3 r j ^ + W 8^ - » ) ’ í5£ / j y ;
L x + V X1J J

105. ( v + 1 + V jf)V h 2/ i
\ y «_ y * { ai / K ' ^

(a — 63) T/3 — feV V a —8(i3 y 2 a -y 2 c


106.
V i ( a - b l f + (2* y 2a)2 j / 1 - j/ 1

107 . { í ^ l + [ ( a 3 - y ) 2 x 3] í } — -JJ-K(«" — * 2) 3+ 4 a V
20 Problems

108. [ f / i - ^ r + ^ + ^ r r 8: 4 yí_}_4 y i
/ a^X-X-~\/t a r —‘1®
109. + 4 {x + i)~ f C^r ^ V xJr l ) 2
y x+ Va :
1 -n-1
3x
110 . 4 1 \ 3 x— 2 /

111. 2 -f c a2— b2— \ a2 — a Y a2— b%

-(r * - r « Y w a f ± ]/ (n, -j- 3a2x ~r 3ax2 4- £3) Itf


112 .

- q / ã + !/> ) " - -( 2 v a ) 2 5 í i 1 (46) *


113. -(a ã -Ô ^ H ^ + ô V 1I

-46
114.
i l '
\ a-}- (ab)- —í>& a 0 (ab)- -f- 9í>)1 t - J
( 3 + 2 o y ã 4-b y ;
M ! </ - \ b 1 y ab—a
115.
ròa-~\~ob ]ab a y ã - b yã
( V/ã - y 6)3 + 2 a- : V'í + 6 V5 , 3 yãfc—3&
116. a—b
a y « + b \/,/i *
1 / V » -l/6 ~
117. I i ( 3 3 (« 6)
(a~ -\-h-)~z ‘ a~~b~
1
118.
Í/A T + 1 ) (»■'“ + ( / T - ' )
a , a -f- , o- 3/
119. ] [ ( / « - i!^ )'2+ 3 ( J / í + V~x

120 .
Chapter I I . Algebraic Transformations 21

1
121. “+ + + - “ K a 2 + x + 2a y z 1
- fly i-f- J /a i J

122.
x y * —x 3
[ ( ^ E f - v = ) ( ^ f f - v = ) J

/ - r = + y ^ + 6 t » í>2+ í>2 ..r 1 , 1


123.
/a L (í^ + y s )2 6J + _í A
a ‘*62
a+ x f ax2— y õ 2x

124.

1 2al - 2
125. 1 1 1 ll l l 1
a4+ a 8+ t a1 a8+, al
„4*—„8 _2
a2—-4 f-2

K V à - l t / ^ 3 + 2 V 2 + ^ ( g + 12) y í - 6r - 8
126.
K y i + Í ^ 3 -2 1 /2
y x —1
‘\/a9bf/r^-\-'\/aJ63: \ 'a
127.
(A2_ fl&__2a2i V Í 6
2
__a ~Z { _______3a2________ a + b______ ab '
\ 36 — 6a+ 2aò—62 * 3a — ab "ã-j-ft
f 10x2+ 3 a x , 6x — x2— a x + a 6 ,, ,
128.
L 4x2—a2 f 2x + a *) - 2 ] x

( a + 2x)
X
(4x2—
129. r x+4 -z + 2 ] 1/9
L 2x2— 2x — 4 f 2 (x2+ 3x + 2) J Vix
x+ 6
- ( K 2 + K 5- h ( z 2- 2 2)2
' V Í + V2

130. (i-^ > 2+ i . y<-


!• x — 2
( i + * í 2+ d - i ) ?

+ <*+ *> M t + ^ -4x x2— 3x —4 ,


22 Problems

131 a2 V ^ " 1 V & V a b — 2 V a 26 a6*>


(a2— ab— 262) i f a56
a—3 r a 4-2 6
-(a — 1) (a2— 4a + 3)_1J
' a + 26 La24 -a 6 --3 a — 36
______ O
.q o V
r a° Vy aa—
^ - ( a\mr
6)T : VKã« I/ *,
\ V « . \ / ”> \
(a2— 62) a~i ' l K 6 + K a J +
, 6 / 2a + 26 , a -f-36
+ a ‘ \ a — 46 + 2a + 26
a2-f-2ia6
2l ” 2a2 — 6aô — 862 )
133, |~ ab ^V b — Y a b V a f ^ . a 4/ 6 + 6 V ã ^
a6 f/"a6 -V»
+
62— 4a2 ______________®
1 ___ \
4a (-624-3a6 + 2a2 2a2-)-a6— 62 /

í3 4 4(2a6)? (a + 26)-l , K 26 V 2a 6 + ^-^2ã56


5 —
1 / ã — 1 /2 6 ' V 2Õ6
fí l a 26 862 \
0 V 6a— 486 — 3a— 66 a2— iOaò+1662 /

C H A P T E R III
ALGEBRAIC EQUATIONS
SoZi;e following equations:j
4*>c 66+7a 3ay ^ ay _ a i — 6 , 6x + a a2-f-62
IdD . õb 262 ~ 1 — 62—ã b ’ l á b * ~ã-t-b n a — 6 ~ a2— 62

c+ 3z c — 2z 2c4-s
138.
4c2 4-Gcd — 9dt— Qcd ~~ 4c2— Õd2
x —1 2a2 {1 — x) 2x — 1 1— x
139.
n—i n*— 1 — 1 — n4 i+ n
3íi6 + l 3a6 , (2a + l ) x . a~
140.
a X ~ a + 1 + a ( a + l ) 2 + (a + 1 )2
3aòc , a2&2 , {2a-\-b)b2x 0 _ , bx
141. a + 6 ^ (a-f-6)3 ^ a(a + 6)2 oCX -] —
x-\-m ax am bzx
142.
T+S (a 4-6)2 ~ a2— 62 “ a»— a624-a26— 6*
Chapter I I I . Algebraic Equations 23

m (z — m) z (2 + m)
143.
z
+1 m
____ _____ ____ _m2-
z(z-j-m)
_ m{z — tn)
a2+ x a2— x 4a6x + 2a2 262
144. &2~ x 62 + x — 6* —
(a-}-n) {anx-f-m 2-!-*3) ax nj2
145.
x34- ax2— a2x — a2n n-f-x ‘ x2— a2
_ I j . j" a-\-\____ a (x-|-li
146. a -\~1
az-j-1
a-f-x
| x-\-i
x~f~a- [. (x-f-fl"1) a
3a
ax-j-1 +‘M
147. a2-j-ax-f-x2 ax — x2— a2 x (a4-}-02x2"{-••r1)

148. c (K x — a) — 6 (K x — 6 )+ a -f-ò = |/ .r
149. JL _ ! ___ I___ !__ =0; 1 5 0 .^ --
a a "j- x a “I- 2x 4 (x2—62)
—b , x
151. 1----- 152. a6 — 262 ac2— 26c2 ' 6c
2 a2 x2— 2ax ’
X 2x 5a2 x2 1 1 x
153. x-}-a + 4 (x2 — a2) ' 154. n2x — 2n 2-
a— i 26 «2 — 62
155. (a — x)2 156.1
a3-—ax (2a — x) ’ a2+ ^2— 2ax
1 i 2<n + 3) . a-\-x — 2n a —2a
157. 2n-f-aar 158. -1
2x ~ x 2 x3—4x ’ 2a —n x

159. -1; 160.


- 2«x2- f n2x2 x2-f- a2— 2ax 9x2
x + x2 i — a2 ab
161. 1 — x2 ' (l+ a x )2— (a-|-x)2 (6 — a)2
162. Factor the íollowing expression inlo linear factors:
iix - 3x2 + 70

163. Factor the expression —-j- into two factors, wliose sum is

±6 +' ±a
164. Factor lhe following expression:
lõx3 + x2 - 2x
165. Factor the following expression:
r> + 2x4 + 4x2 + 2 + x
24 Problem*

165a. Solve the equation


(1 + X2)2 = 4x (1 — X*)
166. Write a quadratic equation, whose roots are

■í. and —
b a

167. Set up a quadratic equation, whose roots are


--------- and ---------- —
to—y n to+ 6 y 2
168. Write a quadratic equation, whose roots are
_______ a_______
"j/ã ± V a— b
169. The roots x, and x , oí the quadratic equation
x1 + px + 12 = 0
possess the following property: xi — x 2 = 1. Find the coeflicient p.
170. In the equation
5x2 - kx + 1 = 0
determine k such that the difference of the roots be equal to unity.
171. The roots xj and x 2 of the equation
x2 — 3ax + n2 = 0
are such that x J+ x| = 1.75. Determine a.
172. In the quadratic equation
x2 + px + q = 0
determine the coefficients such that the roots be equal to p and g.
173. The roots of the quadratic equation
ox2 + bx + c = 0
are x, and x„. Set up a new quadratic equation, whose roots are —
and — .
174. Given a quadratic equation
ax2 + bx + c = 0
Set up a new quadratic equation, whose roots are:
(1) twice as large as the roots of the given equation;
(2) reciprocai to the roots of the given equation.
Chapter I I I . Algebraic Equaiions 25

175. Set up a quadratic equation, whose roots are equal to the


cubes of the roots of the equation
ax2 4- bx 4- c — 0
176. Set up a biquadratic equation, the sum of the squared roots
of which is 50, the product of the roots being equal to 144.
177. Find all the roots of the equation
Ax* - 2Ax3 + 57x2 4~ ÍSx - 45 = 0
if one of them is 3 + £|^6.
178. Determine the constant term of the equation
6x 3 — 7x2 — 16x 4* m = G
if it is known that one of its roots is equal to 2. Find the remaining
two roots.
179. Knowing that 2 and 3 are the roots of the equation
2x3 + mx2 — 13# -j- n ~ 0
determine m and n and find the third root of the equation.
180. At what numerical values of a does the equation
xz4-2ax Ya? — 34-4 = 0
have equal roots?
180a. In what interval must the number m vary so that both
roots of the equation
x2 — 2mx -f m2 — 1 — 0
lie between —2 and 4?

Solve the following equaiions:


181. { / r + 2 - K » - 6 = 2; 182. V 2 2 ~ x - Y W ^ x = 2
183. — l / í —T = 2; 184. K * + 3 + K 3 l ^ 2 = 7
185. | / Í T Í + V/ 2 Í + 3 = 1; 186. Y s Y ^ 2 = 2 Y 7 + 2 - 2
187. Y 2 x + i + V x ~ S ^ 2 Vx\ 188. V 1 + * K *s + 24 = i + 1

,8 9 .

x —5
190. ] / 7+2 ^ V *+ 3 z+ 2 / x -f 2
x-+
V x 2 — 16 7
191. 4" ~Yx 4~ 3 =ss
y7^3 Y7^3
26 Problems

x + "|/x2-f-x x— x
4Qq _____2________________\_______ J_
2 + 1 /4 — x2 2 — 1 /4 — *2 *

194. V 2 ^ 7 + K Í - / 2 / 7 - 1 / i = v^28

195. ] / x + j /a — \Vx — l / x = 4 - l / -----1


f ^ 'X + l/X

196. / ?T..± i ± . l ^ Lz£ = g!; 197. a r = a - ] / a 2- a : y rF + ^


1/27 + 1 — 1/27 — * 1 r
4 q8 V l + r ! J; - x a ~ ‘ 1 . <Qq l / l + a2*2 — a x ___1_
l / T + r ^ + x a -1 4 ’ ' l/l+ S S + o * c2

209 * / - c i~ V **2— c2 _ 9 (J + c)
x + c - l / * 2 — c2 8c

201. | / x - f 3 - 4 F 7 = T + ] / x ^ 8 - 6 l / ^ í = l

202. 2 J/"a + .r + Y a — x — ]/"a — x + Y x (« -f j )


203. ] / a2— ;c + y b 2— x == a + 6
204. J/a — a: + ]/*6 + x = Vr®4*fe
205. K J + 7 = o - l / ' í ; ' 206. 3 Í H I + 3 l í± í = ]/í
- _ 1 1
207. ) / * + y * = 12; 208. (a:— l) 3 -+ 6 (ar — 1)* = 16
209. / 2 + / I Õ + l x = — v^ V 1 5 - 2 x - 9
210. + ^ 2 7 ^ 3 = ^ 1 2 ( x - 1)
211. y d — x -\ -y f b — x — y a r h - 2x
212. y x + 2 y j *== 3; 213. 2 / z2- 3 / " i = 20
_ _ ?
214. V ã {-x — y/~a + x --=0;
215YW~ “ K 2x + 2 12
216. x2 -[• l l + F V - • 11 52 217. ,,
f ' X2— 1 í i+ i
218 T—* — T 8 • 21<l -x ) V “ -—x + {x-- b ) Y ^ b a t
1 X (-2 y r - x + Y * —b
Ckapter I I I . Algebraic Equations 27

* - i _ ..4 t-V *
220.— ~ = ] / 221.
2— y * F 2 i+ V x 2

222. / x 2- 3 x + 5 + * 2= 3x + 7

223. Va3i 2+ 5í - 8 - 1 / ' 3 x'- + 5x + 1 = 1

224. v<f + + 8 + V y 1+ + 4 = V %( f + iy + 8)

Solve the following Systems of equations:


|x2+ t/2= 2(xiH-2) f x 4- xo -f ü — 11
225. 226. T , . ,n
l * + !/ = 6 l x-y + xy- = 30
f x + y* = 7 f x2— y — 23
227.
{ xy1 — 12 228' | x2y = 50
| (x2- y 2)x y = 1 8 0 |3x2— 2xy + 5y2— 35 = 0
229.
lx 2— xy — y2 = — 11 1 5x2— 10j/2— 5 = 0
(x 2+ y'i = - jx y f x2+ xy 4- y2= 13
231. 232. " , .
l x - y = \xy { x+y=A

( X , ir _ 25
lx 2—xy + i/ = 7
233. 234. < » 1 12
1 x—y = l
*■ — y2~ 7

í (t )" -( í ) b- ‘

Give positive Solutions only, assuming that a > 0 , ò > 0 , c > 0 ,


d > 0 and rn^= n.
( x - - x y + y2==7 f x 3 + y> =
1 x3+ y3= 35 ' lx2/(x + y ) = —
Give real Solutions on ly .
'xy(x + y) = 30 x+y
x—y + i+ S
5 51
238. 239.
. x3+ y3 — 35
xy = 6
28 Problems

x + 2y + 3z + 4u = 30
x+ y+ z= i
í 2x — 3y + 5z — 2u = 3
240. ( ax + by + cz = d 241.
3x -J- 4y — 2z — u — 1
[a2x + b2y 4- c2z = d? 4x — g + 6 z — 3u = 8
x+ y+ z= 4 y 4 x + í ,_ 3 z + 7 = 2
242. •/ x + 2y + 3z = 5 243. % 2y + 5x + z 25.5 =
lx 2+ y2+ z2 = 14
Yy+ z—Y 6x = 0
x + y + z = 13 x2+ a2= z 2
244. ^ x2+ y 2+ z 2= 61 245. xy -j- yz -f- zx = 47
xy + xz = 2yz (z — x)(z — y) = 2
r 12 , 5
-+
d? -f- a-x + ay + z = 0 j V ~ i

{
] / y+
6s + 62x + 6y + z = 0 247. {
c3 + c"x + cy + z = 0 | -T7 -+
1 / : 9+^
248. |a: + y - 2 í/ ‘^ = 4
x + y = 10
3x
X + ií' 2+ ] / - 3x
249.
xi/ — í>4 =

250.
f r l '1— y—6
Y x t V -\~Yx — (/ = 4 Y a
251
Y
■{> X2 -r r - Yx?- y2= (K 41 - 3) a
252. { V x- + tf — / xs — f = y | x 2 + xj/ + !,2= 84
t/4 — 144a* ° ‘ \x + Y x y + y = l i
253a. Fiud all the values o{ m for wliich the system of equa-
tions
| x — i/ = m (l + xjf)
t2 + x + t/ + xi/ = 0
has real Solutions.
Chapter I V . Logarithmic and Exponential Equations 29

C H A P T E R IV

LOGARITHMIC AND EXPONENTIAL EQUATIONS


Determine x without using logarithmic tablesi

254*.z + l ( M 0 d 5 loí!S’ ~loe2; 255. x= 1002~1oí!Í 4

1/ 2+4l0Sl8
256. x = r 10 2 ; 257. ar= 49‘ - 1° « 2 + 5 - l°*»*

Solve the following equations:


258. logt log3log2x = 0
259. log„ {1 + log6 (1 + logc(1 + log„ i))} = 0
260. logt {2 log3[1 + log2(l + 3 log2x)]} = j
261. log2 (x + 14) + log2(x4-2) = 6
262. Ioga y + Ioga (y + 5) + log„ 0.02 = 0
263. ■*?8J3,5~ !.3) = 3
log(5 —x)
u (3a-(,)(a2 + a6)->-|
264. 1 - f logx = y logj^ 0 i)-2 J'
4 1
•jlogb + -jlo g (a3— ab-)

265. l o g [ x - a ( l - a ) - 2 ] - j l o g ( l + i )

/ '($ + a
-iog y ^ - « * = 0
■J+T’
266. log* V-> -I- log, (5x) - 2.25 = ( logl ^5)*
267. logwx + lo g 4x - f log2x = 7
268. log0x - lo g ..* 4 -lo g „ « x = -§-; 269. ( i ) 3*~7 = ( D ’ 1" 3

270. 7-3**‘ —5 '+í = 3*41- 5 xt3; 271. 0.125.4“ - 3= ^3^!|^

* Throughout thís book, the Symbol log stands for tho logarithm to tho
base 10.
30 Problems

x+5 «+17
272. 0.5*! .22*+2 = 64-1; 273. 32*~? = 0.25-128*-3

27/l- ( 4 ) X( f r ‘ - » 275' [ 2 ( 2 ^ + 3) ^ ] ^ :
_1
2
276. 2(2> ^ + 3) “ — / * ^ 4 3= 0

277. ^°8_sç_ x — 2
2 7 8 .3 log^sx +
n
279. Iog* (x + 12)•logx 2 = 1; 280. log* (Sx2) •log;x = 1
281. 1 + a + <r+ a3+ ... + a;'~1 + a ''= (l + a ) ( l + a 2) ( l + ff4) ( l + a s)
282 . 52•54•5° ■. . . •5!* = 0.04~S8; 283. 4 * -* -i7 .2 * -* + -.1= 0
284 . 2.4 s* - 1 7 - 4 * + 8 = 0; 2 8 5 .3 ^ 8 1 - 1 0 ^ 9 + 3 = 0
log x+7
286. X~ T ~ ^ iQloe K+i

287. log(4‘ *. 2 >« _ 1) - 1 = log (F 2 ' * - 2 + 2 ) - 2 log 2


288. 2 (log2 - 1 ) + lo g (5 ,/* 4 - l ) = log(5l~ ** + 5 )
289 rjiog«_$ « e x - i __^iog«+i__ -íogx-i

290. + io e »*-!.!> log* = j+ íõ ; 291. log (64 ^ P ^ 55*) = 0


292. !<>gz (9 — 2*) --=3 —x
293. log2 + log( + - - - + ) ) = 1 + log(2I- S+ 1)
294. 2 log 2 + ( l T + ) log 3 - log ( / 3 + 27) =--0

295. log (o 1 Tí+T_ 21" v, 47+T) - 2 = 1 log 10 — V x + 0.2.5 log 4


2 ío£ 2 + loij (x — 3} _ 1
296. Iüíí {l.r - f 1) -+ }og (x —0) 4- log 3 2
297. log, 120 | (,r - 3) - 2 log, (1 - 5*“*) - - log, (0.2 - Ê

Solvc the folloicing systems of equations:


f 3-*n 32-24»'~1 flog3x + log3g = 0
2 98 .
\ r» .5 * -» -= K 2 5 w '* l - * + ir -3 ( 3)
Chapter I V . Logarithmic anâ Exponential Equalions 31

[ 10g„ X + l 0ga !/= 2 f log(x2+ l/2) — l = logl3


300.
[logbX — logb y == 4 ■ llog(x + i/) —log(x—y) = 3log2
[lo g « (* -y ) = l 303 Jlogo ( l + y ) = 2 — loga y
302.
U o g „ (* + !/) = » ' llo g i)3; + logt(g = 4
f Ioga X + Ioga y + Ioga 4 = 2 + loga 9
304.
( x + i/ — 5a = 0
f xy = a2 f 3*.21'= 5 7 6
305.
( log2x + lo g 2 !/= 2.5 log2(a2) V °g v í (y — x ) — í

í log x + log y = log a ^ f Ioga * + IogaSjr = y


307.
12 (log x — log y) = log b |logbix + logj, y = y

| loga X + log»tg = -| 310- |log„U + logaiJ = 2


309.
llog(,i x — log6üy = 1 I u2+ d = 1 2
f x2+ xi/ + p2= a?
311.
l ° g y z V a + lo g y tV b — + 3
Iogax + log41/ + log4z = 2
log,.x — log2y = 0
312. 313. log3y + logo2 + logo x = 2
x2— 5y2+ 4 = 0
.logé 2 + logio X + logKy = 2
í * - ^ x + !/ = 2K 3
314.
l (* + </) 2 "-“ = 3

315. |,X 2 i V/ T ^ = ^ T51


1 log (x + y) = log 40 — log (x —y)
!f /V ^ 3 2 ^ W > 31? f 9"1^ 9 * — 27 ^ 2 7 “ = 0
316.
\ ^ /F = 3 jKíF'7' ‘ U o g ( x - l ) - l o g ( l - j /) = 0
i log x + j log y — log (4 — K l) = 0
318. flog*flg=P
319. < ,
(25 125-5 •'' = 0 Uogí 6x=9
32 Probíems

CHAPTER V
PROGRESSIONS

Notation and formulas


a, = first term of arithmetic progression
a„ — «th term of arithmetic progression
d = common difference of arithmetic progression
us = first term of geometric progression
u„ = «th term of geometric progression
q — common ratio of geometric progression
Sn = sum oí lhe first n terms of a progression
S = sum of infinitely decreasing geometric progression

Formulas for arithmetic progression


a„ = «, + d (« — 1) d)
S, (<n+ "n)"
2 ( 2)
2x1\~f- d (n—1)] n
S, (3)
2

Formulas for geometric progression


ir„ = ir (4)
S u = — IZ iíL ((/ > 1) (ir 5„ (9 < 1) (5)

Sn *■= (g > 1) « Sn = (q < 1) (6)

ARITHMETIC PROGRESSION
320. How many terms of the arithmetic progression
5; 9; 13; 17; . . .
is it necessary to take for their sum to equal 10,877?
321. Find an arithmetic progression, if the sum of its first four
terms is equal to 26, and the product of the same terms equals 880.
Chapter V. Progressions 33

322. In an arithmetic progression a p = q\ aq = p. Express a„


in terms of n, p, and q.
323. Find the sum of all two-digit natural numbers.
324. Find four successive odd numbers, if the sum of theirsquares
exceeds by 48 the sum of the squares of the even numbers contained
between them.
325. An arithmetic progression consists of 20 terms. The sum of
the terms occupying even places is equal to 250, and that of the
terms occupying odd places equals 220. Find the two médium terms
of the progression.
326. Given a sequence of expressions: (a + x)1; (a2 + /*);
(a — x)1-, . . . . Prove that they form an arithmetic progression, and
find the sum of its first n terms.
327. Denoting the sums of the first n,, first n2> and first n3 terms
of an arithmetic progression by St, S2, and S3, rcspectivcly, show
that
(m - n3) + (n3— n,) -j- (n,- n2) = 0
328. Write an arithmeUc progression whose first term is 1, the
|
sum of the first five terms being equal to of that of the ne.xt five
terms.
329. Find an arithmetic progression in which the sum of any
number of terms is always thrce times lhe squared number of these
terms.
330. Find the sum of all two-digit numbers which, when divided
by 4, yiold unity as a remainder.
GEOMETRIC PROGRESSION
331. Insert three geomelric means between the numbers 1 and 256.
332. Find the three numbers forming a geomelric progression.
if it is known that the sum of the first and third terms is equal
to 52, and the square of the second term is 100.
333. Write first several terms of a geometric progression in which
the difference between lhe third and first terms is equal to 9, and
that between the fifth and third terms equals 36.
334. Find the four numbers forming a geometric progression in
which the sum of the extremes is equal to 27, and lhe product of the
means, to 72.
335. Find the four numbers forming a geometric progression,
knowing that the sum of the extremes is equal to 35, and the sum
of the means, to 30.
3-01338
34 Problems

336. Determine a geometric progression in which


l/i -f- U2 4* «3 + «4 + Ui ~ 31
and
«2 r « 3 T iH + Ui - f « 6 = 62

337. A geometric progression consists of live terms; theirsum less


the first term is equal to 19 , and tliat lcss the last one equals 13.
Compute the extremes of thc progression.
338. Find the first term and common ratio of a geometric progres-
sion consisting of nine terms, such that the product of its extremes
is equal to 2304, and the sum of the fourtb and sixth terms equals 120.
339. Tliree numbers form a geometric progression. The sum of
these numbers is equal to 126, and their product, to 13,824. Find
lheso numbers.
340. A geometric progression consists of an even numbcr of terms.
The sum of all the terms is Uiree times that of the odd terms. Deter­
mine the common ratio of the progression.

IXITMTELY DECREASING GEOMETRIC PROGRESSION


341. Prove that the numbers
V f± ). i i.
V 2 -1 ’ 2 -V 2 ’ -
nmslitute an infinitely decreasing geometric progression and find
lhe limil of the sum of its terms.
342. Compute the expression

Í 4 T 3 : 8) [ ] ’ 3 (j 3 - 2 ) -

after proving that iho brackelod addends aro lhe? terms of a decrea-
sing geometric progression.
343. Find the sum of lhe lorms of an infinitely decreasing geo-
irndrie progression in which all lhe lorms aro positivo, th© first
term is 4, and the difference between the third and fifth terms is
equal to ~ .
344. Determine iho sum of an infinitely decreasing geometric
progression. if ii is known that Iho sum of its first and fourth terms
is equal to 54, and the sum of the second and third terms, to 36.
Chapter I*. Progressions 35

345. In an infinitely decreasing geometric progression the sum


of all the terms occupying odd piaces is equal to 36, and that of all
the terms at even piaces equal.s 12. Find the progrossion.
346. The sum of the terms of an infinitely decreasing geometric
progression is equal to 56, and the sum of the squared terms of the
same progression is 448. Find the first terin and the common ratio.
347. The sum of the terms of an infinitely decreasing geometric
progression is equal to 3, and the sum of the cubes of all its terms
equals ^ . Write the progression.
348. Determine an infinitely decreasing geometric progression,
the second tcrm of which is 6, the sum of the terms bcing equal to -~
of thal of the squares of the terms.

ARITHMETIC AND GEOMETRIC PROGRESSIONS


349. The second term of an arithmetic progression is 14, aiul the
Ihird one 16. It is required to sct up a geometric progression sudi
that its common ratio wouid be equal to the common difference of
the arithmetic progression, and the sum of the first three terms
wouid be the same in both progressions.
350. The first and Ihird terms of an arithmetic and a geometric
progressions are equal to each olher, respectively, the first terms
being equal to 3. Write lhese progressions, if the second term of lhe
arithmetic progression exceeds by 6 the second term of the geometric
progression.
351. In a geometric progression the first, third and fifth terms
may be considered as the first, fourth and sixteenlh terms of an
arithmetic progression. Determine the fourth term of this arithmetic
progression, knowing that its first term is 5.
352. Three numbers, whose sum is equal to 93, conslitute a geo­
metric progression. They may also be considered as the first, second
and scventh terms of an arithmetic progression. Find lhese numbers.
353. In an arithmeüc progression the first term is 1, and the sum
of the first seven terms is equal to 2555. Find lhe médium term
of a geometric progression consisting of seven terms, if lhe first
and the last terms coincide with the respectivo terms of the indicated
arithmetic progression.
354. The sum of the three numbers conslituting an arithmetic
progression is equal to 15. If 1, 4 and 19 are added to them, respecti­
vely, we wül then obtain three numbers forrning a geometric progres­
sion. Find lhese numbers.
3*
36 Problems

355. Find the three numbers constituting a geometric progression,


if it is known tbat the sum oí these numbers is equal to 26, and
tàat when 1, 6 and 3 are added to them, respectively, three new
numbers are obtained which form an arithmetic progression.
356. Three numbers form a geometric progression. If the third
term is decreased by 64, then the three numbers thus obtained wili
constitute an arithmetic progression. If then the second term of this
arithmetic progression is decreased by 8, a geometric progression
wili be formed again. Determine these numbers.
357. Can three numbers constitute an arithmetic and a geometric
progression at the same time?

C H A P T E S VI
COMBINATORICS AND NEWTON S BINOMIAL THEOREM
358. The number of permutations of n letters is to the number
of permutations of n + 2 letters as 0.1 to 3. Find n.
359. The number of combinalions of n elements taken three at
a time is five times loss than the number of combinations of n + 2
elements taken four at a time. Find n.
360. Find the médium term of the expausion of the binomial
' i 16

361. Determine the serial number of the term of the expansion


of the binomial j ' " , which contains a7.
362. Find the serial number of the term of the expansion of
/ ^/~ <i f b \si
the binomial ( 1 / —7= H -1 / -*7 = ) , which contains a and b to one
' r yb * y a1
and the same power.
-i a— 1 y
363. SimpUfy the expression ^- and deter­
« -« O
mine the term of the expansion that contains no a.
364. The exponent of one binomial exceeds that of the other by 3.
Determine theso exponents, if the snm of th© binomial coefficients
in the expansions of both binomials taken togother is equal to 144.
365. Find the thirteenth term of the expansion of ^9x — ,
if the binomial coefficient of the third term of the expansion is 105.
Chapter V I . Combinatorics and N ewton’ s Binomial Theorem 37

366. In tho expansion of + thecoeflicients at the fourth.


and thirteenth terms are equal to eacli other. Find the term con-
taining no x.
367. Find the médium term of the expansion of (a ~^a— I./" — ) ,
\ ' ~p a t
il it is known that the coefficient of the fifth term is to the
coelficient of the third term as 14 to 3.
368. The sum of the coefficients of the first, second and third
terms of the expansion of i~ ) ' s equal to 46. Find the term
containing no x.
369. Find lhe term of the expansion of the binomial {x]/' x +~y/ x)"‘
which contains x6, if the sum of aü the binomial coefficients is
equal to 128.
370. Find the sixth term of a geometríc progression, whose first
term is — and the common ratio is the complex number (1 + '/) .
371. Find the sevcnth term of a geometric progression, whose
common ratio is ( l + i ) , and the first term, i.
372. Al what value of n do tho coefficients of the second, third
and fourth terms of the expansion of the binomial (1 + x)n form
an arithmetic progression?
373. The coefficients of tho fifth, sixth and scventh terms of th e
expansion of the binomial (i + x)n constitute an arithmetic pro­
gression, Find n. ^_
f V (l ^ X+ 1 —:---- \ ^
374. In the cxpression \ * /gFT a V a< / determine x sucli
that the fourth term of the expansion of the binomial be equal to
56a5'5. _
375. In the expression ( 2 ^ 2 ~1+ ^ ã ) determine x such that
the third term o! the expansion of the binomial be equal to 240.
376. Determine x in the expression ^ 2 + -p= -)X> if in the
expansion of tho binomial tho ratio of the sevcnth term from the
beginning to the seventh term from the end is equal to — .
377. Find the value of x in the expression ( x - f x 10®*)6, the third
term of the expansion of which is 1,000,000.
38 Problems

1
378. Find the value of x in the expression r ( | /'í) 1<®* + 1-j-y ^ x j ,
the fourth term of the expansion of which is 200.
379. In the expression ^ -p = -+ zl0B ^ j 8 determine x such that
the third term of the expansion of the hinomial is equai to 36,000.
380. The sixth term of the expansion of the hinomial
- xs l0c * is 5600. Find x.
(tÍ .
381. The nintli term of the expansion of the hinomial

L ( V Í ) 51°bx + V x\
is 450. Find x.
382. Determine x, if the fourth term of the expansion of the

hinomial ( IO10» + "iõg^ jj” ) is 3,500,000.


383. Determine at what value of x in the expansion of the bino-
mial the term containing i to a power twice as
large as that of the succeedent term will be iess than the latter by 30.
384. Determine at what value of x the fourth term in the
expansion of the hinomial ^]'r2x~1 -r is 20 times greater
tlian lhe exponent of the hinomial, if the hinomial coefficient of the
fourth term is five times greater than that of the second term.
385. Find out at what values of x the difference between the
fourth and sixth terms in the expansion of the hinomial
( nug + -y~~ ) is equai to 56, if it is known that the exponent
of the hinomial m is less than the hinomial coefficient of the third
term in the expansion by 20.
386. Find out at what values of x the sum of the third
and fifth terms in the expansion of | 2* + ~y===•)"* is equai to 135,
if the sum of the hinomial coefficients of lhe last three terms is
equai io 22.
387. Determine at what x the sixth term in the expansion of
lhe hinomial [ í 'dSo!" :<,X) -J- |// 21*-2' ' 108 3]m is equai to 2i, if it
Chapter V II. Algebraíc and Arithmetic Problems 39

is known that lhe binomial coefficients of lhe second, third and


fourth terms in the expansion represent, respectivoly, lhe first.
third and fifth terms of an arilhmetic progression.
388. Determine al what vaiue of x the fourth term in the expansion
of the binomial
rpog
v j

is equal to 16.8, if il is known that -g- of lhe binomial coefficient


of the third term and the binomial coeffieients of the fourth and
fifth terms in the expansion constitute a geometric progression.
389. Determine at what x the diíference between the rime-fold
third term and the fifth term in the expansion of the binomial

prr+^T
is equal to 240, if it is known that the diíference between the loga-
rithm of the three-foid binomial coefficient of the fourth term and
the logarithm of the binomial coefficient of lhe second term in the
expansion is equal to 1.

C H A P T E R VII
ALGEBRAÍC a n d a r it h m e t íc p r o b l e m s *
390. Find the weight of an arlillery round, knowing that the charge
weighs 0.8 kg, the weight of the projectile is equal to ~ of the total
weíght of the round, and the weight of tho shelí is ~ of the weight
of the round.
391. At a certain factory women makc 35% of all the workers,
llie rest of the workers being men. The numbor of meu exceeds that
of women hy 252 persons. Determine the total amount of workers.

* We do not divide tbe problems into algebraíc and arithmetic ones, sinco
arithraetically solvable problems can aíways be solved algebraically, and vice
versa; the problems which are solved with the aid of equations may often bave
a simpier arithmetic solution. Under “Answers and Solutions” we sometimes give
arithmetic, and sometirnes, algebraíc Solutions, but this sbould not at all lay
any restraint on tbe studenfs initiative as to the choice of the inethod of solution.
40 Problems

392. A batch of goods was sold for 1386 roubles at a 10% profit.
Determine the prime cost of the goods.
393. A factory sold 3348 roubles worth of goods at a loss of 4% .
What was the prime cost of the goods?
394. If 34.2 kg of copper is extracted from 225 kg of ore, what
percentage of copper does the ore contain?
395. Prior to a price reduction, a package of cigarettes cost
29 kopecks. After the reduction, it cost 26 kopecks. What was the
price reduction in percent?
396. One kilogram of a commodity cost 6 roubles and 40 kopecks.
The price was then cut to 5 roubles and 70 kop. What was the price
reduction in percent?
397. The raisins obtained in drying some grapes amount to 32%
of the total weight of the grapes. What quantity of grapes must we
take to obtain 2 kg of raisins?
398. A group of tourists have to collect money for an excursion.
If each pays in 75 kopecks, there will be a déficit of 4.4 roubles;
if each pays in 80 kopecks, there will be an excess of 4.4 roubles.
How many persons take part in the excursion?
399. A number of persons were to pay equal amounts to a total
of 72 roubles. If there were 3 persons less, then each would
have to contributo 4 roubles more. How many people were
there?
400. Sixty copies of the first volume of a book and 75 copies of the
second volume cost a total of 405 roubles. Ilowever, a 15% discmmt
on the first volume and a 10% discount on the second volume reduce
the overall price to 355 roubles and 50 kopecks. Determine the price
of each volume.
401. An antique shop bought two items for 225 roubles and then
sold them and mado a profit of 40%. What did the shop pay for
each item, if the first of them yielded a profit of 25% and the second,
a profit of 50%?
402. Sea water contains 5 % (by weight) of salt. How many kilo-
grams of fresh water shouid be added to 40 kg of sea water for lhe
latter to contain 2% of salt?
403. The hypotenuso of a right-anglod triangle measures 3)/T>
metres. Determine the legs, if it is known tbat when one of them
1 2
is increased by 1 3 3 % and the other, by 1 6 y % ,th esu m of their
lengths *s equal to 14 metres.
404. Two sacks contain 140 kg of flour. Each will contain one anel
the samo amount, if we take 12.5% of the flour of the first sack
Chapter V II. Algebraic and Arithmetic Problems 4t

and put it into the second. How many kiiograms of flour does each
sack contain?
405. Two factories, A and B , undcrtook to fuifil an order in 12 days.
After two days factory A was ciosed down for repairs, whiie lactory B
eontinued fuifiíraent of the order. Knowing that B has an efficiencv
o
o f 6 6 y % of that of A , determine in how many days the order will
be completed.
406. In a mathematics test, 12% of the students of a class did
not solve the problems at all, 32% solved them wilh certain misla-
kes, and the rcmaining 14 students obtaincd correct Solutions. How
many students are there in the class?
407. A piece of a rail making 72% of lhe rail icngth is cut off.
The remaining pari weighs 45.2 kg. Determine the weight of the
cut-off piece.
408. A piece of a silver-copper alloy weighs 2 kg. The weight
2
of silver comes to 14 y % of that of copper. How much siiver is there
in this piece?
409. Three workers received a total of 4080 roublcs for a job. The
sums received by the first and the second workers stand in a ralio
1 3 1
of 7 y to 1 y . The money received by the third worker is 43 y % of
that of the first. What was each worker paid?
410. Three hoxes contain 04.2 kg of sugar. The second box con-
tains ~ of the contents of lhe first, and the third conlains 42 ~ % of
what there is in the second box. How much sugar is there in each box?
411. There is scrap oí two grades of sleel coniainirig 5% and 40%
of nickel. How much of each grade is rcquired to obtain 140 tons
of Steel contaiuing 30% of nickel?
412. A piece of a copper-tin alloy weighing 12 kg contains 45%
of copper. How much pure lin must be added to this piece to obtain
a new alloy with 40% of copper?
413. How much pure alcohol musl be added to 735 granis of a 10%
alcohol solution of iodine to obtain a 10% solution?
414. A piece oí a coppcr-zinc alloy weighing 24 kg was immersed
in water and lost 2~~ kg in weight. Determine the amount of copper
and zinc in the alloy, if it is known that in water, copper loses 11 -i-%
2
and zinc, 1 4 y % oí its weight.
42 Problems

415. Rails are to be Iaid in a 20 km long section of a single-track


railroad line. Rails are available in lengths of 25 and 12.5 metres.
If all 25-metre lengths are used, then 50% of the 12.5-metre lengths
2
will havo to be added. If all 12.5-metre lengths are Iaid, then 66-j- %
of the 25-melre lengths will have to be added. Determine the number
of rail lengths of each kind available.
416. After lhe gradnation exercises at a school the students exchan-
ged photographs. How many students were there, if a total of 870 pho-
tographs were exchanged?
417. The geometric mean of two numbers is greater by 12 than
the smaller number and the arithmetic mean of the same numbers
is smaller by 24 than the larger number. Find the two numbers.
418. Find three numbers, the second of which is greater than the
first by the amount. the third number is greater than lhe second, if
wo know that the product of the two smaller numbers is equal to
85 and the product of the two larger numbers equals 115.
419. The number a is the arithmetic mean of three numbers, and b
is the arithmetic mean of thoir squares. Express the arithmetic mean
of their pairwise products in terms of a and 6.
420. A rectangular sheet of tin with a perimeter of 96 cm is used
to make an open-top box so that a 4-cm square is cut out of each
corner of the sheet and the edges aresoldered together. What is the
size of the sheet used, if the box has a volume of 768 cm3?
421. Find a two-digit number, if the quotient obtained by dividing
2
this number by lhe product of its digits is equal to 2-g and, besides,
lhe difference betwecn the desired number and the number obtained
by reversing the order of the same digíts is 18.
422. Find u two-digit number, if we know that the number of
units lhercin exceeds by two the number of tens and that the product
of the desired number by the sum of its digits is equal to 144.
423. Determine a certain positive integor on the basis of the fol-
lowing data: if we adjoin the figure 5 on the right of it, the resulting
number is exactly divisible by a number exceeding the desired one
by 3, lhe quotient being equal to the divisor minus 16.
424. Find two two-digit numbers having the following property:
if we adjoin 0 followed by the smaller number on the right of the
larger one, and adjoin the larger number followed by 0 on the
right of the smaller one, then of the two five-digit numbers thus
obtained the first number divided by the second yields a quotient
of 2 and a remainder of 590. ít is also known, that the sum of the
Chapter V II. Algebraic and A rilhntelic Problems 43

two-fold larger desired number and the three-fold smallcr desired


number is equal to 72.
425. A student was asked to multiply 78 l)y a two-digil number
in which tho tens digil was three times as large as the units digit;
by mistakc, lie interchanged the digits in tho sccond factor and
tlrns obtained a product smaller than the true product by 2808.
VVhat was the true product?
426. Two raihvay stations are at a distance of 96 km írom each
other. One train covers this distance 40 minutes faster than does
the other. The speed of the first train is 12 km/h higher than tliat
of the second. Determine the speed of both trains.
427. Two persons simultaneously leave cities A and B and travei
towards each other. The first person traveis 2 km/h faster than
does the second and arrives in B onc hour before the second arrives
in A . A and B are 24 km apart. How inany kilometros does each
person make in ono hour?
428. The distance helwccn A and B by railway is 60 km and by
water, 80.5 km. A train leaves A foiir hours afler lhe departure of
a boat and arrives in B 15 minutes before tho boal. Determine the
mean speods of lhe train and the boal, if the former runs 30 km/h
faster than does the latter.
429. A lailor shop has an order for 810 siiits, another shop lias to
make 900 suits in lhe same period of time. The first shop has coin-
pleted its task 3 days beforo lhe target date, aml the second, 6 days
ahead of time. How many suits does each shop producc per day, il
the second shop makes 4 suits per day more than the first?
430. Two ships meei, one going off to the soutli and lhe other,
to the west. Two hours after Iheir encountcr, they are 60 km apart.
Find the speed of each ship, if it is known thal the speed of one
of them is 6 km/h higher than that of the other.
431. A dog at point A goes in pursuil of a fox 30 metros away. The
dog makes 2 m and lhe fox, 1 m long ieaps. If lhe dog makes two
leaps to the íox's three, at wliat distance from A will tíie dog catch
up with the fox?
432. Assuming that lhe hands of a clock move without jerks, how
long will it take for the minute hand to catch up willi lhe hour luind
if it was 4 o ’clock at the starting Ume.
433. A train iefl station A for C via B. The speed of the train in
the section from A to B was as requircd, bul it fell off by 25% in the
scetion belween B and C. On tho rolurn trip, the requircd speed was
maintainod between C and li, but decreased 25% between B and A.
How long did it take for the train to cover the distance from A to C,
44 Problems

if we know that the same time was spent on the A -B section as on the
B-C section and that on the A-lo-C section the train spent ^ of an
hour less than on the return trip (from C to *4)?
434. A cyclist has to make a trip of 30 km. He leaves 3 minutes
late, but traveis 1 km/h faster and arrives in time. Determine the
speed of the cyclist.
435. A fast train was held up by a red-light signal for 16 minutes
and made up for the lost time on a 80-km stretch travelling 10 km/h
faster than called for by schedule. What is the scheduled speed of the
train?
436. A train has to cover 840 km in a specified time. At the half-
distance point it was held up for half an hour and so, in the remaining
section of the route, it increased its speed by 2 km/h. How much
time did the train spend en route?
437. Two trains start out towards each other from points 650 km
apart. If they start out at the same time, they will meet in 10 hours,
but if one of them starts out 4 hours and 20 minutes before the other,
they will pass each other 8 hours following the departure of the
latler. Determine the mean speed of each train,
438. Two trains start out at the same time from stations A and B
600 km apart and run towards each other. The first train arrives
at B threc hours before the second arrives at A. The first train traveis
250 km in the time required for the second to cover 200 km. Find
the speed of each train.
439. A commuter walking to his train had covered 3.5 km in one
hour and then figured out that at such a rate he would be one hour
late. Thereforo, over the remainder of the distance he made 5 km/h
and arrived 30 minutes before the train’s loavingtime. Determine
the distance the commuter had to walk.
440. The distance between A and B is 19 km by highway. A cyclist
starts out from A at a conslant speed in lhe direction of B. A motor
car leaves A 15 minutes later in the same direction. In 10 minutes
it catchcs up with the cyclist and continues on to B, then turns
around and in 50 minutes after leaving A encounters the cyclist
a second time. Determine the speeds of the car and cyclist.
441. A mail train leaves station A at 5 a.m. for station B, 1080 km
away. At 8 a.m. a fast train leaves B for A and runs 15 km/h faster
than the mail train, When do the trains pass each other if this occurs
midway between A and i??
442. A is 78 km distant from B. A cyclist leaves A in the direc­
tion of B. One hour later, another cyclist leaves B in the direction
Ckapíer V II. Algebraic and A rithmetic Problems 45

of A and cycles 4 km/h faster than the first one. They meet 36 km
from B. How long is each one en route prior to the encounter and
what are their speeds?
443. Two hikers start out at the same time and, walking towards
each other, meet in 3 hours and 20 minutes. How long will ít take
for each hiker to cover the whole distance, if the first of thcm arrives
at the starting point oi the second 5 hours after the second arrives
at the starting point of the first?
444. Two hikers start out towards each other, one from A and the
other, from B. The first hiker starts from A six hours after the second
leaves B and when they meet it turns out,that he has covered
12 km less than the second hiker. After the encounter the hikers
continue walking at the same rate as before and the first of thcm
arrives at B eight hours later, the second arriving at A in 9 hours.
Determine the distance between A and B and the speed of the two
hikers.
445. A dirigible and an airplane are flying towards each other,
having left their terminais at the same time. When they meet, the
dirigible has made 100 km less than the airplane, and it arrives at the
departure point of the airplane thrcc hours after they pass each
other. The airplane arrives at the airport of the dirigible 1 liour
and 20 minutes after they pass each other. Find the speeds of the
airplane and lhe dirigible and the distance between the airports.
446. Two hikers leave A and B at the same time in lhe directíon
towards each other. When they meet, it turns out thal the first
hiker has covered a km more than the second. If they continue on
their ways at the same rate as before, the first hiker will arrive
at B in m hours and the second will arrive at A iri n hours after they
meet. Find the speed of each hiker.
447. Two bodies are moving along the circumference of a circie.
The first body inakes the whole circie 5 seconds fastor than the
second. If they both movo in one directíon, they will come together
evory 100 seconds. Wliat portion of the circumference (in degrees)
does each body make in one second?
448. Two bodies moving along lhe circumforenco of a circie in the
same directíon come together evory 56 minutes. If they were moving
with the same speeds as before, but in oppositc directions, they
would meet every 8 minutes. Also, when moving in opposite direc­
tions, the distance (along the circumference) between the approaching
bodies decreases from 40 metres to 26 metres in 24 seconds. What
is the speed of each body in metres per minuto and how long is the
circumference?
46 Problems

449. Two points are uniíormly moving in the same direction


along the circumferenee of a circle of length c and come together
every t seconds. Find the speed of each point, knowing that one of
them makes the whole circle n seconds faster than the other.
450. The distance between two towns along a river is 80 km.
A ship makes a round trip between the towns in 8 hours and 20 minu­
tes. Find the speed of the ship in still water, if the rate of the current
of water is taken to be 4 km/h.
451. A motor boat goes 28 km downstream and then returns irnme-
diately. The round trip takes 7 hours. Find the speed of the boat in
still water, if the rate of the current of water is 3 km/h.
452. A person boats from town A to town B and back in 10 hours.
The towns are 20 km apart. Find the rate of the current of water,
if we know that he boats 2 km upstream during the same time as he
does 3 km downstream.
453. A ship covers the distance between A and B in two days. The
return trip takes 3 days. Determine the time a raft will take to float.
down the river from A to B.
454. Two bodies, M t and M 2, are uniformly moving towards each
other from A and B 60 metresapart. M , slarts out from A 15 seconds
before M 2 starts out from B. At their respective terminais the two
bodies turn around and immediately go back at the same speeds as
before. Their firsl encounter takes place in 21 seconds and the second,
in 45 seconds after the start of M t. Find the speed of each body.
455. A road lcading from City A to city B first runs uphill for
3 km, Ihon it. is levei for 5 km and then runs downhill for 6 km.
A messenger sets out from A in the direction of B and having covered
half the distance, finds out that he must return to pick up some
packages he has forgotten. In 3 hours and 36 minutes after leaving
lie returns to A. Leaving A a second time, he reaches B in 3 hours
and 27 minutes and makes tlie return trip to A in 3 hours and 51 minu­
tes. Wlial is the speed of the messenger when going uphill, over
the levei ground and downhill, assuming that within the bounds
of each road section the speed remains constant?
456. A typist figures out that if she types 2 pages above her work
quota daily, she will complete her work 3 days ahead of schedule,
and if she makes 4 pages extra per day, she will finish 5 days ahead
of time. llow many pages does she liave to type and in what time?
457. A wurker made a certain number of identicai parts in a spe-
cified time. If he had prodnced 10 parts more every day, he would
have completed the job 4 y days ahead of schedule, and if he had
Chapter V II. Algebroíc and Arithmelic Problems 47

produced 5 paris less evcry day, he would have been 3 days beliínd
time. How many parts did he make and in what time?
458. A typist had to do a job in a specified Ume by typing a cer-
tain number of pages every day. Slie calculated tliat if she had typed
2 pages more than required per day, she would have completed
the task 2 days ahead of time, but if she liad turned out G0°ó of her
work quota, then she would have finished the job 4 days ahead of time
and made 8 pages more than required. What was the daily work
quota and in what time had the job to be compieted?
459. Two workers together complete a certain task in 8 hours.
Working individually, lhe first worker can do lhe job 12 hours
faster than can do the second. How many hours would it take eacli
worker to do the job individually?
460. A swimming pool is filled by two pipes in 6 hours. One pipe
alone fills it 5 hours faster than does the other pipe alone. How iong
W'ill it take for each pipe operating individually to fill the pool?
461. Two workers are given a task to make a batch of identical
paris. Aftor lhe first had worked for 7 and the second, for 4 hours,
they found out that SA> of the task had been compieted. Having
worked together for anolher 4 hours, lliey íigurcd out that V,8 of the
job had yet to be donc. How iong would it take each worker to do
the whole job individually?
462. Four identical hoisting cranes were being used to load a ship.
Aftor they had worked for 2 hours, anolher two cranes of a lower
capacity were put into operation, wilh the rcsult that the loading
operation was completed in three hours. If ail the cranes had begun
working at the saine Ume, the loading would have been completed
in 4.5 hours. Determine the Ume (in hours) required for one liigh-
power and one low-power cranc to do the job.
463. A task was set to dolivor a building material from a raihvay
slation to a construetion site in 8 hours. The material had to be
delivered witli 30 thrcc-ton trucks. Tliesc trucks worked for two
hours and then 9 five-ton trucks were added to hclp out. The task
was completed in time. If the five-ton trucks had begun the opera­
tion, and the lliroc-ton trucks had been brought two hours later,
then only 13/ , 5 of the material would have been delivered in the allot-
ted time. Determine how many hours it would take ono threc-ton
truck alone, one five-ton truck alone, and 30 five-ton trucks to deli-
ver ail the material.
464. Two typists underlake to do a job. The second typist hegins
working one hoor after the first. Three hours aftor lhe first typist
has begun working thero is stiil s/ 20 of lhe work to be done. Whon
48 Problems

the assignment is completed, it turns out that each typist has done
half the work. How many hours would it take eaeh one to do the
whole job individually?
465. Two trains start out from stations A and B towards each
olher, the second train leaving half an hour later than does the first.
Two hours after the first train had started, the distance between the
trains came to 19/30 of the entire distance between A and B. The
trains met midway between A and B. How much time would it
take each train to cover the distance between A and B?
466. A rectangular bath 20 cm x 90 cm x 25 cm (a rectangular
parallelepiped) is used to wash photographic negatives. Water flows
in through one pipe and, at the same time, out through another
pipe to ensure its constant agitation in the bath. It requires 5 minu­
tes less time to empty the bath through the second pipe than it does
tojfill it through the first pipe, the second being elosed. If both pipes
are open, a full bath will be emptied in one hour. Find the amount
of water each pipe lets pass through in one minute.
467. A construction job required the digging out of 8000 m3 of
earlh in a specified time. The operation was completed 8 days ahead
of time because the leam of navvies overfulfilled their plan by 50
cubic metres daily. Determine the original time limit for the
assignment, and daily overfulfillment of the plan in percent.
468. A raihvay was being repaired by two teams of workers.
Each repaired 10 km of the track despite the íact that the second
toam worked one day less than did the first. How manykilometres
of lhe track did each leam repair per day if both teams together
repaired 4.5 km daily?
469. Two workers together did a job in 12 hours. If at the begin-
ning the first worker had done half the assignment, and then the
second had completed the other half, the whole job would ha ve been
done in 25 hours. How long would it take each worker to do the
whole job individually?
470. Two Iractors of different performance characteristics, working
together, ploughcd a field in t days. If at firstone tractor had ploughed
half the field, and then the other one had completed the other
Half, the ploughing operation would have been completed in k days.
How many days would it take eacii tractor to plough the field indi­
vidually?
471. Three different dredgers were at work, deeponing the entrance
channel to a port. The first dredger, working alone, would have taken
10 days longer to do the job; the second, working alone, would have
required an extra 20 days, and the third dredger, working alone,
Chapter V II. Algebraic arid Arithmetic Problems 49

would have required six times more time tian needed for ail the three
machines operating simultaneously. How long would it have taken
each dredger io do the job individuaily?
472. Two workers, the second one beginning working 1 -™ days
after the first. can complete an nssignment in 7 days. If each of
them had done the job individuaily. the first worker would have
required 3 days more than would have the second. How many days
would it take each worker to do the job individuaily?
473. Two different tractors, working together, ploughed a field
in 8 days. If at first one tractor had ploughed half the field and the»
both tractors together had ploughed the other half. the whole job
would have been clone in 10 days. How many days would it take
each tractor to plough the field individuaily?
474. A number of men undcrtook to dig a ditch and could have
finished the job in 6 hours, ií they had begun working simultaneously,
but they began one after another. lhe intervals between their star-
ting times being equal. After the last worker had begun working.
a time interval of the same length elapsed and the job was finished,
each one of the parlicipants working till the completion of the job.
How long did they work, if the first worker to begin worked 5 times
as long as the last one to begin?
475. Three workers together can complete a task in t hours. The
first of them. working alone. can do lhe job twice as fast as the íliird
and one hour faster than the second. How long would it take each
worker to do the job individuaily?
476. A lank is filled with water from two taps. At lhe beginning
the first tap was open for one third of the time which would have
been needed to fill the tank, if the second tap alone had been open.
Then the second tap was open for one third of the time required to
fill the tank. if lhe first tap alone were open. This done, the tank
was i3/ 18 fuil. Compute the time required to fill the tank by each
tap separalely, if both taps together fill it in 3 hours and 36 minutes.
477. In lhe cunstruetion of an electric power station, a toam of
bricklayors was assigned the task of laying 120,000 bricks in a spe-
cified time. The learn completed lhe task 4 days ahead of lime.
Determine the daily quota of bricklaying and lhe actual number of
bricks laid, if it is known that in three days the toam laid 5000 bricks
more lha» required by the work quota for 4 days.
478. Three vessels contain water. If V3 of the water of the first
vessel is potired into the second. and then V4of the water now in the
second vessel i.s poiired into the third, and. finaiiy, V|0 of the water
4-0má
50 Problems

now in the third vessel is poured into the flrst, then each vessel
wül contain 9 litres. How much water was there originally in each
vessel?
479. A tank is filled with pure alcohol. Some of the alcohol is
poured out and replaced by an equal amount of water; the same
amount of the alcohol-water misture thus obtained is then poured
out, ieaving 49 litres of pure alcohol in the tank. The tank has a capa-
city of 64 litres. How much alcohol was poured out for the first
time and how much for the second time? (It is assumed that the
volume of the misture is equal to the sum of the volumes of the
alcohol and water; actually it is somewhat lesser.)
480. A 20-litre vessel is filled with alcohol. Some of the alcohol
is poured out into another vessel of an equal capacity, which is
then made full by adding water. The misture thus obtained is then
poured into the first vessel to capacity. Then 62/ 3 litres is poured
from the first vessel into the second. Both vessels now contain equal
amounts of alcohol. How much alcohol was originally poured from
(he first vessel into the second?
481. An 8-litre vessel is filled with air containing 16% of oxygen.
Some of the air is let out and replaced by an equal amount of nitro-
gen: then the same amount of the gas misture as before is let out
and again replaced by an equal amount of nitrogen. There is now 9%
of oxygen in the misture. Determine the amount of the gas misture
released from the vessel each time.
482. Two collective farmers together brought 100 eggs to market.
Having sold their eggs at different prices, both farmers made equal
surns of money. If the first farmer had sold as many eggs as the second,
sho would have received 72 roubles; if the second farmer had sold as
many eggs as the first. she would have received 32 roubles. How
many eggs did each une of thern have originally?
483. Two collective farmers with a total of a litres of milk, though
sclling lhe milk ai different prices, made equal sums of money. If
lhe first farmer had sold as much milk as the second, she would
have received m roubles, and if the second farmer had sold as much
milk as the first, she would have received n roubles (m > n). How
many litres of milk did each one of them have originally?
484. Two internai combustion engines of the same power output
were subjected to an efficiency test and it was found that ono of
them cunsumed 600 grams of petrol, while the other, which was in
operation 2 hours less, consumed 384 grams. If the first engine had
cousumed as much petrol per hour as the second, and the second,
as much as the first, then both engines would have consumed equal
Chapler V II. Algebraic and Aritkmetic Problems 51

amounts of petrol during the same period of operation as before.


How much petrol does each engine consume per hour?
485. There are two grades of gold-silver alloy. In one of them the
metais are in a ratio of 2 : 3 and in the other, in a ratio of 3 : 7.
How much of each alloy need we take to get 8 kg of a new alloy in
which the gold-to-silver ratio will be 5 to 11?
486. One barrei contains a nnxture of alcohol and water in a ratio
of 2 to 3, another barrei, in a ratio of 3 to 7. How many paiis need we
take from each barrei to obtaín 12 pails of a mixturc in which the
alcohol-to-water ratio is 3 to 5?
487. A certain alloy consists of two metais in a ratio of 1 to 2,
another alloy contains the same metais in a ratio of 2 to 3. How many
parts of botii alloys are needed to produce a tliird alloy containing
the metais in a ratio of 17 to 27?
488. Two wheels are sei in rotation by an endless belt: the smaller
wheel makes 400 rcvolutions per minute more than does the larger
wheel. The larger wheel makes 5 revolutions in a time interval that
is 1 second longer than that required for the smaller wheel to makc
5 revolutions. How many revolutions per minute does each wheel
make?
489. Over o distance of 18 metros tlio fronl wheel of a vehicle
makes 10 revolutions more than does the rear wheel. If the circum-
ference of tho front wheel were increasod by 6 decimetres, and the
circumference of tho rear wheel. reduced by (i decimetres, then over
the same distance the fronl wheel would completo 4 revolutions
more than would the rear one. Find t he circumfcrences of both wheels.
490. A barge with 600 tons of goods was unloaded in three days,
2/ 3 of the goods being unloaded during the first and thírd days. The
amount of goods unloaded during the second day was less than that
unloaded ori the first day, and the amount unloaded on the tbird
day was less than that unloaded on the second day. The diffcrence
between the percent reduction of tho amount of goods unloaded
on the thírd day with respect to that unloaded on the second day
and the percent reduction of lhe amount unloaded on the second
day with respect to that unloaded on lhe first day is equal to 5.
Determine how much w-as unloaded each day.
491. Two Solutions, the first containing 800 grams and the second,
600 grams of anhydrous sulphuric acíd, are mixed to produce 10 kg
of a new solution of sulphuric acid. Determino the weights of the
first and second Solutions in the mixturc, if it is known that tho
content of anhydrous sulphuric acp! in the first solution is lOpeVcent
greater than that in the second^^ntion.
/ O r ti ' ■ 4*

52 Problems

492. There were two different copper alloys, the íirst containíng
40 per cent less copper than the second. When these were melted
together, the resulting alloy contained 36 per cent of copper. Deter­
mine the percentage of copper in the first and second alloys, if it is
known that there were 6 kg of copper in the first alloy and 12 kg
in the second.
493. Two trains—a freight train 490 metres long and a passenger
train 210 metres long—were travelling along parallel tracks towards
each other. The driver of the passenger train noticed the freight
train when it was 700 metres away; 28 seconds later they passed
each other. Determine the speed of each train, if we know that the
freight train takes 35 seconds longer to pass the signal lights than
does the passenger train.
494. A freight train consists of four- and eight-wheel tank-cars
with oil. The train weighs 940 tons. It is reguired to determine the
number of the eight- and four-wheel tank-cars and also their weight,
if it is given that the number of the four-wheel cars is 5 more than
that of the eight-wheel cars; the eight-wheel car weighs three times
as much as the four-wheel car and the net weight of oil (that is,
minus the weight of the cars) in all the eight-wheel cars is 100 tons
more than the weight of all the loaded four-wheel cars. The eight-
wheel tank-car carries 40 tons of oil and the weight of the oil in the
four-wheel tank-car is 0.3 of that in the eight-wheel car.
495. The tunnel boring machines, working at the two ends of
a tunnel have to complete the driving in 60 days. If the first machine
0
does 30/0 of the work assigneâ to it, and the second, 26-g- %, then
both wiil drive 60 metres of the tunnel. If the íirst macliine had done
‘“/ 3 of the work assigned to the second one, and the second, 0.3 of
the work assigned to the first one, then the first machine would
have needed 6 days more than would have the second. Determine
how many metres of the tunnel are driven by each machine per day.
496. Two raihvav crews working together completed a repair job
on a track section in 6 days. To do 40% of the work the first crew
alone wouid require two days more than the second crew alone would
require to complete 13 y % of the wholo job. Determine how many
days it would take each crew to repair the wholo track section indi­
vidual iy.
497. Six humlred and ninety tons of goods were to be delivered
from a wharf to a raüway statíon by five 3-ton trucks and ten i-i-to n
trucks. In a fcw hours, the trucks transported 25/ ;a of the goods.
Chapter V II. Algebraic and Arithmetic Problems 53

To complete the deiivery in time, the rcmaining gouds had to be


transported in a time interval 2 hours less than that already spent.
The transportation was completed in time because the Iruck drivers
had begun making one trip per hour more than before. Determine
how many hours it took to transport ali the goods. and also lhe
number of trips per hour that were made originally, if it is known
that the 1 ^--ton trucks made one trip per hour more than did the
tiiree-ton trucks.
Note. It is assumed that ali the trucks were fully loaded on each trip.
498. A sports ground has the shape of a rectangle with sides of a
and b metres. It is bordered by a running-track whosc outer rim is
also a rectangle wbose sides are parallel to and equally spaced from
the sides of the ground. The area of the track is equal to that of the
ground. Find the width of the track.
499. An auditorium has a chairs arrauged in rows, the number of
chairs in each row being the same. If 6 chairs are added to each row
and the number of rows is reduced by c, then the total number of
places in the hall wili increase by one-tenth of their original number.
How many chairs are there in each row?
500. Two bodies spaced at d metres are moving lowards each other
and meet in a seconds. íf they move at the same speeds as before,
but in one direction, they wili meet in b seconds. Determine the
speed of each body.
501. A motorcyclist and a cyclist simultaneously slart out lowards
each other from point-s A and B d kilometros uparl. In two hours
they pass each other and continue on their wnys. The motorcyclist
arrives at B t hours before the cyclist arrives at A. Find the speed of
the two vehicies.
502. A hiker starts out from point A in lhe direction of B\ a hours
iater a cyclist starts out from B to meet the hiker and meets hirn ò
hours after the start. How long wili it take the cyclist and the hiker
to cover the whole distance between A and [B, if the cyclist re-
quires c hours less than does the hiker?
503. Train A , whosc speed is u km/h, departs after train B , wbose
speed is uf krn/h. The di(ference between lhe departure times (the
lag of train A ) is calculated so that both trains simultaneously
arrive at the deslmalion. Train B covers 2/ 3 of the distance and then
has to reduce its speed to half. As a result the trains meet a km
from the destination. Determine the distance to the terminai
stalion.
54 Problems

504. A man puts money in a savings bank and one year later
earns an interest of 15 roubles. Having added another 85 roubles,
he deposits ihe money for another year. After the expiry of this
period the sum-total of the principal and its interest is 420 roubles.
What sum of money was originally deposited and what interest
does the savings bank pay?
505. The output of machine-tool A is m% of the sum of the outputs
of machines B and C, and the output of B is n% of the sum of the
outputs o! .4 and C. What is the percentage of the output of C with
respect to the overall output of A and B ?
506. An increase in the output of a factory as compared to that
in the precediug year is p% for the firsl year and q% for the second
year. What should the percent increase of the output be for the
third year for the average annual increase of the output for three
years to be oqual to r%?
507. a% of some quantity of goods is sold at a profit of p% and b%
of the rest of the goods is sold at a profit of q%. What profit is made
on seiiing the remaining goods, if the total profit is r%?
508. Equai (by weight) pieces are cut off two chunks of alloys of
difforent copper conlent, the chunks weighing m kg and n kg. Each
of the cul-off pieces is melted together with the remainder of lhe
other chunk and the copper conlents of both alloys then become
equai. Find the weight of each of the cut-off pieces.
509. A certain sum of money was arranged in n piles. An ?ith
part of the nnmev in the first pile was taken from it and put into
the second pile. Then an «th part of the money in the enlarged second
pile was taken from it and put into the third pile. The same opera-
tion was continued from the third to the fourth pile, and so on.
Finally, an «th part of the money in the «th pile was taken from it
and put into ihe first pile. After this, final operalion each pile
had A roubles. How much money was there in each pile prior to the
shifting operaüon (you may confine yourself to n —5)?
PART T W 0

GEOMETRY AN D TRIGONOMETRY

CHAPTER VIII
PLANE GEOMETRY
510. The perimeter of a rigkt triangle is equal to 132, and the
sum of the squares of its sidos, to 6050. Find the sides.
511. Given in a parallelogram are: the acute angle a and the
distances m and p between the point of interseetion of the diagonais
and the uneqmil sides. Determine the diagonais and the area of the
parallelogram. ‘ 1
512. The base of an isosceies triangle is equal to 30 cm, and the
altitude, to 20 etn. Determine the altitude dropped to one of the
sides.
513. The base of a triangle is equal to 60 cm, altitude, to 12 cm
and the median drawn to the base, to 13 cm. Determine the sides.
514. On the sides of an isosceies right triangle with the leg b
three squares are constructod oulwards. The centres of these squares
are joined through slraight liues. Find the area of the triangle thus
obtained.
515. The sides of a square are divided in the ratfo m. to n. a large
and a small segments being adjacenf to each vertex. The successive
points of division are joined by slraight, Unes, Find the area of the
quadrilatcral obtained, if the side of lhe given square is equal to a.
516. Inseribed in a square is another square, whose vertíces lie
on the sides of the former square and the sides form 30-degroe angles
With thoso of the former square. What portion of the area of the
given square is the area of the insertbed square equal to?
517. Inscribed in a square with side a is another square, whose
vertico.s lie on lhe sides of the former. Determine the segments into
which the sides of Üie first square are divided by lhe vórtices of the
25
second square, if the area of lhe iallcr is equal lo ^ of lhat of the
former.
56 Problems

518. Inscribed in a rectangle with sides 3 m and 4 m long is another


rectangle, whose sides are in the ratio 1 : 3. Find the sides of this
rectangle.
519. Inscribed in an equilateral triangle ABC with side a is another
equilateral triangle LM N , whose vertices lie on the sides of the
first triangle and divide each of them in the ratio 1 : 2. Find the
area of the triangle LMN.
520. Find the sides of a right-angled triangle, given its perime-
ter 2p and altitude h.
521. Two equal segments CM and CN are marked on the sides CA
and CB of an isosceles triangle ABC. Determine the length of the
segments, knowing the perimeter 2P of the triangle ABC, its base
AB = 2a and the perimeter 2p of the rectangle AM N B cut off
by the straight line MN.
522. Given a right-angled trapezoid with bases a, b and shorter
side c. Determine the distance between the point of intersection
of the diagonais of the trapezoid and the base a, and between the
point of intersection and the shorter side.
523. Find the area of an isosceles triangle, if its base is 12 cm,
and the altitude is equal to the line-segment joining the mid-points
of the base and of one of the sides.
524. The perimeter of a rhombus is equal to 2p cm, and the sum
of its diagonais, to m cm. Find the area of the rhombus.
525. The longer base of a trapezoid is equal to a, and the shorter,
to b; the angles at the longer base are 30° and 45°. Find the area
of the trapezoid.
526. Compute the area of a trapezoid, whose parallel sides are
equal to 16 cm and 44 cm, and nonparallel ones, to 17 cm and 25 cm.
527. Find the area of a square inscribed in a regular triangle with
side a.
528. The base of a triangle is divided by the altitude into two parts
equal to 36 cm and 14 cm. A straight line drawn perpendicular
to lhe base divides the area of the given triangle into two equal
parts. Into what parts is the base of the triangle divided by this
line?
529. The altitude of a triangle is equal to 4; it divides the base
into two parts in the ratio 1 : 8. Find the length of the line-segment
which is parallel to the altitude and divides the triangle into equal
parts.
530. A triangle ABC is divided into three equal figures by straight
lines which are parallel to the side AC. Compute the parts into
which the sido AB, equal to a, is divided by the parallel lines.
Chapter V III. Plane Geomctry 57

531. A straight line parallel to the base of a triangle, whose area


is equal to S , cuts off it a triangle with an area equal to q. Determine
the area oí a quadrilateral, whose three vertices coincide with those
of the smaller triangle and the fourtli one lies on the base of the
larger triangle.
532. Parallel sides of a trapezoid are equal to a and b. Find the
length of the line-segment whích is parallel to them and divides
the area of the trapezoid into two equal parts.
533. Perpendiculars are drawn from the vertex of the obtuse
angle of a rhombus to its sides. The length of each perpendicular
is equal to a, the distance between their feet being equal to b. Deter­
mine the area of the rhombus.
534. Find the area of a triangle, if two of its sides are equal to 27 cm
and 29 cm, respectively, and the median drawn to the third side is
equal to 26 cm.
535. Given two sides b and c of a triangle and its area S — ^ bc.
Find the third side a of the triangle.
536. Given the bases a and b and sides c and d of a trapezoid.
Determine its diagonais m and n.
537. Given a parallelogram, whose acute angle is equal to 60°.
Determine the ratio of the lengths of its sides, if the ratio of the
19
squared lengths of its diagonais is equal to - j .
538. From an arbitrary point taken inside an isosceles triangle
perpendiculars are drawn to all the sides. Prove that the sum of the
three perpendiculars is equal to the altitude of the triangle.
539. Two secant lines are drawn from a point outsíde a circle.
The imernal segment (the chord) of the first secant is equal to 47jm,
and the externai one, to 9 m; the internai segment of the second
secant exceeds its externai segment by 72 m. Determine the length
of the sccond secant line.
540. From a point m cm dístant from the centre of a circle two
lines are drawn tangent to the circle. The distance between lhe
points of tangency is equal to o cm. Determine the radius of the
circle.
541. Given inside a circle, whose radius is equal to 13 cm, is
a point AI 5 cm distant from the centre of the circle.
A chord AB = 25 cm is drawn through] the point U . Find the length
of the segments into whích the chord AB is divided by the point M.
542. In an isosceles triangle the vertex angle is equal to a. Deter­
mine the ratio of the radii of the inscribed and circumscribed circles.
58 Problems

543. The sides of a triangle are: a — 13, b — 14, c — 15. Two


of them (a and b) are tangent to a circle, whose centre lies on the
third side. Determine the radius of the circle.
544. An isosceles triangle with a vertex angle of 120° is circum-
scribed about a circle of radius R. Find its sides.
545. On the larger leg of a right triangle, as on the diameter,
a semicircle is described. Find the semicircumference if the smaller
leg is equal to 30 cm, and the chord joining the vertex of the right
angle with the point of intersection of the hypotenuse and the semi­
circle is equal to 24 cm.
546. In a right-angled triangle a semicircle is inscribed so that
its diameter lies on the hypotenuse and its centre divides the latter
into two segments equal to 15 cm and 20 cm. Determine the length
of the arc of the semicircle between the points at which the legs
louch the semicircle.
547. In an isosceles triangle with the base equal to 4 cm and
altitude equal to 6 cm a semicircle is constructed on one of the
sides as on the diameter. The points at which the semicircle inter-
sects he base and the other side are joined by a straight line. Deter­
mine the area of the quadrilateral thus obtained, which is inscribed
in the semicircle.
548. Given an isosceles triangle with the base 2a and altitude h.
Inscribed in it is a circle, and a line tangent to the circle and parallel
to f he base of the triangle. Find the radius of the circle and the length
of the segment of the tangent line contained between the sides of the
triangle.
549. From a point lying wiíhout a circle two secant íines are drawn,
whose externai porfions are 2 m long. Determine the area of the
quadrilateral, whose verlices are the points of intersection of the
secants and the circle, if the iengths of its two opposite sides are
equal to 6 m and 2.4 m.
550. The sides of a triangle are equal to 6 cm, 7 cm, and 9 cm.
From its vórtices, as from centres, tkrec mutually tangent circles
are described: the circle, whose centre lies at the vertex of the least
angle of the triangle, is internally tangent to the remaining two
circles, the latter bcing externally tangent to each other. Find
the radii of the three circles.
551. An exterior tangent to two circles of radii 5 cm and 2 cra
is 1.5 times longer than their interior tangent. Determine the distan-
ce between the centres of the circles.
552. The distance between the centres of two circles, whose radii
aro equal to 17 cm and 10 cm, is 21 cm. Determine the distances
Chapter V III. Plane Geomelry 59

between the centres and the point at which the centre line inter-
sects a common tangent to the circles.
553. To two externaily tangent circles ol radii ft and r common
tangent lines are drawn: one interior and two exterior ones. Deter­
mine the length of the segment of the interior tangent line conta-
ined between the exterior tangents.
554. To two externaily tangent circles of radii R and r common
exterior tangent lines are drawn. Find the area of the trapezoid
bounded by the tangent lines and chords joining the points of tan-
gency.
555. Two circles of radii R and r are externaily tangent. A common
exterior tangent is drawn to these circles, thus forming a curvilinear
triangle. Find the radius of the circle inscribed in this triangle.
556. Through one and the same point of a circle two chords (equal
to a and b) are drawn. The area of the triangle formed by joining
their ends is equal to S. Determine the radius of the circle.
557. In a circle of radius R three parallel chords are drawn on one
side of its centre, whose lengths are respectively equal to those of the
sides of a regular hexagon. quadriiateral and triangle inscribed
in the circle. Determine the ratio of the area of the portion of the
circle contained between the second and third chords to that con-
tained between the first and second ones.
558. Determine the area of a circle inscribed in a right-angled
triangle, if the altitude drawn to the bypotenuse divides the latter
into two segments equal to 25.fi cm and 14.4 cm.
559. A circle is inscribed in a rhombns with side a and acute angle
equal to 60°. Determine the area of the reetangle, whose vertices lie
at the points of tangency of the circle and the sides of the rhombus.
560. Drawn to a circle of radius R are four tangent lines which
form a rhombus, whose larger diagonal is equal to AR. Determine
the area of each of the figures bounded by two tangents drawn frorn
a common point and the smallor arc of the circle contained between
the points of tangency.
561. The area of an isosceles trapezoid circumscribed about a circle
is equal to S. Determine the side of the trapezoid, if the acute
angle at its base is equal to Jt/6.
562. An isosceles trapezoid with an area of 20 cm* is circumscribed
about a circle of a radius of 2 cm. Find the sides of the trapezoid.
563. About a circle atrapezoid is circumscribed, whose uonparallel
sides form acute angles a and p with the larger of the parallel sides.
Determine the radius of the circle, if the area of the trapezoid is
equal to Q.
60 ProblemB

564. About a circle oí radius r a right-angled trapezoid is circum-


scribed, whose least side is equal to 3r/2. Find the area of the tra­
pezoid.
565. The centre of a circle inscribed in a right-angled trapezoid
is 2 cm and 4 cm distant from the end points of the larger of the
nonparallel sides. Find the area of the trapezoid.
566. A circle is inscribed in an equilateral triangle with side a.
Then three more circles are inscribed in the same triangle so that
they are tangent to the first one and to the sides of the triangle, and
then another three circles tangent to the above three circles and to
the sides of the triangle, and so forth. Find the total area of all the
inscribed circles (that is the limit of the som of the areas of the
inscribed circles).
567. A triangle ABC is inscribed in a circle; through the vertex A
a tangent line is drawn to intersect the extension of the side BC
at the point D. From the vertices B and C perpendiculars are dropped
to the tangent line, the shorter of these perpendiculars being equal
to 6 cm. Determine the area of the trapezoid formed by the perpen­
diculars, side BC and the segment of the tangent line, if BC — 5 cm,
AD — 5[/^6 em.
568. Three equal circles tangent to one another are inscribed in
a regular triangle, whose side is equal to a. Each of them is in con-
tact with two sides of the given triangle. Determine the radii of the
circles.
569. Inside an equilateral triangle with side a there are three equal
circles tangent to the sides of the triangle and mutually tangent
to one another. Find the area of the curvilinear triangle formed by
the ares of the mutually tangont circles (its vertices being the points
of tangency).
570. Inside a square with side a four equal circles are situated,
each of them touching two adjacent sides of the square and two
circles (out of the remaining three). Find the area of the curvilinear
quadrangle formed by the ares ot the tangent circles (its vertices
being the points of tangency of the circles).
571. Find the area of a segment, if its perimeter is equal to p,
and the arc, to 120°.
572. A circle of a radius of 4 cm is inscribed in a triangle. One
of its sides is divided by the point of tangency into two portions
equal to 6 cm and 8 cm. Find the lengths of the other two sides.
573. In an isosceles triangle a perpendicular dropped from the
vertex of an angle at the base to the opposite side divides the latter
in the ratio tn : n. Find the anglcs of the triangle.
Chapter V f í í . Plane Geomeiry 61

574. A chord perpendicular to the diameter divides it in the ratio


m : n. Determine each of the ares (arc measure) into whieh the circle
is dívided by the chord and diameter.
575. Determine the angle of a parallelogram, given its altitudes
ht and h2 and perimeter 2p.
576. In a riglit triangle find the ratio of the legs, if the altitude
and median emanating from the vertex of the right angle are in the
ratio 40 : 41.
577. In a right triangle the hypotenuse is equal to c, and one of
the acute angles. to a. Determine the radius of the inscribed circle.
578. The sides of a triangle are equal to 25 cm, 24 cm and 7 cin.
Determine the radii of the inscribed and circumscribed circles.
579. Determine the radii of two externallv tangenl circles, if the
distance between their centres is equal to d, and the angie between
the common exterior tangents, to cp.
580. Determine the angle of a rhombus, given its area Q and the
area of the inscribed circle S.
581. A regular 2n-gon is inscribed in a circle, and a regular n-gon
is circumscribed about the same circle. The difference between the
areas of the polygons is P. Determine the radius of the circle.
582. The midpoints of the sides of a regular n-gon are joined by
straight lines to form a new regular »-gon inscribed in the given one.
Find the ratio of their areas.
583. A circle is circmnscribcd about a regular n-gon with side a,
another circle is inscribed in it. Determine the area of the annulus
bounded by the circles and its width.
584. A circle is inscribed in a sector of radius R with a central
angle a. Determine the radius of the circle.
585. From one point two lines are drawn tangenl to a circle of
radius R. The angle between the tangents is 2a. Determine the
area bounded by the tangents and the arc of the circle.
586. A rhombus with the acute angle a and side a is divided into
three equal parts by straight Unes emanating from the vertex of
this angle. Determine the lengths of the line-segments.
587. A point is situated inside an angle of 60° at distances a and b
from its sides. Find the distance of this point from the vertex of the
given angle.
588. Determine the area of a triangle, given the lengths of its
sides a and 6, and the length í of the hisector of the angle between
these sides.
589. In an isosceies triangle the length of the side is equal to a,
and the length of the line-segment, drawn from the vertex of the
62 Problems

triangle to its base and dividing the vertex angle in the ratio 1 : 2,
is t. Find the area of the triangle.
590. Given the angles of a triangle, determine the angle between
the median and altitude drawn from the vertex of any angle.
591. The side of a regular triangle is equal to a. A circle of radius
is drawn from its centre. Determine the area of the portion of the
triangle outside this circle.
592. In a right-angled trapezoid, whose altitude is h, on the side,
which is not perpendicular to the base, as on the diameter, a circle
is drawn touching the opposite side of the trapezoid. Find the area
of the right-angled triangle, whose legs are the bases of the trapezoid.
593. Prove that in a right-angled triangle the bisector of the right
angle bisects the angle between the median and altitude dropped
to the hypotenuse.
594. Prove that in a right-angled triangle the sum of the legs
is equal to the sum of the diameters of the inscribed and circumscribed
circies.
595. Determine the angles of a right-angled triangle if the ratio
of the radii of the circumscribed and inscribed circies is 5 : 2.
596. Prove that the straight lines successively joining the centres
of the squares constructod on the sides of a parailelogram and adjíSi-
ning it from outside also form a square.

C H J P T E E IX
POLYHEDRONS
597. The sides of the base of a rectangular parallelepipcd are a and
b. The diagonal of the parallelepiped is inclined to the plane of the
base at an angle a. Determine the lateral area of the parallelepiped.
598. In a regular hexagonal prism the longest diagonal having
lengtli d forms an angle a with the lateral edge of the prism. Deter­
mine the volume of the prism.
599. in a regular quadrangular pyramid the lateral edge of length m
is inclined to the plane of the base at an angle a. Find the volume
of the pyromid.
600. The volume of a regular quadrangular pyramid is equal to V.
The angle of inclination of its lateral edge to the plane of the base
is equal to a. Find the lateral edge of the pyramid.
(501. The lateral area oi a regular quadrangular pyramid is equal
to ,S' cm2, its altitude, to H cm. Find the side of its base.
Chapter I X , Polyhedrons 63

602. Find the volume and lateral area of a regular hexagonal


pyramid, given the lateral edge l and diameter d of the circle inseri-
bed in the base of the pyramid.
603. Find the altitude of a regular telrahedron, whose volume is
equal to V.
604. In a rigkt parallelepiped the sides of the base are equal to a
and ò, and the acute angle, to a. The larger diagonal of the base
is equal Io the smalier diagonal of the parallelepiped. Find the
volume of the parallelepiped.
605. The diagonais of a right parallelepiped are equal to 9 cm
and \ 33 cm. The perimeter of its base is equal to 18 cm. The lateral
edge is equal to 4 cm. Determine the total suríacc area and volume
of the parallelepiped.
606. The lateral edge of a regular triangular pyramid is equal to /,
its altitude, to h, Determine the dihedral angle at the base.
607. Determine the volume oí a regular quadrangular pyramid,
given the angle a between its lateral edge and the plane of the base,
and the area S of its diagonal section. Find also the angle formed by
the lateral face and the plane containing the base.
608. The base of a regular pyramid is a polygon. the sum of inte­
rior angles of which is equal to 540°. Determine the volume of the
pyramid if its lateral edge, equal to l, is inclined to the plane of the
base at an angle a.
609. Determine the angles between the base and lateral edge. and
between the base and lateral face in a regular pentagonal pyramid,
whose lateral faces are equilateral triangles.
610. Given the volume V oí a regular n-gonal pyramid in which
the sido of the base is equal to a, determine the angle of inclinaiion
of the lateral edge of lhe pyramid to the plane containing lhe
base.
611. The base of a quadrangular pyramid is a rectangle with the
diagonal equal to b and the angle a between the diagonais. Eaeh
of the lateral eriges forrns an angle p witii the base. Find the volume
of the pyramid.
612. The base of a pyramid is an isosceles trianglo with the equal
side.s of a and the angle between íhem equal to a. AU lateral edges
are inclined to the base at an angle p. Determine the volume of the
pyramid.
613. The base of o rectangular parallelepiped is a rectangle inscri-
bed in a circle of railius /?, the smalier sido of this rectangle'subten-
ding a circular arc equal lo (2a)". Find the volume of the#parallele-
piped, given its lateral area S.
64 Problems

614. The base oí a right prism is an isosceles triangle, whose base


is equal to a and the angle at the base, to a. Determine the volume
of the prism ií its lateral area is equal to the sum of the areas of its
bases.
615. The slant height of a regular hexagonal pyramid is equal
to m. The dihedral angle at the base is equal to a. Find the total
surface area of the pyramid.
616. Through the hypotenuse of a right-angled isosceles triangle
a plane P is drawn at an angle a to the plane of the triangle. Deter­
mine the perimeter and area of the figure obtained by projecting
the triangle on the plane P. The hypotenuse of the triangle is
equal to c.
617. In a regular n-gonal pyramid the area of the base is equal
to Qr and the altitude forms an angle <p with each of the lateral
faces. Determine the lateral and total surface areas of the pyramid.
618. The side of the base of a regular triangular pyramid is equal
to a, the lateral face is inclined to the plane of the base at an angle
of cp. Find the volume and total surface area of the pyramid.
619. The total surface area of a regular triangular pyramid is
equal to S. Find the side of its base, if the angle betweenthe lateral
face and the base of the pyramid is equal to a.
620. The base of a pyramid is a rhombus with the acuto angle a.
The lateral faces are inclined to the plane of the base at an angle (5.
Determine the volume and total surface area of the pyramid, if the
radius of the circle inscribed in the rhombus is equal to r.
621. Determine the angle of inclination of the lateral face of
a regular pentagonal pyramid to the plane of the base, if the area
nf the base of the pyramid is equal to S, and its lateral area, to o.
622. The base of a right parallelepiped is a rhombus. A plane
drawn through one of the sides of the lower base and the opposito
side of the upper base forms an angle p with the plane containing
lhe base. The area of the section thns obtained is equal to Q. Deter­
mine the lateral area of the parallelepiped.
623. The base of a pyramid is an isosceles triangle with the base
angle a. Each of the dihedral angles at the base is equal to <p. The
distanco bctween the centre of the circle inscribed in the base of the
pyramid and the midpoint of the height of the lateral face is equal
to d. Determine the total surface area of lhe pyramid.
624. The base of a pyramid is a polygon circumscribed about
a circle of radius r; the perimeter of the polygon is equal to 2p,
the lateral faces of the pyramid are inclined to the base at. an angle qi.
Find the volume of the pyramid.
IX . Pohjltedrons

625. Tho lateral edges of a fnistum of a regular triangular pyra-


raid are inclined to tho base at an angle a.The sido of tho lower base
is equal to a, and that of tho upper ono, to b (a >■ b). Fiml tho volume
of the fruslum.
626. The bases of a frusium of a regular pyramid are squares wilh
sides a and b (a > b). The lateral edges are inclined to the base at an
angle a. Determine the volume of the frustmn and the dihedrai angies
at the sides of the bases.
627. The base of a pyramid is a right-angied triaugle, wiu.se
hypotenuse is equal to c and acule angle. to a. Ali the lateral edges
are inclined to the base at an angle (5. Find lhe volume of tho ;>yra-
mid and the face angies at iO vorlex.
628. The base of an oblíquo prism is a right-angied triaugle ABC
the smn of the legs of whicb is equal to m, and the angle at lhe
vertex A, to a. The lateral face oí the prism passiug through the
leg AC is inclined to the base at an angle fT A plane is druwn tbrougii
the hypotenuse AB and the vertex C’i of the opposite trihedral angle.
Determine lhe volume of the eut-ofí triangular pyramid, if il is
known lhal it has equal edges.
629. The base of a pyramid is an isosceles triaugle wilh the base
angle a. All the lateral edges are inclined to the plane containing
the base at equal angies <p ~ 90° — a. The area of the sectiun pas-
sing through the altitude of the pyramid and the vertex of the base
(isosceles triaugle) is equal to Q. Determine the volume of the pyra­
mid.
630. The base of a pyramid is a rectangle. Two of the lateral faces
are perpendicular to the base, the olher two forming angies a and [5
wilh it. The altitude of the pyramid is equal to //. Determine the
volume of lhe pyramid.
631. The base of a pyramid is a square. Out of two opposite
edges one is perpendicular to the base, lhe olher is inclined to it at
an angle ^ and has a lenglh /. Determine the lengths of the remaining
lateral edges and the angies of lheir inclination to the base of
the pyramid.
632. The base of a pyramid is a regular triaugle wilh side a. One
of the lateral edges is perpendicular to the base, the other two being
inclined to lhe base at equal angies (1. Find the surfaco area of the
largcst lateral face of the pyramid and the angle of its inclination
to the base.
633. The base of a pyramid is an isosceles triangle; the equal
sides of the base are of lenglh a and forni an angle of 120*. The
lateral edge of lhe pyramid. passing through the vertex of the ob-
'-0 1 3 3 8
66 Problems

tuse angle, is perpendicular to the plane of the base, the other two
being inclined to it at an angle a. Determine the area of the section
of the pyramid by a cutting plane which passes through the largest
sitie of the base of the pyramid and bisects the edge perpendicular
to the base.
634. A regular triangular pyramid is cut by a plane perpendicular
to the base and bisecting two sides of the base. Determine the volume
of the cut-off pyramid, given the side a of the base of the original
pyramid and dihedral angle a at the base.
635. Through the vertex of a regular quadrangular pyramid a cut­
ting plane is drawn parallel to a side of the base and at an angle tp
to the base of the pyramid. The side of the base of the pyramid is
equal to «, and the face angle at the vertex of the pyramid, to a.
Find the area of tho section.
636. A plane is drawn through the vertex of a regular triangular
pyramid and lhe midpoints of two sides of the base. Determine the
area of the section figure and volumes of the portions of the given
pyramid into which it is divided by the cutting plane, given the side
a of the base and angle a formed by the cutting plane with the base.
637. A regular tetrahedron, whose edge is equal to a, is cut by a
plane containing one of its edges and dividing the opposite edge in
lhe ratio 2 :1 . Determine the area of the section figure and its
angles.
638. Determine the volume of a frustum of a regular quadrangular
pyramid, if the side of the larger base is equal to a, the side of the
smaller base, to 6, and the acute angle of the lateral face, to a.
639. Determine the volume of a regular quadrangular prism,
if its diagonal forms an angle a with the lateral face, and the side of
the base is equal to b.
640. The base of a right prism is a right-angled triangle with hypo-
teniiso c and acute angle a. Through the hypotennse of the lower
base and tho vertex of the right angle of tho upper base a plane is
drawn to form an angle fl with the base. Determine the volume of the
triangular pyramid cut off the prism by the plane.
641. The base of a right. prism is a right-angled triangle in which
lhe smn of a leg and the hypotenuse is equal to m, and the angle be-
t ween them, toa . Through the other leg and the vertex of the opposite
trihedral angle of the prism a plane is drawn at an angle (i to the
base. Determine the volume of the portions into which the prism is
divided by tho cutting plane.
642. The base of a pyramid is an isosceles triangle with the base
angle a. Each dihedral angle at the base is equal to <p = 90° —a.
Ch apíer ! X . Poli/lt edrons 07

The lateral area of the pyramid is S. Determine the volume oí the py-
ramid and its total surface area.
643. The base of a pyramid is an isosceles triangie with the side a
and the base angle a (a > 45°). The lateral edges are inclined to the
base at an angle p. A cutting plane is drawn through the altitude of
the pyramid and the vertex of one of the angles a. Find the area of
the section figure.
644. The base of a right pristn is a quadrilatcral in whicli two
opposite angles are right ones. Its diagonal joining the vertices of
oblique angles has a length l and divides one of tlicm into portions
a and p. The area of the section figure contaíned ín a cutting plane
passing through the other diagonal of the base and perpendicular
to it is equal to S. Find the volume of the prism.
645. The base of a pyramid is a square. Two opposite faces are
isosceles triangles; one of thern forms an interior angle p with the
base, the other, an exterior acute angle a. The altitude of the pyra­
mid is equal to H. Find the volume of the pyramid and the angles
formed by the other two lateral faces with the plane containing the
base.
646. The base of a pyramid is a rectangle. One of the lateral faces
is inclined to the base at an angle p = 90J— a and the face opposite
it is perpendicular to tiie base and represents a right-angled triangie
with the right angle at the vertex of the pyramid and an acute angle
equal to a. The sum of the heights of these two faces is equal to m.
Determine the volume of the pyramid and the sum of the areas of the
other two lateral faces.
647. The base of a pyramid is a rectangle. One of the lateral faces
is an isosceles triangie perpendicular to the base; in the other face,
which is opposite the first one, the lateral edges, equal to b. form an
angle 2a and are inclined to the first face at an angle a. Determine
the volume of the pyramid and the angle between the above two faces.
648. In a regular triangular pyramid, with the side of the base
equal to a, the angles between the edges at its vertex are equal to one
another, each being equal to a (a < 90°). Determine the angles bet­
ween the lateral faces of the pyramid and the area of a section drawn
through one of the sides of the base and perpendicular to the opposite
lateral edge.
649. Determine the volume of a regular octahedron with edge
a and also the dihedral angles at its edges.
650. The dihedral angle at a lateral edge of a regular hexagona!
pyramid is equal to <p. Determine the face angle at the vertex of the
pyramid.
5*
08 Problems

651. The base o£ a pyramid is a regular hexagon ABCDEF. The


lateral edge MA is perpendicular to the base, and the opposite edge
MD is inclined to the base at an angle a. Determine the angle o!
inclination of the lateral faces to the base.
652. The base of a pyramid is an isosceles triangle ABC in which
AB ~ AC. The altitude of the pyramid SO passes through the mid-
point of the altitude AD of the base. Through the side BC a plane is
drawn perpendicular to the lateral edge ,45 and at an angle a to the
base. Determine the volume of the pyramid cut off the given one and
having vcrte.v 5 in common with H, if the volume of the other cut-off
portion is equal to V.
653. The side of the base of a regular triangular pyramid is equal
to a. A section bisecting an angle between the lateral faces repre-
sents a right-angled triangle. Determine the volume of the pyramid
and the angle between its lateral face and the plane containing the
base.
654. Through a side of the base of a regular triangular pyramid
a plane is drawn perpendicular to the opposite lateral edge. Deter­
mine the total surface area of the pyramid. if the plane divides the
lateral edge in the ralio m :n, and the side of the base is equal
to q.
655. The diagonal of a rectangular parallelepiped is equal to d
and forms equal angles a with two adjacent lateral faces. Determine
the volume of the parallelepiped and the angle between the base
aml a plane passing through the end points of three edges emanating
frum one vertex.
656. In a reclangnlar parallelepiped the point of intersection of the
diagonais of the lower base is joined with the midpoint of one of the
lateral edges by a siraiglit line, whose lenglh is equal to m. Tbis line
forms an angle a with the base and angle ($ = 2a with one of the la­
teral faces. Tíiking lhe other adjacent lateral face for the base of the
parallelepiped. find its lateral area and volume. (Prove that a < 30°.)
657. The base of a riglit prism is a trapezoid inseribed in a semicir-
cle of radins B sn Ibat its larger base coincides with the diameter.
aml t iiosmailer one subtends an are equal to2a. Determine the volume
of the prism. if lhe diagonal of a face passing through one of the
imiiparaliel sides of the base is inclined to the latter at an angle a.
65S. The diagonal of a rectangular parallelepiped, equal to d.
forms an angle p Ü05 — o with the lateral face. The plane drawn
through this diagonal and the lateral edge inlersecting with it forms
an angle a with the saine lateral face (prove that a > 45°). Determine
lhe volume uf the parallelepiped.
C h o p lc r I S . I' u h/hcdrt CO

659. In a regular triangular prism two vertices of the uppcr base


are joined with the midpoints of the opposite .sidos of the lower base
by straight lines. The angle between these lines which faces the base
is equal to a. The side of tlie base js equal to b. Determine the volume
of the prism.
660. In a regular triangular prism the angle between a diagonal
of a lateral face and anolher lateral face is equal to a. Determine the
lateral area of the prism. if the edge of the base is equal to a.
661. The base of a righl prism is a right-angled trjangle ABC
in which C —D0C. /_A — a and the leg AC — b. The diagonal of
the lateral face of the prism which passes through the hypotenuse
A B , forms an angle p with the lateral face passing thrmigh fhe ieg
AC. Find the volume of the prism.
662. The total surface area of a regular quadrangular pyramid is
equal to.S\ and the face angle at lhe vertex, to a. Find the altitude of
the pyramid.
663. In a regular n-gonal pyramid the face angle at the vertex is
equal to a, and the side of the base, to a. Determine the volume of (he
pyramid.
664. In a regular quadrangular prism a plane is drawu through
a diagonal of lhe lower base and one of the vertices of the upper base,
which cuts off a pyramid with a total surface area S. Find the total
surface area of the prism, if the angle at the vertex of the triangle
obtained in the section is equal to a.
665. The lateral edges of a triangular pyramid are of equal length
l. Out of the tliree face angles formed by these edges at the vertex of
the pyramid two are equal toa , and the third, to p. Find the volume
of the pyramid.
666. The base of a pyramid is a right-angled triangle, which is
a projeclion of the lateral face passing through a leg. The angle oppo-
site this leg in the base of the pyramid is equal to a, and the one lying
in the lateral face is equal to p. The area of this lateral face exeoods
that of the base by S. Determine lhe difference between the areas
of the other two faces and the angles formed by lhe lateral faces with
the base.
667. In a triangular pyramid two lateral faces are isosceles right-
angled triangles, whose hypoteriuses are equal to b and form an
angle a. Determine the volume of the pyramid.
668. ín a pyramid with a rectangulur base each of the lateral edges
is equal to /; one of lhe face angles at the vertex is equal to a, the
other, to p. Determine the area of lhe section passing through lhe
hisectors of the angles equal to p.
70 Pmblems

669. In a parallelepiped the lengths of three edges emanating from


a common vertex are respectively equal to a, b and c. The edges
a and b are mutually perpendicular, and the edge c forms an angle
a with each of them. Determine the volume of the parallelepiped, its
lateral area and the angle between the edge c and the plane containing
the base. (For what values of the angle a is the problem solvable?)
670. All the faces of a parallelepiped are equal rhombuses with si-
des a and acule angles a. Determine the volume of the paralle­
lepiped.
671. The base of an oblique parallelepiped is a rhombus ABCD
with the side a and acute angle a. The eágeAAi is equal to b and
forms an angle q>with the edges AB and AD. Determine the volume of
the parallelepiped.
672. In a reclangular parallelepiped a plane is drawn Ihrough
a diagonal of the base and a diagonal of the larger lateral face, botli
emanating from one vertex. The angle between these diagonais is
equal to p. Determine the lateral area of the parallelepiped, the area
of the section figure and the angle of inclination of the cutting plane
to the base, if il is known that the radius of the circle circumscribed
about the base of the parallelepiped is equal to R and the smaller
angle between the diagonais of the base, to 2a.
673. The base of a right prism is a right-angled triangle ABC.
The radius of the circle circumscribed about it is equal to R. the
leg AC snhlemls an arc equal to 2p. Through a diagonal of the late­
ral face passing through the other leg BC a plano is drawn perpendi­
cular to Ihis face and inclined to the base at an angle fS. Determine
the lateral area of the prism and the volume of the cut-off quadrangu-
lar pyramid.
674. The base of a pyramid is a trapezoid, whose nonparallel sídes
and smaller base are of equal length. The larger base of the trapezoid
is equal to a. and the obtuse angle, to a. Al! the lateral edges of the
pyramid are inclined to the base at an angle p. Determine the volume
of the pyramid.
675. The base of a pyramid is a trapezoid, whose diagonal is per­
pendicular to one of the nonparallel sides and forms an angle a with
the base. All lhe lateral edges are of equal length. The lateral face
passing through the larger base of the trapezoid has an angle <p = 2a
at the vertex of lhe pyramid and its area is equal to S. Determine
the volume of lhe pyramid and the angles at which the lateral faces
are inclined Io the base.
676. The base o[ a pyramid is a regular triangle, whose side is equal
to a. The altitude, dropped from the vertex of the pyramid, passes
Chapter ÍX . Polt/hedruiis 71

through one of lhe vertices of thc base. Tbe lateral face passing
through tho side of the base opposite this vertex is at an angle g> to
the base. Determine the lateral area of the pyramid, if one of the
equal lateral faces is taken as the base.
677. The base of a right prism is an isosceles triangle witli the equal
sides of lengtli a and lhe base angle a. Through the base of the trian­
gle, which is the nppcr base of the prism, and the opposite vertex of
the lower base a ctittirig plane i? draxvn at an angle f) to the base.
Determine tbe lateral area of the prism and the volume of the cut-off
quadrangular pyramid.
678. The base of a pyramid is a squarc. Its two lateral faces are
perpendicular to the base, and the remaining tno are inclined to
it at an angle a. The radius of the circle circumscribed about the late­
ral face perpendicular to the base is equal to R. Determine the total
surface area of thc pyramid.
679. The base of a right prism is a right-angled triangle with a leg
a and angle a opposite it. Through thc vertex of tbe right angle of
the lower base a plane is drawn which is parallel to the hypotcnuse
and intersects the opposite lateral face at an angle P — 90° — a.
Determine tbe volume of the portion of the prism contaíned between
its base and the cutting plane and the lateral area of the prism. if
the area of the lateral face passing through the leg a is equal to
the area of the seclion figure. Determine the value of thc angle a at
which thc cutting plane intersects the lateral face passing through
the hypotcnuse of the base.
680. The base of a pyramid is a rectangle. One lateral edge is per­
pendicular to the base, and two lateral faces are inclined to it at
anglesa and p, respectively. Determine the lateral area of the pyra­
mid, if its altitude is equal to //.
681. The base of a pyramid is a right-angled triangle with an actite
angle a; the radius of the inscribed circle is equal to r, Each lateral
face is inclined to the base at an angle a. Determine the volume and
the lateral and total surface areas of the pyramid.
682. The base of a prism ABC A i s an isosceles triangle ABC
{AB — AC and ,^ABC = a). The vertex B, of thepipper base of the
prism is projccted iulo the centre of the circle of radius r inscribed
in the lower base. Through the side AC of the baso and the vertex
a cutting plane is drawn at an angle a to the base. Find the total
surface area of the cut-off triangular pyramid ABCB, and the volume
of the prism,
683. The base of a pyramid is a right-angled triangle. The altitude
of thc pyramid passes through thc point of intersection of the liypotc.
72 Problems

nuse and the bisector of the right angie of the base. The lateral edge
passing through the vertex of tlie right angie is inclined to the base at
an angie a. Determine the volume of the pyramid and the angles of
inclination of the lateral faces to lhe base, if the bisector of the right
angie of the base is equal to m and forms an angie of 45° + a wilh
the hypotenuse.
684. The base of a pyramid is a rhombus with the side a. Two adja-
cent faces are inclined to the plane of the base at an angie a, the third
one, at an angie p (prove that the fourlh lateral face is inclined to the
base at the same angie). The altitude of the pyramid is H. Find its
volume and total surface area.
685. The base of a quadrangular pyramid is a rhombus, whose side
is equal to a and acute angie, to a. The planes passing through the
vertex of the pyramid and diagonais of the base are inclined to the
base at angles cp and i|>. Determine the volume of the pyramid, if its
altitude intersects a side of the base.
686. The base of an oblique prism is a right-angled triangle A B C
with the leg5C = a. The vertex of the upper base is projected into
the midpoint of the ieg BC. The dihedral angie formed by the lateral
faces passing through the leg BC and hypotenuse A B is equal to a.
The lateral edges are inclined to the base at an angie p. Determine
the lateral area of the prism.
687. The base of a prism A B C A tB tCi is àn isosceles triangle A B C
(AB — A C and £_ BAC = 2a). The vertex A t of the upper base is
projected into the centre of the circle of radius R circurnscribed about
lhe lower base. The lateral edge A A t forms with the side A B of the
base an angie equal to 2a. Determine the volume and the lateral
area of the prism.
688. Determino the volume of a regular quadrangular pyramid,
whose lateral edge is equal to l and the dihedral angie between two
aiijacenl lateral faces is equal to p.
689. Oiven iu a frustum of a regular quadrangular pyramid: diago­
nal d, dihedral.angie a at the lower base and altitude //. Find the
voíume of the frustum.
690. The lateral edge of a frustum of a regular quadrangular pvra-
inid is equal tu l and inclined to the base at an angie p. The diagonal
of lhe pyramid is perpendicular to its lateral edge. Determine the
volume of the pyramid.
691. The altitude of a frustum of a regular quadrangular pyramid
is equal to / / , the lateral edge and diagonal of the pyramid are
inclined to the base at angles a and p, respectively. Find the lateral
area of the frustum.
Chapttr i X. l'o! tjheúrons 73

692. The sidos of lhe bases oí a fruslum of a regular quadrangular


pyramid are equal to a and a { 3, rospectively; the lateral face is in*
clined to the base al an angle y. Determine the volume and total sur-
face area of the fruslum.
693. A cube is inserihed in a regular quadrangular pyramid so
that its four vertices are found on the lateral edges. and the remaining
four, in the plane of its base. Determine the edge of the cube, if the
altitude of the pyramid is equal to //, and the lateral edge, to /.
694. A cube is inserihed in a regular quadrangular pyramid so that
its vertices lie on the siant heighls of the pyramid. Find the ratio of
the volume of the pyramid to the volume of the cube, if t he angle bet-
ween the altitude of the pyramid and its lateral face is equal to a.
695. The base of a pyramid is a right-angled triangíe withlegs equ­
al to 6 and 8, rospectively. The veriex of the pyramid is at a dislance
of 24 from (he base and is projected uniu its plane at a point lying
inside the base. Find the edge of the cube. whose four vertices lie in
the plane of the base of the given pyramid, and the edges joining these
vertices are parallel to the corresponding legs of the base triangíe
of the pyramid. The otlter four vertices of the cube lie on the lateral
faces of the given pyramid.
696. In a regular quadrangular pyramid the dihedral angle at lhe
base ís equal to a. Through its edge a cutting plane is drawn at an
angle {5 to the base. The side of the base is equal to a. Determine the
area of the .seclion figure.
697. íu a regular quadrangular pyramid the side of the base is equal
to a> and the dihedral angle al the base. to a. Through two opposite
sides of the base of the pyramid two planes are drawn at right angíos
to each other. Determine the length of the line of interseetion of the
planes contained inside the pyramid, if it is knoun that it intersecís
the axis of the pyramid.
698. In a regular quadrangular pyramid a plane is drawn through
a vertex of the base perpendicular to lhe opposite lateral edge.
Determine the area of the seclion figure thus obtained. if the side of
the base of the pyramid is equal to a, and the lateral edge is inciíned
to the planecontaining the base al an angle (p ((p>45°; prove this).
699. It isrequired íocul a rogularquadrangular prism with a plano
to obtain a seclion vielding a rhomhus with the acute angle rx.
Find lhe angle of inclinalion of the cutting plane to the base.
700. The base of a right parallelepiped is a rhomhus with the acute
angle a. At whnt angle to the base must a cutling plane be drawn to
oblain a seclion yíelding a square with its vertices lying on tho late­
ral edges of the parallelepiped?
Problems

701. A right parallelepiped, whose base is a rhombus with the side


a and acute angle a is cut with a plane passing through the vertex
of the angle a, the sectionyielding a rhombus with the acute angle ~ .
Determine the area of tliis section.
702. The edge of a tetrahedron is equal to b. Through the midpoint
of one of the edges a plane is drawn parallel to two non-intersecting
edges. Determine the area of the section thus obtained.
703. The base of a pyramid is a right-angled triangle cvith a leg a.
One of the lateral edges of the pyramid is perpendicular to the base,
the other two being inclined to it at one and lhe same angle a.
A plane perpendicular to the base cutsthe pyramid, yielding a square.
Determine the area of this square.
704. In a frustum of a regular quadrangular pyramid the sides of
the upper and lower bases are respectively equal to a and 3a and the
lateral faces are inclined to the plane containing the lower base at
an angle cc. Through a side of the upper base a plane is drawn paral­
lel to the opposite lateral face. Determine the volume of the quadran­
gular prism cut off the given frustum and the totaf surface area of
the remaining portion of the frustum.
705. Two planes are drawn through a point taken on a lateral edge
of a regular triangular prism with the side of the base a. One of them
passes through a side of the lower base of the prism at an angle a
to the base, the other, through the parallel side of the upper base and
at an angle (1 to it. Determine the volume of the prism and the sum of
the areas of the sectiong thus obtained.
706. In a regular quadrangular prism a plane is drawn through
the midpoinls of two adjacent sides of the base at an angle a to the
latter to intersecl three lateral edges. Determine the area of the sec­
tion figure obtained and its acute angle, if the side of the prism’s
base is equal to b.
707. The base of a right prism is an isosceles trapezoid (with the
acute angle ct) drcumscribed about a circle of radius r. Through one
of lhe mmparallel sides of the base and the opposite vertex of the
acute angle of the upper base a plane is drawn al an angle a to the
base. Determine the lateral area of the prism and lhe area of the sec­
tion figure thus obtained.
708. */rhe base of a right prism ABCAiB,C, is an isosceles triangle
A BC withjunglo a at the base BC. The lateral area of the prism is equ­
al to S. Fim! the area of lhe section by a plane passing through a dia-"
gonal of the face BCCiB{ parallel to the altitude AD of the base of
the prism and at an angle ji to the base.
C h a p ter /.V . í ‘ o li/ h e iir o n s

709. The base of a right prism ABCA,BiC, is a right-angled trian-


gle ABC with an angle p at the vcrtex B (p < 4 5 °). The diffcrence
between the areas of its lateral faces passing through the legs BC
and AC is equal to S. Find the area of the section by a plane forming
an angle tp with the base and passing through three points: the vertex
B t of the angle P of the upper base, midpoint of the lateral edge .4.4,
and point D situated on the base and symmetrical to the vcrtex B
with respect to the leg AC.
710. Non-inlersecting diagonais of two adjacent lateral faces of
a rectangular parallelepiped are inclined to its base at angles a and p.
Find the angle between thesc diagonais.
711. Given three plane angles of the trihedral angle SABC:
/_ BSC — a: ^ CSA = P; < ASB = y. Find the dibedral angles of
this trihedral angle.
712. One of the dihedral angles of a trihedral angle is equal to A;
the plane angles adjacent to the given dihedral angle are equal to a
and p. Find the third plane angle.
713. Given in a trihedral angle are three plane angles: 45°, 60° and
45°. Determine the dihedral angle contained between the two faces
with plane angles of 45°.
714. A line-segment AB is given on lhe edge of a dihedral angle.
In ono of the faces a point AI is given, at which a straight linc drawn
from A at an angle a to AB intersects a line drawn from B perpendicu­
lar to AB. Determine the dihedral angle, if the straight linc A. 1/ is
inclined to the second face of lhe dihedral angle at an angle p.
715. Given two sketv lines inclined at an angle q> to each othcr and
having a eommon perpendicular PQ = h which intersects botli of
them. Given on these lines are two points A and B, from which the
line-segment PQ is seen at angles a and p, respectively. Determine
the length of the line-segment AB.
716. Given on two mutually perpendicular skew lines, the perpen­
dicular distancc between which PQ — k, are two points A and B,
from which the line-segment PQ is seen at angles a and P, respecti­
vely. Determine the angle of ínclination of the segment AB to PQ.
717. A cutting plane divides lhe lateral edges of a triangular pyra-
mid in the ratios (as measured from the vertex): — , -Ílí-, — .
"1 "2 ><:)
In what ralto is the volume of the pyratnid divided by this plane?
718. From the midpoint of the altitude of a regular quadrangular
pyramid a perpendicular, equal to h, is dropped to a lateral edge, and
another perpendicular, equal to b, to a lateral face. Find the'volume
of the 'pyramid.
76 P n ib lem s

CHAPTER X
SOUDS OF REVOLUTION

719. The generator of a cone is equal to I and foras an angíe of 60°


with the plane of the base. Determine the volume of the cone.
720. The length of the generator of a cono is equal to /, and the
circumference of the base, to c. Determine the volume.
721. The lateral surface of a cylinder is developed into a square
with the side a. Find lhe volume of the cylinder.
722. When developed, the curved surface of a cylinder represents
a rectangle, whose diagonal is equal to d and forms an angle a with
the base. Determine the volume of the cylinder.
723. The angle at the vertex of an axial section of a cone is equal
to 2cc, and the sum of the lengthsof its altitude and the generator, to
m. Find the volume and surface of the cone.
724. The volume of a cone is V. Its altitude is trisected and through
the points of division two planes are drawn parallel to the base. Find
the volume of the médium portion.
725. Determine the volume of a cone, ií a chord, equal to a, drawn
in its base rirclo sublends an arc a, and the altitude of the cone
forms an angle p with the generator.
720. Two cones (one inside the other) are constructed on one and
the same base; the angle between the altitude and the generator of
the smaüer cone is equal to a, and that of the larger cone, to p. The
diííerence between the altitudes is equal to h. Find the volume of
the solid bounded hy tlio curved surfaccs of the cones.
727. The curved surface of a cone is equal to S. and the total one,
to P. Determine the angle between the altitude and lhe gene­
rator.
728. When developed on a plane, the curved surface of a cone
represents a circular sector with the angle a and chord a. Determine
the volume of the cone.
729. A plane, drawn through the vertex of a cone and at an angle
cf tu the base, culs off the circle of the base an arc a; the distance bet­
ween the plane and the centre of lhe base is equal to a. Find the volu­
me of the cone.
730. A square, whose side is equal Lo a, is inscribed in lhe base of
a cone. A plane drawn through the vertex of the cone and a side of
the square intersects lhe surface oí the cone along a triangle, the angle
at the vertex of which is ct. Determine the volume and surface of the
cone.
C h a p ter A‘ . S o lid s of fie v o lu iú

731. The element. I of a fruslum of a cone forms art augle a with


the plane of the lower base and is perpendicular to the straight line
joining its upper end point with the lower end point of lhe opposite
element. Find the cnrved surface of the frustum.
732. Given a cone of volume V, wbose generalor is inclined to the
base at an angle a. At what height sbould a cutting piane be drawn
perpendicular to the axis of the cone to divide the cnrved surface of
the cone into two equal parts? The same question for the total surface.
733. Determine the volume and surface of a spherical sector cut
off a sphere of radias R and having an angle a in the axial section.
734. The surface of a spherical segment of radius R is S. Find its
altitude.
735. The area of a triangle ABC is equal to S, the side AC ■■■-■ b
and /_ CAB — a. Find lhe volume of the solid fe.rmed by rotating
the triangle ABC about the side AB.
736. Given in a triangle ABC: the side a. angle B and augle C.
Determine the volume of a solid obtained by rotating the triangle
about the given side.
737. A rhombus with the larger diagonal d and acute angle y ro-
'tates about an axis passing outside it through its vertex and perpen­
dicular to its larger diagonal. Determine the volume of the solid tinis
obtained.
738. Given in a triangle: sides 6 anilc and the angle a beUveeu lhein.
The triangle rotates about an axis which passes outside it through
the vertex of the angle a and is inclined to the sides b and c at equal
angles. Determine lhe volume of the solid thus generated.
739. In an isosceles trapezoid a diagonal is perpendicular to one of
the nonparallel sides. The side is equal to b and forms an angle a
with the larger base. Determine the surface of the solid generated by
rotating the trapezoid about the larger base.
740. Two planes are drawn through the vertex of a cone. One of
them is inclined to the base of the cone at an angle a and intersccts
it along a chord of length a, the other is inclined to the base at an
angle p and intersects it along a chord of length b. Determine the
volume of the cone.
741. A sphere is inscribed in a cone. Find the volume oi the sphere.
if the generator of the cone is oquai to l and is inclined to lhe base at
an angle « .
742. A straight line, tangent to the curved surface of a cone, forms
an angle 0 with the element passing through the point of langoncy.
What angle (<p) does this line form with the plane of lhe base P òf
the cone, if its generator is inclined to the plane P at an angle a?
78 Problems

743. An obtuse triangle with acute angles a and P and the smaller
altitude h rotates about the side opposite the angle p. Find the sur­
face of the solid thus generated.
744. In a cone (whose axial section represents an equiiateral tri­
angle) installed with its base up and filled with water a bali of radius
r is placed flush with the water levei. Determine the height of the
water levei in the cone after the bali is removed.
745. In a cone, the radius of the base circle of which is equal to R
and whose generator is inclined to the base at an angle — , a right
triangular prism is inscribed so that its lower base lies on the base of
the cone, and the vertices of the upper base are on the curved surface
of the cone. Determine the lateral area of the prism, if the base of the
prism is a right-angled triangle with an acute angle a , and its altitu­
de is equal to the radius of the circle along which the plane passing
through the upper base of the prism intersects the cone.
746. In a triangular pyramid, whose base is a regular triangle with
t hc side tf, a cylinder is inscribed so that its lower base is found on the
base of the pyramid, its upper base touching all the lateral faces.
Find the volumes of the cylinder and the pyramid cut-off by the plane
passing through lhe upper base of the cylinder, if the altitude of the
cylinder is equal to 4- , one of the lateral edges of the pyramid is per­
pendicular to the base, and one of its lateral faces is inclined to the
base at an angle a (define the values of a for which the problem is sol-
vable).
747. A right triangular prism is inscribed in a sphere of radius R.
The base of tiie prism ís a right-angled triangle with an acute angle a
and its largest lateral face is a square. Find the volume of the prism.
748. The base of a pyramid is a rectangle with an acute angle a bet-
ween the diagonais, and its lateral edges form an angle cp with the
base. Determine the volume of the pyramid, if lhe radius of the cir-
cumscribed sphere is equal to R.
749. The radius of the base circle of a cone is equal to R and the
angle aí lhe vertex of its axial section is a. Find the volume of a re-
quiar triangular pyramid circumscribed about the cone.
750. A sphere of radius r is inscribed in a frustam of a cone. The
generator of the cone is inclined to the base at an angle a. Find the
curved surface of the frustum.
751. Circumscribed about a sphere ís a frustum of a cone, whose
elements are inclined to the base at an angle a. Determine the sur­
face of the frustum, if the radius of the sphere is equal to r.
(liu / ite r X . S o lid a of iie v o lu iio n 79

752. A sphere of radius r is inscribed in a frustum of a cone, whose


generator is inclined to the plane of the base at an angle a. Find the
volume of the frustum.
753. From a point on the surface of a sphere of radius R three equal
chords are drawn at an angle a to ono another. Determine their
lengths.
754. A frustum of a cone is inscribcd in a sphere of radius R. The
bases of the frustum cut off the sphere two segments with ares in the
axial section equal to a and p, respectively. Find the curved surface
of the frustum.
755. The lateral faces of a regular quadranguiar pyramid are incli­
ned to the base at an angle a. The slant lieight of the pyramid is equal
to m. Find the surface of a cone inscribed in the pyramid and the an­
gle of iriclination of the lateral edge to the base.
756. A cone is circumscribcd about a regular hexagonal pyramid.
Find its volume, if the lateral edge of the pyramid is equal to /
and the face angle between two adjacent lateral edges is equal to a.
757. A cone is inscribed in a regular triangular pyramid. Find the
volume of the cone if the lateral edge of the pyramid is equal to (
and the face angle between two adjacent lateral edges is equal
to a.
758. A cone is inscribed in a sphere and its volume is equal to ono
fourth of lhat of lhe sphere. Find the volume of the sphere, if the al­
titude of the cone is equal to H.
759. A sphere is inscribed in a regular triangular prisrn. lt touches
the three lateral faces and both bases of the prism. Find the ratio of
the surface of the sphere to the total surface area of the prism.
760. A sphere of radius R is inscribed in a pyramid, whose base is
a rhomhus with the acute angle cc. The lateral faces of the pyramid are
inclined to the base at an angle (p. Find the volume of the pyramid.
76Í. A hemisphere is inscribed in a regular quadranguiar pyramid
so that its base is parailel to the base of the pyramid and the spheri-
cal surface is in contact with it. Determine the total surface area of
the pyramid, if its lateral faces are inclined to the base at an angle
a and' the radius of the sphere is equal to r.
762. A hemisphere is inscribed in a regular quadranguiar pyramid
so that its base lies on the base of the pyramid and the spherical sur­
face touches the lateral faces of the pyramid. Find lhe ratio of the
surface of the hemisphere lo t he total surface area of the pyramid and
the volume of the hemisphere, if the lateral faces are inclined to the
base at an angle of a and lhe difference between the lengths of the
side of lhe base and the diameter of the sphere is equal to m.
80 Problema

763. In a cone, with the radius of the base circle R and an angle a
between the altitude and generator, a sphere is inscribed which tou-
ches the base and the curved surface of the cone. Determine the volu­
me of the portion of the cone situated above the sphere.
764. The surface of a right circular cone is n times as large as the
surface of the sphere inscribed in it. At what angle is the generator
of the cone inclined to the base?
765. A sphere is inscribed in a cone. The ratio of their volumes is
equal to n. Find the angle of inciination of the generator to the base
(calcuíate for n ~ 4).
766. Determine the angle between the axis and generator of a cone,
whose surface is n times as large as the area of its axial section.
767. Inscribed in a cone is a hemisphere, whose great circle lies on
the base oftiie cone. Determine the angle at the vertex of the cone,
if the ratio of the surface area of the cone to the curved surface area
of the hemisphere is 18: 5.
768. Determine the angle between the altitude and generator of a
cone, if the volume of the cone is 1 -rr times as larjre as that of the
.1 a
hemisphere inscribed in the cone so that lhe base of the hemisphere
lies on the base of the cone and the spherieal surface touches the cur­
ved surface of lhe cone.
769. Determine the angle between the altitude and generator of
a cone, whose curved surface ís divided into l\vo equal paris by the
tine of its intersection with a spherieal surface, whose centre is loca-
ted at the vertex <>f the cone and lhe radius is equal to the altitude of
the cone.
770. A cone with the altitude / / and lhe angle between lhe genera­
tor and altitude equal to a is eut by a spherieal surface with the centre
at tho vertex of the cone to divide the volume of the cone into two
equal portions. Find the radius of the sphere.
771. On the altitude of a cone, equal to / / , as on the diameter. a
sphere of radius™ isconstructcd. Do termine the volume of the portion
of the sphere situated outside the cone, if the angle between the
generator and altitude is equal to a.
772. Given two cxternally tangenl spheres O and Oi. and a cone
circumscribed about them. Compute the area of the curved surface of
the frustum, whose bases are the circles along which lhe spheres con-
tact the surface of the cone, if tho radii of the spheres are equal to R
and R^
773. Four balis of ono and the same radius r lie on a table so that
they touch one another. A fifth bali of the same radius is placed on
Chapter X I - Trigonomelric Transformalions 81

them at the centre. Find the distance between the top point of the
fifth bali and the plane of lhe table.
774. Determine the angle at the vertex of the axial section oí a cone
circumscribed about four equal balis arranged so that each of them
is in contact wilh the three remaining ones.
775. The faces of a frustum of a regular triangular pyramid touch
a sphere. Determine the ratio of the surface of the sphere to the total
surface area oí the pyramid, if the lateral faces of the pyramid are
inclined to the base at an angle a.
776. Inscribed in a cone is a cylinder, whose altitude is equal to
the radius of the base circle of the cone. Find the angle between the
axis of the cone and its generator, if the ratio of the surface of the
cylinder to the area oí the base of the cone is 3: 2.
777. The radius of a sphere inscribed in a regular quadrangular
pyramid is equal to r. The dihedral angle formed by two adjacent
lateral faces of the pyramid is equal to a. Determine the volume of
the pyramid, whose vertex is al lhe centre of the sphere and the vér­
tices of the base lie at the four points oí tangency of the sphere and
the lateral faces of the pyramid.
778. A sphere of radius r is inscribed in a cone. Find the volume
of the cone, if it is known that a plane tangent to the sphere and per­
pendicular to the generator of lhe cone is drawn at a distance d
from the vertex of lhe cone.
779. The edge of a cube is a, A B being its diagonal. Find the radius
of a sphere tangent to the three faces converging at the vertex A and
to the three edges emanating from the vertex B. Also find the area
of the portion of the spherical surface outside the cube.
780. In a regular letrahedron, whose edge is equal to a, a sphere is
inscribed so that it is in contact with all the edges. Determine the
radius of the sphere and the volume of its portion outside the tetra-
hedron.
CHAPTER XI
TRIGONOMETR1C TRAN SFORMATIONS
Prove the jollowing identities:

a
783. 2 (esc 2a -f- cot 2a) = cot rg— tan 2
«-•1338
82 Problems

cosa-j-sma cos a-f-sina


784. cosa—sina = tan (45°-f a); 785. cosa—sina" : tan 2a 4- sec 2a
/ n , \ . 2/ n sin 2a
\ sin2e
786. SÍO1 ( T + a ) - sin ( t - a ) = - y 5

2 cos2 a —i
787. -= 1
2 tan —a j aj

1—-sin 2a
788. tan' ' 14- sin 2a
(t - ) '
cos 2a 1 . «n
789. — Tõ------;— s— — t s i r 2 a
cot2 a —tan2 a 4
si n a + c o s (2g - a ) t
790.
cos a — sm (2$ (2p — a) \4 ■/
l-f-s in 2a _ _ i + tan a
791. tan |-|-t a j

sin z-j-cos (2y — x) 1-j-sin 2y


792. co s z ~ s in (2 y — x) ~ cos 2y

793. tair a —- tan2 p — sin (a - f p) sin (a — p) sec2a sec2 p

,an ( t — f " ) •(1+ s ia a >


794. =cot a
sin a
/ tl , fa \ 1— sin a
795. tan V 4 1 2 / :1
cos a
:2 (sin 2a + 2 cos2 a — 1)
796.
cos a — sin a — cos 3a - f sin 3a
sin a —sin 3a-{-sin 5a
797. tan 3a
cos a — cos 3a -j- cos 5a "
. / , . . . , . . , a—b . a —c b— c
798. sin (a — ü)4- sjii (a — c)+ sin (ò — c) ~ 4 cos sin — -cos——
799. 2 (sin6x + tos6ar) — 3 (sin4x cos4x) 4- 1 — 0
800. sin a j - sin ^a 4 -*^ - j *4*sin ^a -J- — 0

801. sin2 {45° f a ) — sin2 (30o —a) —sin 15° cos (15°- f 2a) — sin 2a
802. Show that
1— 2 cos2q) .
—----------- = tan q) — cot (p
sin 9 cos tp
Chapter X I . Trigonometric TransjormaUons 83

803. Show that


. 2 — — 2 sin g —sin 2ct
2 ~ 2 sin a + sin 2a
804. Prove the identity
cos2 (p4- cos2(a + fp) — 2 cos a cos q>cos (a -f cp) —sin2a
805. Simplify the expression
sin2a -f sin2p ~j- 2 sin a sin p cos (a + p)
806. Prove that
sin2a -}- sin2P -{- sin2y — 2 cosa cos P cos y = 2,
if a - f p - f y ^ n .
807. Prove that
cot A cot B -}- cot A cot C 4- cot B cot C — í,
if A 4-B + C - Jt.
808. Prove that
2n_ i_
cos ^r-
o cos 5 4
809. Prove that
. 3n }
C0S— f-CÜS—

Reduce to a form convenient for taking logaritkins:


810. 1 + cos a “p cos-y-; 811. 1— }/*2 cos a -f- cos 2a
812. 1 — sin2(a~f P)~~ sin2(a-—p)
813. í +1 sin a + cos a 4- tana; 814. —. aa" CQSa
s in -y

815. 1 — tana + seca; 816. cosa-j-sin 2a — cos3a


817. tan ( a 4 - + tan ^a —— )

818 ^ sin ft—sin 2ft . gjg ~|/2—cos a —sin a


* 2sinp4sin2p ’ * sina —cosa
820. cot a -f cot 2a 4- esc 2a; 821. cos 2a 4-sin 2a tana
822. 2 sin* a + V 3 sin 2a — 1; 823.
6*
84 Problems

824. 2 + tan 2a + cot 2a

825. tanx — 1 + sinx (1 — tan x) + 1+ tana~


l-t-cosa-)-cos2g~f cos3g
826. 827. 1 — sin2 2a— sin2 P— cos4a
cos a 4- 2 cos2a —í
sin (x + y + z)
828. tan x + tan y + tan z — cos x cos y cos z

829. sina + sinp+siny if a + p + y = 180°

CHAPTEE XH
TKÍGONOMETRIC EQÜATIONS

Solve the following equalions:

830. 1 — s in 5 x = I cos *2---- sin -^ -j


831. sin x + sin 2x 4- sin 3x + sin 4x = 0
832. sin (x + 30°) + cos (x + 60°) = 1 + cos 2x
833. sin x 4 - sin 2x + sin 3x = cos x + cos 2x + cos 3x
834. cos2x — cos 8x f- cos Gx — 1; 835. co s x — cos2x = sin3x
836. sin(x — 60°) = cos(x + 30°); 837. sin 5x + sin x -{-2 sin* x = í
838. sin2x (ta n x-{-1) = 3 sin x(cos x — sinx) + 3
839. cos 4x = — 2 cos2x; 840. sin x -f-cosx — si* -

841. sin 3x = cos 2x; 842. sin4 y-f- cos1— =


843. 3 tan2x — sec2x = 1; 844. (1 + cos 4x) sin 4x = cos2 2x
845. sin4x + cos4x = cos4x; 846. 3 cos2x — sin2x — sin2x = 0
847. cos2 x + 3 sin2x + 2 Y 3 sin x cos x = 1
848. 6sin2x - f 3 sin x cosx — 5cos2x = 2
3 5
849. sin2 x + -vj-cos2x = -jS in x cosx

850. sinx 4 -V °3 co sx = 1; 851. sin x c o s x = 1


852. sinx + cosx = l 4-sin 2x; 853. sin 3x -f cos 3x = Y%
Chapter X I I . 7'rigonometric Equations 85

854. sinxsin7x = sin3xsin5x; 855. cosxsin 7x — cos 3x sin 5x


856. sinxsin2xsin3x —-|-sin4x; 857. 2eo$2x + 4cosx = 3sin2x

858. 5 co s 2 x ~ 4 s in x ; 859. tan i- x| 4 tan x ~ 2 = 0

co s (™ — x\
860. 8 tan2tt- = 1 + secx; 861. — r--^--------—sec2— — 1
2 i 4- cos x 2
862. 1 —cos (jt— x) sin —0

863. 2 ^ 1 — sin x j j = J^3 tan


864. sin x — cos x —4 cos2x sin x = 4 sin3x
865. c o t x + 1^ =2
' 14-cosx
866. 2 cot (x — Jt)— (cos x -f sin x) (esc x — sec x) = 4
Q£*im - / \ 1 . / Jt \ SCCJT— cos X
867. sm(n — x) + cot ^ — x ) = — asínV-

i —tan ~-
868. ----------- = 2 s in 4
i - c o t í

869. sin (n — x) + cot (-^ -4 x ) = secj — x )— cos(2jt — x)

870. sec2x — tan2x 4~ co t ( -y- 4~ x ) ~~ c°s 2x sec2x


871. sin3i(1 4-cota :)4 -cos:li-(l i tan x) = cos 2x
872. sin3x cos 3x 4- sin 3x cos3x - 0.375
873. tan x -j- tan 2x — tan 3x
874. l4 -s in x 4 -c o s x = 2 cos — 45°)

875. 1— cos22x = sin3x — cos(-^-4 x j

876. 1 _ 3 c o s x + cos2x = í | | i^

877. [cosx — sin(x—n ) f + 1 = j|— ^


86 Problems

878. (sin a: + cosz)2= 2sin ( -j- + x } sin(-|- — x j


879. 2 — sin x cos 2x — sin 2x cos x =

#4*
= [ C0S( x - T - ) - s in (- )]'
880. (1 — tan x) (1 + sin 2x) = 1 + tan x
. . cos 2x
881. COS X 4- Sin X = ;----:—s -
1 1— s m 2x
882. (1 4 -sin 2x) (cosx — sin x) — 1 — 2sin2x
cos2 x —sin22x
883. 4 cos2 x sin (x 4- 30°) sin (x — 30°)

sin (60° 4-^) 4- sin (60° — x) tan x_____ _____ cot x


884.
2 (14- tan2 x}2 ' " ( 14- cot2 x)2
sin (x — 30°) 4- cos (60°
885. sec2 x — ( cos x !- sin x tan
r)- cos x

tan y + cot y
886. sin ( 4 + ^ ) - s i n ( x _ 'r ) =
21/2
887, 2 V
f 2 sin v
(45“ +i x)/ = í 4-s m x
. 2 (sin 2x — cos2x tanx) . . ,
888. 1 ----- i------- -=-------------- ' = cos4x — sin* x
1 /3 sec2 x
889. sin 3x — 4 sin x cos 2x

890. sec x + 1 = sin (rt — x) — cos x tan —


tanr-----:--
— 2x tan x2 sin
o •(45// cro i sin
4- x) \ • / / ro \ a
891. t
tan 2x — tanx \\ (45
/ —\ x) = 0 /

892. tan (x — 45°) tan x tan (x > 45°) 4 cos 1

tan (x + 4 5 1 + ta n (x -4 5 ° ) ^ tan _ 450) , an ^ + 450) tan%


893.

894. tan (ar 4- a) + tan (x — a) — 2 cot x


/ . x x \2 2
895. \
Sin^
2
— COS-Tr)
2 /
—------------------
. X z_+r-r-
n
' ' tnn __- _ tnn _ 1...
2
Chapter X I I I . Inverse Trigonometric Funclions 87

896. .;...v-^.- - z - ....= 1 -f tan ( Z f- 45°) - tan í 45= - 4 \


sin (30 -}- x) “I- sin (30 -■ x) V2 / \ 2 /

897. sin4x + s in 4 ( x + -| -j —j

897a. sin4x-\- sin4 (x j -j sin4 (x — i ) = -|

Solve the following Systems of equalions:


ono X -j-U x —y 1 1
898. c os —^ - c o s — = y ; cosxcosz/ —-^-
899. £~[~i/ = a; sinxsin t/ = m; 900. x - t y = a; tan x -f tan // — m

901. x - \ t a n x -f tan y = 1

902 2sin I+cos y= 1* I6sin2 x+cos2y = 4

903. sin a sin t/^ ; im xU n y = ~

904. s in x = 2 s in y ; cosx = cos í/

C H A PTE R X III

INVERSE TRIGONOMETRIC FUNCTÍONS

905. Compute

2 arcsin ( — ^ ) + arcco t ( — 1) + arccos ~ = + ~ arccos ( — 1)


906. Prove that
tan (arccos x) = ——- -----
907. Prove that
tan (arcsin x) = -y J Í = -
y i —x2
Compute:
908. s in ^ a r c c o t ( — |-) j ; 909. sin [ —arcsin ^

910. cot ^-“-arccos (•—y ) j ; 911. tan ^5 arctan — arcsin


88 Problems

912. sin ^3 arctan }^3-|-2 arccos

913. cos |^3arcsin-i^4-arccos ^

Prove the following identities:

914. arctan (3 + 2 V”2 ) — arctan = -2-

915. arccos — arecos = -2-


' 3 2yã 6
ni a » 4 . • 5 , • 16
91o. arcsin-^--f arcsm^ + arcsmgg=s-j-

917. arccos j - + arccos ( — f ) = arccos ( — j|)


1 1 32
918. 2 arctan -g- + arctan ^ - = arctan ^

919. arctan - j + arctan i + arctan ~ + arctan ~

Solve the following equations:


920. 4 arctan (x- — 3x — 3) — it = 0
921. 6 aresin (x2— 6x + 8.5) = n
922. arctan ( x + 2) — arctan ( x + 1) = -^-
1 Jl
923. 2 arctan y — arctan x = -j-
2 ^
924. aresin-----— — aresin V 1 — x = aresin —
3 V i 3

925. arctan í?— arctan = aretan x

926. aresin 3x = arccos 4x; 927. 2 aresin x = aresin'-^-


928. Solve the System ot equations
x + y = a r c t a n t a n x t a n y = a 2 (| a | < l)
ANSW ERS AND SOLUTIONS

PA R T ONE

ARITHMETIC AND ÁLGEBRA

C H A PTE R I

ARITHMETIC CALCULATIONS
1. 6.5625 ..o 15 31. 4
15.
ot>64
3 2. 4000
2. 29* 16. ()
33. 66
5 17. 700
3. 365-1- 34. O
8 18. 100
35. 9.5
4. 3 l 5 19. 10
3 6. 0.09
— i1
20.
5. 18 { '2 35
3 7.
48
21.
6 . 50 2
38.
22. 3
7. 23.865 I
39.
23. 92 “ Ic
8 . 36 | "80
xi
40. o _L
24. 5 “ 3
9 . 5 9 9 .3
1
10. 8 4 .0 7 5 25. 41.
1 84 8
11. 2 .5 26. 10 42. 1301

12 . 2
M ^21
— 27. 1
43. -2 0 .3 8 4
28. 1320
13. 0 .0 1 1 5 44. 2.25
29. 11
14 15! 45.
280 30. 250 JT
90 Answers and Solutions

C H A P T E R II

ALGEBRAIC TRANSFORMATIONS
Preliminaries
ln solving probiems of the present chapter (beginning with Problem 62)
the foilowing should be taken into consideration.
1. The radicai is called principal, or arithmetical, if the radicand o is
positive (or equal to zero) and if, furthermore, the root itself is taken positive.
Examples. The expression y —27 cannot represent an arithmetical root,
since the radicand is negative. The expression y 16 is an arithmetical root, if
we consíder onlv its positive value (i.e. 2). The expression 27 represents an
arithmeíifal root (i.e. 3), if we consider only its real value ^ it also has two

imaginarv vaiues —■~ -y i and — ™ i ^ . The expression —16


cannot represent an arithmetical root, since the radicand is a negative number.
2. The rules for transformation of radicais set forth in algebra are true only
for arithmetical roots.
For instaure, the equality \~x = >r x~ is not true for negative vaiues of x.
Thus, at x —8 the left meniber of the equality has only one real value £ —8 =
—2, while the right member has two real vaiues p' 64 — ± 2 (if imaginary
vaiues are also eonsidered, then y ~ 8 has three vaiues, and y G4-~six vaiues).
In view of this, in the present section dcdicated to the identity transformations
«if irrational expressions we assume that all radícands inay have only positive
(and zero) vaiues*, which mearis that literal quantities entering the expressions
to be simplified should meet some additional conditions. In a number of cases
(see, for instance, notes to Probiems 65 to 71) we indicate sueh conditions.
Sometimes the conditions to be satisfied by literal quantities are given in
the problem itself. Then in solving such a problem one is to prove that under
lhese conditions all radícands are positive. __
3. it should be particulariy noted that the equality ~\/x~ — x (where l/x^
is an arithmetical root) holds true only_ for x > 0. For a negative x it is inva-
lid; instead we have the equality ~[/x2 - - — x. Both cases may be ^covered by
the equality "\/x2=: |x|. Thus, if x — — 3, then V (~~ 3)2= = — ( — 3)
(and f/ ( — 3)2 is an arithmetical root, since the radicand ( — 3)2 is positive
and the root is taken in its positive value). We may also write '\/( — 3}2=
The iniportance of this note is seen from the foilowing examples.
Example 1. Simplify the expression — 2mn~\-n2. The solution
~\/m- — 2mn -{* n2= "\/(m — n)- — m — n
is true only for m > « . For it should be written instead
"\/m‘~ — 2mu-f-n2— — (m — n), i. e. m2— 2mn -f-n2= n — m
* With the exeeption of Problem 64, where the radicaud of the cube radical
eau never he positive (see lhe solution of this problem on pages 96-97).
Chapter I I . Algebraic Transformations 91

Thus, ií m = 2 and n = 3, then m — n = — 1, whereas


1/m » - 2mn + = V 4 - J2 + 9 = V I = 1
A general formula has the form
■y' m 2 — 2m.n -f- n2 -■=[ m — n\ or \ m-— 2nm ( n —m \.
Example 2. Simplify the expression
V 4 + 4p + Pa — V-'* — 4p - f P"
~\f4 - f 4p + pz t *V 4 —
Denoting (for the sake of brovity) the given expression by A, we have
<for p=y= ~ 2):

l-|2=ü|
12 -f-p 1— [2 — p | Í2 + p|
|2-f-/>|-f |2 — p} , , f 2 — />j
*U + pI
I f the fraction is positive, then
2+ p
i_ ? ^
. _ 2-f-/> __ p

1-f
2+ P
and if it is negativo, then

1+ _____
2 -f P _2_
2—P ~ P
2-f p
Let us analyse what values of p yieJd the first or/and the second cases. The
fraction ; is positive when 2 — p and 2 + p are of the samc sign. Lei us
2-f-p
first require that both quantities 2 — p and 2 - f p be positive. The quantity
2 — p is positive for p < 2 , the quantity 2 p is positive for p > —2. Con-
sequently, both quantities are positive for —2 < p < 2. Now u c reauire that
both quantities 2 — p and 2 - f p be negative, but soon find out that this requi-
rement cannot he fulfilled, since 2 — p is negative for p > 2, and 2 -f- p
negative for p < 2, but these conditions are incompatible. Ilence. the fraction
r-r-~ is positive only for 2 < p < 2. For p > 2 and p < —2 this fraction
~>P
is negative.
Thus, for } p l < 2 and íor jp | > 2 . At |p| = 2 both e\-pres-
sions are valid. __
Example 3. The equality V fl8~ fl3 is true only for a > 0 . For negative
values of o we have instead of it the equality V a° — —a3. Thus, at as=-i
92 Answers and Solutions

we have V ( ~ 1 )6= — ( — 1) = . Herc is an arithmetical root,


since the radicand { — 1)* = 1 is positive and we take the positive value of
the root.
Example 4. Take the factors outside the radical siga in the expression
V ( a ~ 5)«» (a — 3)3.
The given root may be arithmetical only for a > 3 , since for a < 3 the
factor (n-~3)3, and, hcnce, the whole radicand are negative. The equaiity

'\/(a — 5)8 (a — 3)3~ (a — 5)3 (a — 3) ~\/a— 3

holds true for a 5. For a < 5 the following should be written instead

— 5)8 (a—3)3= — (a 5}3 (q — 3) V a “ ^


A general formula is
V (a —5)6(a —3)3— |a— 5 j3 {a — 3) ~\/a—3 (for « > 3).
4. In general, the equaiity " x» — x (where the left member denotes an
arithmetical root) holds true only for positive values of x (and at x — 0). If n
is anjn en number, then at a negative x instead of j / ’x” = x we have the equali-
ty í .rn ^ —x. if n is an odd number, then at negative values of x tliere exists
no arilhmetical root at all.
40. Grouping the last three terms of the expression in parentheses, factor it
— h- — c~ -f- 2bc — a* — (b — c)- -- (a -f- b — c) (a — b -f- c)
The given expression lakes the form
{a -f- c b) (a -f* c — b) = {a -f- c)" — 62
Of
Answer: (a-i-cp — b2\ 139 ~ .
1
47. The expression in parentheses is equal to - — - . It is convenient to rever­
so aii the stgns hoth in the numerator and denominator of the last fraction, and
then to factor the numerator; the fraction takes the fonn
(fl4.n)(W^ l ) f ns + n.}-i)
a2— 1
"Imít/w n2^~n *
4S. The denominator of the secund fraction is equal to (1 -f x) (x — 2a).
The parenthesized expression is equal to 1 -j- x. The given expression is equal to
x________ 2_
a {x —2d) x — 2<i a
\
Ansuv
t!
49. .-iHsuvr: — -— .
a-f- 2x
Chapter I I . Algebraic Transformations 93

50. Represent the secood addend ía the form ~ . Heduce the paren-
3a — 1
- 3 (2<j2-f9<z-f 10)
thesized fractions to a common denominator; this yields „
J a (3a — 1)
•equating the trinomial 2a2 + 9a-f l O to zero and finding tlje roots a ,= —2;
az — — — , factor it

2a=+9a+ i0=2(a+ 2) +
Now tho expression io parentheses takes the form
— 3 (a -f- 2) (2a 4* 5)
a (3a — 1)
MuUipiying it by
3a3 + 8ag —3a 4a (a + 3) (3a— f)
, 1 o (2-f a) (2 — a)
l“ T
12 (2a + 5) (a+ 3)
Ansuer: --------------- 75--------- .
a— 2
5Í. Uedace eacli fraction, íactoring botb the numerator and denominator.
ab
Answer:
a-\-b
52. Factor the denominator of tbe second fraction and reduce lhe latter
x , y ( x — j /) 1

x2+ í/2 (x2-j-y2Mx+W


1
Answer.
53. After simplificalion the dcnotninalors of the fractions take the form
4(*2 + *-M ) , 4 (*2- x + t)
-------- 3-------- and ---------3--------
The given expression is transíormed in the followíng way
JL ± ( j __ *__ 1 *2-H
3 * 4 V x24 -* 4 -i -x + l ( x 2 -f* 1 ) 2 — X2 X4 4 " * 2 4 - i

x24 -l
A/wwer: + +l ■
54. Factor the denominators of the first four fractions, reduce the first
fraction by a — í. The expression in parontheses takes the form
i 2 (q —l) 4(a + i)
a—1 (a + 2 ) ( a - 2 ) < a - l ) (a + 2) + ( a - 1 ) ( a -2 )
2 (° + 3 )
( a - l ) ( a + 2 )(a -2 )
94 Answers and Solutions

. v. u u í»* t- j v. *1 t .. 36a3— 144a— 36a24-144 „


This should be multipliea by the fraction----------- aà-fTf------------• Factor
the numerator of the last fraction by grouping the terms, and the denomi-
nator, as the sum of cubes a3-f-33; then this fraction takes the form
36 (a — 1) (a-f-2) (a— 2)
(a-J-3) (a2— 3a-f-9)
72
Ansioer:
-3a 4" 9
55. Let us denote the dividend (the sum of fractions) by A and the divi­
sor, by B. Factoring the poiynomials whick enter A, we find
3 (* + 2) , (x-f-2) (2x— 5)
2 (x 4 -l)(x 2 4 -l) 1 2 (x — 1) (x24 -l)
x 4-2
Taking outside parentheses, we have
2 (**4-1)
A *+ 2 / 3 2x — 5 \ (x -f2 )(x 2- 4 )
2 (x2 + i) ' \ x + l i ~ * - i I ~ ( X2 + l ) ( * + i ) ( x _ i )
Then we fínd
p 2 (x2 -4 )
(x2 + t) ( , + i ) ( x _ i )

Dividing A by B, we get .

Answer: .
56. Let A denote the dividend and B, the divisor. Equating the trinomiaí
x2 — Xy — 2y2 entering the expression A to zero, we solve the obtained equation
for one of the unknowns, say, for the unknown x; on finding xi — ~~y and
*2 = 2y, we get the following factorization of the trinomial: x2 — xy — 2y2 =
— (x 4- y) (x — 2y). Now we have
X —y XZ-'r yt-\-y~~2
2y~x (*4-y)<x — 2y)
Write in the subtrahend 2y — x instead of x — 2y, simultaneously reversing
the signs in the numerator of this fraction. Reducing the fractions to a common
denominator, we get
2x24 -g -2
(2y —x) (x-i-y)
In the expression B factor the numerator by representing it in lhe form
(2x2 4- y)2 — 2S, and the denominator, by grouping x2 4 - xy and j + z. Then
(2x24- y -4- 2) (2x24 - 2)
(x + y) (x 4 -f)

Dividing X by B, we get y + 2) .
Chapter I I . Algebraic Transformations 95

*+ i
Answer:
(2y — x) (2*2 -f- y 4- 2) •
57. Factoring the polyDomials contaíned in the given expression, we obtain
<a+ 2)(a — 1) f 4(a-f-i) 3 ']
an(a—3) i_ 4(a-f-l) (a — í) a (a — 1)
a-j- 2
./Insujer: flfi+i
58. Let A denote the dividend and B, the divisor. The numerator of the
fraction A is

■j- I4aMI>■+')%n - -1) = y l2“ <*>+')" + 5H2« (t ■+ ■-1!


and the denominator
a — q2— 2a ■— — — {d-)-l)^)™ a(íi-f-a-{-l){n — a — 1)
Leave the numerator of the fraction B unchanged and write its denominator
in thie íorm — ac(n —a— 1).
[2a (6 + c)n-f 1) c
Answer: — — -— ----—
2 (n-)~ a-f-i)
59. First method. Reduce all the fractions to a coinmon denominator:
bc (b — r) — ac (a — c) -j- ab (a — b)
(a)
abc (a — b) (a — c) (6 — c)
Having multipiied the binomials in the denominator, we gct azb — afr
- f 62c — azc -f- ac2 — bc2, i.o. the same expression as in the numerator. After
reduction we obtain - 4 -.
abc
Second method. Putting in the numerator of the fraction (a) a = b, we íitid
that in this case the numerator vanishes. Consequently, according to Bézout s
theorem, it is divisiblc by (a — b). The quotient is
a {b — c) — c (b — c) = (b — c) (a — c)
Thus, the numerator is equal to (a — b) (b — c) (a — c).
Third method. Let us reduce to a common denominator only the íirst tuo
íractions of the given expression. We get
(,2 — — a 2 _j_ a c

ab (a — b) (a —c) (b— c)
Grouping the terms in the numerator (the first one with the third and the
second with the fourth), we arrive at the expression
(b + a) (b — a) — c (b — a) = (a — b) (c — a — 6)

Now reduce the fraction by (a — b) and add lhe third fraction of the given
expression.
Answer: - 4 — .
abc
96 Answers and Solutions

60. The first factor is equal to ■ The expression in brackets is

equal t0 Ç” ! * ) 2- 1 _ ,{ ° ± £ + ,1) f e ± , £ - <' . Multiplying the given expres-


2ax 2ax
1
find (<1+ x + 1 )2 . Substituto x - the numera tor takes
a— 1
2a
the íorrn the denominator becomiug equal to
(fl-ip
Ansiver:
Ua — 1)
01. Let us denote lhe expression in brackets by A and the expression in
parentheses, by D. We have A i B ^ — AB. Getting rid of the powers with
negativo exponents in the expression A, we have
2ft2-_3aft_2qg (b — 2a) (2 6 + a) b — 2a
•'l a (a+ 26) ( 2 6 — 0 ) a(a-{-2b)(2b— a) a (2b— a)
Fransforming B, we get

B --,an ( 2b -p3a — \ - 0n. M «-2 6 ).


2a — ò ) 2a— b
Finailv, we find AB — an~1b (reverse the signs of the terms both in the numera-
tor and denominator in one of the fractions to be multipiied).
Ansirer: an~íb.
62. The numcrator is transformed to the form a2 — i 2, the denominator,
to a -j- b.
Answer: a — b.
Xote. For the roots to be arithmetical ones, the numbers a and b must not
be negativo.
63. The first radical is equal to
]" (a — b}3 (a -j- b)“ — (a — b) f (a -j- 612
.4nsífcr: h (a3 — b3).
Xote. It is assumed that a > 6 (olherwise the first root will not be arithme­
tical).
64. This is an excepfion front tbo role considered on page 90 whieh States
that radicands may have only positive vaíuos. The thing is that the radicand of
the cube radical is always negativc. Indeed, we must consider the expressioDS
V b * and \/2x (whieh have reajbvalues only forx > 0) to be positive (olherwise
the expression 2 "j/Üx — 4 ~\/'lx loses its uniqueness). But then the difference
2 V(17 — 4 — V 2 4 r — ~[/32x is negative.
And so, we admit that the radicand of the cube radical is a negative nurnber.
Then the cube root itseif has a negative value. Prior to applying the ruJe for
transíorming radicais, we have to accomplish such a transfonnation:

y 2 Yüi—i y s = —J / T y s — 2 y s
Now the radical on the right is an arithmetical root. After reduetion to the
same index as that of the first of the given factors we obtain

- Y ay s - 2 ysi= - Y (ky 2S— 2 y 5 ) a - — ( 7— 4 y ã )


Chapter I I . Algebraic Transjormations 97

Muítiplying lhe roots, ve get: — y/~64x2 [49 — (4 3)2J= — 2 ]/ x.


Answer: — 2 f x.
Note. I{ no attcution is paid lo the fact that lhe radicand of the cube
root is negative, one obtains tlie wrong answer 2 f x.
65. The first radical is cqual to y (a -f- l ) 4 {a — 1). Taking tiie factor (a -f- 1)
outside the radical sign, \ve get a —-1. The given expression is
cqual lo ____
i _____________
2 1 ' 1r <a+ l)(a + 2)
Bring the radicais to a cornmon index:
a_ |a+ * IV (a~ U3
2 (fl + l ) ( « + 2)
If lhe numher a -j- 1 is positive, lhen |a -f 1 |— a -}- 1, and on reducing, \ve get
o ] { a — l)3
~2 7+ 2 “ '
Note. The numher a -}•• 1 is just positive. Indeed, since the radicand
(a _i_ _ i) ís assumed to be positive (or cqual to zero), and the íaetor
(a -f* l}4 is nonnegative in all cases, thcn « — 1 > 0, i.e. a ^ 1, and under
tljis condition a + 1 ^ 2.

Answer: -tj- —
(o -l)3
a 2
66. Assuming all the roots to be arithmet ical, brins the íactors

(i-h lÜ ± i \/3 {/ V3«»


~lia " " " y 9+18a~t + 9 a -2 V (J d + « )-
to a cornmon index 6. The first and second factors take the respectivo íorm
í/ (I -r- «)3(1 -r «) . \
V 27a3 ’ V 8 1 (i+ a )4

Muítiplying thern, \ve get ~ y^ã.


Note. The first factor is an arithmetical root only if a > 0 (if a < 0, the
radicand is negative, at a =•- 0 it loses its senso). The second factor is an arithme­
tical root at any « (oxcept for a = —1). Consequently, the quantity <2 may have
any positive value.
Answer: — y <>■
67. Place ah under lhe first radical sign. The given expression takes lhe fonn

"/»
Note. For tlie given radicais to be arithmetical roots the following condi­
tion must be observed: a > b. Tlie case a — b is excluded, since the second
factor loses its sense.
7 —0 1 3 3 8
98 Answers and Solutions

A n sw er: 1.
68. Rationalizing the denominators, \ve get
(V «-H ) ( V ã + íi)= -iiõ
A n sw er: — 115.

69. The dividend ia equal to — .; the divisor, to


~\/a—b-l-'\/ a-\-b , . *|/a—è
—....... ...! t.— !— ; the quotient, to -~L-r ----- •
y a~-b b
Note. For all the given roots to be arithmetical the íollowing three conditions
must be simultaneously satisfied: a ^ 0, a — è > 0, a -{■ b ^ 0 (they may be
replaced by the following two conditions: a ^ 0, j b |< |a } ).
A n m e r -. .y * j p L . '
0
70. The dividend is equal to
b- ——
a ; the divisor, to b- ——
a ; the quo-
H
o
tient, to — . The quantity a may have any positive value; b may have any
value, except for ± ~\/a.
A n sw er: — .
71. The uumecator of the íirst fr&ction is reduced to the form

( V ~ ) 3-j-(y n r ^ )2 iiia i^ ii-a i


V l —a~ yi—

lí the exprosdons í + a and i — a are positive, then (see Prelimiiuu ies


on page 90, Item 3) the mmierator is equal to . 1'nder the same
y i-« a
! 1_l_« |—i i —„ |
condition the dcnominator is equal to
VI- VI-
fraction is equal to ~ , and the given expression, to 0.
Note. For lhe radicais contained in the given expression to be arithmetical
roots it is necessary that the quantities 1 - f a and 1 — a be of the samo sign.
Rut it is impossible that both of them are negativo, since 1 - f a < 0 ií a < — 1
and 1 - - a < 0 if a > 1, but these conditions are ineompatibie. For the quanti­
ties t f a and 1 — a to be simultaneously positive the following condition
should he fulfilled: — 1 < a < 1, i.e. j a [ < 1 (the values a = ± 1 are exclud-
ed. since at each of them one of the given expressions ^-L~~ , | loses its sense;

the value a — 0 is also exduded, since the fraction — loses its sense).
Answer: 0.
Chapter I I . Algebraic Transformations 99

72. Substítuting x — i nto the expression ~[/x2~ Í , wo get

= / | ( . + - i ) ! - i = | / { (■■ l ) : - 4 - j— 1 1

Since, by hypothesis, a > 1, then a — ^ - > 0 . Therefore

V 3 = T -y (— t )
Similarly \ve finei
V V _ ! = i-(6 -.-i)
Substitute the values of the radicais found into the given oxpres>ion.
Answer: 3262+1 *
2 am
73. SuLstituting x into the expressions ~\/a - bx and
6 ( l + m2)
~\/a — bx, we finei

V a+ t>i—y r “ + 1 ^
2om .
i+m|, t/
y 11

and
~\/'a—6.r = |I — m J
+ '»-
Since 1 + m2 is always positive, then o rtiusl aiso be positive (al a < 0
both roots are imaginary; at o -- 0 they are equai to zero and the given expres-
sion is indetermiiiate). Since, according to the additional condition, | m | < i,
then both 1 -p m and í — m are positive.
The given expression takes the forra

i « + ' > o / T + r - ( t - " ) | / T+ r


Answer: ---- (for a > 0).
74. The problem is similar to the previous ono. We have

{m— x) ~ =
2 mn 2 Vo-»)2 -i|
/12+ 1) p n 2+ 1 V « 2+ i
Since, by bypothesK n < i , then
JL fii, 2 (1 — n)
(m—x) -
Vnã+T
100 Answers and Solutions

By analogy,

(m-f x),2 ” 2 {! + ")


>v+i

2 T/k
75. Substituting x = into the expression 1— x2, \ve get
1+ *

(H-*)* (Í + A)2
isTo\v we find

(l-x 2 ) H+ M
=1 :1 /1 - *
|i —*1
Since, by the additional condition, fc> l,^ th en the quantity 1+ A is positive

and 1 — k, negativo. Therefore (1— x2) 2 — ^ ^ ^ . Inside the first brackets


k— i
we <>et - — j - , inside the second, -7- •. The given expression is equaPto
“ k—í k

V A -1
+Vk=*
Vk
Answer: ~]/k~~ 1 ( l - f y =~) *

76. The expression ín the first parentheses i$ equal to ------ ^— — • (the


exponent — 2 refersVonly to the numerator oí the third addend!). Simplifi-
cations yieid —— ^ ---------------
4a 4«
The expressions
1
f ( « + l)-3= a-f-1 and (a-fl) = V ( a+1)3
wilt be arithmctical roots only if « > — 1. At this condition the radical
V ( o 2— 1) (a— l ) = l / ( o — i)2 ( a + l )
will also be an arithmctical root (since the factor (a — l)2 cannot be nega­
tivo). The equality
1/(a — i)2 (a-f-1) —(a— í) y j + í
is troo only for a > 1. If a < l , then
y i a - l ) 2 (a + l) = —(« — 1) 1/r+T
(see Preliminarics, Item 3, page 90).
Ckapter I I . Algebraic Transjormations 10!

The given expression is equal to


(a—1)2 f a—í o+l 1
Ta LT+1 |« - í IJ
Note. At o — ± 1 the expression loses its senso.
, « —1 , (o2+ 1) (1 — a) f
Answer: -------— for a > 1; ---- ■%
—— r r ;----- for - 1< « < 1, i.e. for
o+i 2e(a + l)
M d -
77. The given expression may be represented in the form

2f / $ J --------- / p — ,-n
2M ( M f )
It is assumed that J2-~a2> ü , i.e. jx | > ja | {olherwise the root \'s~ — a-
will not be arithmetical, lhe case |x| — jo| is excluded, since the seeond
radicand loses its sense).
The íirst factor is raiuced to the form
h «2 |_a2 t2—0/í
2|x| y .í2_-a2 Vx2—al
(sinco x2— a2> 0 , then |x2— a2 J= x2—az).

The expression - y — J2 is transíormed to lhe form

/r ( x 2 - 2 a2 \2 j xz 2o21
V V— ^ J M M
Here lhe numeralor can be written in the form x2— 2o- only if x2— 2a'- i
i.e. if |x |> j o | V - -
Now the given expression is written in the form
x~ — 2a2 ~ f x2 — a2 j a j •|x j
2 |x[
‘ y x2— a2 2ox |x2— 2a2 j
Taking into consideralion that j x j-| x |= |x |2— x2 and reducing, we obtain
x2— 2o2 tal x2 — 2o2 | a I
-----------. — _!— + — x, or -------------- —õ— j x, wluch is lhe same.
a |x2— 2«21 a I x2— 2a2 |
Answer: lí | x| > }a| , the given expression is equal to ± x; the plus sigo
x2— 2a2 x2 _
is taken when-----------> 0, and the minus one, wiien--------- <u.

J[ .„xt ,..^ L = u, i.e. ií ] x |~ |a |~\/% the given expression loses its seus
‘use.
78. Get rid of negative exponenls. The numeralor takes the form
2ab~\/a 2a6 T/ò _
yi+yi“ i 1yz+yz
" 1 /- ***2ao,
102 Answers and Solutions

the denominator becoines


2ab ( JL_)
>a-fV ab 64-Vafr
-V ab />
Noting that
a+ V ^ —V a (V fl+ V*) and b+ Yãb —V^(V^“r V fl)
represent the denominator ín the form
1 2aô
2afc T+“ =2 Vaò
^V a (y â + V ò ) V 6 (V « + V 6)0 / - V ab
Now the given expression is equal to
2ab
— = Vab
2~]/ab
Ansiaer: ~\/ab•
79. The expression in the first parentheses is reduced to the form
2 ']/ãx
V a+ i ( V a+ V i )
Raising it to the power — 2, we get
(a + *)(y°+lA)2
4ax
Simiiarly, the expression in the second parentheses is reduced to the form
( « + * ) ( y » —~VxY
4 ax

When subtracting, factor out ■ (simplificat-ions yield 4 V õ x inside


the parentheses).
. a-i- x
v «
80. Aíter simpliíication the last addend takes the form
2Y x
Reducing all the fractions to a common denominator and summing, we get

2< ^ ± l - y g + ;
•>y^+a
A u s iv e r : _______
81. Ansiver: 2 (x 1 / x2— i ).
_ 3 1 2 _4
82. Inside the brackels we have a 2 ba 2 ba'* — a 3 b-. The given expres-
sion is equal to a~lbr>. Substituto into it
Y 2 ,, 1
a— and b~
2 >' 2
Chapter I I . Algebraic Transformations 103

Answer: 1.
1
83. We represent the given expression in the form and
«+ ! *>+1
make the followtng substitutions
1
a = --------- — V 3 and 6— --------- =* 2+ V ‘à
2+V3 2— V 3

We find a -f-1 —3— V 3 i — \-r~ - and so on.


v a-j-1 6
Answer: 1. ______
84. Answer: '}/x- — 4x.
85. Answer: n.
80. For all the roots to be arithmeUcal it is necessary that x— a2> 0 .
4 V x y .r —
The expression in parentheses is reduced to the form —
«2
The given expression is equal to

}/x / _____ a2_____ \ _ 112


\ ~ 4 \,'x \ /” 4{.f —«2)
,
A n sw er. — .
a2 ,t ~ .
4(x —ü2)
87. The denominator oi the socond fraeliun is equal to
ü 1 1 i
X“ — i - l* -)3 — 1 ^ (X - — 1) (X - l - X 2 -J -ij

Answer: x — 1.
88. The dividem! is equal to
3 3 l i i i i1 2
25 “ 27y5 (2=)* f (% 5)3 = (2® + 3»5) ( 2 - 3 . 2 J» 5 + % 5):
1 l
the divisor is equal lo 22-f3 yb.
Answer: 2 — 3 1/ 32i/2-|-0j, i/2.
89. Let us get rid oí negativo exponents in the socond teiin by multiplying
hoth the numerator and denominator by «2.
We obtain in the numerator «3— 1, and in the denominator
i _í 2 i i _i 3
a’ (a2 — a 2) = a2 { a 2 (a 2 — a -)] — a 2 {a — i).

On reducing we gel a • Similarly, tbe third tcrm is equal to

1
104 Answers and Solutions

90. The dividend and divisor are respectiveiy transformcd to the form
3 3 i
<z2-f-62 (a— b)3
i i”; ~~í i r
a3 (a — 6)3 a3 (a2— b)~
3 3 3 3
Taking into account that (a2 - f 62) (a2— b2) = a3— ò3, vre obtain the quotient
az -$-ab-{-bz, At a — 1.2 and ò = -| - we get 2.52.
Ansuwr: a2-f- ü6-j-62; 2.52.
91. Removing brackets and collecting like terms, we represent the dividend
1 l i i í i i i
in the form 6a262 -}-96 = 362 (2a2-}-362); and the divisor, a2(2a2 -f-362). The

quotient is 3 V \ ; at the given values <z= 54 and 6— 6 it is equal to i .

Answer: S j / " 1,
92. Multiplying both the numerator and denominator of the given fraction by
_l _i i
l(a-}-6) Z~r(a — 6) 2H(a + 6) ~~~{a — b) 2]

we get íor the numerator


_t _i _ _t i
{(« + &) 2- ( « - 6 ) 2H - [ (a -f5 )~ 2 - h í « _ 6 ) 2] = 2{tf + 6 f 2
and for the denominator

-i -i i _i
l(« + 6) 2—(tf—6) 2]~~ [(a+ 6) 2-{-(a_fc) 2] = —2{tf~6) 2

Answer: —\ / ~ ~ .
V a-j-b
93. The first factor o! the subtrahend is reduced to the form i —theu
d-
we have

1 7 -i
n?.(1 „, a~2 (1— ° a) t ( l — ggi^ + q g d — a3) -|
' 1—a2
Redueing the fraction by (1 — a2), we obtain
_ J. i _i l
a2 (i — a2) ~ — (i — 2 = ~ ( 1—u2)2
.•tmii-rr: —']/1 —«2.
Chapter I I . Algebraic Transjormations 105

94. The given expression is equai to

j » — i________ y r + 3 —i
] / i ( j - f - i } ( i 2+ l ] \/s (1 -(--x2) t 1 —j -
________ i 3- i i V í
T / x ( * + l ) ( x 2 + l) l / x ( l + x2) ■r + 1

,
Answer:
Vx
---- —7 - .
x -í-1
_ 1_
95. The numerator ot Uie th ml term is reduced to the form'R 1 {R 2 — x-)
The denominator is equai to The given expression takes the form
1 _1 _i
( i ? 2 _ * 2 ) 2 _ x 2 { / i 2 _ x 2) 2 + ^ 2 ( / í 2 „ x2) 2«

1 _1 1
= (/i 2 _ x 2 )2 + (R i _ x 2) - 2 (R 2 _ x 2) — 2 ( /? 2 _ T 2 )2

Answer: 2~\/í& — x2.


90. The first and second addends are reduced, respectively, to the form
l l
P+q . 2(P2+ r ) .. 2
1 \ ’ i i 11 í i i 1
p q (p l j r < T )2 (p 2 + < r )3 p 2q2 f p 2 -\-<i 2)2 p 2q2
Reducing these addends to a common denominator, \ve get
1 j:
p -\ -q + 2 p 2q2

i i
pq (/>2~f?2)2
1 1
The numerator oí this expression is equai to (/>2-f<?2)2.
Answer: .

97. Introduce hcre fractionai exponents. Factor out a3 in the expression


2l 2 12
a-f-a3x3, and x J in lhe expression x-f-a3x3. Then the numerator of the
first fraction will lie
2 2 i i i i
a 3 — x :i ( g 3 H - x 3) ( a 3 — j 3 )

X' x•
106 Answers and Solutions

and thus the first fraction is reduced to the form The bracketed

expression takes the form


i
3 1
2~

Answeri .
x4
98. Represent the binomial a — ~\/ãx in the form The
numerator of the fraction will be

( 1/ õ - f l ) 2— V 5 « a + V í + 1

The denominator is equal to 3 (a -f* V a + t)«


Answer". 27.
99. Get rid ofjnegative exponents; reduced by 2a — 3, the first addend
takes the form — ; reduced by a — i , the second addend yields .
a '* a 12
Answer" 9a.
100. Take a — b outside the brackets in the first factor. The quantity
ti -T- b
cannot be negati ve (otherwise the roots are not arithmetical).
The given expression then takes the form

.insuw: 26 (a —6).
101. Represent the dividend and divisor, rospectively, in tiie form

a yÕfe a~\/b t 6 ( } a — y ò)
a-f-l/írà y a + 'i /6 ’ a—b
The quotient may be reduced by — taking into
account that
fl-6^(yí4-y6)(v'ã-i/& )-(yã-fi/6)(yã+y6)(ys-5/6)
Answer: a y 6 (y 'a -f-y 6).
102. Represent the given expression in the form

( V ^ + C lA )3 . a - V a V b + b - f
Va ‘ y'a(\/a — ~\/b)J
Chapier I I . Al^ebraic Trnnsformaiions 107

Factor the nutneratur oí the dividcnd as the sum of cubes. On reducing


we have in the brackets
W ã + y b ){y z -y b )= a -b
Answer: y (a—ò)2.
2 s
103. Heduce the fraction ------------ 37= - by y x. The bracUeted expression
x y' x — 4 y x

is reduced to the form ~ . Reduce thís fracüon by V ^ + 2- The


given expression is equal to

(■- y f z r ) 2-- v íy + W 1 =■( y i - - * r-._ 1*.+.41

It is assumed that x > 0 (at negativo x the root > x wiü not be arith
metical; at x - Q the given expression loses its sense). Thercíore x -j-4 > ü.
>tnsu>er: — k \ fx .
104. The bracUeted fraction is equal to
2(Vx + Vi/) 2
v*(y*+y») y*
The given expression is equal to

z ^ { ~ = Y y iy í= x3-32x 2
' x~ ^ 3 2 i

Answer: 32x.
105. Factor out y ax in the numerator of the first fraction. Taking into
account that j x - — y a- ~\/.t — ~\/a, reduce the fraction. The first faelor
takes the form __
r 4 — , 1 + V a x ]“ 2 / 1 t/—
- > ax - f ---- „ = ( ^ V ax
L | ax J ' y ax >

The second factor is y ( 1+F 7 ) ■ si,,cc / is an arithmetical root,

the expression t + >s aiways positive.


Answer: a(^ x V a ).
106. The quanlities a and c must hc* positive. Thereíore. the denominator
of the first fraction, whicJj is reduced to the form
1/2 (a — b2)2 + 8aí-2 y 2 {a + **)*,

is equal to y 2 (a -\ -b 2). The numeralor of thís fraction is equal to \ ^ («


l /õ __
The second fraction i s -----~ r - \ ac.
ya
108 Ansieers and Solutions

Answer: — ^ ac.
107. The mtnuend is equal to
2
X2
5- l ]
1 a2
The radicand of the subtrahend is equal to (a- - f x2)2 the quantity az -\-xz
being positive.
Answer: — 1.
2¥7
108. The bracketed expression is equal to — ——— : raisíng to the

(V x -y j)2
powei - 2, \ve get - On reducing by V x -f \r a the divisor is
4 ]/i

equal to —------ -----— .

A„ ^ er, .
V*
10Í). Factor out > x ín the nuinerator; reduce the fraction; the given expres-
sion takes lhe form
(2 > x )3 + 4x + 4 - f ( V x + l ) " ^ 5 x - f 10 V x - f - 5
Answer: 5 ( x -j-1)“ .

110. ín tiie First fraction transpose x J from the numerator to the deno
minator (vith a positive exponent); the fraction turns out to be equal to
3 3
-— - . Heducc the second fraction by x . The given expression takes the form
/ 3 1 \ -i /l-2x\ -l (x — 2) (x — 1) 3x — 2
l x~2 x-1 / \ 3x — 2 / " 2x — 1 l-2 x
x2

111. The First factor is equal to Squaring the bracketed expression, wo


get a -—- 2 ab.
2
Answer: 2 (a— b).
1 i
112. Cube the difference j.'x — y « = x ,J— a3; the rmmerator of the fraction
is Oiiunl to
2 1 12 1 2 t t 2
3x — S x V + 3 i V = 3x:l (x3 — x V + a3),
lhe dcnnminator being equal to
21 12 1_ 2 2 l l
— 3 «— 3x’V K íxV * ~ — 3« 3 f«3-}-x :! — x3o:!)
Ckapter I I . Algebraic Transformations 141

Reducing the fraction, \vc get — . The given expression is equal to


ã*
i ,3 ______ -
- • 4 1 + y « o + i )3^ “ =>

Answer: i.
113. The numerator of the first fraction is equal to
a + 2 V ã b — 3 b * = ( V Ã ) 2 - f 2 V ã Y b — 3 ( l / 6 ) “ = ( l / 7 - f 3 \ , ' T ) ( \ / 7 ~ yr>)
1
Answer:
2b •
114. The denominator of tiie first fraction in parentheses is
i i i i j i i t
(a2)2 - f « V — (i (í,-)2 ^ (a2 — 2ò-) (a2 4 36*)
li 1!
The denominator of the second fraction is equal to (a24 3b2)2. The numerators
aro factored in a similar way.
Answer: ------x r .
a — 9b
115. Reduce the fraction in parentheses by V a + V^* The first of the
fracüons enlering the given expression is equal to
3 ‘\/a (a — 1 ah 4 fo) _ 1
3 V í i n ^ y + u / b ) 3] yT +yi
The second fraction is equal to
y ã ( y b - y a) _ 1
yã(a~b) 1 /7 4 1 / 6
Answer: 0.
11G. Answer: 3.
117. Get rid of negativo exponents. Factor the expression
3 a t t
a~—62= (a2)3— (62)3
Answer: i.
118. Transforming the first hracketed addend, \ve get
_______________ l-fl2 ______________
yã\( Í'7if-\- . 'í + i l |( í í p — i 'í+ l|
The numerator of this fraction is equal to
( i ~ fl)( i 4 « ) = í í - ( ^ i H 4 í r « í 3i
Factor the sum and difference of cubes.
110 Answers and Solutions

Aiiswer: a, _
llí). Factor out { a in tho numerator of the fraction. The multiplicand
is equal to y 'ã — and the multiplier, to
4 ( y az -}- y ax + y' a:2) .
Answer: 4 (a — x).
120. Represcnt the fraction in the form
V ã W V ã f-V b f]
= a (a V a b -j- Y è 2)
V a— b
The expression in the first parentheses (the dividend) is equal to
yã(a + 2 yã y b-f yp) = yã(yã+ ysf
The divisor is equal io y a ( y a -}-y/" b).
Answer: a,
121. The denorainator of the minnend is equal to V a * ) , and
the numerator, to ~|/a y x (( y a ) 9 -j- { í Jr)'5]- Reducc the minuend to the form
a — ^/a Jr x - f y x . The subtrahcnd is equal to V {a 4- V * )“ = j a 4* ~Vx \*
Instead of the latter expression \vc may write since a -\ ~ yx is a posi­
tive quantity (the quantily a cannot be negative, since the given expression
contains V « ) -
d»i'U’ír: a'*x.
122. The factors of the deuominator are equal to !-{-> x and 1— { x.
The numerator may be represented in the form — x(t — V x ) .
A n sw er: — x 3.
123. The numerator of the bracketed fraction is equal to

í «3({ « + V*)+* v + a+ Vfc)[(y «)3+ (Yh)3] -


= (> af v ^ r V 0—i'’ « ys-j-ô)
The given expression is equal to

« y r ) ' 1-'— v
1b-l a » (í n—y *) >/ ã—y ^
.• l»síi’<*r: 0 .
124. The numerator of the minuend is equal to
(r iy + c r x f
( y a ) 3 — (5 x)2 ( f « - Y ) 2
4
A n sw er:
, j6 a.
Í2Õ. First add the first Uvo fractions; the commoR âenominator is equal to

1 i 1 1 i 1 1 1
|(a'M t) + « 8][(o'i + !) — «*] — (o4+ !)= — « ' = o 2+ a‘ + l
Chapter I I . Algebraic Transformaiions íll

2(<z4 + i)
We get . Now subtract the tbird fraction; tbe comraon denotnina-
1 i

i
tor is a - f az - f 1.
4
Answer:
a-\-ar-\-i
126. V V í - 1 / V í - 2 V 5 = ^ ( 1/2 - 1)'2 (3 + 2 1/ 2) = 1. A similar
transformation is performed in the denominator. Theá literal radicand in the
numerator is equal to (V-*—2)3. The fraction ^ X is reduced by
Vx-Í
1 /x -t .
Answer: 1.
Í27. The numerator oí the first fraction is equal to

a -y -j- ab y — a (a -f- b) y

The denominator is transformei] to the forra (ò-j- a) (b — 2a)|j! a^bJ. Thus, the
first fraction is equal to ♦ The dividend in parentlK'ses is equal to
3a^ a~\-b ____, 4_ 3a3
- . Dividing it by we obtain Subtra-
tb -2 a ) (3 — b) 3a — ab (b — 2a! la-j-b)
a (3a2 + 2ab — Ifi)
cting then ^-pj-.wofin,! (ÍTinS=r.U]
The given expressio» is equai to

a ^alSa — b)
b^2ã
Answer:
2x-f-a
128. The multiplicam! is equal to . The oxpression in the secoml
brackets is equal to j/2 x — a.
Answer: 2x + ®*
12Í). Answer: ]/ l.
130. Answer: f ^ ... .

131. The minuend is equal to — ^-r-, and subtrubend, to — ______


, a+ b (a + b) (a + ib )‘
Answe.:
a -f 2b
112 Answers and Solutions

b 2a-4-6
Í32. The first addend is equal to — r-r-; the second, to
a-\-b « a-j-b
a-f-6
Answer:

133. The first addend is cqual to - ■■■■;•^ : the second, to -i*. Answer. 4--
^ ao a b
2 b— a
134. Answer: -7-r - — .
Io-\-a

C H A P T E R III
ALGEBRAIC EQUATIONS *

133. Write the fraction in lhe form l + T r - r Then the given


6o 6 6
equation takes the íorm
a (h — 3a)
262(6-0) y ~ ¥ T
76 (6 —a)
vhence y
3 (6- 3 a ) '
76 (6 —a)
Answer: y
3 (6 — 3a) ‘
136. Gel rid of the denominator (the common denominator is a2 - b ~ ).
Answer: x ~ 0,
Í37. Solving the given equation by the general inethod, \ve get
3aòc -4- ab (a -f-6) - f 6c (6-f-c) -f-ca ( c - f 0)
ab 6c -f- ca
This fraction may be rednced by íactoring the numeralor (represent the expres-
sion 3a6c in the form of the trinomial abc -f abc ~r abc and group each of the
suceessive terins with a6c). We get
x — a -f- 6 -}- c
The solution is simplified with the aid of the following artificial method.
Represent the addend jn the form - — -j- 1 and perform

* In solving problems of the present chapter we do not consider singular


values of the known quantities. at which a given equation loses its sense or lias
no sohition. or acquires more Solutions.
For instance. in Problem 135 the given equation loses sense at 6 ~ 0 and
at 6 - a ----- U. since at 6 - 0 the denominators of the first and second terms
vanish. and at 6 — a --- 0 the same happens to the last term. Furlhermore, at
a 0 the given equation has an infinito numher of Solutions, becausc it takes
t)ie form i -- 1» becoming an identity. Finally, at 6 = 3a the given equation
has no Solutions at all, since it is rednced to the form 0 -y — -L
Chapter I I I . Algebraic Equatioi 413

similar transformations with the othcr two addends of the loft member. The
equation takes the form:
(x -ía + O + c)] ( I + ± + | ) = 0
Answer: zr=a-\-b-\-c.
138. The common denominator is 6cd (2c-|-3d) (2c- •3i).
C(4c2 — 9Í2)
Answer: z =
8c2+ 27d2
n2 (1•- x ) . , . 2n2 ( x -
the form ------1— i) (to obtain
2
139. Represent the fraction —- id
- i 4 -
the sarae denominator as in the next fraction). It is advisable to transform
the fraction-------r to —t-------. Transpose ali the terras lo the left and group
n—1 1— n r
them (the first ono with the fourth, and the second with the third one).
We get

(1- x )( _ ± _ + T ^ r ) + r J - r !2„2(x - i ) - (2x - 1,) = o


1 1 I
Transíormini! ------- h -7- 7— to tlie form ------ v . "'e get iirf of the denomi-
° i —n 1-{- n *—
nator.

4
140. Transpose all the terras containing x to the left side of the equation,
and all the known ones, to the right side. Reducing each member to a common
denominator, we get
(3afc-f-4) (a-f-1)2— (2a -f* 1) 3ab (a-|-1)2-\-a~
a (a-f-1)2 (a-j-1)3
or
3sfe (a-f- l)2-ha2-j-2a-f- 1 — 2a— 1 _ a |36(a-f- l)--f- a]
_ _ X (1+1)5
Whence
a (36 (a-4-l)2- f fll a [36 (a -|-1)2 + aj
a(o + l)2 x- ( a + i)3

Reducing it, we find

a-f-1

Answer: x -- — r~r
a-f -1
141. Group the terms as in Problem 140; on transíorraing we get
abf3c(a + 6)2+ fl6] a (3c(fl-f 6)2+ a6]
(a Ar 6)3 ~ a (a 6)2 X
8-01338
114 Answers and Solutions

Answer; x = — —7- •
a -f b
142. The common denominator is (a -f b) 2 (a — b).
, m(a + b)
Answer: x —--------------
a
143. Rewritethofraction in the forjn ( - .■"'*„,!■) ■Th™ thecommon
denominator is mz (z2 — m2). Getting rid of it and collecting like terras, we get
m3z2 _ 4m3z ~ o. This equation has two roots: z — 0 and z = 4m. But when
rejecting a denominator containing an unknown quantity, extraneous roots may
appear; and namely, these are the roots which, when substituted into the com-
mon denominator, nullify it. In the given case z = 0 is an extraneous root.
It does not satisfy the given equation, since the íirst and third terms lose their
sense al s = 0. The root z — 4m does not nullify the common denominator,
therefore it is not an extraneous one.
Answer: z — Am.
144. The common denominator is bi — x2. Getting rid of it, we obtain
2x (a2-f-ô2 — 2afc) = 2 {c2— fe2), whence There are no extraneous

roots, since the denominator b* — x 2 does not vanish at x = g ~^~f .


a—o
, a-f-i
Answer: x .. .....- - t- •
a—o
145. The common denominator is (x2— a2) (x-f-n). Getting rid of it, we
n2
íind x = — . At this value of x the denominator does not vanish. Hence,

x— — is the root of the given equation.


a
Answer: x = —— .

146. Rewrite x-fo*"1 in the form x - f ~ * Aíter transformations we get

2a _ 2 _x
ax-f- 1 a x -f 1 ~ 2

Reducing by a x -f 1, we find x = 2a.


Note. The' reduetion by ax-f-1 is possible, provided a x -f i is not equal
to zero. But at x = 2a we liave a x - f 1 — 2a2- f 1 > 0. Therefore, the obtained
root is not an extraneous one. But suppose, for example, we have the equa-
‘)a o z
tion — — r— •----- s— in this case the reduetion by x — 2a would also
x — 2a x — 2a 2
give x — 2a. However, this root is of no use bccause the fractions------
2 a
x — Za
and — lose their sense at x = 2a. Thus, the equation — : --------— —
x — Za x — 2a x — Za
=.- -í- !ias no solution.
Chapter I I I . Algebraic Equations 115

Answer: x«=2a.
147. Revvrite lhe equation in the form
a-f-x a—x _ 3a
a2 ~\-x2 -j-ax a2-f-x2— ax x (a4 ~f a2x2 + x4)
The comnion denominator of lhe left member (a2-{-x2-f-ax) (a2-f x2~ ax )
may be transíormed to
(a2 + x2)2— (az)2 a4 a2x2 -f- x4
We get
2a3________________ 3a________
a * a 2 x2 x i x (a4-|~a2x24 -x 4)
3
Anmer:
2a2 -
148. Transpose the terms containing the unknown to the left side of the
equation, and the constant terms, to the right:
(a _ 5_ \)~)/x = (a2 —62) — (a-f-6)
After factorization of the right member we obtain
(a —b — \)~\/x =(a-\'b}(a — b— 1)
Whence we have ~/z — a-f-6.
Since the exprcssion ~\/x means the positive value of the square root, íor
a-f-6 < 0 the problem has no solution.
Answer: x — (a-f-6)2 (if a-f-6;> 0).
149. Getting rid of the denominator and collecting like terms, wc get
2x2 + 6ax-f-3a2 = 0.
tt (V3 —3) x2
fl(V 3 + 3 )
Answer: *t--
2 2
150. The comnion denominator is 4 (x-f-6) (x —-ò). Sirnplification yields
12x2—46x— 62 = 0
. b b
Answer: xj = y ; x2 — — -g-.
151. The commoD denominator is (x —a)2. Getting rid of it, we obtain
(x — a)z — 2 a (x — a) -f- (a2 — b2} = 0
From this quadratic equation we find
x— a ~ a b
Answer: X j = 2 a - } - 6 ; x 2 — 2 a — ò .
152, The common denominator is òe2 (a — 26). Rejecting it, we get
(cx)2— (a — 26). (cx) — 6 (a — b) = 0
From this equation we find
(a — 2 b )± .a
c x - ------- r-------

8*
146 Answers and Solutions

a —b b
Answer: x* = --------; x2-- ------ .
c c
153. Rejecting the denominator, we obtain the equation 4x (x-~a)-j~
-}- 8x (x ^ -a )= 5 a 2 or, alter simplification,
12x24-4ax— 5a2 = 0
.
Answer: =
a r
;x 2 =
5a
-----g - •
154. The common denominator is n(nx — 2). After simplifications the equa­
tion takes the form
(n—1) x2—2x—(n-f-l^O
Answer: x« = 4» • x2~ — 1.
n— 1
155. The common denominator is a (a — x)a. After simplifications we get
the equation
(a -f- i) x3 — 2ax -{- (o — 1) — 0

Answer: Zf. = 1: x2-


a ~f~1
156. The common denominator is (x a)2. Getting rid of the denominator,
we obtain the equation
(x —- a)2 — 2b (x ~ a) — (a2 6a) = 0
Solving it, we find
b ±(
Answer: xt = 2a -f- b; x2 — &•
157. The common denominator is nx (x *— 2) (x - f 2). After simplifications
we obtain the equation
x2 — (2 — n) x — (2n2 -f- 4n) ~ 0
A nswer: xi — n -f- 2, x2 == —2n.
158. First method. After standard transformations we get the íollowing
equation
x2 - f (<* — 2n — 2a -f- u) x — (a — 2n) (2a — n) = 0
Jts solution can be found at once, if we draw our attention to the fact that
the constant term is the product of the quantities —(a — 2n) and (2a — «),
and the coefficient at x is tne sum of the same quantities taken with the reversed
sigD.
Second method. Transposing unity from the right to the leít, we get
a - f x — 2n a— 2n-}-x
2o— n x —

whence: (1) a —2 n -fx = 0 or x1==2n—a


4 1 A
------------= 0 or x3 — 2o — n
0
<2>ãT
Chapter I I I . Algebraic Equations 117

Answer: x\ — 2n — a; x 2 = 2a — n.
159. We get the equation
(n — l)2 x2 — a (n — 1) x + (a — 1) = 0
to avoid operations with fractions it is advisable to put (n — 1) x — z or directly
íind (« — 1) x írom the equation
|(n — 1) arp - a [{« — 1) x] + a - 1= 0
We get
(n — 1) X{ — a — í; (n — 1) x2 = 1

. “l i . 1
n— 1 n— 1
160. The denominator of the left member is equal to (a — x)2. Multiplying
both members o£ the equation by it, we find
/ a — x \2 / a \2 5 / a — x \2
l" 7 " ) :
4 f a — x \2 / a \2
9 l x / l a+ 6 )
Taking the square root, we get one of the two equations:
2 a —x _ a , a—x _ a
3 x ~ q+ 6 ân 3 x a+ 6
2a
(a-f- b) 2a<a-f 6)
Answer: x f ■
5a+ 26 * ' 26-
161. First transform the expression
(1 + ax)2—(a+ x)2= 1+ a2x2 x2
Grouping the first term with the iast one, and the second with the third
one in tfie right member, we get (1 — x2) (1 — a2). Now the given equation is
rcduced to the form
ab
x (x + l) =
(a—bfi
a a
Answer: x« — -------; x2= ------- — .
6
* 6—a * a— 6
162. The trinomial ax1 -j-òx-f-c is factored into íirst-degroe factors in the
following way: ax2 + 6x + c = a (x — xj) (x — x2), whore xt and x2 are the roots
ol the equation «x2-j-6 x + c —0. In this caso a = ~ -3 ; Xi = 7; x2= ------ jiL ;

thus we get — 3 (x— 7) ^x-j--—- j .


A n sw er: (7 — x )(3 x -}-!()).
163. Sínce
a___6__ a2—62 _ (a+ 6)(q—6)
b a ab ah
118 Answers and Solutions

we may, by guess, factor into - l í í - and ° ^ b- |their sum isequa!

to ~ - f . — j . Now it is necessary to find out whether this solution is unique.


Let u and v be the required íactors. By hypothesis,
a b , , a 6*
b a b a
Consequently, u and v are the roots of the quadratic equation

The expressions for u and v will contain the radical

/G f+ T )-«(T -7 )
Knowing íor sure tbat a rational solution is possible here, let us try lo get
rid of the radical. For this purposc ínstead of + wrile the expression

( ’F ~*o*)2 an^’ *or con?Peasat'*on’ add *-G- 4» then under the radi­
cal sign we get a perfect square ^ ~ _ 2 j Z.
a J~b a — b
Answer: — 1— •— j---- .
a b
164. 15z3-f j 2— 2x — x (15.c2-f-x— 2). The roots of the equation iõ r2 -}-
—2 = 0 are and x.» — — Consequently,

15z2 -f x — 2 ~ 1 5 j (a-H—=-) = {a r — 1) {5^-h2>

Answer: x (3.r — 1) (15x -j- 2).


165. First method. Represent the sum 2x4 + 4x2 -j- 2 in the forni 2 (x2 + l)2.
Second method. Arrange the polynomial terms in the order of decreasirig of
their exponents and break up the term 4x2 into two summauds 2x2 - f 2x2; then
group the first three terms and lhe last three ones and carry out factorization.
Answer: (x2 + 1) (2x2 -j- x -f- 2).
165a. Rewrite the left member in the foilowing way
(1 - x2)2 -j- 4r«
The equation takcs the form
(1 — x2)2 - 4x (1 ~ x2) -f- 4x2 « 0
or
1(1 - x2) - 2x)2 - 0
Answer: x t --- — 1 + V ^ - x2" — 1 — V 2 -
166. The required equation is ^x— ~ ^ ^ x—
Chapter I I I . Algebraic Equalions 119

Answer: abx2 — (a2 + b2) x + ab = 0.


167. By Viète’ s theorem the sum of lhe roots x x and x2 of the equation
x 2 + px + q — 0 is —p, while their product is equal to q. Hcnce,

___________ _ _ _ i ____________ —
—2-10 2ü
i o — y ?2 1 0 + 1/ 7; 100 — 72 2S
1__________1 1
10— Y n ’ 10 + 1 / 7 2 2tí
20 1
The required equation is x2— 28"x ^" 28’ ” ^'
/ íjísojít : 2 8 x 2 — 2 0 x + 1 = 0 .
168. Solved like the preceding problem.
Answer: bx2— 2a '[/ax-\-a2 ~ 0 .
169. According to Viète’ s theorem, x\i% — 12; by hypothesis, x s — x2 = 1.
From these equations it is possible to find xt aod x2 (4 and 3, or —3 and —4)
and then p — — {xj + x2) — ± 7 .
But to íind xí + x2 there is no need to determine separately x, and x2. We
may compute
(x { + x 2)2 = ( x , — x 2)2 + A iii-i — *2 + 4*12 ” 49
whence p — — (xt + x2) = ± 7 .
Answer: p = ± 7 .
170. We have
1 .
xix 2 —-^ \ x t —x2— 1

Then, as in the previous problem, find xl -{-x.,-—± and tako into

account that x, + x 2 —
Answer: k dt 3 ~]/h.
171. We have
x'f + x| = 1 .7 5 ; x , x 2 — a'-; x, + x2 = 3a
There aro Ihree unknowns herc: x t, x 2, a. We have to íind a. Squaring the third
equation and sublracting twice the second one, we find x\ + xíj — 7a2. Compa-
ring this with the first equation, we find 7al = 1 .7 5 .
Answer: a — ;± — .
172. By Viète’s theorem
p + q = —p, and pq = q
This system has two Solutions: (1) p ~ 0, q = 0; (2) p = 1, q — —2. In the
first caso we have the equation x 2 — 0 , in the second, x 2 + x — 2 = 0 .
Answer: (1) p — 0; <7 = 0
(2) p — 1; q — —2
173. The roots of tho required equation are: and = -^x_.
Express y t + y2 in ternas of coefíicíents a, b, c. For this purposo transforin
120 Answers and Solutions

*f-f *j (*i-f-ga)2—2* 1*2 and substitute —•


— for (xf ¥ x2)
yi+y* = a
*i*2 *1*2
62 — 2 ac
and for x\x2. We get . Besides, we have yiy2= *i x2
*1
Consequently, the required equation is
* fc2_2ac
y + l= 0
^4nstaer: ocy2 — (ò2 — 2ac) y - f ac — 0.
174. This problem may be solved like the preceding one, but a shorter way
is preferable.
In the first case both roots of the required equation must be Iwice as large
as tbose of the given equation. Hence, we have to find the unknown quantity y
whose value is twice the value of the unknown quantity x satisfying the equation
ax2 -f- bx -f- c = 0. From the condition y — 2x we find x = -|~; substituting
it into the given equation, we get

° ( t ) " + 6 ( t ) + c=0
In the second case make the substitution x = — . We get

“ ( t ) 2 + 4 ( t ) + c==0-
Answer: (I) ay~ - f 2by -j- 4c — Ü
(2) cy- + 6y - f a = 0
175. First method (see solution of Problem 173). We have
y i+ í/2 — * f - f *2 - ( * í - f * 2 > 3 — 3 x ^ 2 ( X j - f X o)

r, , ......... b c ,. , b3 — 3abc
Substituting xj-F xo— — — and xjx.> = — , we fmd yi + l/a — ---------^3----- •
c3
Then, y xy 2 ~ (xjx2)3~ , and, using Viète’s theorem, set up the required
equation.
Second method (see solution of Problem 174). By hypothesis, y —x3, i.e.
x ~ f ry. Substituting it into the given equation, we get
a f y2+ b f y = —c
To rationalize this equation raise both_members to the third power and
transform the sum 3 (a f y-}-b f "y-f-3a f y~ (b f y)- to 3aby {a (}r y)~ + b f y],
By virtue of the found equation the bracketed expression is cquat to — c.
Answer: n:iy~ (/;3 — 3aòc) y - f c3 = 0.
176. Any equation of the nth degree having the roots xit x2, . • xn,
may be representod in the fonn
(x — xt) (x — X o ) . . . (x — xn) = 0

A biquadratic equation always lias two pairs of roots of the same absolute value
and of oppositc sigas. Putting x 3 — —x, and x4 = —x2, we may write the bi-
Chapter I I I . Algebraic Equalions 121

quadratic equation ín the íorm


(x — x,) (x — x2) (x + Xi) (x - f x2) = 0, i.e. (xJ — xj) (x2 — x|) = 0
or
X4 — (x| + x2)x2 + xjx; = 0.
But, by hypothesis,
*? + xS + ( - x ,)2 + ( - x2)í = 50
and
x 1x2 ( — *i) ( — * «)= 144
Hence,
xf-}~xt — 25 and xj\r3 = 144
Answer: x* — 25x2 - f 144 = 0.
177. If an algebraic equation (with real coefficients) has a complex root
a + bi, then the conjugate complex number a — bí will also be its root. Thus,
we know two conjugate roots of the given equation: 3 -+■ i ] / ü and 3 — i VÕ.
Botb oí them can be verified directly, but it is símpler to accomplish the follow-
ing transformation beforehand.
According to the remainder theorem, the left member oí the equation must
be divisible by the expressions z — (3 + i T/Ô) and x — J3 — i *J/Õ), and con­
sequently, by their product aswell, i.e. by |(x — 3)-—i l/(>] ((x ~ 3 ) 4- * V 6 ) =
— z2 — 6x 4~ 15. On dividing, we factor the left member into two factors:
4z4 — 24x3 -j- 57x2 -f- 18x — 45 — (x2 — üx -f- 15) (4x2 — 3), and the given
equation decomposcs into the following two:
(1) z2 — 6x + 15 = 0 and (2) 4x2 — 3 = 0

The íirst one lias two roots X| = 3-H~|/8 and x2 = 3 — i VÕ, -r3— and
t/3
x4 = ----- \L— being the roots of the second.

Answer. xi = 3 4-í'V / õ; x2 = 3 — í ~J/Õ; x3 ; x4=^---- 3^5-_


178. By hypothesis, x = 2 must satisíy the given equation. Therefore we
have 6-23 — 7*22 — 10*2 + m = 0, whencc m — 12. We get the equation
Gx3 — 7x2 — 16z -f- 12 = 0, one of the roots of which is equal to 2. By the
remainder theorem the left member must be divisible by (x — 2). Dividing, we
íind 6z2 + 5z — 6. Consequently, the equation may be represented in the form
(x — 2) (Óz2 4* — 6) = 0. In addition to lhe root .rt = 2, its roots are also
the roots z2 and x3 of the equation 6x2 4- 5* — 6 — 0.
2 3
Answer: m = 12; x2 — ; x3 = — .
179. Substituting x — 2 and x — 3 into the given equation (seo solution
of the preceding problem), we get
4m 4- n = 10 and 9m 4* n = —15
From this system we find rn — —5, n =-- 30 and obtain the equation 2x3 —
— 5x2 — 13x 4- 30 = 0, whose left member must be divisible by z — 2 and
x — 3, and, consequently, by the product (z — 2) (x — 3). The equation is then
122 Answers and Solutions

rewritten in the form (x — 2) (x — 3) {2x + 5) = 0. lts roots are: xj = 2;

5
Answer: m = —5; n — 30; x3 = — y .
180. The quadratic equation x- + px + q = 0 has equal roots when^the
radicand | y ^ — q ís equal to zero. In this case it raust be that (o ”] / o 2 — 3) —
— 4 = 0, i.e. a* — 3<z2 — 4 = 0. This biquadralic equation has two real roots
(a = 2 and a ~ —2) and two imaginary roots (a = t and a — —i). Confining
ourselves to thc real roots*, we get the following pair of equations: x* + 4x +
+ 4 = 0 and x2 — 4x + 4 = 0. Thc first equation has the roots zi — xz = —2,
the second one, x$ = x2 — 2.
Answer: at a -- 2 and a = —2.
180a. The roots oí the equation are
xlt2= m ± y /» 2 _ m 2 + l = m ± 1
By hypothesis, we have

Answer: — í <[ m < 3.


181. Isolate one of the radicais, for instance, the first one. We get
"]/í/ + 2 = 2 + Ví/ — 0.
Square both members. Collecting like terms and reducing by 4, we have
V y — 6 = 1, whence y = 7. A chcck shows that this root is valid.
Answer: y = 7.
Note 1. Here and henceforward we consider square roots and, in general,
roots of even imlices to be arithmetical roots. See Preliminaries for Chapter II
(pages 9U to 92). For roots of odd indices see the footnote to Problem 209.
Note 2. A chcck is carried out lo reveal extraneous roots (they may appear
as a result of squaring both members of the equation). There are no extraneous
roots in lhe given problem. But let us take the equation ~\/y + 2 + ~\/y — 6 =
= 2, which differs from the given one only in sign. Solving it in the same way,
we get \/y — ü = —1. Squaring the latter, we íind the same root y — 7. It is
invalid; the taken equation has no solution at all. Ilere it is not necessary to
carry out a chcck, sincc it is obvious that ~\/y-~ 0 cannot be equal to —1 (see
Note 1). But in other cases (see Problems 184 and 130) such a check is necessarv.
182. Soived in the same way as the preceding problem.
Answer: x -- 6.
183. Isolate thc first radical and square it. Aftcr simplification we obtain
x — 1 = 2 V -T ~ 1* Squaring once again, we find (x — í)2 — 4 (x — 1) = 0.
This equation may be divided by x — 1, laking into account that x — 1 is
one of its roots. We find then thc other root x — 5. We may also remove the
parentheses and solve the quadratic equation thus ohtained. A check shows
that both roots are valid.
.dnsuw: xj = í; x2 = 5.

* We assume that the coefficients of the given equation are real numbers.
Chapter I I I . Algebraic Equations 123

184. Proceeding in the same way as in the previous problem, we íind x -f


-|~ 22 = 7 y 3 x — 2, hence, x2 — 103x -{- 582 = 0. Tliís equation has two roots:
xt — 6 and x% ~ 97. The given equation is satisfied oniy hy the first root, the
second being an extraneous one (it satisfies the equation ~\/‘Ax — 2 — y x -t- 3 =
= 7, which difíers from the given one in the sign at the radicai).
Answer: 2 = 6.
185. Soived in the same way as the preceding problem. Out of the two roots .
= —1; xz = 3 the second is extraneous.
Note. x — 3 is a root oí the equation
~ Y2 x+ 3 = í
Answer: x = —1.
186. Answer: n — 34; x 2 — 2.
187. Answer: 2 = 4.
188. Square it. We get the equation
* y i í + 2 4 -x 2 —2 *= 0
which decomposes inlo the following two:
x = 0 and '}/xz -{-'M-~x — 2 — 0

The second one gives x — 5. Períorm a verification.


Answer: x\ = 0; — 5.
189. Reduce the given equation to the form

z 3-2

Square it and multiply by x2 (through which an extraneous root x = 0


may be introduced). We obtain the equation

Square it once again.


Answer: z ~ — .
4 ___________
190. Multiplying both members of the equation by y (x -j-2 ) (r4-3), we get
V íjr -â H z + ãj + y < j r - 4 ) ( í + 2) = 7
Proceeding in the same way as in Problem 181, we íind

Y f x — 4 )(i + 2 )= 4
Ilence, we get two roots xj=:6, x2~ —4. A check shows that x<> is invalid
7 - 7 -
sínce ít yíelds a wrong equahty - y y 2 = — — ]/ 2 .
Answer: x —(j.
191. Getting rid oí the denominator, we get
424 Answers and Solutions

This equatioc bas two roots x — 5 and x — —5. But at x = —5 the expression
Y x — 3 bas no real value (see Note 4 to Problem 181).
Answer: x — 5.
192. Reduce the left member of the equation to a common denominator

3 x -í\Y]EEà,^JL-
—x x
Hence
3 ( i - f 1) = 5 ~\/x (x-f-4)
Squaring it, we obtain
9 (x 4- l)3 - 25 (x + 1) x = 0
or
(x + 1) [9 (x - f 1) - 25*! = 0

Answer: xj = — 4; x2” ~ r -
193. Solved in the same way as the preceding problem.
Answer: Xj — 2; x2 = —1.6.
194. Square both members of thegiven equation. After the identity transfor-
mations we get V 2 8 —x = V 7 . In squaring this equation an extraneous
root may be introduced which satisfies an equation differing from the given
one only in the sign of the right member. The equation 1 /2 8 — x = y 7 has
the only root * = 21, which is not an extraneous one, since 2 l/7 -f-l/2 1 >
> V 2 1 /7 -1 /2 1 .
Answer: * = 21.
195. Rewrite the equation in the following way:

V x a - Y x — "V x — y * = — ^ ■
2V * + V i
Get rid of the denominator; this may result in that an extraneous root x = 0
will be introduced {since the denominator vanishes at x = 0). There cannot be
other extraneous roots, because * = 0 is the unique root of the equation
V x + "Yx — 0 (see solution of Problem !43).
After simplification we get the equation
2* — 2 l/x^ZT^ — ~Yx = 0
one of the roots of which is * = 0 . Rui this root is an extraneous one,
since at x~-0 the right member of the original equation loses its sense.
Factor out Y x -
y * {2 V i-2 y /^ í - i ) = o
Solving the equation 2 V * — 2 ~[/x— 1 — 1= 0 (see solution of Problem
181), we find — Carry out a check.
25
/l«su5<?r: x —
16
Chapter I I I . Algebraic Equations 125

196. First rationalize the denominator. To this end multipiy both the
numerator and denominator by V 2 1 x + ' } / 2 1 — x\ we get
{-y w V x A -y fr ^ f 27
2x ~ x
or, after simplifications, ______
27 -f- ~\/272— x2 __ 27
x x
wherefrom we íind x = ±27. Both roots are vaiid.
Ânswer: x = ±27.
197. Isolating the radical, square both members of the equation. We have
x2— 2«x = — x "j/x2 4- a2
One of the roots of this equation is x — 0. To find other roots divide both
members of the equation by x (it can be done, since now x ^ 0). Then square
both members once again. We get x = a.
When veriíying the result, one may arrive at the wrong conclusion that the
3
values x = 0 and x = -7- a ahvays satisfy the given equation. For a better
4
understanding of the essence of the error let us consider a numérica! example.
At a — —1 the given equation has the íorm
X= — í — V 1 — X ' f X2 ± 1

3 3
Neither x = 0, nor x = a — — T satisfy this equation {it has no solu-
-7-
4 4
tion). The same result is obtained for any oth£f negativo value of a.
And here is the mistake: the quantity V a 2 is considered to be equal to a,
whereas it is true only for a > 0. For a < 0 we have ~\/a2= —a: for instance,
V O — = *—( — 3).
The correct general formula (see Prclimioaries, Item 3 on page 90) is:
Y ã 2 = |a |
Using this formula, we find that at x — 0 (when the ieft mcmber of the equation
vanishes) the right member is equal to a — y a2 — a — \a\ . For a > 0 this
expression is also equal to zero, but for a < 0 it is equal to 2a. Consequently.
ií a > 0, the value x = 0 is a root of the equation; but if a < 0, then x — 0
3
is aot the root. The same refers to the value x — — a.
3
Answer: if a > 0, then xj = 0, x2 = -^a\ if a < 0, the equation has no
solution.
198. Writtcn wilhout powers having negative exponcnts the given equation
has the forra _________

/• H r M ‘
126 Answers and Solutions

First method. Reject the denominator: S j / ”l + The Ie£t


member is positive; hence, the right member is also positive. Squaring
yieids ^JL J2 = - A , whence ^ the value — is rejected, since

T > ° ) ‘
Second method. Rationalize the denominator

[/ <^ (T r -r r - r

The bracketed expression cannot be negative; therefore

/ ' + ( í - ) ™ T - / > + (v ) '-í+ V

Squaring yieids: 1 4. j 2 == , whence


4 '
Answer: x =
199. Soived in the same way as the preoeding problem. Using the second
method, we íind

n / l + a2 í2 -a x)2 = -i-

The expression ~\/í-{-azx~ — ox is always positive; thereforc

"|/1-j- a2x2— ax- i.e. V * + a“x~= ax +


M M
I Cl2—1 c2_j
Squaring it, we get x = or J — r t "'hich is the same.
2o |c | ’ v* 2a |c |
c2 — 1
Check. Substituting x = ^ , _ , , we find
2a|c|
4C2 4-(C2_1)2 <*a+<)3
1 -(- a2x2 —
4c2 4c2
Tafcing into account that e2-f-l is always positive, we find

Further computations show that the given equation is always satisfied.


Answer: x ----,^ 7~ 1 i.e. for c > 0 we have ------ f for c< 0 we
2 a jcj
have x -
Chapter I I I , Algehraic Equations 12?

200. Factor out the expression "[/x 4- c both in the numerator and denomi-
Dator of the left member, and reduce the fraction bv this expression*.
After performing these operations we get
Y x -f- c 4- V -T~ c _ 0 (x -fr)
y j+ l-y z -r ; £
Then rationalize the denominator. After simplifications we fjnd
8"^/x2— c2 = x-f-9c. Henre, z —- ~ or i = — ^— c.
A check shows that both values satisfy the equation when c > 0 and do
not satisfv it if c •: 0.
5 29
Answer: At c > 0 \ve have xi c and r*— — c: the equation
has no solution.
201. Transform the first radicand in the following way:
x 4 -3 _ 4 V ^ l = U - l ) - 4 y í 3 T + 4 = ( V J 3 T - 2 ) 2

Similarly, the second radicand isequal to ( y * —1—3)‘ . The given equation


takes the form
j y ír rr_ 2 | + | y j^ T ~ 3 | ^ i fa >
(see Prcliminaries to Chapter II, Item 3). The following tliree cases are pos-
sible: (i) y i ^ Ã > 3; (2) Y ~ \ < 2; (3) 2 < Y ~ i < 3.
In the first case the equation (A) takes the form:

y j r í — 2 + y í ^ I —3 = 1 , or Y ~ 1 = 3
This result does not agree with the condítion Y x — 1 > 3 .
In the second case the equation (A) takes the form:

_ (y x _ i_ 2 )-(y x -l~ 3 ) = l or y * ~ l = 2

This result does not agree with the condition Y x — 1 < 2 either. Consider,
finally, the third case, when the equation (A) takes the form:
( y — i^ 2 )~ {Y ~ l-Z )^ i (B)
This equalíty is an identity, hence, the equation (A) is satisfied by all x
for which
2 < y * _ i <3
Since Y x — 1 > 0, all the three mombors oí the inequality may be squa-
red, and we find
: x < 10,

* Reducing by Y x + c, we assume tbat x =£ —c. If the solution of the ob-


tained equation had yielded x = —c, this value would not have been a root of
the given equation. But, as we see below, we do not obtain such a root.
428 Answers and Solutions

i.e. the Solutions of the given equation are contained within the range bounded
by 5 and 40 (the values 5 and 40 included). All of them are the Solutions of the
given equation, since they suit the third case, wben the given equation (A)
becoraes identity (B).
Answer: 5 x -< 10.
202. Square both merobers of the equation, transpose all the terras to the
left and factor out ']/a - f x:
— x — 'l / x ) = 0
This equation decomposes into two. From the first one: ~\/a -f- * = 0, we find
x = —a. A check shows that with a ^ 0 this value satisfies the given equation.
Ií a < 0 , the equationloses its sense{since ~\/a — x becoraes an imaginary value).
The second equation is 4 ( V a + x + V a — '*) ^ "V x. ^ ^ is solved as in
Problems 183 to 187, we get (besides the extraneous root x — 0) x = .
A check will show that this root is also an extraneous one, which raeans that
the second equation has no Solutions at all. We may make sure of this fact more
easily, if the following raethod of solution is applied. Let us transform the second
equation to
________ 8x
— ~\/a— x
which is done by raultiplying and dividing ~\/a-rx-\-~\/a — x by the conju-
gate expression ~[/a-{-x — ~\/a—
Dividing ít by ~\/x (which is possible wíthout losing roots, since x = 0
is not a root), we get ’\/a"^x— ~)/a—-x —8 ~\/x. Subtraoting this equation
from the above obtained + ~\/a — x = ^ "y/x, we find

2 ~\/a—x ——— ~\/x

But this equality is impossible, since its left member is a positive number,
whereas the right one is negative. Suppose, negíecting this íact, we square both
64
merabers. This operation would yíeld an extraneous root x — a.
lí a is positive, then x = —a; if a is negative, the equation has
no solution.
203. Here we may successfully apply the method of transferring the irrationa-
lity to the denorainator (see the preceding problera).
Answer'. x — 0.
204. Answer. xt ~ a; x2 = —b.
203. Answer: x — (for a > 1).
4
For a < 1 the equation has no solution.
206. The given equation may be represented in the forra
Chapler I I I . Algebraic Equationx 120

or
3 3

2 3 a
Raising ít to the power we get o - f ;r — a x, whence x~.= ~ ------.
aTt- 1
Check:

(rt-r-r)”
a6 — 1
3

(g-í-j)' .

(a3- l f
A n s u ie r : 7 - - — ; ií a < í , the equation has no solution.
n3- l
207. Put \ h = z. Tlien
y 'í =. (i '.r)2=7e-
The equation takes tlie form
32+ s „rj,-_„
Hence, zt —3, ~ 4 . Since y x musl be a positivo nuinber, the second
root is an oxtraneous one.
Answeri x ~ á l .
i
208. Put (j•— 1)4= 2 . Tlien proceed as in the previous probleiu.
Ansuier: x = i l .
209*. Cubing hoth members, we get
y ro + 2 x + y i 5 - 2 x =7.
Here the isoiation of one oí the radicais is not essential.
Ansuier: x t - 3; x> =-: — — •
210. Cube both members of the equation by using the formula (a-^b)3=»
= a3-|-3ab(a-l-b) + b3. We obtain
i + 3 { z (2 x -3 ) ly x + í '2 x - 3 l + 2x—3 = 12 (,t - 1 )

• Here and henceforward we do not consider cube radicais, and, in general,


radicais of odd indices to be arithmeíical, assuming that the radicand ntay be
negative as weü (but obiigatoriiy real). The value of the radical is also considcr-
ed to be a real munber.
0 -0 1 3 3 8
130 Answers anã Solutions

By virtue of the given equation the bracketed expression may be replaced


by the expression >/12 (x —-1). We get
^ ( 2 i —3)-12(x—l) = 3(2r—i)
Cube it. Transposing all the terms to the left, we find
( x <— 1 ) í 1 2 z ( 2 x —3) - 27 (x — 1 )2 ]= 0

This equation is decoraposed into the íoUowing two:


x — 1 = 0 and Í2x {2x— 3 ) — 27 (x — 1)2= 0
Check the found roots.
Answer: xj = l ; x2 = 3.
211. Solved in the same way as the preceding problem.
Answer: x i ~ a ; x2= o ; x3= — ,
212. Put y"x== z\ then Y x2— ~2'‘ Substituting into the original equation,
we get 2z2- f z — 3 = 0, whence z i = í ; z2— — •

Answer: X i ~ l ; x2 — — -j~ .
o
213. Solved in the same way as the preceding problem,
Answer: zj — 64; z2= -----.
o _____
214. Put y ra -r x = z; then ~]/a-\-x — z3 and y rã~^fx = z2.
Answer: x i = - ~ a ; x2 = 1 — <*• _____

The second solution is rejected as a negative one (see Note 1 to Problem 181
en page 122). To determine x we get an equation
A n sw er: x — 7.
216. A n s w e r : x = = ± 5 .
217. Put y^x — z; then y/rx2 = z2 and x = z3. We obtain:

Reduce the first íraction by z3 — 1, and the second, by : + l . We get: z2 —


_ - _ 2 --- 0. Rut the reduction of the first íraction is lawful, provided z - —
— 1 ==£=<), and that of the second íraction, if z -f- 1 ¥* 0. Meanwhile, out of tho
two roots z t ~ 2 and z2 — —1 the second one gives z 4- 1 = 0. It does not suit,
since at z = — 1 we have x = — 1, and the left member of the given equation
loses its sense.
C h a p ie r 111. A lg eb ra ic liq u a tiu n s 131

Answer: x — 8.
218. Putting ~\/x — z, transform the equation to

Reduce the fraction by z -j~ 2 (see lhe ixplanalion to the preceding problem).
We gel z- — z — i> ----- 0, whence zs — 3, z2 — —2. The second root does not
suit, because, íirstlv, the expression -— ^ luses its senso and, secondly, z cannot
2 T 2.
be a negativo munber.
A n sw er: x — 9.
219. Here lhe introducticm oi an auxiliarv uiiknown. u<ed in the previous
problems, is oi no help. Represeni the equation in the íorrn
(•y^=r?V'‘ 4 -(y.c- i . ) 3
V a -s + y .r -b
and reduce it by ~\Za —x -f \/x~-b (the reduction is lawful, since this nuiu-
ber cannot beequal to zero). A í ter simpiiiications we obtain \ {a — x){x — b) = 0.
A n sw er: x i = a ; xa = b.
229. Represent the given cquation in the form

This equation decomposes into the foüowing two: the first equation i-
\ 2 — xv-0 , its root being j-j 2; the second one is \ 2 (2 —x) = 2 — y x
íafter gettíng n'd of the denominaíor). íts roofs are: x2= 0: .

Answer: X\
221. Answer: x = 81.
222. Isolating the radical and squaring both membcrs of the obtained equa­
tion, we get a íourth-dcgree equation. But in the present case it is possible
lo apply an artificiai raethod. Rewrite the equation in tbe form
y X2 _ + 5-f- 3x -f 5 = 12
Puttíng V * 2 — 3x-|-5 = 2, wo get z2-\-z— 12 = 0. Take oniy the positive
root 2— 3.
Answer: xi = 4; x2 — —1.
223. We may use the saine method as in the preceding problem. But it is
obvious that the equation has no solution. índeed, the quantity 3x2 + 5x -f- i
exceeds 3x2 -f- 5x — 8 at any x. Therefore

V 3z2-f- 5x -f-1 > V 3x2-j- 5x — 8


which means that the left member of the given equation is negative at any x
and, consequently, cannot be equai to unily.
Answer: The equation has no solution.
9'
132 Answers and Solutions

224. Denote one of the radicands by 3 ; it is most convenient to put yz -f-


-f- -f- 6 — 2. The equation takes the form
y i+ 2 + V 7 = 2 = y r z
Getting rid of the radicais, we find z2 = 4. Only the root 2 = 2 suits (at z —
= —2 the Uvo radicands are negative). Solve the equation y* + 4y - f 6 = 2.
Oheck on solution.
Answer: y = —2.
225. * It may be solved by using the substitution method (from the second
equation fiud y — G ~ x o t x — 6 — y and substitute it into the first equation),
but the following artificial method is somewhat more effective. The first equati­
on is trausforraed to (x — y)a = 4, wheuce x — y — 2 or x — y = —2. We
obtain two systems of equations:
(1) f x — y = 2 (2) / x y— 2
l x -j-(/ = 6 \ x y —G
A n sw er:(l) x t — 4, í/ j -—2
(2) x., --- 2, 1/2 4
220. Represent the given systern in the íorm
/ xy + ( x + y ) ^ í l
l xy (i -J-j ) = 30
Put for the sake of brevity xy — zi, x - y y —z^. Then we have
Í*i + S í= li
\ Z\Z-i — 30

By Yiète’s theorem and z2 are the roots of the quadratic equation z2—
— U z -f3 0 = 0. We find: 21= 6, 22— 0 or zj = 5, z2= 6. We get two systems:

( * + » = !? and ( x+ » = *
t xy = b t xp= b
Each of them may be solved by applying Viète’s theorem or the substitution
method.
Answer: ( l ) r ■
-« 5, / / — i (2) x = 1, // = 5
0 ) « 2 , w= 3 (4) x = 3, y — 2
227. Put y--. - then we have lhe following system
f 3+ == 7
l « = 12
Answer: (í) x- , 4 ,y^y:i
(2) J- ■ 4. ,, = _ V 3
(.'!) « - 3 , ;, = 2
- 3. „ ■- - 2
228. Put x2 zj and 2. We get the system
/ zi “f H—23
\ ^ , = -5 0
* Most prohlems of this chapter are successfully solved by using artificiai
inethmis. The main difficulty liere is to find out an adequate artificial method.
Chapter / / / . Algebraic Eyuations 133

Answer. (1) x = 5 , y —2
(2 ) * = - 5 L y= 2
(3 ) i = i l / 2 , _ »=-25
(4 ) x — — i 1 / 2 , jf= — 25

229. Put —aryssrzj; x2— y2= z 2* We obtain the System


f 1 ^ = — 180
\ 2l + 22= - l í
We find Z! = 9; z2= — 20 or ; 4 = 9. Now we have l\vo systems
(1) r xy =
~ 9 and
(2) í „ •í!' “ 2,
\ X2— y2— - 2 0 dna
l x 2 -j2 = 9
9
Solve the first System. From its first equation we find y — — — Substitute
it into the second one and find the biquadratic equation x4-4-
Its roots are:
*,,, = ± 1 / — 10 + 1/181 % ± 1 /3 .4 5 ^ + 1.88

* 3 ,4 = ± V - 1 0 - V 181 % + V ^ 2 3 l 5 * ± 4.841
Now we find
+ 9
Sfl.l y
- 1 0 + 1 /1 8 1 ‘ -86

~ ± ,m
Solve the second system using lhe sarae raetbod.
A nsw er: (i) * « 1 .8 6 , y « -4 .8 4
(2) x xx — 1.86, y « 4.84
(3) * « 4.84i, y « 1.86í
(4) x « —4.84i , y « — 1.86í
(5) x = 5, 4
(6) * = —5, y = —4
(7) x = 4i. y = —5i
(8) x = —4i, y —5í
230. Eliminate the constant terrns by multíplying the second equatiou by 7
and subtracting the result from the first one. We get
—32x2 - 2zy + 75jr = 0
Tbis is a homogeneous equation oí the second degree (i.e. an equatiou con-
taining only tenns oí the second degree). Dividing both members oí the equation
by x% (this may be done since x = 0 is not a root), wo transform it to —32 —
™2-|--f-75 = 0 , and, solving tbis quadratic equation, we find X - A or
434 Answers and Solutions

or — = — ~ . B y usiag this method we can íind the ratío ~ írom any honioge-
z 25 x
neous equation of the second degree.
N ow w e s o iv e two Systems:
(D 5 x 2 — 10y2 — 5 = 0 (2 ) 5x2 — 10y2— 5 = 0

and

jl -JL y_ _16
x 3 x 25

(b y th e s u b s t it u t io n m e th o d ).

Answer: (1 ) x — 3 , y = 2 (3 ) x =

( 2 > x = - 3 , y= — 2 (4 ) x

231. R e w r ite th e fii* s t e q u a tio n : x2— 2 x y -fy 2 = Zy. T hen we have

1 1
( x — y ) 2 = -^- xy. W rite the second equation in the form 2 (x — y) = -^- xy.
H en ce, ( x ~ y ) 2 — 2 (x — y )~ 0, w h e re fro m x —-y = 0 and x — y= 2. W e get two
Systems:
(1 ) f z -y 0 (2 ) f x — y = 2
< and
V xy~=Ü l xy = 8
Answer: (1 ) x = y = 0 ; (2 ) x = 4 , y = 2 ; (3 ) x = — 2 , y = — 4.
232. R e w r it e th e fir s t e q u a t io n in th e fo llo w in g w a y :

{z- -r 2xy -h y2} = 4 3 ■+• zy or (x - f y )2 — 43 = xy

F r o m th e s e c o n d e q u a t io n : x -f y = 4; s u b s t it u t in g , we get 16 — 13 = xy.
N o w w e s o iv e th e s y s te m

( xy = 3
l x+y=4
Aíwu?er: (1 ) x = 3 , y = l ; (2 ) x = i , y = 3.
233. S o lv e d in th e sam e w ay as th e p re ce d in g p r o b le m . W e get a new
sy stem

r x y =* 6
X x y = 1

Answer: (1 ) x = 3 ( 2 ) x = — 2
S - 2 y—— 3

234. P u t- ~ ~ z: th en — and th e fir s t e q u a tio n ta k es th e fo rm


y x 2
pj or 4 — 25z 12 = 0. Its roots are zt = ~ and z2 = - ~ . N ow we
Chapter I I I , Algebraic Equations 135

have tw o S y stem s:

(1 ) f <2 ) f
< y ' 3 < y 4
(, x 2— y2 = 7 [ x2 — y2 = 7

T h e y a re s o lv e d l>y s u b s t i t u t i n g th e v a lu e of x o b t a ín e d fr o m t h e fir s t equa-


tio n in to th e s e co m l on e.
Answer: ( J ) x -~ 4 , .V = 3
(2) x — —4, y = -3
{3) jc=r 3i, y = 4i
(4 ) x~ — 3i, j/ = — 4t

235. T h e system can be w r itte » in th e fo r m

( x my'n = cambn
(_ x"y™ = dambn
M u ltip ly th e s e e q u a t io n s a n d d iv id e one of th em 'b y th e o th er. W e get

(zy)m+n = c d ( i2mò 2 ,‘ and - ; w h en ce

j J, = K ' i r + V " + ' V ' + " and

M u lt ip ly in g th e se e q u a t io n s . w e fin d

x2 — e '" 2 - '1'

y2 m ay be ex p tessed in a s im ila r w ay p r o c e e d in g fr o m th e e q u a tio n

(■s-r
o r d e r in w h ic h
í~ . U

fo llo s v
d iffe r s fr o m

t h e le t t e r s
th e c o r r e s p o n d in g e q u a tio n fo r

c and d.
x o n ly in th e

Answer. x = d” 1-™ 1

y —c»4-m4 ~ni a »/ym+n


2 3 6 . I n t h e s e c o n d e q u a t i o n w e f a c t o r x 3 - j - y 3 i n t o (x - f - y) (-r2 — xy - f j r )
a n d d i v i d e t h e s e c o n d e q u a t io n b y t h e fir s t o n e . W e g e t z - f y = 5 . A d d in g
3 xy t o b o t h r u e ir ib e r s ol t h e f i r s t e q u a t i o n , w e o b t a i n (z - f y)2 = 7 - f 3 xy.
S u b s titu tin g 5 fo r (r -j-
y) i n t h i s e q u a t i o n , w e f i n d xy — 6 . W e s o l v e n o w t íie
sy stem
( * + y -5
l xy=z 6
Answer: (i) x = 3, y~ 2
(2 ) x 2, y = 3
2 3 7 . M u lt i p l y in g th e s e c o n d e q u a t io n b y 3 a n d a d d in g it t o th e fir s t o n e , w e
g e t ( x - f y f - - 1 . C o n f i n i n g o u r s e l v e s t o r e a l S o l u t i o n s , w e f i n d x - j - y ---- 1.
S u b s t i t u t i n g 1 for x + y i n t h e s e c o n d e q u a t i o n , w e h a v e xy — — 2 . We solve
136 Ânswers and Solutiotis

dow t h e s y s t e m

l x y = —2
Answer. (1 ) x = 2, y—— 1
(2) £-■= —1, 2
238. Solved in t h e s a m e way as the precedtng problem.
Answer: {1 ) x = 3, y —2
( 2) j/ = 3
239. Put The fir s t e q u a t io n ta k e s th e fo r m
y
1 .
H en ce, 5 and z-
T ’ 1*6*
•r-r-y =5 and

F rom th e e q u a t io n - = 5 we fin d y = — x. S o lv e t h is e q u a t io n to g e
—y 3
x- y 1
th c r w ith th e g iv e n e q u a t io n x y ~ h j. U se th e e q u a tio n th e
x — y ~~ 5
sam e w ay.
Ansiver: (i) 1 = 3, » = 2
(2) 2 = — 3 , y= —2
(3 ) x = 3 t, y = -2 i
(4) x = —3í, ij —21
240. E lim in a te th e u n k n o w n z fr o m th e g iv e n sy stem . T o th is e n d (1 ) s u b »
tra ct th e se co n d e q u a t io n fro m th e fir s t o n e m u lt ip lie d by c, and (2 ) s u b tr a c t
th e th ir d e q u a t io n f r o m th e s e c o n d o n e m u lt ip lie d a ls o b y c. A s a r e s u lt, v o get
th e ío U o w in g s y s te m

( (c— a ) x — (c — b)y = (c — d)
X a ( c ~ a ) x-—b (c— b) y —d (c—d)
w h e r e fr o m wc fin d x and y: z is fo u n d in a s im ila r w a y .
(r — d) {b — â) a d)(c — d)
(a — (b — d) {a —d)
Ansiver: r ■-
(c — a)(b — a )' (a — ò) (c — 6 )’ “ (b — c)(a — c)'
241. F ir s t e l i m i n a t e u: fo r th is p u rpose: (1 ) m u ltip ly lh e secon d e q u a tio n
b y 2 a n d a d d it t o t h e f ir s t o n e ; (2 ) m u l t i p l y t h e t h i r d e q u a t i o n b y ( — 2 ) a n d a d d
it t o t h e s e c o n d o n e ; (3 ) m u l t i p l y t h e t l ü r d e q u a t i o n b y ( — 3 ) a n d a d d t h e r e s u lt
t o t h e ío u r t h o n e . F i n a l l y w e o b t n i n t h e f o l l o w i n g s y s t e m

( 5.r — Ay í" 13z 30


J — 4x— \\y r 9 z = 1
[ — 5x — 13y 123 — 5
E lim in a te x fr o m th is s y s t e m , s u b t r a c t in g th e t h ir d e q u a t io n fr o m th e se co n d
one h e ío r e h a n d . W e get
(a i Ar — Ay - j-' 13= = 30

(r) 12: = õ '


C h a p ter 111. A lg eb ra ic E q u a tio n s 137

Add (a) to (c), multiply (b> by 5 and add ít to (c). We obtain the íoilowing;
j — 17y -{-253 = 41 J — í7 y-r 202 = 41
\ — 3y — 3 z = — 15 0i l y~j~z^5
Hence we find z — 3 and y — 2; x is íound from (b) and u, from the third o[
the given equations.
Answer: x — 1; y — 2; z — 3; u — 4.
242. Subtracting the íirst equation from the second mie. we get y - - 2z -- 1.
Hence, y — 1 — 2z. Substituting this value of y into the first equation. we find
x = 2 ~f 3. Substituting then the found vaiues of a and y into the third equation,
9
we get 3z2 - f 2 —- 2 = 0. Its roots are r, = ~ and 22 — — 1. Substituting now
the vaiues of z into the equations x = s -f- 3 and y — 1 — 2z, we find two valu-
es for each of tlie unknowns x and y.
. 11 1 2
Answer: (1) x = -^-, y = — ^
(2) x = 2, y - 3, 2 --1
243. Square the first equation. cube the second one, and square the third
one on having transposed the second term to the right member of the equation.
And so we get the following system.
/ 4 x 4 ~ y — 3z = — 3
< 5x~f- 2y - f ’ — 1-5
1 6x— y — 2= 0.
, 9 6 33
Answer. i = 2 = 2§ ■
244. Squaring the first equation and subtracting the second one from it, we
obtain xy -{- rz 4- //- 54. By virtue of the third equation the first two addends
may be rcplaced hy 2yz. We get 3yz -- 54, i.e.
yz = 18 (a)
Now the third equation may be wriíten in the forrn xy - f xz =■• 2-18, i.e.
x (y -f- z) = 36 (h)
Since the first equation lias the form
X - f (y -r 2) = 13 (c)
x and y 4- z may be found from (b) and (c). We get
r x=9 f x=4
i í,- f* = 4 °r l y+* = 9

To find y and 2 separateiy, make use of (a). Thus we obtain two Systems:
(1) S !>+ ■-= i and (2) f J +
i, yz = 18 f yz— iS
Note. When squaring the first equation there appears a danger of introducing
extraneous roots. But if Ihcy had appeared, they would have satisfied the equa­
tion x — j- y -f- r —13, which contradicts the equation (c).
138 Answers and Solutions

Answerl (1) x = 9, p = 2 4 -iV l4 ( z —2 —*V l4


(2) x = 9 , y = 2 - i Í / l 4 , 2 = 24" í 1 /Í 4
(3) x = 4, y — 6, s —3
(4) x = 4, y = 3, 2= 6
245. Represent thej third equation in the form
z2 — xz — ítz + xy = 2
Adding it to the second one, we get
zz -f* 2xy = 49 (a)
whence z2 — 49 — 2xy. Substítute this expression into the first equation. We
get (x + y)2 = 49, i.e. x 4- y — ± 7 . First put x 4* y — 7.
Represent the second equation in the form
xy 4- 2 (x -r y) ~ 47

and substitute into it the expression xy — —-9 z- , obtaíned frora (a), and the
value x + y — 7. We get z2 — 14z4-45 = 0, whence z\—b and z*>= 9. If z —5,
49 ——Z“ 49 ••2“
then xy = — ^— = 1 2 ; but if z = 9, then x y ~ — - — = — 16. And so we have
two systems
(1) = 7 (2) f x-\-y — l
and
l xy ss 12 \ xy — — 16
each having two Solutions. Thus, we obtain four Solutions:
(1) x = 3, y = 4, z= 5
(2) x = 4 , y = 3, z= 5
7-4- y H3 7 -V 1 1 3
(3) x = , , = — s— z=9
2
7— 1 /ÍÍ3 7+ynã
<4) x = z= 9
2
Now put x y = — 7 and find four more Solutions by using the samo method.
Answer:
(1) i = 3, í/ = 4, z=5
(2) 1 = 4 , y * 3, z= 5

<3>* 7+y®. 7 -1 /5 1 3
2= 9

7 + 1/513
<4)X 7- > / r a , y= 2= 9
2
(5) x = — 3, í/ = - 4 , 2= —5
.(S) x = - 4 . — 3, z = -5
Chapter I I I . Altiebraic Equatii 139

— 74-1/113 -7-VU3
(7)
2 2
- 7 4- V'Ti3
(8) x = - 7 — V Ü 3 -9
246. Subtract íirst the second equation and then the tbird from the first
one. We get
(a 3 — 6 3) 4 (a 2 — b“) x 4 (« — b ) // - - U (a)
(a 3 — c 3) 4 ( fl2 — c‘2 ) .r 4 (« — c) U — O (b)
Reduce the equation (a) by (e — b) and equation (b), by (a — r). We have
(a á 4 «b 4 b -) 4 (a 4 b) * 4 = 0 tc)
(a- 4 ac 4 c~) 4 ( a 4 c ) * 4 y — 0
Subtracting (d) from (c), we get
(ab—a c 4 b 2~ f 2) 4 ( b — c) x = 0
Hence,
ab— a c 4 b 2— c2 , , , , .
* = ----------0j4—---------
c ——( « 4 b 4 c)
The unknovvn y is found from (c) or (d). Now find z from any of the given
equations.
Answer: x — — ( a 4 b 4 c)
y=zab-\-àc-j-ca
z — —abc
1 1
247. Putting — ■— u and — ■............= r% we get the following system:
Ví 1
y y-
( 12u 4 = 5
\ 8u4i0a —6
Its roots are:
L
'' 4 ’ V*
V ry+T
Hence, x — 17; y = G.
Answer: x = l 7 ; y — G.
248. By virtue of the second equation the first one mny be rewritten in the
form 10— 2 l / x y = 4. Hence, xy = 9. We get the system
/ x4 jf —10
\ xy^ 9
^4naaer: (1) x = 9, y = l ; (2) x — í, y — 9.
4
— z. The first equation takes the form i — 2 4 0.

Hence, z = 1. i.e. 1 / - ^ - = 1. From this equation we find u = 2xand substi


r x4y
tute it into the second one.
140 Answers and Solutions

Answeri (1) x = 6 , y ~ 12; (2) x — —4.5, y — — 9. ______


250. The first equation is reduced to the form y x2+ y2 = 2 y i 7 , wheace
x2- f y 2— 136 (a)
Square the second equation to obtain + X2— y2 = 18— x, whence
if- = z&x — 324 (b)
Substitute this expression into (a). We get x- + 36x — 460 = 0, whence x — 10
and x = —46. Substituting into (b), we find y and obtain four pairs of Solutions:
(1) x — 10, y = 6; ( 3 ) x = — 46, y — Gl/Sbi
(2) x = 10, y = — 6; (4) x — --4 6 , y — — 6~}/55i
The third and fourth pairs of Solutions do not suit, since the expressions
"VT^Fy and ~\/x —Ty, where the radicais must mean arithmetical values of
the root (otherwise they are indefinite since the root has two values), make
no sense at complex values of x -f- y and .r — y. The first and second pairs of
Solutions should be checked.
Answer: (1) x = 10, y — 6; (2) .r — 10, y = —6.
251. The system makes sense only if a > Ü (see the preceding explanation).
Square the first equation:
y x2— y2 = Sa — x (a)
Substituting this expression into the second equation, we get
y x - + y2= ( l / 4 1 4-5) a — x (b)
Square the equations (a) and (b):
r/2~ — 64a2+16ax (a')
y2— C|/4[ a " ~ 2 ("V"41 4-5) ax (b'>
Eliminating y from (a') and (b')« we get
(130 + 10 1/41) = (26 + 2 y i i j a x
whence x~-5a. From (a') we find y — ±:4a and then perform a check.
.-lnstrer: (1) x — 5s, y —4a; (2) x-=5a, y — 4a.
252. Square the first equation: 2x2 — 2 \ x* — y1— y2. Substitute here the
value x4— y4-:144a4 from the second equation. We get
y2 =s 2x- — 24a2 (a)

wberefrom we find y4 and substitute it into the second of the given equations.
We get
x4 — 32a2x2 + 240o4 « 0
Hence. x ± y 5 Õ « and x - ^ ± : l / l 2 a . We find y from the equation (a). For
each of the values x - - ± y 2 0 a we have y ----- ± 4a, and for each of the values
x - 'sjz + '12 a we have y = 0 . A check sbows tbat out of the six pairs of obtained
root» some are extrancous for a > 0 , others, for a < 0 . Let us take, for instance,
the pair x — y ã õ u , y —4a. Substituting it into the first equation, we find
Chapter I I I . Algebraic Equations 141

*V^3t>a2— ~\Zkàí = £ia, i.e. 61a j — 2ia| = 4a. This equality is an identity for
a ;> 0 , but it does not hold true for a < 0.
Answer: For a > 0 the Solutions are:
(1) z = “|/20a, y — 4a; (2) x = — V720 <*> y = 4a
(3) x=~\/i2 a, y — 0; (4) x — — 1/12 a, y —0
For a < 0 tbe Solutions are:
(5) x = 1 /2 0 a, U ~ — 4a; (6) x = — l/2U a, y = — 4a. __
253. First method. From the secoiul equation we find x - f y = 14 — 'V-ry.
Squaring it, we get _
x- y2 -j~ 2.ry — 196 - f - xy — 28 f/xy
whenco _
■c2+ ! /2'T-xy--= 196— 28 \/xy
By virtue o! the first equation we have 8 4 — 196 — 28 y'xy , whence •xy--=4,
i.e. xy = 16. Substituting V x y = 4 into the second equation, we find x4-y = 10,
and then solve the system
( x ~ y — 10
\ x y -íü
Second method. Factorizc the left tnember of the first equation:
X2 4 - xy + y - — (x y )2 — ( l / x y ) “ V x y ) (x + y— V xy) = 84

Hence, by virtue of the second equation, we get


14 (x -r tj — \yx y ) —84
i.e. x-f-y — V ^ y — 9. From the System
r .r ~ -y — V Tu -=6
. I -r -í-S -r T xij — 14

\vc* may find x^-ij and yxij.


Answer: (1) x ^ 2 , y — 8
(2) 1 = 8, y —2
253a. From the first equation we find y ^ ^ , from the second y —
2-4-x -r • 1
..1 2“J~X
~ ----- _L_ • equating these expressions. we get — — - 7—— ; hence. we
14- x 1 H * 1+~mx
n 1 -4- x
have the following equation:
(1+ m ) x2-f (2 -f- m) x -f (2 — m) = 0
This equation has real roots, provided
(2~j~m)2— 4 (1 -j-m) (2 — m) >• 0
Simplifying the left meraber, we get the expression 5m'1— 4 > 0, whence
o
|m |> — . Under Uiis condition x has rea! vaíues, which raeans that y~~

ilso has real values.


1-
Answer: |m j
142 Answers and Solutions

C H A P T E R IV
LOGARITHMIC AND EXPONENTIAL EQUATIONS
Preliminaries
To solve equations containing logarithms to different bases (see, for example,
Problems 267, 268, 309 to 313) it may turn out to be convenient to reduce alí
the logarithms to ono base. So, let us introduce some relevant formulas supplied
with necessary explanations.
1. The formula

toft* “ Í s È 5 (8)
enables us to change the roles of the logarithmic base and the number.
Exarnple.
1 1
logs 2 =
log2 8 3

Explanation. According to the definition of the logarithin, log2 8 is the expo-


nent indicating the power to which it is necessary to raise the base 2 to obtain
the number 8. Thus, svmbolicallv, log2 8 = 3 is equivalent to 23 = 8. But
l
the last equality may be written in a different way: f / 8 = 2, i.e. 83 = 2. Hence,

1h«-'2 “ ¥•
1
In general, the equality ax = b may be written as bx = a. The former equa--
lity means that loga b = ar, the latter, that logb a — ~ -, wherefrom the formu­
la (a) is derived.
2. The formula (a) is a particular case of the general formula

loga.V= logb Ar
logb a (b)

which expresses the following important fact: if \ve know the logaritlims of
various numbers to the base 6, we can find the logarithms of the same numbers
to the base a; to this effect it is sufficient to divide the former by logj, a fi.e.
hy the logarithra of the new base to the old one). Instead of dividing logb N
by log5 a we may [by virtue of (a)J multiply it by loga b:
logn Ar “ loga 6 *logj, IV (c)
The number by whicir logarithms in one system are multiplied to give lo-
garilhms in a second system is called the modulus of the second system with res-
pect to the first. That is to say, the factor loga b is the modulus of the system
of iogarithms to the base a with respect to the system of logarithms to the base b.
Example. Having a tahle of common logarithms, we can compile a table
of logarithms to the base 2. To this end it is sufficient to perform division by
C h a p ter / I. i .a ^ u n t h rn tc and h .r p o u e n ti< il l-.q iu ilio ii^ 143

log 2 — 0.3010 or irmltiplication by log2 10 = 03ÕIÕ = ^hus,


. * log 3 0.4771
l0g23 = W = O Õ ! Õ = , -5to
Explanation. By the definit-ion of the logarithm we have 2,0ír-3—3. Take
tlie logarithms of this equality to the base 10. We «et log23-log 2----iog 3.
whence log->3 — -r— t ■ Just in the «ame wav we ohtain the formula ib) from
b” log 2
the identity o,og° A = A r by taking the logarithms to the base b.
In order not to ronfuse the notations, it is advwible to use the follmving
method íor a check: write the fraction — instoad of the oxpre-ooon log. M(of
a *
course, these expressions are not equal to each other); treat the expressions
logft a, loga N and so or» in a similar way. Then in.-tead of the formulas (a),
(b), (c) we get other formulas, which are alsu true. Tinis. instead of (e) we get

IL -!l £
a a b
254. First method
2 (iiog 9 - I 0S 2 ) . „ loe?
1 = 10-10 ^2 / = 10 - 10lo s9 ~ 2 ,o " 2 = 10 .| 0 '■

log t q 9
By definition. 10 — therefore x ~ 10— = 22.5.
Answer: = 22.5.
Second method. Taking the logarithms, we have

log x = log IO -- iog 9 — log 2 1 log 100

10.Q
log x 2= log 10 - f log 0 — 2 log 2 = log - g j -
Answer: x == 22.5.
255. As in Problem 254 (second method), we have

l o g 2 = | i - ~ i - l o g 4 j log 100

, . 1 . . , 10 10
log x — 1 — - log 4 == log — ; x= — -
2 y 4 y i
Answer: x = 5 .
25C. Proceeding in the sarae way as in the previous problems, we} have

log x = 1 ( 2 + 4- log f « ) 1»B 10 = l°g 16 = log (■10 ^ 16);

*=10 {/TB
Answer: 2 = 20,
144 Answers anà Solutions

257. First methoá


_2-2 iogj2 1 _ 72 f 1 49 1_25
+ 5io g 5 4 ~ 7 lo g 7 4 ' r 4 ~ 4+ 4 ~ 2

(cf. solution of Problem 254 by tbe first method).


Second method
Let us denote 0 = 49* l0í?7“ , and s = 5 l°°5 ‘ ; tlien
X—IJ--Z
Taking tbe logaritkms, \ve íind that log7 y = {1 — iog7 2) log7 40, or
7 49
log7 y = (log7 7 — log7 2)2 = 2 log7 ~ = log7

49 1 25
■whence similarly, \ve find that . Hence, x = .

Answer: x = ^ •
258. We have log-, log3 log2* ~ lo?4 1, whence log3 log2 * = 1; log2r = 3.
stnsíx-er: x = 8.
259. Like in the preceding problem, we have
1+ logi, |14- logc (1 + logp x)| = 1
Íogíi 11-r it>gc(1~r iogp x)] -0
then
1— lugc (1 -T- Í0R,, x) ^ i; loec (I -x logp x) = 0
1 f logp x — 1; logpjrr.: 0: x= l
A n sw er: x --- 1.
2G0. The expression in braces must be a positive nnmber since a nogative
number has no (real) logarithm to base 4. Therefore, having rewritten the given
equation in the form
t
2 log3 |1 ~ log2 (1 3 log2 x)\ ---- 42* = V'4
we shouhi take only the positive value of V 4. i.e. 2. Applying similar transfor-
matíons for the second time, we tlien obtain
log3 [1 logn (1 -p 3 logo *)) ” 1» 1 *r log2 (t - f 3 log2 .r) = 3,
log2 (i -f- 3 log;. x) ~ 2
hence, i H- 3 log2 x 4. log2 .r = 1.
.•Insüvr: .r — 2.
2(»i. Rcpresent the given equation in lhe íorm log2 (x -4 14) (x -f- 3) = G,
or (x --r 14) (.v 2) — 26 -- 64, whence x- -r 16x — 36 -- 0, -••= 2, x2 ~=
.: is, The second root does not suit. since the left member ccmtains the expres-
sions íog.> (x -f 14) and íog, (x -[- 2), which have no reai value at a negative .r.
Anstrcr: x 2.
262. Rcpresent the given equation in the forrn
log(1 (y (y -r 5)-0.023 --- 0
Chapter I V , Logaritkmic and Exponential Equations 145

hence,
y (y + 5) *0.02 = i or y2 - f 5y — 50 — 0
\ve get two roots: yt = 5, y2 — —10. The second root does not suit (see solution
of the preceding problem).
Answer: y = 5.
263. We have
log (35 — x3) — 3 log (5 — x) or log (35 — x3) — log (5 — x)3
hence,
35 — x3 — (5 — x)a or x- — 5x -f- 6 — 0
Answer: x t = 2, x2 —■ 3.
264. Transforming the bracketed expression, we get
h &) (q2 ____ b(a — b)2
ò- 2 a(a-\-b)
Then the given equation takes the form

i + logx = - i - l o g i ^ ^ - — y l o g 6 + -i-log[a (a + 6 )(o—ò)l

Applying the theorem on the logarithm of a product (and of a fraction) to the


rignt member, we obtain
1 -f- log x — log (a — b) — log b
Substituting log 10 for unity, rewrite the equation in the form

log 10 + log x = log ( a -b )~ - log b or log (lOx) = log - — -


b
hence, 10x= a- - — .
b
A
Answer: Ü~~b .
x = -■^
265. The given equation may be represented in the form

log ( * ~ y f e " ) =log V '


wherefrom, taking antilogarithms, we find

/ H- í / i+ a
y r ^
or
= V l_ a
y t-a
whence
i

1
Answer: x
y t-a
10—01338
146 Answers and Solutions

266. The given equation may be written in a different way:

y log* 5 + log* 5 - f log* * — 2.25 = ( y log* 5 ) 2

since logx x — i , after simplifications we get


log| 5 — 6 logx 5 + 5 = 0
Solving the quadratic equation (in the unknown logx 5), we find two roots:
logx 5 = 5 and logx 5 = 1 .
Answer: xj = |/"5; x 2 = 5.
267. First method. Putting log10 x ~ z, we have x = 162, hence,
logt x = 2 log4 16 = 2z and log2 x = z log2 16 = 4z
The given equation takes the form z + 2z + Az = 7, i.e. z = 1.
Second method. Reduce all the logarithms to the base 2 by the formula (b)
(page 142}. We find Íog4 xx=-j^|2 * = ; similarly, Iogig x = . We
1 l
get the equation -^-log2x + y l o g 2x + log2x = 7, whence log2x = 4 .
Answer: x — 16.
268. Solved in the same way as the preceding problem.
Answer: x = a.
269. Rewrite the given equation in the form

whence 3x — 7 = 3 — 7x.
(irMfr*
Answer: x — í.
270. Represent the given equation in the form
7.3X+1 _ zx+i ™ 5X+2„ 5x+3
Factoring out 3X and 5V, we have
5
3* (7-3 — 34) = 5* (52— 53) or ( — )*

whence x = — 1.
Answer: x — — 1.
271. Rewrite the given equation in the form

2~3.2ix-s: 2 2 24X-9—. o-
1 2_3x
Hence,
4x— 9 = ^ -z

Answer: x = 6.
272. The given equation may be wntten as
2- x2<22x + 2_ 2~ â or 2~xZ*~x 'i'2 — 2~6
Consequently, —x2 + 2x + 2 - - —-6.
Chapter I V . Logarithmic and Exponential Equations 147

Answer: xj = 4; x2— — 2.
273. Represent the given equation in the forrn
5 (« + 5> 7 { x + 17)
2 *“ 7 =2-2-2 x-3
whence
5 (.!-{-5) o , 7(x-f-17)
x -7 - ^ x —3
Answer: x = iO .
274. Since ~ ™ s the given equation may be rewritten ia
the followiag way:
/ 2 \ix í l \ {i~x)3 ~ 1
\ 3 ; \3 / 3
Hence,
2-c + 3 (f — *) = 1
Answer: x = 2.
275. Represent the given equation in the form

A 2 Vx / Vx - 1 . = 22,
equating the exponents, we find
3 1 /7 + 3 = 2, or 2x— 5 1 / * — 3 = 0
V * (V x -i)
Let us denote V * by z; then sve have
2z2— 5z — 3 = 0 , whence zj = 3, z2~ —

But the second root does not suit, because the quantity z (which represents
the arithmetical valuc of the radical V x ) must be positive. Thus, we have
i*=~\/x\ hence x = 9.
Answer: x — 9.
276. The given equation may be representcd in the form
» s+ 3
t+
2 “ 1 * =2 Y x -i
Hence,
, 'V x + 3 4
21/7 1 /7 -1
therefrom 3x — 8 l / * — 3 = 0. Putting we have 3z2 —8z— 3 = 0* z, = 3*
1 1 1
«2— — y ; the second root z2~ — y does not suit (see solution of Problem 275).
Consequently, x = 9.
Answer: x — 9.
10*
148 Answers and Solutions

277. The given equation may be written as


3 , i___JL
a x 2 -i 2x-2 4 =a0

Consequently,
3 i 1
X 2-Í + 2x — 2 4

After simpiifications we get x2 —2 x —1 5 = 0 .


Ansioer: x * — 5 ; x 2 = — 3 .
278. Usíng the formula (a) (page 142), we obtain

log* * + 21o* «• !« * ,« ) '

1
1 + 2 log* a +1 -T
2T— logx a
Solving for Iogx a, we get
7 ± V 4 9 -4 8 7± 1
Jog.r a =
8 8
4
Answer: xi = a; x2= a .
279. By formula (b) (page 142) we find
log4 2 1
log,.2- log4x 2 iog4 x

Then the given equation takes the form log4 (x -f- 12) == 2 log4 x, whence x +
-j- 12 = x2. We take only the positive root x = 4; at negative x the expression
logx 2 has no real value.
Answer: x 4.
280. Wriíe the given equation in the form:
(log.v 5 -j- 2) logg x — 1

Since logx we get the equation

( 1_ L 7 + 2 ) !of?l, , 1

Solving it for log5x, we find

(logr,x ) | ^ ~ and (log5x)2= — 1

1
.4nsu>er: xj “ “\/5; x2
Chapter I V . Logariihmic and Exponential Equaíions 149

281. The left member of the equation is the sum oí x + 1 terms of a geomet-
ric progression, and therefore \ve have (for a ^ i)

' , " '7 =-(!+<■) (l+ ^ H i + o*) (i + oe)


or
l — a*+t = ( I — a)(l + a) (1 + a-) (1 + a4) ( i + a®)
or
1 — a*+i = 1 — ais
whence a*+1 — a16; x + 1 — 16; x — 15. At a — 1 the general formula for
the sura of the terms of a geometric progression is not appíicable. In this case
the left member of the given equation is the sum of x + 1 addends, each being
equal to 1, and so, the equation takes the form x -f- 1 = 16, hence, x — 15.
Answer: x = 15.
282. Rewrite the given equation in the form
52-H+tí-K. .4-2.r_.55G
whence
2 + 4 + 6 + ... + 2x=56orl + 2 4 - 3 + .. . + *=28
The left member of the equation is the sum of the lerms of an arithmetic pro­
gression. Therefore we get the equation

Í * ± í > í = 28

whence x t — 7, z2 — -—8. The second root does not suit, since the number x
must be a positive integer.
I Answer: x — 7.
283. Rewrite the given equation in the form
2ü*2-4 —'1 7 -2^2~4 + 1 = 0
Denoting 2* by z, we get
z" — 17z + 10 = 0; Z\ — 16; z2 = 1
whence x x = 4; x2 — 0.
Answer: x t = 4; x2 = 0.
284. As in the preceding problem, putting 4* — z, we have 2z2 — 17z +
+ 8 = 0.
Answer: x2= — — .

i
285. Putting 9 * ~ z , we ohtain the equation
3z2 — lOz + 3 = 0
Answer: x t = 2; x2 — —2.
286. Taking the logarithms of the given equation (to the base 10), we obtain

,‘ og X&1. log I - log a: +1 or log2jr + 31ogx —4=0


whence log x t = 1; log x2 = —4.
150 Answers and Solutions

Answer: x t = 40; x2 — 0.0001.


287. Transform the given equation so that either of its members represents
the logarithm oí a certain exprcssion. For this purpose substitute log 10 for
unity in the left meraber of the equation. Now the given equation may be writ-
ten in the form

log
« -* 2 ^ -1
= lo g
V2Vx-z-f- 2
10 22
Since the logarithms are equal, the numbers are also equal

4“ *2 — —1 * V~2l'*“
-
2-{-2

After simplifications we get the equation


Yx
2 V x ~S<2 2 - 2 4 = 0
r« Yx
Since 2 1 x (2 2 )2, then putting 2 2 —z, wc have the equation z2— 5z-
— 24 = 0, whose roots are zj==8 and z: -3 . Taking zj = 8, we get the
Yx
equation 2 =8, whencc
i/i =3, i.e. x = 30.

The second root z — — 3 leads to the equation 2 2 — — 3 which has no


Solutions since no power of the positive number 2 can be a negative number.
Answer: x — 36.
288. Find successively (see solution of the preceding problem):

2 1 og (| + Iog (5 ,,x + l ) = log ( —P=- + 5 )

l = g [ ( | ) 2(5’ - + l ) ] = l o g ( Í ( l ± | ^ )

hence,
S ( l - f 5* x)
{5 V x+ i ) = (A)
25

After siraplification we get

521 * - 1 2 4 - 5 1‘ * — 125 = 0

whence 5 ] * = 125, or 5* x = — 1. The second equation has no solution; the


first oue gives ~\/x = Z\ x = 9.
The equation (A) may be solved in a different way. It may be reduced
bv 5* x -f-l^= 0, and then we get ^ = ~ — rr-; hence, 5 1^ = 125 and x = 9.
Chapter I V . Logarithmic and Exponential Equations 151

Answer: x = 9.
289. Represent the given equatioo in the form
gíog *^_ 5 log x - l _ glOg x+i x -1

Factoring out 5logx and 3IogJfl . we have


5|0S * (1 + 5~') = 3105 * (3 + 3~J)
or
5Iogx 25 / 5 \ i o g * / 5 \2
3log x “ 9 '* \ 3 / " V3 )
whence log x — 2.
Answer: x — 100.
290. Taking the logarithms to the base 10, we get

2 iog4 x — 1.5 log2 x — 4 “

This biquadratic equation (in the unknown log x) has two real roots: log x = 1
and log x = —1; hence, xi = 10, x2 = 0.1.
Answer: x4 — 10; x2 = 0.1.
291. Taking antilogarithms, we obtain
=) or 2-J- i0* = ( i j ) 2\

i.c. 2x2-40 x = 2 " ü' 24; hence, i » — 4 0 i + 144 = 0.


Answer: xj — 36; x2= 4 .
292. By the definition of the logarithra, the given equation is equivalem
to 9— 2 * = 2 3-*, or 9 — 2* = , whence 22x~~ 9-2*4-8 —0. Solving this equa­
tion (quadratíc equation in the unknown 2*), we find
xj = 3; x2= 0
Answer: xj= :3 ; x2 = 0.
293. As in Problem 288, we get
2 (4*-2_5_ 9) - 10 (2X~2 -j- 1)
Noting thal
2=c-2 = 2x -2~2 = -i-2 * , and 4*-2 = 4*.4-2=-L .4*
4 16
we obtain the equation
22* — 2 0.2*4-64=0
whence, like in the preceding problem, we find *1 = 4 ; x2= 2.
Answer: x j —4; Xo = 2.
294. It is convenient to trauspose the last term to the right. Then, as in
1-f- ~ - i+J. _L
Problem 288, we get 4-3 = 27. Noting that 3 2x = 3-32x, we get
the equation
_L i
12.32x= 3 x - f 27
152 Answers and Solutions

JL i -L
Putting 32x = z, we have 3x = (32x)2, and so, we get the equation z2-í2 z ~ j-
-4-27 = 0, with zj = 9; z2= 3 being its roots.
Answer: x\ — — \ —
295. Taking antilogarithms (cf. solution of Problem 288), we have
3 „ 2 4~ y r4x+'í
100 4y ‘x+0.25
The equation may be represented in the form
j ( 3v n r r 16 \ 2
100 \ oyTic+7 /
Getting rid of the denominator, we obtain
gV4x-f 1_ 1g __200, i.e. g V ^ + i ^ 63t
whence x — 2.
Answer: x — 2.
296. Represent the given equation in the form
4 log 2 - f 2 log (x — 3) = log (7x - f 1) - f log {x — 6) + log 3
whence, taking antilogarithms, we find
2* (x — 3)2 = 3 (7x - f 1) (x - 6)
The roots of this quadratic equation are x\ ~ 9; x2 = —3.6. The second root
does not suit, since it yields x — 3 = —6.6, which means that the expression
log (x — 3) has no real value {the same can be stated about the expressions
log (7x 4- 1) and log (x — 6)].
Answer: x = 9.
297. Represent the right member in the form
—log5 (0.2 — 0.2 -S*-3) = — Iog5 0.2 — log5 (1 — 5*~3)
Represent the addend (x — 3) in the form log6 5*-3. Transposing the terms, we
get the equation
log5 120 + log5 5*~3 - f log5 0.2 = 2 log5 (1 — 5*~3) — log5 (1 — õ*"3)
or
120.0.2*5*“* = 1 — 5*“*
Answer: x — 1.
298. The given equations may be represented in the form
( 26x+3~ 2 4í''^^
\ ^+2
l 5•+*-»_ 5 2
Equating the exponents, we get the following systera
í 6x — 4y = 1
\ x—3y=0
4
Answer: x= 3 1 .
; y = -r?
14 14
Chapter I V . Logarithmic and Exponential Equations 153

299. Taking antilogarithms <>f lhe íirst equation, \ve gel the following
system oí equations:

300. In algebra, consideration is usually given only to the logarithms of


positive numbers to positive bases, otherwise a number may have no (real)
iogarithm. Therefore, we consider the known quantities a and b (logarithmic
bases) to be positive; the unknown quantities x , y (“numbers” ) must also be posi­
tive.
Taking antilogarithms, we find
x
xy ... a-, — — b*

The system has two Solutions:


(1 ) x ^ a b 2, y =

(2) x — — «ò2, y——

But the second soiution does not suit, since at positive values oí a and b, it
yields negative values of x and y.
Answer: x —ab2; y = - p - .
301. Taking antilogarithms» we get the system
x2-hy2 13 x ++yy .
10 * —y *
from the second equation we find |/ = -|-x; substituting it (into the first
equation, we have two Solutions:
(1) x, = 9, iji — 1: ( 2) x2= —9, y2= — 7
The second soiution does not fit, since it yields x -[- y < 0 and x — y < 0
(see soiution of Problem 300).
Answer: x — 9; y — 7.
302. Taking antilogarithms, we have

This system has two Solutions:


1- f 1 /5
2

-1 -1 /5
2
154 Answers and Solutions

The first solution yields


x — ys=sxy— — 2 ~ f-V 5 > 0

The second one gives us


x — y —x y — — 2 — l/S < 0

The second solution does not suit, since the base of the logaritbms xy must
be positive (see Problem 300).

A— n , = ,-2 = ^ L .
303. Taking antilogarithms, we get the following System
a2
1+ - xy — b*
~y
or
/ x + y = a2
X xxj~b*
This system has two Solutions:
a2-h y a* — 464 a2— \/a4— 4ò4
(1) xl 2 ’ Jí 2
a2— V o 4~ 4ô4 a2-f- \/a4—464
(2)
y t~ 2
Considering the given quantities a and b to be positive (as the logarithmic bases),
we must distinguish between the following two cases:
(i) a4 < 464, i.e. a < ~\/26, and (2) a4 ^ 464. i.e. a ^ "1/26. In the first
case the system bas no solution, since x and y are imaginary numbers. In the
second case x and y are not only real, but also positive, since both the sura
x -r y — a~ and the product xy — 64 are positive.
a2 4- "1/ a4~ 464 a2 — a* — 464
Answer: x —----!—----- ——: u—---------- -—-
2 * 2
304. Taking antilogarithms of tbe first equation, we obtain the system
/ 4x y = 9 a 2
\ x-)-y = 5a
Both Solutions are suitable.
. . . . a 9 9 a
Answer: (í) = — (2) x2 = — a, —.
305. Since in the second equation the unknowns x and y are preceded by the
logarithm symbols, both of them are positive (if a solution exists). As far as the
quantity a is concerned, it may be negative as well (since the logarithm Symbol
is followed by the positive number a2). But in this case it should be written
log (a2) 2 log j a| instead of the equality log (a2) = 2 log a. For the sake
of brevity let us denote log x -- A'; log y ~ Y ; log j a |= A . Taking loga-
rithms of the first equation in the given system, we get the following system
a* + r = 2.4, a 2 - f y 2 = 10a *
Chapter I V . Logarithmic and Exponential Equations 455

Squaring the first equation and subtractiDg tbe second frora it, \ve get X Y —
~ —ZA2. Thus, we have an equivalent system
X + Y = 2A, X V ^ —3A2.
Consequently, X and Y are the roots of the equation z2— 2Az — 3A2~ 0. Hence,
4
one solution is X — 3A, Y = — A, i.e. x = |a|3, í/ _____. Xhe other soiution

is 2 = p j , J/ = |a|3.
A check shows that both Solutions suit.
Answer: x t = \a[S, ; x 2^ j j j , y2= U|3.
306. From the second equation we have y —x = ( V 2 )'1= 4. Hence, y = x - f 4.
Substituting it into the first equation, we get 3x -2x+4 = 576 or Gx -24= 576.
Answer: x = 2; y —6.
307. The given system may be written as
xy=a

{
Since x and y must l>e positive, we get the following System
( xy — a

7 ^
Answer: x — ~]/a y lr, y - VjL
\n>
308. The given system may be written in the form
. ■ 4 . 3
Ioga i-r -2 *°6a y = Y ’ 4-
2 log&x4-logby = 4
whence
3 3

x}/ 'y = a2, y r7y — 62


3 3 3 3

Multiplying them, we have x2y2 —a262 or xy — ab. Divide the last equation
by each of the previousones.
. a2 62
Answer: x = - T- ;y = .
b a
-

309. The solution is similar to the preceding one.


Answer: x~aV~&> H— —57=? •
6y b
310. Using the formula (a) (page 142), write the first equation as

í0g)jU whence log0 u ^ l ,


156 Answers and - Solutions

i.e. u — v. Substituting it into the second equation, we have u* -f- u — 12 — 0.


Only positive solution is acceptable (see solution ol Problem 300).
Answer: u = v — 3.
311. Put y^a = u; then

a =u?-
and

logj, y W lo g u U ^ -í-
yf a 6
similarly.

Consequently, the second equation may be written as


* , y
2+ * “ ys
We get the foliowing system:
”1*xy -f~yz ~ d)
, 2a
( 2)
I+ !,“ y r
which is equivalent to the given one. Squaring the equation (2), we get
4a2
+ + (2a)

Subtracting (i) írom {2a), we find

And so we obtain the system


1 , 2a
(2)
\x + ^ y f
i fl2
(3)
1 " " T
which has only one solution

I= !,= V í
Note. When squaring an equation there is a probability of obtaining extra-
neous Solutions. It is just the case here: equation (2a) has extraneous solu-
tions as compared with equation (2). For instance, the vaiues x - y -
Chapler I V . Logarithmic and Exponential Equations 157

satisfy equation (2a), faut do not satisfy equation (2). In other words, thc
equation x 1-j-2xy -f- V2~ —r — is not equivalent to the equation x -s- y — ■'^a - •
3 2 V3
it is equivalent to two equations: x-f-j/----— — and x-'r y ~ — ■ —— . Nevar­
ia 3 1/3
theless, the given system is equivalent to the systern of equations í - f a =
2a a2 2a
= — r=-, xy = - ^ - , since the latter contains the equation x-\-y —— —-----the
V 3 á 1 /3

íact which eliminates the possibility of equality x-\~y for i 0


1 /3
2a
^at a ~ 0 the equations x-H / =
v a and c" ) '
But had we taken instead of the system (2)-(3) the system (l)-(3), i.e. the
system

{
x2 + xy + y2 ~ a z (1)
fl2

I,J=~ «)
it wouid not have been equivalent to the given one. Indeed, in addition to
the solution = r , it wouid have had another soiuiion x — y = — .
Therefore, when squaring one or several equations, it is aivvays necessary
either to clear out the problem of equivalency, or to chec-k by substitution
the Answer:
suitability
x — yof= the Solutions.
V »'
312. Taking into consideration the formula (b) on page 142, we have
1
Iog4 x = ~ 'lo g 2x\ therefore, the first equation is redueed to the form x - y - .
Now we solve the system
r - y1
\ x2— 5y2+ 4 ™0
Answer. x\ — 4, r/t == 2; x2 —- 1, y2 -- 1.
313. With the aid of the formula (b) on pago 142 we rnay write the given
system as
1 1
iog2 x 4- — log2 y + y log3 2= 2
1
log3í/ log3 2 + 4 log3 x = 2

log.i z + y log4 iog4y =


Taking antilogarithms, we find
x V yz —4
y = (a)
z Vxiy = 10
158 Answers and Solutions

Multiplying ali the equations (a), we obtain


(xyz)z = 4*9‘ 16
whence
xyz — 24 (b)
(we take the arithmetical value of the root, since by virtue of the given equa­
tions x, y, z must be positive). Square each of the equations (a) and then divide
them by (b).

314. From the first equation we find x-\-y — 2x~v 3 2 , from the second,
x-f-y = 3-2*-v, consequently,
x -y
3— = 3 or - ^ - = 1
Hence, x-\~ y ~ Z - 2 z ~ 12.
Answer: x ~ l \ y = 5.
315. The given System is reduced to the foliowing one:

s-i-y 7 x2— y2 = 40
10 x
Dividing the second equation by the first one, we get x — y=s.t - y . Solving
the system
70 4x
a?4-y —■ and x — y =
x T
we have x t = 7, y, = 3; x2 — —7, y2 = —3. The roots x2, y2 do not satisfy
the second equation of the given system, since the numbers x2 - f y2 and x2 —
— y2 are negative.
Answer: x = 7; y — 3.
316. Represent the given system in the form
2x . , 3u Z l+ ix = !í
— 5-?-—-
o í/ _ 2 * 3" = 3 »
wherefrom we get
^ = 5+ - ^ , '- 1 + 2 - 2*
y * y y
Put — then from the first equation we have 2í2—-5í — 3 = 0; íj = 3,
1 X T 1
-—— i i-e. 3 or — = —— . Hence we find the expressions x — Zy

and x — — y; suhstituting them into the second equation, we find

Answer: Xj — —2.
Chapter V. Progressions 159

317. The given system is reduced to the foilosving one:


í — — — —5
< tf *
L £-j~ y — 2
From the first equation (see solution of Problem 316) \ve íind ~ = 3 or — = -
1
y y
3 1
The second equation gives zj — — , y i = — ; xs --~ —2, w2= 4. The roots z2,
tf2 rejected.
Answer: z = — , y — \p.
318. The given system is reduced as follows:

l2 y íj= 3 + V í
Putting ~\/'x — u; Vtf —*>» we get uy = 4 — u; 2ue = 3-fi\
A nsw er: X f= = 4 , t f t ~ 1; z2 —í , y2~ 9 -
319. Rewrite the given system in the form
ay — zP, bx — yq
Since x and y must bc positive (as the logarithmíc bases), the original system
is only solvable ai positive values of a and b. From the first equation we find
p
y = — ; substituting it into the second equation, we get xpq — aqbx. Rejecting
a
the root x — 0 (since x must be positive), we obtain the equation xP9~l — aqb.
If pq — i , then this equation either has no Solutions (for aqb =£ 1), or ís an
identity (at aqb = 1). In the latter case the original system has an iníinite
xp
number of Solutions (z is an arbitrary number, and y — — ; or y is an arbitra-

ry number, and x ■ — ). If pq 1, then we get the following solution:


pq- t/
V '6 ,

- y aqb. =n ~ V b ‘‘a (pq 1).

CHAPTER V
PROGRESSIONS
Arithmetic Progression
320. By hypothesis, at == 5, d — 4. Substituting these values into (3) and
performing some transformation», we get the equation
2n- + 3n — 10 877 =* 0
Its roots are: n, = 73 and n2 — —74.5, only the former being suítabie.
Answer: 73 terms.
160 Answers and Solutions

321. By hypothesis,
<*i + («i + d) + (ai + 2d) + (ai 4- 3d) = 26
üj (bj + d) (^t + 2<í} (fli -j- 3d) = 880

The first equation gives 4at + 6d = 26, whence ai — — . Substitutíng


it into the second equation and simplifying the parenthesized expressions, we
get
13 — 3d 13- d 13 + d 13+ 3d oon
2 ' 2‘ 2 * 2 ~ 880
Getting rid of tke denorainator and multiplying the numerators (it is most
convenient to muitiply the first numerator by tke fourth and the second by the
third), we íind:
9d4 _ iegOd2 + 14 481 = 0
Denoting the roots of this biquadratic equation by d', d", dm, d " " , we find

d ' ~ 3; d"— — 3; — and d " " = — ^ -— 2.; from the equation a* —

= — — we find tke corresponding values of the first term:

13 — V l6 0 9 _ 13- I - y 1609
= 11;
2
Answer: the problera has four Solutions:
(1) 2; 5; 8; 11; 14; . . .
(2) l í ; 8; 5; 2; - 1 ; . . .
I3-VTÕ Ü 9 39 — V Í6Õ » 394-VTÕÕ9 1 3+ V 1609 .
{à) _ ; ------ g------- , ------- g------- ;
1 3 + V 1609 3 9 + y 1609 39— y 1609 43 — 1/I6Õ9
(4)
2 6 tí ’ 2 ’
322. Denoting ap and aq by ax and d, we get (by hypothesis) the foilowing
system:
( a, + d (p — i) = q
\ at + d (q ~ —p
Ifencc, d — — 1 and ax ~ p + q — 1. By the formula (1) we find:
an = (P <1 ~ i) — (« — 1) — p + q — n
A n sw er: an —- p + q — n.
323. Natural two-digit numbers fonn an arithmetic progression with com-
mon difference d — 1; its first term «i • Jl), and the last one an — 99. By the
formula (1) we find the nmnber of tenns n — 90. The formula (2) gives:
(10+ 99)-90
— 4905
An swer: 49U5.
Chapter V. Progressions 161

324. Let us denote the odd numbers by n, (n -f- 2), (n -f- 4), (n -f (»). Tben
the even numbers contained between them wiü be ( tt -f I), (« -f 3). (« 4- 5).
By hypothesis,
na -{- (« 2)2 -f* (n -t* 4)2 -f- (n *r 6)’ = (n -r O2 + (>< ~r 3)* -r (n-f5)2~f-48
or
«* + ((« + 2)2 — (« +1)2) -f [(n + 4)2— (n + 3)2] -{- ((o -j-Ol2 - (« 4 -5t-| - 48 - 0
vrhence
n- + (2n - f 3) + (2n - f 7) + (2n - f 11) - 48 ^ 0
or
n- - f f'm — 27 =- I1
Hence, n ~ 3 or n — —0.
Ansu-er: (1) 3; 5; 7; 9 or (2) - 9 : —7; — —3.
325. The tcrms a2\ o*; «u. • • «20 eonslitute an arithmetic progression
with cornmon difference 2d and tbe number oí tcrms 10. Usine tlie formula (3)
(in which oj should be replaced by a2 and d, by 2d), wi* fim!
f:>„, + 2rf-9)10 _

Substituting a.2 — (ti -j- d, \vc bave


*1 - f 10d ^ 25 (a)
In the same way, proceeding from tbe progression at\ a:i: n0: . . a,?< we find
*n the same way
a, 4 9d - 22 (b)
From (a) and (b) we may find a, and d, and tben all tbe terms of the progres­
sion. But since it is required to find tbe médium terms oniy, i.e. <7,0 = at -4- 9rf,
and a,, = a, 4- 10d, tben from (a) and (b) we imniediately bave: a,n = 22 and
«n = 25.
Answer: the medi um terms are equal to 22 and 25. respectively.
326. Put bi — {a x)'1. bz (n~ + -r2). b?. — (a — j )2. Wo find bz —
— 6, = 63 — b2 = — 'lar. Heriee, the terms bf, bz, b3 constitute an arithmetic
progression with cornmon difference d = —2ax. By the formula (3) we bave
\2<a + r)2— 2ar (n — 1)]*
S; — [a2- f (3 — n) az-*r x-\ n

Answer: Sn— [a2-f(3 — n) axA- r 2) n,


327. By the formula (3) we bave

2at d (n: —-1)

2«, -f-rf-----------
-------- (/».•{—1) n3
.7.

t i—01338
Í6 2 A n sw ers and S o lu tio n s

or

A . = 0, + - í ( n, - i )

£ - .,+ 4 < * -1 1

- ^ - . , + 4 (» , - i )
M u líip ly in g th e o b t a in e d e q u a litie s by ( « 2 — « 3) , {n3 — n * ) and ( « j — n 2)> res-
p e c tiv e ly , a n d a d d in g th e p rod u cts, w e íin d :

S1 S■> S-'
— («2— « 3) + ~ ( " 3 — « l J + V 1- ( ' * 1 — n z) =
«i «2 «3

= « i ({«2 — « 3 ) -h (« 3 — » i ) - f ( « i — "2)1 +

-f" T Í(,;í — (n-2— « 3) — («2— 1) ( »3— («3— 1) — «•>)]

The b ra ck eted e x p r e s s io n s are id e n t ic a ü y equ al to z e r o , c o n s e q u e n tly ,

5 . S-»
-J -(n 2— n3) -f (n 3 — ( « ! — n 2) = i 0
n l «2 '*3

w h ic h c o m p le te s th e p r o o f.
328. By h y p o th e s is , 5 10 — õ S 5. E x p r e s s in g S s and Su> by th e fo r m u la (3 )
and t a k i n g in t u a c c o u u t t h a t at ~ 1 , w e íin d

(2 -f-ítfH Ü r (2 -í r/) 3
•> J* 2
w henee d - — 3. _

329. By iiy p o th e s ís ,

S n= 3 „s or 1 3 -+ ■ !,(;-D l- = 3 ,,2

Since n ^ Q, then, reducing this equation by n, we get 2ax - f dn — d — tí« or


2<ii — d = (6 — d) n (a)
B y h y p o ih o s is . th e e q u a lit y (a ) m u s t. b e s a t i s f i c d at. a n y n, b u t th e le ft m e m b e r
o í (a ) c o n t a i n s n o n , w h e r e a s t h e r i g h l m e m b e r v a r i e s w i t h n , p r o v i d e d t h e f a c t o r
(3 _ d i s n o n - z e r o . O n l y i n t h e e a .s e 0 — d -- 0 th e r ig h tm e m b e
of n (is equal to z e r o ), th e r e ío r e we m ust have d — C .T h e n fr
d
2«, — d -- O, ue. di = y =r' «>•
/iíirnvr: 3; 9; to; 21; , . .
3 3 0 . T h e n m n b o r s w h i c h a r e n o t e x a c t l y d i v i s t b l e b y 4, y i e l d i n g t h e r e m a i n -
d e r 1, h a v e t h e fo r m 4 k -j- 1 (A — a n y n a t u r a l n u m b e r ). T h e y fo r n i a n a r it h m e t l c
p r o ç r e s s í o n w i t h c o m m o n d i f f e r e n c e 4. T h e f i r s t t w o - d i g i t n u m b e r o f í b i s f o r m
is Í 3 (it is o b t a i n e d a t k 3 ) ; t h o la s t o n e is 9 7 . B y t h e fo r m u la ( l ) , w h e r e
. i ; ; , a ri .... 9 7 a n d d 4 , w e íin d » = 2 2 . T h e fo r m u la (3 ) y ie ld s th e r e q u ir -
ed Mim.
C h a p ter V. P r o g re ss io n s 163

To determine for what values of k the numbers of the form kk 4- 1 will be


two-digit, we may also make use of the following system of inequalities:
/4 fe -f 1 > 1 0
14A-+1 < 1 0 0
1 3
wberefrom we find 2 < k < 24 —- ; heoce, k may have values equai to 3; 4i
5; . . 24, the total number of whíck n ~ (24 — 3) -f- 1 = 22.
Answer: 1210.

Geometric Progression
331. The geometric mean of two (positive) numbers a and b is a positive
number x determined froin the proporlion a : x = x : b. To insert three geomet-
ric means between the numbers 1 and 25G mcans to find the three numbers
“ 3» u* which satisfy the conditions:
1 : u2 — u2 : us ~ us '■ui — ui '•256
Hence, the numbers = 1, uz, u3, u4 and u5 — 256 forin a geometric
progression. By the formula of the «th term of the progression, 256 = l*q*.
This equation has one positive root 9 — >/ 256 — 4 (—4; - f 4i; —4i are discard-
ed, sínce they are D O t suitable). Now, by the same formula, we find: u2 — 4;
“ a = 16; u4 — 64.
/Insiüer; 4; 16; 64.
332. By hypothesis, u, -f- u3 — 52 and u§ — 100. or u2 — ± 1 0. By the
property of the geometric progression, «ju3 = u| = 100; hence, «, and u3 are
the roots of the equation u2 — 52u ± 100 — 0, whence u, — 50 and u’3 — 2.
or u" — 2 and u3 = 50.
Answer: (1) 50; 10; 2, or (2) 50; —10; 2, or the saine numbers following
in the reverse order.
333. By hypothesis: (1) u3 — u, — 9 and (2) m5 — u3 — 36. Using tho for­
mula un = uign~1, rcwrite these equations in the forra: (1) utq3 — u, = *j;
(2) mq* — Uí?2 — 36. Dividing (2) by (1), we get qz — 4. hence, q = ± 2 ; from
(4) we find: u, = 3.
Answer: (í) 3; 6; 12; 24; 48; . . .
(2) 3; —6; 12; —24; 48; . . .
334. By hypothesis, uj-j-u4= 27 and u2u$ — 72; but since —z=züi
lt u3
or u2u3= uju4, we have a system of two equations:
(1) «i -■{- Ui — 27 and (2) uju4 — 72
whence u, = 3 and u4 — 24, or ut = 24 and u4 — 3. From the formula u4 =
— Ujç3 we find q — 2, or q — 1/ 2.
Answer: 3; 6; 12; 24, or in the reverse order: 24; 12; 6; 3.
335. By hypothesis: (1) ut -f- u4 — 35 and (2) u2 -f- u3 = 30. As in Pro-
blem 333, for determining q we get the following equation
1+ ? 3 35
?(1 + ?) 30
11*
164 Answers and Solutions

or, when reduced,


1— ç-f-92 7
? " 0
We find:
(1) q = 3/2; — 8; (2) q — 2/3; uj = 27.
We get two progressions:
(1) 8; 12; 18; 27; 40.5; . . .
(2) 27; 18; 12; 8; 5-|-; . . .
whose íirst four terras are equal, but follow in the reverse order.
Answer: 8; 12; 18; 27.
336. In the second of the given suras we replace each term by the preceding
one multiplied by q (by the definitíon of the gcometric progression). we obtaín
mq -b uzq - f Uzq - f UiQ + us>Q= 62
or
q (uj ~b un -f- U3 -f- -b «s) “ 62
By hypothesis, the parentbesized expression is equal to 31; hence, 9 = 2. Using
the formula Sn = ** , we have 31 = , wherefrom ux = i
Answer: 1; 2; 4; 8; . . .
337. By hypothesis, we have:
(1) U2 + 1/3 + u4 + u5 = 19.5; (2) üj — «2 -b U3 ~b «4 = 13
The problem is similar to the preceding one.
Answer: U\ — 1.6 and U5 = 8.1.
338. The terms n4 and uti are equidistant from the beginning and the end
of lhe given sequence; therefore u4u6 — u ,«9. Since. by hypothesis, utM9 = 2304,
then UiUfí — 2304; besides, also by hypothesis, u4 -b uc =■- 120. From these
two equations we find i/4 — 24; u'6 = 96, and = 96; = 24. Let us take
the íirst solution. By the formula un = uIç n' 1 we have:
(1) 24 = n,?3; (2} 96 = utç5
Dividing (2) by (1). we find q- = 4, whence q = 2 or q ■■= —2. In the first case
the equation (1) yields ií4 — 3, in the second, itj = —3. In the íirst case the
nine terms of the progression are:
3; 6; 12; 24; 48; 96; 192; 384; 768;
in the second:
—3; 6; —12; 24; —48; 96; — 192; 384; —768
Taking the solution u\ 96; = 24, we find the same two sequences of the
terms, but in the reverse order.
Answer: (1) — 3; q ~ 2
(2) «, - - 3 ; 9 = -2
(3) n, = 76<S: q - 1/2
(4) uj = —768; 9 = - 1 /2
339. By hypothesis: (1) ui - f u2 •+• «3 — 126 and (2) utu2u3 13 824.
Since u2 is the geornetric mean of ut and u3, we have » io 3 — nf; therefore instead
of (2) we may write «| = 13 824. whence uz ~ V 16 824. In the given case, by
factoring 13 824, it is easy to find that u2 = 24. Substituting it into (1) and (2),
C h a p ie r V. P r o i ’ r es sio n s 165

we get the following system of equations: u, -f- « 3 ~ 102; ntu3 = 576. Solving
it, we find: = Ü; u:i — 96, and ut — 96; u3 = 6. Thus, we get two progres-
sions: 6; 24; 96, and 96; 24; 6, which differ only in the order of the tenns.
Answer: 6; 24; 96.
340. It follows (by hypothcsis) that the sum of the even terms is twice that
of the odd terms, i.e.
U2+ t*j + » e + •••-f-»2n _ o
wlH-w3H‘w5+ •••+ u2n-l
Replacing the terms n2; ij,ú . . uin by the expressions u2 — utq\ u4 =
— uzq\ . . u2jl ~ ü2n_,f/t we find q 2.
Answer: q = 2.

Infinitelv Decreasing Geometrie Progression


341. To prove the fact that the given mimbers constitute a decreasing geo-
metric progression we ha ve to check whethcr the ratios — and — are eqnal and
Uj u2
whether each of them is less than unity. We have:

02 1 __________ ____ I
______ . V2 - H _ _t _
d)
«I 2 - V 2 ' V 2 - i ‘ V 2 ( 1 /2 ~ 1 ) ‘ V 2 - I 2 + 1/2
u3 1 1 _ 2- 1 / 2 (2 ~ l/2 )(2 + V 2 ) _ 1
(2) 2 •>_ 1/2
«2 2 2 ( 2 + 1 /2 ) 2 + 1/2
1
Since u2 _ . < 1 , the given numbers form a decreasing
2+ V 2
geometrie progression. By the formula of its sum we find

._ V 2+ 1 , ( V | ± 0 í 2 ± j / ? > = 4+ 3 V
( 1/ 2- 0 0 /2+ 1)
( V 2 - 1 ) (l
2+V^
<4nsu>er: S —4 + 3 1 /2 .
342. As in the preceding prohlem, we find that the bracketed expressiou

is equal to — ™ . The who!e expressiou lhen takes the form

- ( > 0 /3 + 1 )
....
Answer: — G ( l / 3 + í ) .
343. By hypotnesis,
32
« ! = 4 and 03— « 5—gj

By the formula Un — uj?” - 1 we get from the second equality


. 32
166 Answers and Solutions

Taking into account that uj = 4, weget the biquadratic equation 81 g4 — 81ç2 4-


2V2 i
4 - 8 = 0, whose roots are: çJj2 = ± —~ ~ and ?3,* = ± y * The negative
roots are not suitable, since, by hypothesis, ali the terms are positive; botb
positive roots are suitable, since they are less than unity. And so, \ve get two
infinitely decreasiug progressions.
Answer: S ' = 12 (3 -f- 2 l / 2 ) and S° = 6.
344. By hypothesis,
ui + ut = 54 and u2 -f- u3 — 36
Using the formula un = Ujç71-1, \ve get a system oí two equations:
/ iií 4~n1g3“ 54 rwi í l + ? )(l — ?4 -g2 )==54 (1)
\ \ (1 4 “ Ç} — 36 (2)
Dividing {{) by (2), we obtain the equation
1 — g-f-g2 _ 3
q ~~2
whence — 2 and qz — 1/ 2. The suitable root is g2 = -~ < 1, We find frora.
(2) Ui = 48.
Answer: S = 96.
345. First method. By hypothesis,
(1) ut 4" «a 4- 4" • • • = 36
(2) u2 4* u* 4* «« 4* • •• — 12
The terms of the first and the second sums also constitute iufinitely decreasing
progressions with one and the same common ratio ç3. The first term of the first
progression is equal to uít the first term of the second “progression, to u2, i.e.
to Uiq. Expressing the sums of the first and second progressions by the formula
for the sum of an infinitely decreasing progression (where ínstead of q we take
5 a, and ínstead of ut in the second case we take uj?), we obtain (1) ^ = 36

and (2) ™ 12. Dividing (2) by (1), we get q — 4/3, and from the first
equation we find ut = 32.
Second method. Since u2 — uí?» = u3q and so on, then ínstead of u2 4-
4* u4 4* ü6 4- • • • = 12 we get utq 4- u3q -j- u5g 4- • ■ . = 12, or
{1(“ 1+ “ 3 4- u* 4- . . .) — 12 (i)
Dividing the expression ut 4- n3 4- ub ~r * . . — 36 by (1), we find q — *1-.
On the other hand, the sum of all the terms of the progression is 12 4- 36 = 48.
By the formula for the sum of an infinitely decreasing progression we have

whence u j« = 3 2 .
, QO 3 2 32
Answer: 32; — ; ; ...
C h a p ter F. P r o g re ss io n s 167

346. By hypothesis,
ui H~ u2 H- w3 ~t~ • • • — 56; « i -{- -r ^3 “T • • • ~ 448
The addends of the second sum also forni an infinitely decreasing geometric
progression with the first term uj and common ratio q2. Expressing the sums of
these progression», \ve get

= 56, —- í i — = 44 8
1— 7 1 — 72
or
uj = 56 (1 — 7) d)
K? = « 8 (1 - 92) (2)
Dividing (2) b y (i), we find
UI = 8 (1 + <;) (3)
Eliminating ut from the equations (1) and (3), \ve get
8 (1 -r q) = 5 6 ( l - 7)
whence q — — . Frora (1) we finei </t = 14.

Answer: «i = 14, q — .
347. Solved in the same way as the preceding problem. For determining u,
and q \ve get the foílowing system of equations:

1-7 U)
108
(2,
Í3
Eliminating Ui from these equations, \ve get the equation 3q- — IO7 -j- 3 ~
= 0. Otit oí its two roots oniy one, nameiy q — y , is suitabie (the other, q =
- 3, being more than unity). From the equation (1) we find ux = 2.
Answer: 2; ; —; ...
348. Solved in the same way as ProbJems 346 and 347. To determine ut
and q we obtain the foílowing system of equations:
d)
< » i __ 1 , ür
l 4- q ~ 8 1 ( 2)

Equation (2) is equivalent to the equation ut — 8 (4 - f 7). Eliminating u,


from the system utq — ü, ut — 8 (i -f- 7), we gel the equation 47a -f- 47 —
— 3 = 0 with two roots: 7, = —3 2, qz = 1/2, but the first root is not suitabie,
siace its absolute value is more than unity. From (4) we find u, ~ 42.
Answer: 12; 6; 3; . . .
168 Answers and Solutions

Arithmetic and Geometric Progressions


349. By hypothesis, we find:
d = 16 — 14 = 2; a, = 14 — d = 12
aj - f a2 - f a3 = 12 -t- 14 - f 16* = 42
Consequent.lv, in the required geometric progression (1) q — 2 and (2) u, -f
4- «g/ 4- ut<72 = 42, whence = 6.
Answer: 6; 12; 24; . . .
350. The first three terms of the geometric progression are: 3; 3g; 3q-. 13y
hypothesis, a, = 3; a2 = 3q ~r 6; since 0.3 — a2 = « 2 — au we have a3 = 2az —
— a, — 6q 4- 9. By hypothesis, th is third term is equai to the tbird terra of
the geometric progression, i.e. 3q2. Thus, we get the equation &q - f 9 — 3ç-,
who.se roots are: q — 3 and q — —1. In the first case the geometric progression
is 3; 9; 27; . . . and the arithmetic one is 3; 15; 27; . . . . In the second case
we get two sequenccs of numbers: 3; —3; 3; —3; . . . and 3; 3; 3; . . . which
may be corjsidered rcspectively as a geometric progression with common ratio
q = —1, as well as an arithmetic progression with common difference d — 0.
Answer: (1) 3; 15; 27; . . .; 3; 9; 27; . . .
(2) 3; 3; 3; . . .; 3; - 3 ; 3; —3; . . .
351. The problem is similar to the precedíng one. By hypothesis, aj = «1 —
— 5; consequently, i.3 — 5 u5 ~ 5q4. Then. also by hypothesis. o4 — u3 —
— 5r/2; <j16 —- «5 = 5fy4. Henee: (1) 5q- -- 5 -f- 3d, (2) 5q* — 5 4- 15d. Elimi-
nnting d, we get the equation q4 — 5qz 4- 4 = 0, whence q2 — 4 or q- — 1.
Since a4 — 5q2, the fourth terra of the arithmetic progression is equal to 20 in
the first case. and to 5 in the second.
Note. In either of these cases we obtain two different geometric progressions,
the arithmetic progressions being the samo. Namelv, in the first case we have
the fotlowing geometric progressions: 5; 10; 20; . . . and 5; — 10; 20; . . the
arithmetic progression ^with common difference — '2Í-2ZÉ == 5 j being 5;
10; 15; 20; . . . . In the second case we get 5; 5; 5; . . . and 5; —5; 5; —5; . . .
the corresponding arithmetic progression containing eqtiaí terras: 5; 5; 5; . . . .
Answer". 20 or 5.
352. By hypothesis. aK— ut; a2 — uiq; n7 — utq2, wherefrom we find:
0 ) d ~~ a2 — a, - «i (q — 1) and (2) 6d = a7 — a, — ut (q2 — 1). Eliminating
d, we gel iíi (gz — 1) — 0»j {q — 1). Since tt, 0, then q2 — 1 = 6 (q — 1),
whence q — 5 or q — i. From the condition «j q 4- «iç2 = 93 we find
«i = 3 and «1 — 31, rcspectively.
Answer". (1) 3; 15; 75, (2) 31; 31; 31.
353. By the formula (2) on page 32 we find a- — 729; consequently, in the
geometric progression we have; «, — at — 1; « 7 — a1 — 729. lt is required
to find the médium term which is the fourth one both from the beginníng and
from the end. Hence, the first term uu the required médium term «4 and the
last one « 7 form a continued proportion: : « 4 = «4 : Hence, « j = « i« 7
and «| = 729.
Answer: «4 = ±27.
354. By hypothesis, a, 4- a2 4" as 15. Since a2 — = a3 — a2. then
2a2 — a, 4- o 3, and from the given condition we have 2a2 4- az — 15. Hence,
rt2 = 5. Then at = 5 — d; a2 — 5; 03 = 5 4* d and. by hypothesis, «j — a, 4"
1 = 6 — d; u2 r- az 4- 4 = 9; u3 = a3 -r- 19 = 24 4* d. Since «5 = ui«3.
C h a p ter V I. C o m h in a to r ic s and X c w to i iíu to n n til T h eorem fO<>

we have
cj2 (G „ d) (24 -r d)
whence we find d — 3, a, — 2 or d — —21, a, = 26.
Answer: (1) 2; 5; 8. (2) 26; 5; — 16.
355. By hypothesís, at — nj 1; a2 — n2 -4- G;Ta3 = u3 -f- 3; hence <z} -f-
- f a2 + a3 — («, - f u2 + u3) + (1 + 0 4 - 3), or, by virtue of the^condition
ÜJ 4- u2 -j- u3 = 26, we get
a4 -f- a2 4* a3 ~ 26 - f 10 — 36
Then proceed in the same way as in the preceding prublem.
Answer: 2; 6; 18 or 18; 6; 2.
356. Suppose the required numbers are: n,; j/jiy; then. by hypothesis,
the numbers u u tq and (wji/2 — 64) constituiu an aríthinetic progression and,
consequentiy,
Ui<] — uf — (wj<73 — 04) — « ií ü)
Furthermore, by hypotliesi:s, the numbers ui'. (" ií — 8): («,o! -- Cí) form a
geometric progression and. consequentiy,
(UiQ — 8) : ut — {u,ç2 — 04) : (« ií — 8) (2*
After simplificatioris the systeiu oí equations (1) and (2) lakes the íorm
u t ( f ] - — 2r/ - j - 1) — 04, »1 (q — 4) - - 4

whence q = 13 and u, — ™ or q ^ 5 and u{ - 4.

Answer: (1) ^ . (2) 4; 20; 100.


357. Suppose the required numbers arc: ut. u2 and u3. lí these numbers are
lhe terms oí a geomelric progression, then
"1 - (D
and ií they are the terms of an arithmetic progrcssion, then
2u2 uj 4* us (2)
Eliminating uz from (1) and (2), we íind (u, - f u3}- = 4uju3 or (*,, — u3)3 » 0.
whence u, — u3, and from (2) we íind u2 — zzj. Hence, ü! — u2 t/3.
Ansu»fr: possible, if the three numbers are equal.

C HA P T E H Vi
COMBINATORICS AND NEWTON S BINOMIAL THEOREM
Notatíon:
™ total number of perrnutalions of m eícments takon n at a time
Pn — total number of pcrmutatious of n elemetns
6’^ = total number of corabinations oí rn elements taken n at a time
T h + i~ C ^ a ^ x m -h |8 (fr -f-l)t h lerrn oí the oxpansion of Ibe binomial (x -f-ap*
470 Answers and Solutions

.358. By bypothesis,
Pn 0.4 1 - 2 .3 ... n 1
í>n42 “ 3 0I' 1 - 2 - 3 . . . » ( o + l ) ( n + 2)- 30
whence (n-f-1) {n + 2) = 30. The roots of thís equation are: n* = 4, n2= — 7.
The second root does not suit.
Answer: n = 4.
359. By bypothesis, 5C^ = C^+2 or
5n (n— 1) {n— 2) (n-f-2)<n-M ) n (n— 1)
1-2-3 ” 1.2-3-4
whence
5 ( n - 2 ) = Sn+ 2)4(n + 1)

Answer: nj — 14; n2 = 3.
360. The required term

16-45.14-13.12-11-10-9 a*
r 0= ( —i)*cf, (-£-)8(*2)*=
1 .2 -3 -4 .5 .6-7-8 * x*

Answer: 12 870 -
361. We ha ve

^ , = c ?=( 4 v s ) " ( | ^ ) 12- "

This cxpression coatains a to the power —* - ———— . By hypothesis,


n , 2(12— n) _ . . . , , _
— !___----- = 7, whence n — 6, i.e. n-j-1 — 7.
A rsunt: the seventh term.
362. We liave

. n 21 — « 21— n n , n
«By hypothesis, ---------g— = — ^--------- — , whence n = 9 .
Answer: the tenth term.
t _ 1
363. After siiuplifícation we get (a3 — a 2)í0. We have:
JL t 10-n _
Tn*l = ( - D " C g, (a ” 2) » ( J ) 10- " = ( - 1 ) " C g a - ^- " 2

By hypothesis, -----------------™ = 0, whence n-=4.


Answer: T$ = 210,
364. Let x be lhe exponent of the first hinomial. Th eu the sum of the bino-
tnial coefficients is 2*. The sum of the binomial coefficients of the second bino-
Chapter V I, Combinatorics and N ew tons Binomial Theorem 171

«nial is 2X+3. We get the equatíon


2* -b 2*+3 = 144; 2X (1 -j- 8) = 144; 2* = 24; z = 4
Answer: 4 and 7.
305. We have ~ Í05, whenee m = 15; then

455
r 13= ( - i ) < 2 C í ? ( - J = ) 12(9z)3^
' ]/3x x3
. 455
Aníujgr: -~r- .
j:3
366. By hypothesis, hence, m = 15. Then we have

r n+ = Cfj ( - i ) “ (2 2)'5->> = CJ‘5o"l3 0 -3 n

8 y hypothesis, 30— 3n = 0, n = 10.


Answer: T,* — 3003a10.
ri 4a
367. By hypothesis, i.e. m2—-5w— 50 = 0, whenee m = 10 (the
*oot m a - 5 is not suitabie). The médium terrn

r . = c í 0( — 1)51 Y ” 252

Answer: the médium (sixth) terrn is —252.


368. By hypothesis, 1 + mH— !Ití^ p lL = = 4 6 . Then solved in the same way
•as Problem 367.
Answer: the required (seveulh) terai J 7— 84.
369. By hypothesis, 2m=128, whenee m —1. We have

Tn+i — C!7lz
" T x |t7_"ní

4íy hypothesis, — (7 — «) = 5. "henco n = 3.


Answer: the required (íourthj term 7’4= 35 z6.
370. We have u„ = u,ga= ( l - f í ) 5. The multiplicam] y is cqual lu
( ;|
- g r = -.J = — j. According to the binomial theorem the factor is
«qual to i - f . 5 í 1 0 * 2 i0t3 5i4- f í5. Henco,
U(J ^ _< - 5í2 - 10i3 - lOi4 - 5ia - i8
Now replace the powers ol imaginary unit by their expressions:
(S — j3 _ j2 j = — i; i* — t3i = — u = -j-l;

= í<í = i; is = - 1
172 Answers and Solutions

Note. lü the given example, where the base is 1 - f t (or, in general, when the
base is a binomial of the lorm a ± aí), ínvoíution may be done in a raucb
simpler way. Namely, square 1 -f~ i. We get (1 -f- O2 — 2i, hence (1 -f- i)6 =
= (1 + 0 4-(l + 0 = (20M 1 + i) - - 4 (1 + í).
Answer: u0 = —4 + 4i.
371. We have u7 = i -f- — j . Since ™ «7 “ * (1 ” 0 e* Then
we may proceed in the same way as in the preceding problem. We may find the
modulus and argument of the product of six factors, each being equal to i — i.
The modulus of the quantity 1 — i is V 2 , the argument being equal to —45o.
Hence, the modulus of the product is equal to ("V^)6 — 8, and the argument,
to 0 { —45°) = —270°. Consequentiy,
(1 — í)» as S fcos ( —270°) - f i sin (—270°)j —
Answer: u7 = —8.
372. By hypothesis, the numbersCI; C\: C2 form an arithmetic progression.
Hence, Cln + 2C%, i.e.

n ’ n (n — l)(n —2) n n fu— 1)

Since n ~ 0, boih numbers of thr equality cau be divided by n. We get the


equation n2 — 0» 4 -1 4 = 0 with the roots m — 7 and nz = 2. The second root
is not suitable, since at n — 2 the expansion of the binomial has only three
terms, whereas, by hypothesis, there is a fourth term.
Answer: n — 7.
373. Solved in the same way as the preceding problem. On reducing by
n {n — 1) (n — 2) (n— 3)
(this number is non-2ero, since, by hypothesis,
4!
we get n2 — 2tn~- 98 = 0.
Answer: n — 14 or n = 7.
4_ ,t-l 5»at
374. Rewrite the first addend in parentheses in the form a5 x = a ox ;
ar-1 2x
. -TXT £ Z 7 . The fourth term of the expansion is
the second, m the form a>ax+ l = a x+1 1
5 -x j 6x 5- x ^ 6x

56a *v *'i' 1 . By hypothesis, 56a x *+* = 5 6 a 5-5. Consequentiy,


5 —x
x + _ 6x_ = 5.5

Answer: x~ -2 or x — — 5.
375. Reprcsent the given expression in the form
«-1 2(3- x )

[2 * 4-2 * - * 1«
By hypothesis,
4< x-l) 4(3-x>
Chapter V I. Combinalorics and N ew tons Binomial Theorem m

4 (x-l) 4 (3 -x)
2 * + 4“ * = 2*
Hence,
4 (x — 1) , 4 (3 — x) ,
X ^ 4—X *
Answer: x = 2 .
i _i
376. The seventh term T-, of tho expansion of the binomiai {2^ -f-3 3)x i

T7= C «{ 23)x _ 6 (3 3)«


aiul the seventh term from the end is
i _ t
T~ = C%(23)0 (3 3)x~8
onsequently,
1 _ 1 *-12 x - 12 *-12
T-j ‘ T ' — (2 3 )^*- 8 ^—8 (3 3^6 —(.v—6) = 2 3 3 (j 3

-^-'2 (2
Ôy hypothesis, 6 3i ==-g-,i.e. íj 3 — 6~í. Hence, — ^---- — — i.

Answer: x = 9.
377. By hypothesis.C|j-3(i 1o5*)2=10«, í.c. 10i3+ 2los*=10« or i 3+= k," I =10\
Taking logarithms ol Ihis equation, we get (3 -f 2 log x) iog x -=5. Solving
tho Jast equation, we get

(iog xj) t-, 1 and (log x2) = — —

Answer: x« = 10; x*= 1 0 2 —------- 7==-


JOOj/jÜ
378. By hypothesis,
3 logx+7

C g (V 5 ) ,w#*+1 C> x)3 — 200, i.e. 20x4<los*+*> = 200


Dividing both members of this equation by 20 and then taking logarithms, we
get after símpliíicalions
(log x)2 - f 3 log x — 4 = 0
Hence, (log xj) = 1, (log x2) — —4.
Answer: x, — 10; x2 = 0.0001.
379. Solved ín the same way as the preceding problom. We get the equation
xiogx- 2 fo00. Taking logarithms of the obtained equality, we find
(log íj) =•• 3 and (log x2) = —-1.
Answer: xs = 1000; x2 — 0,1.
174 Answers and Solutions

380. Solved in the same way as the two previous problems.


Answer: x t = 10; z-» = ■■. — .
1 V ío
4
381. Answer: £* = 10 0 ; x2=
y m
i
382. Answer: x! = 1000; zs =
' V io '
383. By hypothesis,
Tk+z— Tk+i = 30 (a>
rhere

Tfe+j —C\2X and .- r'-'12


k+iT
x 3
g _2fc 4 _2k
By hypothesis, the exponent — is twice as large as the exponent— - —
o o
i;_<>£ 4 _27-
i.e. — = 2— j —■ , whence fc= l . Then, after simplifications, the equa-
tion (aj takes the form:

2z^ — i i x 3 -f-5 = 0

Vse the suhstitution =


/- '1/2
.4nsa'er: z j -- 5 *V 5 ; z2 = —j — .

384. By hypothesis, 5C*n = consequcntly, we have the equation


„ m (m — 1} (m— 2)
om ----------- ---------------

Since m 0, both members of the equation can hc divicied by m. We get m, =


— 7 and m2 —4. The second root is not suitable, because rn should be a posi­
tive integer.
By hypothesis, T* ~ 7-20; hence,
X X- 1

C fl 2 3)3 (2 2 )* — 140
Answ er: x ~ h .

3NÕ. By hypothosís, we have C”^ — m = 20; m t).— m = 20. Out of the


two roots nij ==S aml mz ~- — 5 only the first one is suitable, since the expo-
nent of the binômia! is assumed to be a positive integer. Rewrite the bino-
x _ _3_ _x
uiial in the form {2~ 1"_[-2lü ~)8, By hypothesis,
r 4_ f fl= 56

k
Chapter VI. Combinatorics and A'ew tons Binomial Theorem 175-

2
After simplifications \ve get 56-2v — — 56. Putting 2* = y. \ve obtain
the equation y- — y — 2 = 0. wherefrom yx = 2 and y2 = —1. Since 2X — y
caimot be a negative number, the only suitable root is y* = 2 and, hence, 2* ~
— 2, i.e. x — 1.
A nswer: x — 1.
386. Since the binomial coefficients of the tenns equidistant from the be«
ginning and the end are equal, instead of the coefficients of the last three tenns
we raay take those of the first three tcrms, i.e. 1 -f- m -f- ~ 22, whence

~ 1—*
m = 6 (see the preceding probiem). Hence, the binômia! is (22 - f 2 2 )«. By
hvpothesis,
r 3 + n = 135
or
1 —X X 1 —X X

c\ { 2 ~ ) 2 ( 2 h * + c i (22)2 — 135
After simplifications we obtain
2*+i-f-22-a: = 9 or 2-2* + ~
Zx = 9
As in the preceding probiem, we find: (1) 2 * ~ 4 and (2) 2X= ~ .
Ansieer: x j = 2; x2 — —1.
387. The nurnbers a,, a3, a5, which are rcspectivcly the first, third and fiftb
terms of an arithmetic progression. form an arithmetic progression themselves,
so that 2a3 = ax + a5. Since, by hypothesis, a, = a3 — C'jn; ab — £'£,* then
_ _ _ — = ,„ — ^ — -
2m{m— i) . m (rn — t) (m— 2)
+

Reducing by m (m =£ 0), we find the equation m! •— 9m ~j~ 14 = 0. whose


roots are mx = 7, m2 — 2. Since, by hypotnesis, the expansion of the binomial
contains at least six terms, then m > 5, hence, only mt — 7 is suitable. The
binomial is
lo g O O -3 * ) : tog 3
(22 +2 r
By hypothesis

or
£!>2(x-2) los 32log (to-3-v>_2j
Hence we have
2 ( * - 2 ) lo g 3 + f o g ( i o - 3 x ) „ . { 2«
consequently,
(x — 2) log 3 + log (10 — 3*) = U
Taking antiiogarithrns, we get
3X~- (10 — 3 V) = 1
176 Answers and Solutions

or
- g ( 'i 0 - 3 * ) = l

Theu proceed in the sarae way as in Probiem 385.


Answer: — 2; x2 = Ü.
14
388. By hypothesis, the numbers -g- C~n, 0%, and Cf,t constitute a georae
ric progression; consequently,

^ a c s , = (c»,)í

Both members of the equality can be divided by m2 (m — l ) 2 (m — 2) siuce


none of tho factors (m, m — 1, m — 2) is equal to zero (it follows from the given
conditioo that m > 3); we get m = 9. By hypothesis, 7* — 16.8, or

c p 3 1| i ° e 1 l0* 51S6 1'- l ">* <«- » = 16>8

Hence we get the equation

- i l o g { .c — 1) — logo — lo g (6 — V S j: ) = — 1

Taking antilogarithms, we have


io y r ^ T « 5 ( 6 — y s x )
Hence, — 50 and x2 = 2. The íirst root is not suitable, since at x = 50 lhe
oumber 6 — ~\/bx is negative and, conscquently, has no iogarithra
Answer: x =- 2.
389. By hypothesis,
log (3-C3,) — log C\H— 1
or
, 3C3
m . t.
l o g = 10» 10
cm
;3C3
hence - - 10. After simplifications we find the equation m -—-3m — 18 — 0,
^m
whose rools are m j--6 and m->~ — 3. Consequentiy ?a = 6. From the eondi-
tion 97*3 — Tf,— 240 \vo get the equation

wlience
g.o3x-2_ 23x+i = 18
or
9-23x
02 23x*2 = 16
Hence,
23.v 2« and x — 2
Answer: x ~ 2 .
C h a p ter V IJ . A l^ e b r a ic and A r ith m e tic P r o b le in s 177

C H A P T E R VII
ALGEBRAIC AND ARITHMETIC PROBLEMS
3D0. The xveight of the round is made up of the weights of the projectile,
charge and sheü. The weights of the projectile and shell taken together make
y - f - ' g = J2 ^5e toía^ weíg^!t of the round. Tbus, the weight of the charge
li 1
makes i — ^ == _ of the weight of the round. which amounts to 0.8 kg. Hence,

the weight of the round is equal to 0.8 kg: ^ — 9.G kg.


Answer: 9.6 kg.
391. Out of the total number of workers, men make 10090 — 359o = 6590.
The number of men exceeds that of women by 65% — 3594 = 3094. which
252-100
amounts to 252. Consequently, the total number of workers is equal to — —— =
= 840.
Answer: 840 workers.
392. The profit percentage is calculated in relation to the prime cost (taken
for 10094). Hence. lhe selling-price (1386 roubles) makes 1009o + 1094 — 11094
of the prime cost. Consequently, the prime cost is equal to
1386-100 40_A , .
---- — =1260 (roubles)
Answer: 1260 roubles.
393. The loss is calculated in relation to the prime cost (taken for 10094).
Hence. 3348 roubles make 10094 — 494 = 9C?0 of the prime cost. Consequently,
the prime cost of the goods was equal to

Í ^ | i 152. = 34S7.5 (roubles)


9o
Answer: 3487 roub. 50 kop.
34.2-100
394. The copper content of the ore is — r— 94.
Answer: 15.2°ó.
395. The price was reduced by 29 kop. — 26 kop. = 3 kop., which amounts
to % of the old price. The number - “ 40 ^ is approximately repla-
ced by a decimal fraction.
Answer: 10.34%.
396. Solved in the same way as the previous problem.
Answer: 10.94%.
397. By hypothesis, 2 kg of raisins make 3294 of the total weight of tho gra-
pes. The weight of the grapes is equal to = 6.25.
Answer: 0.25 kg.
398. Let us denote the number of lourisls by x. In the first case the collected
money amounts to 75x kop.; hence, a sum of (75* -f- 440) kop. ís needed for
12-01338
178 Answers and Solutions

organizing the excursion. In the second case the collected money amounts to
80x kop.; hcncc, (80x — 440) kop. is needed. Consequently, 75x -f- 440 =
= 80x — 440.
Answer: 176 persons.
72
399. Let us denote the number oí persons by x; then each was to pay — .
By hypothesis»

Answer: 9 persons.
400. Let the price of one copy of the first volume amount to x roub., and
that of the second volume, to y roub. The first condition yields the equation
60x -f- 75y — 405. With a 15®ó discount the price o! one copy of the first
volume amounts to 0.85x roub.; with a 10% discount the price of one copy of
the second volume comes to 0.9y roub. From the second condition we obtain
the equation

Solving the system oí the two equations, we íind x = 3, y — 3.


Answer: the price of the first volume is 3 roubles; the price of the second
volume is also 3 roubles.
401. Let the first item be bought for x roub. Then the second item was bought
for (225 — x) roub. The first item yielded a profit of 25%. Hence, it was sold
for 1.25x roub. The second item which yielded a 50% profit was sold for
1.5 (225 — x) roub. By hypothesis, the shop sold the two items for 225 roubles
and made a total profit of 40%. Hence, the two items were sold for 1.40*225 =
= 315 roub. We get the equation

l i j + i 4 - ( 2 2 5 — z) = 31S

Answer: the first item was bought for 90 roubles, the second, for 135 roubles.
402. 40 kg of sea water contain 40*0.05 = 2 kg of salt. For 2 kg to amount
to 2% of the total weight, the iatter must be equal to 2 : 0.02 — 400 kg.
Answer: 60 kg of fresh water should be added.
403. Let us denote the lengths of the legs (in meíres) by x and y. B y hypo­
thesis, xs -f* y2 = (3 T/5ja. After the first leg is increased by 133 -i- % , i.e. by
1 1 1 2
133 — : Í00 = l — of its length, it is equal to 2 -^x. Increased by 16 -=-% the
3 3 3 o
1 1
second leg is equal to 1 -g* y. Thus, we get the equation 2 j i - f 4 *g* y = 14.
Answer: 3 m and 6 m.
404. If we take 12.5% of the flour contained in the first sack, then 87.5%
of the flour is left in it, which amounts to 440 kg : 2 = 70 kg. Consequently,
t f. . , . 70*100
the first sack contains ^ .
Answer: the first sack contains 80 kg, the second, 60 kg.
Chapter V II. Algebraic and Arithmetic Problems 179

405. Both factories together could fulfill ~ of the order per day. By hypo-
2 2
thesis, B has an efficiency of 66-5- % , i.e. of that of A ; consequently, the
o 3
2
efíiciency of both factories is 1 — of that of A. Hence, A can daily fulfill
1 2 1 1 2 1
22: 1-£• == of the order, while B , ~ = ^ of the order. Before A was
2 1 5
ciosed down rs ==-tt of the order was fulfilled. To fulfill the remaining — of
12 o b
5 1
the order B needs another ■*- : rr: = 25 dav?-
b 30
Answer: the order will be completed in 27 = { 2 5 + 2) days.
406. The 14 students, who ontaiued correct Solutions, make 100%
— (12% + 32%) — 56% of the total number of the students of the class.
The total number of the students is ~ 25.
Answer: 25 students.
407. The weigbt of the cut-ofí piece makes 72% of the total weight of the
tail, hence, the weight oí the remaining part (45.2 kg) amounts to 100% —
— 72% = 28% of the total weight of the rail; 1% of this weight is and
2o
72% amount to •72 = 116 ~~ kg m 116.23 kg.
Instead of determining one percent of the weight of the rail we may set up
the proportion x : 45.2 = 72 : 28.
Answer: the weight of the cut-off piece is (approximately) equa! to 116.2 kg.
408. The weight of the piece of the alloy (2 kg) makes 100% + 14-~ % —

= 114-™% of the weight of the copper. Hence, 1% of the weight of the copper
o
amounts t o ------ — kg. Consequently, the weight of the silver, which comes to
114 ■ —
2
14 y % of the weight of the copper, is equal

kg

Instead of determining one percent of the weight of the copper, we may set up
the proportion

z :2 = i 4 y :1 1 4 y

Answer: the weigbt of tbe silver is -jr kg.

12*
180 Answers and Solutions

3 1 7
409. The money received by the second worker amounts to : ^ '2’ =s30
7 4
o! that of the first, or, in percent, ~ .1 0 0 ?o — 23-g- % . The total received
by the three workers (4080 roubles) makes

100% + 23 i % + 43 i % - 1 6 8 J %

of what was paid to the first worker. One percent of the money received by
the first worker comes to roub-, hence the first worker was paid
i64

-2 ^ . 1 0 0 = 2448 roub.
1661

The second worker received 23-J-% of this sum, i.e.

2448-23j
_ = 571.2 roub.

The third worker carned


2448-43j
= 1060.8 roub.
1ÕÕ
Answer: 2448 roub.; 571 roub. 20 kop.; 1060 roub. 80 kop.
410. If the first box contains x kg of sugar, then the second contains
42— ^ ^
i - x kg, and the third, y x * ~ 5Õx hypothesis, x-f--g-x-í-ggX =
4 17
= 64.2, whence x = 3 0 (kg). Of this number we take first -g-, and then .
Answer: 30 kg; 24 kg; 10.2 kg.
411. Let us take x tons of the first grade; it contains 0.05x tons of nickel,
and then it is necessary to take (140 — x) tons of the second grade, containing
0,40 (140 —x) tons of nickel. By hypothesis, 140 tons of Steel contains 0.30-140
tojjs of nickel. ConsequentJy, 0.05x-f- 0.40 (140 — x) = 0.30-140. Hence,
x — 40.
Answer: 40 tons of the first grade and 100 tons of the second grade.
412. The piece of the alloy contains 12 kg-0.45 — 5.4 kg of copper.
Since in the piece of the new alloy this amount of copper makes 40% of its
weight, the piece weighs 5.4 : 0.40 = 13.5 kg. Hence, it is required to add
13.5 kg — Í2kg = 1.5 kg of pure tin.
Answer: 1.5 kg.
413. Solved in the same way as the preceding problem: (1) 735 g-0.16 —
== 117.6 g; (2) 117.6 g : 0.10 == 1176 g; (3) 1176 g — 735 g = 441 g.
Answer: 441 g.
Ckapter V II. Algebraic and Arithmetic Problems 181

414. Let x denote the weightof copper (in kg). Then 24 — x is the weight
of zinc.The loss of weight is -i- x (for copper) and y (24 — x) (for zinc). Conse-

quently, z - f y (24 — jr) = 2 y . Hence, x = 17.


Answer: 17 kg of copper, 7 kg of zinc.
415. Let us denote tne number of 25-metre lengths by x, and that of 12.5-
metre lengths, by y. For a 20-kilometre (20 000-metre) track 40 000 m of rails
are needed (two Unes). By hypothesis,
*>
06 f
2 5 z 0.50*12.5y = 40 000 and Í2.5y + ■— *25x - 40 000
Answer: 1200 pieces of 25-metre rails and 1600 pieces of 12.5-metre rails.
416. Let the number of students be x. During the exchange each student
reeeivedx — i photographs and all the students reeeivcd x (x — 1) photo-
graphs; by hypothesis, we have the equation x {x — 1) — 870.
Answer: 30 students.
417. Let us denote the smalier number by x and the larger number, by
y {x < y). The first condition yields ~\/xy — x-f-12, and the second condition,
= y — 24, i.e. y — x — 48. Solving the system, we fínd x — 6, y — 54.
Since 6 < 54, this solution is suitable.
Answer: 6 and 54.
418. Let the smallest number be x, the next one, y, and the largest, z. We
have three equations
y — x — z — y; xy — 85; yz = 115
From the first equation we find z = 2y — x; substituting it into the third
equation, we get 2y2 — xy — 115 or, by virtue of the second equation, 2y- —
— 200. Out of the two Solutions (xj = 8.5, yt = 10, z, = 11.5; x2 — —8.5,
Vz = —10, z2 = —11.5) the first one is suitable (since x t < y, < z,), and the
second is not (since x2 > yz > z2).
Answer: 8.5; 10; 11.5.
419. Given
5 ± | ± l = a and ± t ! Ç ± £ = b

It isrequired to find X- , From the first equation we have x2 -f- ir-J-


H- z2-j- 2 (xy-\- yz-+- zx) = 0 a1. By virtue of the second equation we have
* * + y2-f- z2 == 3b. Hence, 36-f- 2 (xy -f- yz -f zx) — 9 a2.
Answer: * » ± f ± J Z = .
420. If the length of the sheet is x cm, and width, y cm, then lhe box has
the foliowing dimensions: length—(x — 8) cm, width—(y — 8) cm and height—
4 cm. By hypothesis, 4 (x — 8) (y — 8) = 7Ó8 and 2x -f* 2;/ — 96.
Answer: 32 cm X 16 cm.
421. Let the tens digit be x, and units digit, y (x and y are positive integers
less than 10). We have the foliowing systern of equations:
m ’r 'J ... 2 - | ; < 1 0 i+ » )-(1 0 » + *) = 18
xy á
182 Ânswers and Solutions

Out of the two Solutions = 6, y = 4 and x — - i , y — — only the first


one is suitable.
Answer: 64.
422. If the number of tens is x, then the number of units is equai to x - f 2.
We get the equation
[ÍQz + (x - f 2)) [x + (x - f 2)] = 144
2
whence x — 2 and x = —3 ; by hypothesis, the second solution is not suitable.
Answer: the required number is 24.
423. Let the required number be x. If the figure 5 is adjoined on the right
of it, then we get the number lOx -f- 5. By hypothesis, we have
40x - f 5 = (x - f 3) (x - 13)
Answer: 22.
424. Let the larger number be x, and the smaller, y. If three digits (zero
and the two digits of the smaller number) are adjoined to the larger number,
then the digits of the latter express the number of thousands, and tbus, finally
we get lOOOx + y. And from smaller number we get the number ÍOQOy -j- lOx.
By hypothesis,
lOOOx + y « 2 (lOOOy + 10x) - f 590; 2x + 3y = 72
Solving the system, we find x ~ 21, y — 10. Being two-digit numbers, they
satisfy the condition of the probiem.
Answer: 21 and 10.
425. If the units digit of the factor is x (x an integer, less than 10), then the
tens digit is 3x. The factor is eaual to 3*10x + x = 31x. The incorrectly writ-
ten factor is lOx -f- 3x = 13x. The true product is equai to 78*31x, the product
obtained by mistake is 78*13x. By hypothesis, 78*31x — 78*13x = 2808.
Hence, x = 2.
Answer: the true product is equai to 4836.
426. The speed of the first train is x km/h, that of the second, (x — 12) km/h.
We have the equation
96 96 2
x — 12 X ~ 3
Answer: the speed of the first train is equai to 48 km/h, that of the second,
to 36 km/h.
427. Let the rate of the first person be v km/h. then the rate of the second
is equai to (u — 2) km/h. The first spends ~ h, the second, h- We obtain
the equation
24 24
v— 2 v ~ l
Answer: 8 km/h; 6 km/h.
428. Let the speed of the train be x km/h; then the speed of the boat is
(x —30) km/h. The train spends ~ h, and the boat, h. We get the equation
80.5 66 , 15
x — 30 x 4 + 60
Chapter V I I , Algebralc and Arithmetic Problems 183

Answer: the speed of the traiu is 44 km/h, that of the boat is 14 km/h.
429. Let the first tailor shop produce x suits a day; then tbe second shop
makes x -f- 4 suits per day. The first shop has completed its order in days;

hence, the time given for the fulfilment of the order has faeen -f- 3 j days.
The time given to the second shop has been the same. Consequently,
810 . _ 900 ,
------ ^3 = ----r-r + O
x x-j-4
Answer: the first shop produces 20, and the second, 24 suits per day.
430. Let the speed of the ship going off to the south be x km/h, and that o
the other ship, (x - f 0) km/h. Since the directions of their travei are mutually
perpendicular, by the Pythagorean theorem we have
Í2x)- - f [2 (.r + ü)i- - 60-
Answer: the speed oí the first ship is equal to 18 km/h, that of the second.
to 24 ktn/h.
431. Tvvo dog’s leaps cover 4 metres; three fox s ieaps, 3 metres. Consequent-
jy, each time the dog runs 4 metres the distance between the dog and the fox
is reduced by 4 m — 3 m =■-- 1 m. The initiai distance between them is 30 times
greater. Hence, the dog will catch up the fox, when it covers 4 m *30 = 120 m.
Answer: at a distance of 120 m.
432. In one minute the minute hand turus through 6°, while the hour hand,
only through ~ . At four o'clock the angle between tbe hands is equal to 120°.

During x minutes the hands turu through 6x and ~ x degrees, respectively.


\
By hypothesis, Cr — j - x — 120.
g
Answer: 21-rr minutes.
11
433. Let us denote the time spent by the train to cover the A-C section by
(hours) and therequired speed, by v (km/h). By hypothesis, tbe distance A D was
covered by the train in ■
— h at a speed of v km/h, and BC, in —- h at a speed of

0.75-u km/h. Hence, AB ~ and BC = 0,l h - v ~ km. By hypothesis,


on the return trip the C-B section was covered at a speed of v, and the B-A sec­
tion, at a speed of 0.75e. Hence, the time spent on thè C-B section was :^

i,e, * h, and the time spent on the B-A section was : 0.75 v, i.e. - * . h.
2 2 2-1). /S
By hypothesis,

2*0.75n *75 + *
Answer: 10 hours.
184 Answers and Solutions

434. Suppose the cyclist travelled at a speed o! v km/h; then the required
30
speed was (v — 1) km/h. The cyclist actually travelled —• h, while the schedu-
30
led time was h. By hypothesis,
30 30 __ 3
v— \ v 00
The negative solution v — —24 is not suitable.
Answer: 25 km/h.
435. Let the scheduled speed be x km/h. The actual speed was (x -j- 10) km/h.
The scheduled time is ^ h , but actually it was h. By hypothesis,
80 80 16 _
x x-f-10 ~6Ü
Answer: 50 km/h.
436. The first haif of the distance was covered by the traiu in x hours. Then,
to arrive in time the train had to cover the remaiuing portion of the route io
x— hours. The speed of the traiu in the first half of the route was km/h,

in the second, — km/ h. By hypothesis,


x —~2
420 420 0
1 x
x~ 2
The equation has only one positive root.
Answer: 21 hours.
437. Let the speed oí the first train be x km/h, that of the second, y km/h.
1eí the first case the first train covers Uli kilomelres before they meet, the second,
lUy kihmietres. Consequonlly,
!0.r ÍOy - - 050
in lhe second case lhe first train covers 8x kilomelres, and the second (which
spenl en route 8 h -f~ 4 ii 20 min — 12 — b), 12 — y. Consequently,

8 . r - f 12 y = 650

Answer: the mean speed oí the first train is 35 km/h, that of the second is
30 km/h.
438. Lei the speed of the first train be x km/h, and that of the second, y km/h.
The distance of 600 km is covered bv the first train in ^22 hours, and by the
x
sccoml, in 222 hours. By hypothesis,
n
600 , ^ 600 250 _ 200•
•r r ' ” y x ~~ y
C h a p ter V II. A lg eb ra ic and A r ith m e lic P ro b lem s 185

Auswer: thc speed of the first traiu is 50 km/h, that of the second is 4U kimh.
439. lf the distance is x km, then at a rate of 3.5 km/h the commuter wouíd
cover this distance in hours. And since he wouíd be one hour late, the momeot

he started out was separated from the train ieaving time by ^ ~ — 1J hours,

In an hour, duringwhich hehad walked 3.5 km, there remained — 2 j hours


till the traia departure, and he bad to cover the rcmainder oí the distance of
(z — 3.5) km. At a rate of 5 km/h the commuter covered this distance ia
r _3 5
— hours. Since he arrived half an hour before the train Ieaving time, w©
have
__ „ J--3.5 1
3.5 ~ 5 2
Anstvcr: 21 km.
440. Let the speed of the cyelist be x kmmin, and that of the car, y km/min.
The car had travelled 10 minutes, and the cyclist, 10 4- 15 = 25 minutes when
he was caught up by the car. By this mornent they had covered one and the
same distance. Consequentiy. 25r 10y. By the time the car on its retura
trip encountered the cyclist the car had covered 50y km, and the cyclist, 65z km.
The sum of the-u* distances is equa! to twice the distance between A and B.
Thereforc G5.r 5(0/ 38. Solving the system of equations, we find x =■ 0.2;
II --- 0.5.
Auswer: the speed of the cyclist is equal to 0.2 krn/min — 12 km/h. and
tliat of the car. to 0.5 km/min = 30 km/h.
441. Let the trains pass each other in x hours after the fast train departure.
Then. by the time oí the encounter the mail train had travelled {x - f 3) hours.
Each train had covered 1U80 : 2 — 540 (km) before they met. Hence, the speed
of the first train is equal to km/h and that of the second, to km/h. By

hypothesis, — ' — ~ r ^ — 15. Only one root is suitable: x 9.


Auswer: in 9 hours after the fast train departure.
442. Let the first cyclist travei x hours. Reasoning in the same way as in
the preceding prohlcin. we set up lhe equation
30 42
X— 1 X

Auswer: the sneed of the first cyclist is equal to 14 km/h and that of the
second. to 18 km/n; the first is 3 hours cn route prior lo the encounter, the second,
2 hours.
443. Let the distance AB between the starting points be x km, and let
the first hiker cover it in y hours. By hypothesis, the second hiker covers
the distance BA in (»/---5) hours. Hence, the first covers— kilometros ner
V 1
hour, and the second, ------r* kilometros por hour. During an hour the distanc©
tj—o
186 Answers and Solutions

between the hikers ís reduced by ( ) km, during 3-|- hours,


v + lí— 5
\ IX X \ 1 i X
byji3y j * Since they meet in 3-^- hours, we have 3-g-^— 4-
H---- } = x. Since x
y — í> /
0, we can divide both members by x. We get:

4 (fh^H
Hence, we find y. The value of x remains undetermined.
Ansiver: the first hiker covers the whole distance in 10 hours and the second,
in õ hours.
444. Let us denote the point of encounter by C. Let AC — x km; then, by
hypothesis, CB {x -f- 12) km. Furthermore, bv hypothesis, the first hiker
x _l i*>
covers the distance CB in 8 hours. Hence, his rate is equal to ^ km/h.
o

fn the same way we find that the rate of the second isiker is -í- km/h. Conse-
x f 1 8x
querttly, the distance AC is covered by the first ono in x : — ~ hours,
9 (x 4-12)
while the second covers the distance BC in —— - — - hours. And since the second
x
traveis 6 hours more than the first one, we have
9 (x -f 12) 8x
X x 4“ 12
When solving this equation we mayintroduce an auxüiary unknown = s.

\Yeget9s — — 6. Out of thetwo roots ^ = y and z2 = — — ^ the second


one is not suitable. since both quantities x AC and .r -f- i2 = CB must be
positive. From the equation ~ we find x - ■ 36. Hence, AC = 36 km,
CB = 48 km.
Ansiver: AB ■— 84 km. The rate of the first hiker is 6 km/h and that of the
second one is 4 km/h.
445. The prohlem is similar to the preceding one. Let the dirigible fly
to the passing point x km: then the airplane has made (x-4-100) km. The
speed of the dirigible is equal to km/h and that of the airplane, to

— — km/h. From its terminal to the passing point the dirigible flies
1j
x 100 3r
x : — — — — —— hours; whereas the airplane covers the distance between
3 x -t -lU U r

1"3" (z + 190)
the airport and the passing point in ------------------ hours. We obtain the
Chapter V II. Algebraic and Arithmetic Problems 187

equation
-(x + 100)
3x
•r+lOO '

Consequently,. - , — whence x — 200; the second root is not suitable.


x -p lUO o
Answer: the distance between the airports is equal to 500 km; the speed
of the dirigible is 100 km/h and that of the airplane is 150 km/h.
446. First method. The problem may be solved in the same way as the pre-
ceding one. We get the equation
i * \2 " * V"
\ x —a } m ’ * —« Ym
Hence,

y » -y *
Then -we find the speeds of the hikers:
x —a ,
uj = ------ and

Second method. Let us denote the point of encounter by C. Since the first
hiker covers the distance CB in m hours, we have CB — vim km. Similarly,
CA = vzn km. By hypothesis, CA — CB = a. We get the equation nvz — mo| = a.
The sectíon AC is covered by the first hiker in —— hours; hence, the dis-
v2n
tance between the starting and the encounter points he covers in hours.

Similarly, for the second hiker: hours. Since they start out at the same

time, we bave n — ~ m — , whence iq : v2 — Y n ' Let us solve this


Uj
equation together with the first one. For the sake of symmetry it is useful
to make the following substitution: t —-^7=-==—— ■. Substitutingtheexpres*
y/n y m
sions vi =
Y * i and v2= Y ^ t into the first equation, we get (n ] / m - m V n ) f =

= 0, whence t = — ; now we find


n ]/m -m Y n

a Y n v _ a Y m

n y ín —m Y n * n Y m ~ mY n

Note. The problem is solvableonly if n "|/m > m ] /n ; dividing both mem-


bers of thia inequality by the positive num berVm V/T, we get Y n > V m ,
i.e. n > m. This condition can be obtaiued immediately from the given one:
since the first hiker covers a greatcr distance before they meet, his rate is higher
than that of the second hiker. On the other hand, the first hiker has to cover
188 Answers and Solutions

a shorter distance to arrive at B than the second to arrive at A . Consequently,


the lirst will reach B faster than the second will reach A.
Answer: the speed of the first hiker is ----- — — km/h and that of
n y m —m y n
the second, ------IL— ——-p=~ km/h.
n ym —myn
447. Let the lirst body make x degrees in one second, and the second, y deg-
360 360
rees. Frora the first condition we find --------------= 5. Each second the distance
y x
between the bodies (as measured along the arc) is increased by {x — y) degrees.
Every 100 seconds the distance rnust be increased by 360°. Therefore,
100 (x — y) = 360. The obtained system has two Solutions (xt = 18, yt = 14.4;
x 2 = —14.4, y2 = —IS). Both of thern are suitable, since they have one and
the same physical meaning, only the numbers of the bodies and the direction
of motion being changed.
Answer: 18°; 14°24/.
448. Let us denote the speed of one body (in ra/min) by x and that of
the other, by y, assuming that x > y. Let the bodies move in the same direc­
tion and come together at some point A . Let the next nearest encounter take
place at a point B (these points may coincide: for instance, in case the speed
of the first body is twice that of the second; in this event by the moment they
come together once again the first body will complete two revolutions, whereas
the second, only one).
Moving fron\ A to B (this path may overlap itself for one or both bodies),
the second body !ags behind the first one so that by the moment of the nearest
encounter the delay will be equal to the full circumíerence. Since the bodies
come together every 56 minutes, during which tbe first body covers a distance
of 56x metres, and the second one, 56y metres, the circumíerence is equal to
56x — 56y.
Let bow tbe bodies move in opposite directions. Then the sum of the dis-
tances covered by them during the interval between the two nearest encounters,
i.e. during 8 minutes, will make the whole circumíerence. Consequently, the
circumíerence is equal to 8x - f 8y. Tbus we have the equation 56x — 56y =
- 8.r - f 8y.
By hypothesis, in 24 seconds the distance (along the circumíerence) between
tbe approacbing bodies decreases by 40 — 26 = 14 metres. During this time
the bodies do not come together; therefore the decrease in the distance bet­
ween the bodies equals the sum of the distances covered by them in 24 seconds =
-= -í- minute. And so we get the second equation

5 z + 5 y 14
Answer: 20 m/min; 15 m/min; 280 m.
4411. Let x and tj be positive numbers expressing the speed of the points
in corresponding units (if c is the circumíerence in metres, then tbe unit of speed
is 1 m/sec, and so on; it is not clear from the givea conditions in what units the
length is measured). Assuming that x > y, we have the system of equations
c c
tx— ty = c;
Chapter V II. Algebraic and Arithmetic Problems 189

(íor the setting up of the first equation see the preceding problem). Making
a substitution, we find the equation
nty* - f ncy — c3 = 0
c ( V n 2-f4nf —n)
íts positive root is y- (the second root is negative )
2nt
c (V n 24 4n t - f « )
Answer: the higher speed is numericaUy equal to the
c ( "\/n24 4n t — n)
lower, to
2nt
450. Let the speed of the ship in stiil water be x km/h. Then we have the
80 . 80 e 1
equation — r-7 -+ ------r - —° 'õ - '
z-{-4 x —4 à
Answer: 20 km/h.
451. Answer: 9 km/h.
452. Let the rate of the current of water be x km/h, and the speed of the boat
20 20
in stiil water, u km/h. The first condition yields the equation —------j---------— 10;
9 y + x y —x
2 3
the second condition, the equation = —:— . Iu solving this system it is
y — x y -fx
conveníent to put
1 1
y+x ' y —x
Solving the system,
20u + 20r= 10; 2t>= 3k ,
we find
10
y= io * i*e* y ****5; y- =3

whence x — -~ ■
b
5
Answer: -z- km/h.
6
453. Let the raft float down the river over the distance (a km) between A
and B in x days. Then its rate, equai to the rate of the current of water is
~ km/day. By hypothesis, the speed of the ship going downstream is equal

to ™ km/day. Consequently, the speed of the ship in stiil water is ( y ~ ~

— — j km/day. And since the speed of the ship going upstrcam is equal

to ~ kra/day, its speed in stiil water is j km/day. We have the equation

o a __ a a
~2 x T + T
Answer: 12 days.
190 Answers and Solutions

454. Let the speed of the body be x m/s, and that of M2, y mls. By the
moment of the first encounter M t has heen in motíon during 21 seconds, and
M2, during 21—15 == 6 seconds. Thus, we get the equation
2ix + 6y = 60
By the moment of the second encounter Mi has been in motion during 45 se­
conds, and M 2, during 45 — 15 = 30 seconds. Let C be the point of the second
encounter; then Mi by the moment of the second encounter has covered the
distance AB + BC, and M 2, the distance BA + AC . The sum of these distan-
ces is ZAB, i.e. 180 m. And so we obtain the second equation
45x 4- 30y = 180
Answer: the speed of the body Mi is equal to 2 m/s, and that of the body
M2, to 3 m/s.
455. Let the speed of the messenger when going uphill be equal to x km/h,
over the levei ground, to y km/h, and downhill, to z km/h. Before returning
the messenger has covered half the distance, i.e, 14 : 2 = 7 km; he has gone
3 km uphiil, 4 km over the levei ground, then (on his way back) another 4 km
over the levei ground and, finally, 3 km downhill. By hypothesis,
, 3_
i + A + A + £ _ „3•«- 3 8 , 3 ,3
« i.e. — -4-' ■■ I —
—— 3-=-
* x ' y z 5
The other two conditions yield:
9_ 3_ q !7
X ‘ y 1 z 20' x y 1 z " 20
1 1 1
We find — , — , — and then x, y, z.
x y z
Answer: uphill—3 km/h, over the levei ground—4 km/h, downhili—5 km/h.
456. Let the quota be x pages a day and the time lirait, y days. Then, by
hypothesis,
(x 4- 2) (y — 3) = xy and (x *f 4) (y — 5) = xy
Answer: 120 pages, 15 days.
457. Let the operator make x parts in y days. Then he produced —•parts per

day. By hypothesis, if he had turned out “ —r~ 10 parts, he would have comple-

ted thejob in y — 4 days. Hence, lo j “ Xm The other condi-

Üon yields the equation - s j ( y f 3) = We get the following system,


of equations
f 10ií-4 4-j-=45

I — 5j/-|-3 — = 15
k y
Multipiying the second equation by 2 and adding the product to the first one,
we get — — 50. Substituting this value into the second equation, we find y =
= 27. Consequentiy, x = 50y = 1350.
Chapter V II. Algebraic and Arithmetic Problems 191

Note. Tbis problem may be solved in the same way as tbe preceding one, if
instead of the unknown x we introduce the quantity z denoting the number of
parts produced daily. We would obtain the same system of equatioDS, where
the quantity — would be replaced by z.
Answer: the worker made 1350 parts in 27 days.
458. Let the daily quota of the typist be r pages, and the lime limit, y days
then the job involves the typing of xy pages. By hypothesis, typing x + 2 pages
per day, the typist would spend y — 2 days. Hence, the job involves
(z + 2) (y — 2) pages. Consequently,
(x + 2) (y — 2) — xy
Reasoning in the same way, we get another equation:
(j: - f 0.60x) (y — 4) — xy + 8.
Answer: the quota was 10 pages per day, and the time limit, 12 days.
459. Let the first worker complete the task in x hours. Then we have the
equation
Answer: the first worker can do the job individually in 12 hours, the second,
in 24 hours.
460. If the first pipe fills the swimming pool in x hours, then tbe second
1 1 1
fills it in (x 4- 5) hours. Thegiven condition yields the equation — - f = g- -
Answer: the first pipe fills the pool in 10 hours, the second, in 15 hours.
461. Let the first worker, working individually, be able to complete the
task in x hours, and the second, in y hours. Then in one hour the first ful-
fills JL of the whole assignment, and the second, — . By hvpothesb,
x y
Since then they worked togcther for another 4 hours, they

did of the job, which was equal to 1— — Thus,

we have the second equation Subtracting it from the first


3 3 1 1
equation, we get — — jg i w^ence x ~ T^en we — = 24 an<* P —2<4,
Answer: it would take the first worker 18 hours and tÜe second, 24 hours
to do the whole assignment.
462. Let us denote the required numbers by x and y. Four high-power cranes
operated 2 4 - 3 = 5 hours; two low-power cranes worked three hours. There-
fore (see solution of the preceding problem)

4 .5 ._ i-j_ 2 .3 .-i-= l
* y
The second condition yields
4*4.5*— 4-2-4.Õ* — = 1
x y
Answer: 24 hours, 36 hours.
192 Answers and Solutions

463. Let one three-ton truck be able to deliver the load during x hours, and
one íive-ton truck, during y hours. By hypothesis (see Solutions of Problems 461
and 462),

30.8-— + 9 -6 .— = 1 and 9-8-— + 3 0 .6 - —


x y y x lo

Answer: x = 300; y = 270; 30 five-ton trucks will deliver ali the material
in 270 : 30 — 9 hours.
464. Let it take the first typist x hours and the second, y hours to do the
whole job. When the first was busy typing for three hours. the second was busv
9 ii
only for 2 hours. Botb of them did 1 — ^ = ~ of the whole work. We get
the equation
3 ,2 11
T + T =S
When the assignment was corapleted, it turned out that each typist had done
half the work. Hence, the first spent hours, and the second, -|- hours. And
since the first had begun one hour before the second, we have

The system has two Solutions, but one of them is not suitable, since it yields
negative valuc for y.
Answer: 10 hours (the first typist), 8 hours (the second typist).
465. The problem is similar to the preceding one. We get
1.5 l i . JL
y S3Ü; 2'
where x and y are the times (in hours) for each train to travei the distance
between A and B. Out of the two Solutions yieldcd by the system only one is
suitable.
Answer: 10 hours; 9 hours.
466. Let x litres of water per minute flow in through the first pipe, and y
litres per minute flow out through the second pipe. By hypothesis, a full bath
containing 2 X 9 X 2.5 = 45 litres can be empticd in one hour, i! both pipes
" 3
are open. Hence, the amount of water is reduced by litre per minute.
60' 4
Consequently, y — x = On the other hand, when only the first pipe is
45
used, the bath can be filled in ~ minutes; when only the second pipe is used,

the bath can be empticd in — minutes. By hypothesis,

45 45
y
Chapter V II. Algebraic and. Arithmetic Problems 193

The System of equations


3

Ü _ Ü _ 5
* y “

has two Solutions íx j = 2 y - ; y j = 3 and z2 = —3; y2= ~~2 ~ \ , The second


solution is not suitabie (x and y must be positive numbers).
Answer: 2 — 1/min; 3 1/min.
4
467. Let the time limít be x days; tben the daily plan is ~ ~ cubic metros.
The team of navvies completed the job in x — 8 days; hence, the daily output
8000 . n . . 8000 8000 ,n . t . .
was ------ cubic metros. By hypothesis, ----- - -------- --- 50. Out of the two roots
x —8 x—8 x
yielded by this equation (xt — 40 and x2 = —32) only the positive one is sui-
table. Hence, the daily plan amourited to —— = 200 cubic metres. The ovor-
fulíilment by 50 m3 made
50-100
25%
200

Answer: the original time limit was 40 days, the daily overfulíiíment oí
the plan being 25 por cent.
468. The first team repaired x km per day; then the second repaired
(4.5—x) km per day. The first workH ^ days; the second, - days. By

hypothesis, — ^ —^ L — = 1. This equation yiclds two roots: xi = 2 and x£=


— 22.5. The second root is not suitabie, since the number 4.5 — x must be
positive.
Answer: the first team repaired 2 km, and the second, 2.5 km daily.
469. Let the first worker be abie to do the wbole job in x hours, and the se*
cond, in y hours. Hence, half the assignment was done by the first in -£• hours;

the remaining half will be dono by the second in —■ hours. By hypothesis,

= 25. The other conditíon (see solution of Problem 461) yields

Answer: one oí the workcrs (either the íirst, or the second) can do the whole
job in 20 hours, the other, in 30 hours.
13—01338
194 Answers and Solutions

470. Let one tractor be able to plough tbe field in x days, and the second
in y days. We have (see the preceding problem) the system of equatíons:

— ;—M
x y
It can be replaced by the system x-\-y = 2k\ xy — 2kt._______
Answer: it would take one of the tractors {k-\-~\Zk'2’— 2ki) days and the
_______ k
other, (k — ~\/k2— 2kt) days. The problem is solvable for > í.
471. Let all the three dredgers working togelher be able to complete the job
in x days. Then the first one working aione can do the job in (x -f- 10) days, the
second aione, in (x 4* 20) days, and the third alone, in 6x days. In one day the
1 1
first dredgeralone fuífills — — of the job, the second alone, — 7- ^ and the third,
0 x -f- 10 x-j-20
* and all of them together, — of the job. Thus we have the equation
6x

x-j-iO ■+ x 4-20 ' tíx


Answer: the job can be done by the first dredger alone'in 20 days, by the
second, in 30 days, and by the third, in 60 days.
472. The second worker can complete the assignment in x days, the first
7
can do it in (x4-3) days. In 7 days the first worker will fu lfill------ — of the job,
*4 -3
1
in 7 — 1 5 -~days the second worker will do —— of the whole job. Thus,
we obtain the equation

•4~
*43
Answer: it would take the first worker 14 days and the second one, 11 days
to do the job individuaily.
473. Let it be possible for the first tractor to plough the whole field in x
days, for the second one, in y days. The first condition yields

x ' y 8
The first tractor can plough half the field in — days, the reraaining haíf will
be ploughed by both tractors in 4 days (the whole field was ploughed by them
in 8 days). And so we have the second equation “ + 4 = 10, hence x — 12
(days). From the first equation we find y — 24 (days).
Answer: it would take the first tractor 12 days, and the second, 24 days to
plough the field.
474. Since the workers began working one after another, the intervals bet-
ween the starting times being the same, and the first to begin worked five times
Chapter V II. Algebraic and Arithmetic Prablems 195

as long as the last to begin, lhe number of workers was equal to 5. lí the Jasl
lo begin worked x hours. then the total oí the working hours amounted ío
x -f~ 2x - f 3x -f- 4x -j- 5x = 15x. By hypolhcsi.s lhe inen could have fínishcd
the job in 6 hours, if they had begun at the same tinie. Consequently, ícj —
= 5*0, whencc x — 2. The job lasted as long as the íirst worker digged. i.e.
íor 5x hours.
Answer: they worked 10 hours,
475. Let the first worker be able to complete the task in x hours; we gel
the equation
! . i , 1 _í_
- 4 . _ L . - 1- J
2x t

Answer: x —~ (5í — 2-f- V - ^ 2 + 4f + 4).


476. Let the íirst tap íill the tank in x hours. and the second, in y hours.
In one hour the íirst tap íills — oí the tank, and, by hypothesis, it wasopen

/1 \ 3 ^
í —y j hours; hence, the water from the first tap filled —— oí the tank. Similarly,
1
3* *
we íind that the water from the second tap filled ----- of the tank. Since.when
y
13
tbis was done, the tank was ^ we ^iave

3 x 3 y 18

The second condition yields tho equation

T+T^Ts

The System may be solvcd in the follotving way. If we put — -~z, then the
first equation takes the íorm
_1_ J_ __13
3 24_ 3 ' z “ 18

whenco zjs z2= - y . Transíorm the second equation to

-+ t ‘

Substituting -$- — -77* we find y = 9; hence, x~ - y — 6. Substituting

•—-g-, we find y = 0 and x = 9.


Ansujer: one of the taps fills tho tank in 6 hours, the oíher, in 9 hours.
13*
196 Ansieers and Solutions

477. If the daily quota of bricklaying was x thousand bricks, and the actua
mimber of bricks laid daily was y thousand, then we have the system of equa-
tions
['120_120==4
\* y
{3y — 4x = 5

Anstver: the daily quota of bricklaying was 10 000 bricks and the actual
number of bricks laid amounted to 15 000.
478. The consecutive amounts of water (in íitres) in the tbree vessels
(I, II, III) are tabulated bclow:

, 1 ri /i \ . -1 . 2
I X
T 1
3/1 , \ 3/1 , \
11 y T
9 r i/i \ , -1
III z Tõ Lt ( t x~ !,) + z J

By hypothesis, each of the expressions in the last column is equal to 9.


Alteniate solution. First fina the araount u of water contained in the second
vessel after the jirst pouring. By hvpothesis, the second pouring reduced this
1 3
amount by-^-u, leaving there 9 Íitres of water. Consequently, = 9, i.e.
u — 12. Now find the original amount z of water in the third vessel. The first
pouring left it unchanged; the second one increased it by-^-u = 3 Íitres, so that
the third vessel turned out to contain (z -f- 3) íitres. The third pouring reduced
1 9
this amount b y ^ (= - f 3). Consequently, - f 3) = 9, i.e. 2 = 7 . Then
find the original amount of water x in the first vessel. The first pouring left
2
in it - x Íitres; the second pouring left this amount unchanged; the third pouring
1 2
increased it by (3 -f- 3) = 1. Consequently, 1 = 9 , i.e. x — 12.
Finally, find the original amount y of water in the second vessel. After the first
pouring it was increased by -^ x = 4 and becamc equal to 12 Íitres (as it hes been
found). Consequently, y — 12 — 4 = S.
Answer: 12 Íitres; 8 Íitres; 7 Íitres.
479. If for the first time x íitres of alcohol was poured out, then (64 — x)
64 —x
íitres of alcohol was left; for tho second time —^ — x Íitres of pure alcohol was
_x |
poured out, leaving 04 — x ------- — x = ^ (6 4 — x)2 Íitres of pure alcohol.
C h a p ter V II, A lg eb ra ic and A r ith m e tic P ro b lem s 19 7

We get the following equation

± (0 4 -* )* = «
Answer: for the First time 8 litres of aicobol was poured out and for the
second, 7 litres.
480. Having poured x litres of alcohol into the second vessel and made
it fuil by adding water, we have in the second vessel ^ litres of alcohol per

litre of the mixture. Then x litres of the mixture, containing


2õ-r = 55 lilrt's
of alcohol, is poured back. As a result, the íirst vessel now contains | 20 — x j
9
litres of alcohol, Then 6 — litres of the mixture is poured out from the íirst

( 4 , )
vessel 1 - make ^ of the total amount of the mixtureJ . Thus, the annuint

of alcohol is reduced hy , i.e. now the First vessel contains ^2ü — x-f-— j
litres of alcohol. Since the amount of alcohol containcd in hoth vessels is con-
stant and is equal to 20 litres, and hy hypothesis, hoth vessels now contain
the samo amount of alcohol (i.e. 10 litres each), we have

t ( 20- í + s ) = ,°
Answer. 10 litres.
481. Let x litres of air be let out of the vessel, and the samo amount of nitro-
gen put in. The remaining amount of air of (8 — x) litres contains
(8 — x) 0.16 litres of oxygen. Thus, 8 litres oi the mixture contains this amount
oí oxygen, i.e. 1 litre of the mixture contains — — ^ ■— -■■■1 of oxygen. Conse-
quently, when for the second time r litres of lhe mixture is repiaced hy x litres
of nitrogen, the remaining amount of the mixture oí (8 — x) litres contains
.( 8 —x) (8 — x)~ 0.02 litres of oxygen. Hence, in relalion to

the total amount of the mixture (8 litres) the oxygen content is— — ^ x
8
X 100 — . By hypothesis, .... 9. Out of the two roots (x, 2,
x2 ~ 14) only the íirst one is suitablc, since it is impossible to let out mure
than 8 litres.
Answer: 2 litres.
482. Let the first woman have x eggs, and the second one, y eggs. If lhe íirst
had sold y eggs, then, by hypothesis, she would have received 72 roubles. Guise-
72 79
quently, she sold her eggs at — roub. per piece and received ~ x roub. Reasoning

in the same way, we find Ibat the second woman received — y roub. Thus we
198 Answers and Solutions

have two equations


32 72 , 4ftA
— y = — x ; x - f u = i0 0
x y

From the first we find j , whence -“ ■==— ^the negativo value

is not suitablej .
Answer: the first had 40 eggs; the second, 60 eggs,
483. With the notation of the preceding problem we get the following
System
x
m— X ; * + , = «.
J
From the first equation we find x : y~~\/n: ~\/m. Divide then a into parts
proportional to ~/n and ~[/m.
2 ~\/~n a ~\/m
Answer: the first had litres; the second, litres.
1 /m + V n ’ Vm + V n
484. Let the first engine consume x grams of petrol per hour, and the
second, y grams; then 600 grams of petrol was consumed by the first engine
in 559 hours, and 384 grams, by the second in hours. By hypothesis,

595
x
_ 35í
ij
==2. If the first engine had consumed y grams of petrol per hour,

then during 559 hours of operatíon it would have consumed . y grams of

petrol, and if the second had consumed x grams per hour, then during —
384
hours it would have consumed — -x grams of petrol; by hypothesis,
OOOy 384.r
x ~ y
Answer: the first engine consumes 60 g/h; the second, 48 g/h.
9
485. Supposc we need x kg of the first alloy. Then x kg will contain ^ x kg
3
of gold, and (8 — x) kg of the second alloy will contain ^ (8 — x) kg of gold.
5
By hypothesis, 8 kg of the new allov inust contain kg = 2.5 kg of gold.
2 3
Conscquently, y i (8 — x) = 2.5. Hence, x = 1 (kg) and 8 —x = 7 (kg).
Answer: 1 kg of the first alloy and 7 kg of the second.
486. See solution of the preceding problem.
Answer: 9 pails from the first barrei and 3 paüs from the second.
487. Let the third alloy contain x parts of the first and y parts of the second
alloy, i.e. x kg of the first and y kg of the second alloy. Then (x -f- i/) kgof the
C h a p ter V II. A ly cb ra íc and A r ith m e tic P ro b lem s 199

/ 1 2 \
third alloy wiil contain ( "3 •'‘ + - 5'# ) kg °f ^ie ^ rsl nudal and kg
( W » )
oí the second. By hypothesis,

(- M !/): ( 4 i + 4 í/ ) = i 7:2 t
Reducing the dividend and divisor Io a common donominator (15) and dividing
them by y, we gel

( 5 7 +GM i0 7 + 9 H 7:27

whence — =
tj
í3o •
Ansu-cr: 9 paris of the íirst alloy, and 35 paris of the second.
488. Let the larger wlieel make x revolutions per minute, and lhe smaller
one, y r.p.m., y > x. We have two equations:
_5 J_
y — x — 400;
!/ ’ bO
The sccond equation rnay be transfonncd to x y — 300 (y — x), i.e. xy -■ 120,000.
Atmcer: The larger wheel makes 200 r.p.m.. the smaller one, 000 r.p.m.
489. Let the circumference of the front wlieel be equal to x dm, and that
of the rear wheel, to y dm. We have two equations:
180_180__|0 anâ 180 180 ^
x y x -|-6 y — ü
The first one is transformed to 18 (y — x) — xy, the second, to 39 (y — x) —
— xy -f 504. From them we find y — x — 24; xy -- 432.
Answer'. the circumference of the front wheel is 12 dm; that of the rear wheel.
36 dm.
2
490. 000• -77- — 400 tons was unloaded during lhe first and the third davs;
000 tons — 400 tons == 200 tons was unloaded during the second day. Let x
tons be unloaded during the first day; then (400 — x) tons was unloaded during
the third day. The rcduction of the amount of goods unloaded on the second day
(asctmipared with the first day) camc to (x—200) tons, whichmade ——2^ 21122.%
of the amount unloaded on the first day. The rcduction of the amount of goods
unloaded on the third day reiative to that on the second day wa>
200~{4G0 — x) — (x~-200) tons, which rnade ®o or %
200 2
oí the amount unloaded on tlie sccond day. By hypothesis,
x — 200 ^ (x —200) 100 5

We find two roots: x, — 250; x2 = 160. The second one is not suitable, since,
by hypothesis, the amount of the unloaded goods was reduced from day to day,
whereas at x — 160 the amount of the unloaded goods would be 160 tons 01,
the first day, 200 tons on the second day, and 240 tons on the third.
200 Answers and Solutions

Answer: 250 tons was unloaded during the fírst day, 200 tons during the
second, and 150 tons during the third.
491. Let the first solution weigh x kg, then the second weighs (10 — x) kg.
The percentage of anhydrous sulphuric acid in the first solution is =»
80 , „ . k. . 0.6-100 60 ,
= —- and that m the second, -jrr------ = tk------ * Bv hypothesis,
x 10 — x 10 —x *
80 60
= 10
x
The equation has two positive roots rj = 20 and x2 — 4. Since, by hypothesis,
x < 10, the first solution is not suitable.
Answer. 4 kg and 6 kg.
492. The first alloy contained x% of copper, the second, (x -f- 40) % . The
first alloy weighed .. kg, and the second, kg. We get the foílowing
.• 600 , 1200
e q u a tio n :------ \-- = 50.
x-{-40
Answer: 20% and 60%.
493. Let the speed of the freight traio be x m/s, and that of the passenger
train, y m/s. In 28 seconds the freight train covered 28x (m), and the passenger
traín, 28y (m); we obtain the first equation
28x + 28y * 700

The freight train passes the signal lights during -1— seconds, and the passenger
210
train, during - — seconds. Thus, we get the second equatitn

x y
Answer: the speed of the freight train is equal to 10 m/s, i.e. 36 km/h and
that of the passenger train. to 15 m/s, i.e. 54 km/h.
494. If the mimbcr of the eight-wheel tank-cars is equal to x. then that of
the four-wheel cars, to (x -f- 5). If one fonr-wheci car weighs y tons. then one
eight-wheel car weighs 3y tons. The net weight of the oil contained in a four-
wheel car is equal to (40-0.3) tons, i.e. to 12 tons. An eight-wheel car filled
with oil weighs (3y -f- 40) tons, and a four-wheel car, (y - f 12) tons. We have
the first equation
x (3y + 40) -f- (x + 5) (y + 12) = 940
The weight of oil contained in all the eight-wheel cars is 40x tons, and that of
all the loaded four-wheel cars is (x 5) (y -j* 13) tons. Thus, we have the second
equation
40x - (x - f 5) (y - f 12) - 100
Answer: there are 10 eight-wheel] tank-cars, each weighing 24 tons, and 15
four-wheel cars, each weighing 8 tons.
495. Let the first machine drive x in per day, and the second, y m. In th8
first case the first machine would have done 30% of the work, i.e. it would
have driven — 18x (m), and the second, = 16y (m). We have
Chapter V II. Algebraic and Arithmetic Problems 201

the first equation


18z + 16y = GO
In the second case the first machine would have driven y 6 0 y (in ) in ~ 0O--~

days. The second machine would have done the work in 2L-60-— da\». And
10 y
so we have the second equation
4 0 y _ l 8x
x y

The obtained system is easily solved, if we put ~ ~ z . Only the positivo value
3
z — — is suitable.
4
Answer: the first machine drives 2 metres of the tunnoi por day. the mcoikI.

1-tj- metres per day.


496. Let the first crew be able to repair the whole section of the travk in
x days, and the second, in y days. Dy hypothesis. we have the íollowing »y»tem
oí equations
i+ L
* y
40x 40y
100 “ 3UU ~ ^
Answer: the first crew can complete the whole repair job in 10 days, the
second, in 15 days.
497. Let the first portion of the goods |araounting to — .690--= 375 toiis|
be transported in x hours, and oach thrcc-ton truck accomplish y trips por hour.
Then each l y t o n truck made (y 1) trips por hour. By hypothesis. the romaín-
ing portion of the goods (i.e. 690 — 375 ~ 315 tons) was transported in (x —- 2)
hours, the three-ton trucks making (y -f- 1) trips per hour, and 1^-ton trm ks,
(k + 1) 4- 1 = (y + 2) trips per hour. Wo get the system of equation»
5-3XJ+ 10-i 4 --r (!/+ l) = 375

\ 5 .3 ( x - 2 ) ( » + i) + 1 0 .1 - i ( x - 2 ) ( ! , + 2) = 315

After simpíifications these equations take the form


f 2xy-{-x —25
\ 2xy-\-3x— 4y —27
Subtracting the first equation írom the second one, we get 2x — 4y ~ 2. iionco,
2y ~ x — 1. Substituting it into the first equation, we get z- = 25, i.e. x — 5.
202 Answers and Solutions

The íirst portion of the goods was transported in 5 hours, the second, in 5 — 2 —
=- 3 hours.
Answer: ali the goods were transported in 8 hours; at the beginning the three-
ton trucks made 2 trips per hour and 1-^-ton trucks, 3 trips per
iiour.
498. If x is the width of the track, then the area of the sports ground together
■\vith the track, is equal to (a 4- 2x) {b + 2x) m2. And so we have the equation
{a -f- 2x) (b -f- 2x) = 2ab.
Answer: ™ [ ‘| /( a - j~ ò ) 2 4 -4 a 6 — ( a - j - b ) ] .

499. Let x denote the number of chairs in each row; then the number
o f rows is — . We get the equation
x
= l.t a
(x + b ) Í T
After simplifications we have
10cx2 -}- (a - f 106c) x — 10aò — 0
Hence»
— (a + iOfce) ± V ( ^ + Í06cí2 + 400a6c
20c
If the radical is taken with the minus sign, then x < 0; if it is taken with the
pius sign, then x > 0.
Answer: the number of chairs in each row is equal to
V (a + lOfrc)2 -f-4Q0abç — (a - f lQòc)
20c
500. Let us denote the speeds oí the bodies (in m/s) bv n, and iy, let t>t be
higbcr thau r2. The first condition yields the equation* avt 4- avz = d; the
second, 6r, — bv2 — d.
Answer: t-j ^ ~ 4 ~ ™ ^ ; i/2 = -íL^-i------L j . The problem is solvable only
if a < 6 .
501. Let us denote the speed of the motorcyclist (in km/h) by x, and thai
o í the cyclist, by y. We get the following system of equatious
o , n , d d
2x>t-2y = r f ; ----------
V x 1 ______

Answer: the speed of the motorcyclist is equaí to d ' - ^ Í ± ^ ! £ ± f km/b,

and that of the cyclist, to d V m + t " km/h.


502. If it takes the cyclist x hours to cover the whole distance, then the
hiker requires (x-f-c) hours. Let us denote the distance AB (say, in kilomet-
res) by y. The hiker coverod km bofore lie met the cyclist, wbile
Chapter V II. Algebraic and Arithmetic Problems 203

one positive and one negative root (since the product of the roots is equal
to the negative number — bc). Here only the positive solution is suitahle:
— c - f V t a -r26— c)2-\-4bc
2
The distance y romains undctennined. The quantity x + c may be found either

tive solution.
Ansuer: it takes the cyclist
g+ 2fr—c-f-V(a + 2&—c)2+ 46c |[ourg

and the hiker


a + 26 -f" c -)- ~\f(a -\-2b — c}2- j- ^ c „ (a + 2í<*f~c) -}- 1/(a -f- ‘tt>+ c)2— 4 (« -f-foj c j

to cover the wholo distance AB.


503. Let us denote the distance (in kilometros) by x. By hypothesis, accor-
ding to the schedule traiu A must cateh up wíth train B in -j— hours aíter
departure. Actually, it caught up with traiu B after having covered (x — a) km.
i.e. in - — - hours. Conscquentiy, Loth trains travelled — hours less thau

required by the schedule before they met. Traiu li had to travei — hours to
v\
2
meet train A, but actually it covered a distance of ~ .r at a speed of r ( and
1 i
a distance of ~ r —a at a speed of rj, covering the whole distance in

Answer: the distance to the torminal station is equal to — — — — km.


y
The problem is solvable only if e j< 2 e .
204 Answers and Solutions

504. Let the interest be x % . Then the originally doposited sum was
i i í í ! roub. At the beginning oí the second year tha total sum was —- —

+ 154-85, i.e. | — 5 + 1 0 0 ^ roub. At the end of the second year this sum

turned inte ( - J - + 100) ( í + j^q ) r0BÍ>- Hence, we get the equation

( 1? 2+ 100) ( 1+55 õ) = 420


Answer: 300 roub., 5% .
505. Let us denote the output of machines A , B, C by x, y, z, respectively.
By hypotheais,
r(y + s), y= \ (x ~b2)
100'* f '* * 100'
We find x and y in terms of z from these equations; adding them, we get
, 100 {m -f- n) -f- 2mn
z ~r y 10 000 — mn 3

The required percentage is equal to — . 100.


, 10 000 — mn
Answer: 100 •- .---r-;--- ;— ——7:------ .
100 {m -f-n) + 2mn
5Ut>. Let us take for the unit of measurement the output for the preceding
year. Then the output for the fírst year is l + And compared witb it, the

output for the second year is increased by q% , i.e. by 11 ~f ” j ^ . to be equal

If the output for the third year is increased by x % , the increase amounts

t0 ( 1+ í ® ) ( 1+ i 5 õ ) i ® - B>’ h" othesis’

3r — p — q II
100
.•Insu-er:
( 1 + í®>) ( 1+ ÍBõ)
507. Let the prime cost of the total quantity of goods arnount to m
ma
roubles. Then the prime cost of the first bateh sold makes a% of m, i.e.
roubles. By hypothesis, the profit ma de by seiling this batch is p% of this
Chapter V II. Algebraic and Arithmetic Problems 205

sum, i.e. roub. The prime cost of the rest of tho goods is equal

to m— ~ - = m ( ^ ) roubles. The prime cost of the second batch sofd

amounts to 6% of this sum, i.e. to m ^1— roubies. The profit made


on seliiog the second batch is g%; consequently, this profit amounts to

m ^1 — y ^ j Í5õ‘ íl õ rou^ cs* The prime cost of tho remahmig goods is equai to

m- m - m( t - i ® ) i ® = m ( 1- i 5 õ ) ( 1-I 5 õ ) rou1'-
Let the remaining goods be sold at a profit of x \ . Then the profit rnado on
their selling amounts to m ^1 — y ^ j y^~ roub. The total profit ís

m[í5 õ " í ü õ ~^"( 1 ~Tüü) tüu' íoõ“• ' ( 1 TOõ) ( 1 _ r S ) ) í 5 i ]

By hypothesis, the total profit must be r"ó of m roub» i.e. -yyy roub. Con­
sequently,

LIOU 1UU ‘ \ 1 0 0 / 100 1 0 0 1 \ lü ü M lO U /lU ü J HK>

The quantitv m is reduced.


^ _ ± L ( i ___
~100 100 \ 100 0'
Answer:

( *-rô)(‘-râ)
508. First method. Let us assume that cach of the cut-uff pieccs weighs x kg.
For lhe sake of brevity, let us cali the first aüoy (weighing m kg) "alloy
and lhe second, “alloy B". Out of the two newly produced ingots the first one
contains {m — i) kg oí alloy A and x kg of alloy J3, and the second. .r kg of alloy
A and (n — x) kg of alloy B. By hypothesis, the copper content in both alloys
is the sarne, which is possible only if the amounts of alloy A and alloy //, contai-
ned in tlie new alloys, are proportional. \Ve get the equation
m—x mn
whence x
x rn + «

Stcond method. Let u kg he the wcight of copper in 1 kg of alloy A , and u —


the weight of copper ín 1 kg of alloy B. Then lhe first ingot contains (m — x) u -j-
- f ii» kg of copper, i.e. 1 kg oí the first ingot contains u3 ’ x0, kg of
copper. The weight of copper contained in i kg of the second ingot is expressed
in a similar way, Equating the two expressions thus íound, we get the equation
206 Answers and Solutions

in three unknowns (x, u, i>):


n {(m — x) u xv\ — m {(n — x) v -f- *«]

which may be translormed to


(u — v) (mx -f- nx — mn) — 0

By hypothesis, alloys A and B are of different copper content, i.e. the quan-
tity u — v cannot be equal to zero. Consequently,
mx + nx — mn = 0

Answer: each of the cut-oíf pieces weighs —^ — kg.


r m-j-n a
509. Let there be originally x t roubles in the first pile, x2 roubles in the
second and so on, and x n roubles in the «th pile. As is obvious, the first pile
is treated in a special way, since at first an nth part of the money is taken from
it and only by the last shifting operation an nth part of the nth pile is put into
it, whereas each of the rest of the piles first is enlarged on the account of the
preceding pile and then an nth part of it is taken away. Therefore, let us consider
any pile, exccpt for the first one. Let k designate its number. Originally. it had
Xh roubles, then some amount of y roubles of (k — i)th pile was put into it,
and, finally, an nth part of the total sura y - f x * was taken from it. After this
operation pile k had (y -f- xk) roubles. By hypothesis, we have the equation

+ = 4 (*>

A roublesmust remain in the preceding {k — l)th pile, if itis not the first
(i.e. if k =£ 2) (the money in the first pile amounts to Aroubles onlv after it
is replenished from the «th pile). Hence, prior to the shifting operation it had
A -}- y roubles. By hypothesis, the money taken from it makes an «th part of
A 4- y, i.e.
y = k (A + y) (2)

\
Hence, y --~ — j A . Substituting it into (1), we get =
Tlms, each of the piles, except, perhaps, for the second and first (previously
cxchided from consideration) originally had A roubles:
x3 — j 4 — . . . ~ xn ~ A (3)

The unknown .rj may be found in the following way. By hypothesis, at first
an nth part is taken ont of the amount of zi roubles. There remains Xi n ^ ■ rouh-
les. At the end of the shifting process a certain amount of money (y roubles)
from the last pile is put into the first pile. We obtain the equation

(4)
Chapter V II. Algebraic and Arithmetic Problems 207

Reasoning (coníormably to the nth pile) in the same manner wo find as


before y —- — ^A. Substituting it into (4), \ve get

_ (n — 2) n
A, (5>
1 ( n - 1)2
to find *2 we have the equation

( i *1+* * ) ' l ^ — a (6>


where is determined by lhe formula (5). Solving the equation, we find
n (n — i) — (n— 2) ,
X- < « - 1)2
y4nsit’er:
n2 — 2/i »2— 2w+ 2
PART TW O

GEOMETRY A N D TRIGONOMETRY

CHAPTER VIII
PLANE GEOMETRY
510. Lei a and b be the legs of lhe right-angled triangle and c, its hypotenuse.
Bv hypotbesis, a -f- b - f c = 132 and a2 -f- ò2 - f c2 — 6050. Since ar -f~ 62 =
= c2, t-hen 2c2 = 6050, whence c — 1/3025 = 55. Therefore a -f- b = 77.
Squaring this equality and taking into account the relation ar - f b2 = 3025,
vvo get ab ~ 1452. Consequently, a and b are the rooís of the equation
x2 —- 77x - f 1452 — 0
Answer: the legs of the triangle are equal to 44 and 33 respectively, the
hypotenuse, to 55.

511. The altitude BK (Fig. i) of the parallelogram ABCD is equai to


20A'=5=2p. Since /,BAK — <Zi AB = — -,
A ^ — . Similarly, AD — A - - ■ We find:
r csin
,n "a sin a
Amp
> = A D -B K =
sin a
The diagonais are found by the law of cosines.
Answer: S=
sina
2 1/ p 2 -f- w2 — 2mp cos a
BD--

2 *\/p2 4- m --f- 2mp cos a


sin a
512. By hypotbesis, A C = 30 cm and BD — 20 cm (Fig. 2). The altitude AB
may be found proceeding from the similarity of the right-angled tríangles BDC
C h a p ter 1 7 //. P la n e G eo m etry 209

and AEC (havíng the common angle C)> or. which is easier, by comparing two
expressions of the area S oí lhe triangle ABC. Namely,
S —- — AC ■ÍW and S - BC-AE
Hence,
AC-BI) 30-20
^24 cm
1 BC

Answer: 24 cm.
5Í3. Frorn the triangle BDE, where B D — 12 cru and BE 13 cm, \ve find
DE — \' \'Sl — 122“=5 (cm) (Fig. 3). Consequently, AD — AE — DE — — AC—

Fig- 3
- Z ) £ =--4- 6 0 - 5=.25 (cm) and DC~EC- -D E —. 35 (cm). The sides are íound
from tlie trianglcs ADB and DCB.
Answer. A// —*\/7ü9 27.7 cm, BC - V13Ü9-.37 cm.
514. Lct ABC be the given triangle
(AC~ CB — b). It is required to deter­
mine the area S of the triangle OtO-,0:i
(Fig. 4).
We have O^O^-OiC, where 0.^ 3—

AB and 0,C = AB. Hence. S <™1 Alfi .* IA.


Alternate sotulion. The triangle
is equal to the triangle O^BC. since they
liave the common base OtC and equal
altitudes. The triangle O^Ü^C is equal
to the triangle 0,AC (íor the same rea-
son). Hence. lhe triangle 0 t(920 3 is equal
to the square 0\BCA.
Answer: S h2.
515. J3y hypothesis, lhe line-segment
AB - a is divided by the point M in
tho ratio tn : (Fig. 5). Therefoie AM ~
same wny
BS ~ CK - DL = and SC — K D LA -
’*+ «
1 Üí 338
210 Answers and Solutions

Consequently,
m 2fl2 n 2a2
LM = MN = NK = KL =
( m - f n)2;+' (m-f-n)2
r m -f-n ‘]/m 2-f-n2
Furthermore, all the angles of lhe quadrilateral LM N K are the right ones (since
the triangles A LM and BMN are congruent, we have /, LM A = ^M N B=9Q ° —
— /.NMB\ hence, ALMA + Z.NMB = 90°; tkereíore /_LMN= 90°).
Consequently, the quadrilateral LM N K is a square.
fl2 (w2-f-*2)
Answer: S •
(m - f n)2
Alternate solution. Subtract the total area of the four triangles írom the
area of the square ABCD.
U H C

A L ^ -^ -M L and A M -^ X ^ M L
Hcncc,
AB = rl/V + /Vi? = AM + AL = - i (1 - f y j ) ML
Consequently,
area ABCD : area LMNK = A & : ML"- = (1 + V 3 ) 2 :4 ,
4
area A' = ^ + y ^ 3 area ABCD

Answer: the ratio is - j ~ 2 (2 — V -í) ^=0.54.


U + V M
517. Let us denote AM (Fig. 5) by x, Then AL — M B ~ a — x. Consequently,
area KLMN ~ LM - ~ AL% -f- AM - = {a — x)2 -f-
25
By hypothesis, (a — x)2-j-x2 ~ ^ a 2. Solve thís equation.

Ansirer: the required seginents are equal to y and ~ .


518. A preliminary. It will become clear frotn the solution how to find the
position of the vertices of the inscribed rectangle KLMN (Fig. 6). At the moment
Chapter V I I I . Plane Geometry 211

it is necessary to carry out the drawing schematically, beginning with the con-
struction of the rectangle KLMN.
Soíuíion. Find the iine-segments MB — x and BN = y. Since AB 4,
A M — 4 — ar. The triangles DLK and BNAf are congruent (prove it!); conse-
quently, DL = BN = y and LA = 3 — y. The triangles LAM and MNB
are similar, since theír acute angles A LM and NMB are equal (as angíes with
mutually perpendicular sidos). And since, by hypothesis, ML is three titues
greater than MN, we have LA = 3MB, and, also, A M = 3BN, i.e. 3 — y -= 3x

3 VlOÜ
3,87 m.
8
519. The area of the equilatcral triangle ABC (Fig. 7) is equal to
a-. y A «2. The triangle ANL, in which, by hypothesis, AL a
2 2 4 •>
and AN 1 a has its angle A in common with the triangle ABC. Home,
3
their arcas are in the saine ratio as the products of the side>: ^ANL
S ABC
2
3 3 a
----- . Thercfore

s ANL —*31 2_
3 SABC ~ 7J" A«C
Hence,

S.
1/3
12

Note. The triangle LMN, as well as the triangle ABC, is an equilateral one
(prove it). The same method may be used for determining the area of the triangle
LMN in a general case, when the triangle ABC is an arbitrary one and its sides
are divided in arbitrary ratios.
. y i
Answer: —~ — .

14*
242 Answers and Solutions

520. We have (see Fig, 8) a + b - f c — 2p; hence a 4- b = 2p — c and


a2 -j- 2ab - f b- = (2p ~ c)a. But az -j- 6a = c- and ab = ch (see solution of
Prob!eni 5Í2). Therefore c2 -f- 2ch — Ap- — Apc -f- c~, whence

Now we have a-f-ò — and ab — r ~ ^ r > Hence, a and b are the roots
A-f- 2p h-\-2p
of tlio cquation
x * - -2p ( i - r P i . , 2pZh =0
h-f-2p h~~ 2p
2P*
Answer: c =
h -f- 2p

Í L _ |/í -f- p4- y (p — /i}^ — 2/í^]


- ~ 2 j V‘ + p - V ( P - A ) 2-2*3J

T lie p r o b le m is s o lv a b le o n ly if p > fc C ]/2 + 1 .

521. Either of the sidos AC and BC (Fig. 9) of the triangle ABC is equal
to -- p — a. Let x be the length of the line-segment CM (.r — CM-=CN).
The perinieter 2p of the trapezoid A M.XB is obtaincd from the perimeter 2P
of the triangle ABC by subtrncting CM - - í ’.V - 2.r from 2P and then adding
MX to lhe difference thus obtained. From the similarity of the triangles ABC
and MA C we find
AB-MC 2ax
M X - -----— -----
Aí P —a
Hence,

whence
{P -Q ){P -P )
P — 2a
(P -a )(P -p )
Answrr: CM - CN P — 2a
C h a p ie r V I I I . P la n e G eo m etry 2 13

522. It is required to determine thc dístance NP —x botween the point N


(Fig. 10) and the base AD=*a*, and lhe distance NM = y between the point .V
and the sido AB — c. From the similarity of the triangles AMN and ABC

Y \

y ?
X \

/ P

Fig. 10

, , AtN AM . y
(where B C~b) we fmd ’ ,'e‘ T and from the similarity

oí the triangles NPD and BAD we have , i.e. — = - — - Then we


BA AD c a
sol ve thcse two oquations.
ac a6
Answer: x---------r ; y - - —r~r •
a <b a-j-o
523. Let ABC (Fig. 11) is the given triangle. Since DE is a midline of
lhe triangle and D E --C D , we have CD — -i- AC. Consequently, i. CAD —30'.

Thcreforo CD .- C D lÍ = 2 1 / 3 cm.

Answer: S 12 V 3 cin2.
524. i»ut x 130, y - AO (Fig. 12).
Then the aroa S of the rliombus ABC D
is equa! to 2xy. By hypothesis, x y =■-
~ ; besides, from lhe right-angled
1 p
triangle AOB, where A B ^ -j-2 p —~,\\c

find jr2 ( yz r“ ^-Tr] •Squaring both mera- Fig. 12

bers of the first equation and subtracting the second one, we find 2xy - '■■■■■.H ff. ,

Answer: S = cm2.

• The solnüon is indepeudent of whother a is thc larger or the sinaller base.


214 Answers and Solutions

523, Let x denote the altitude BE (Fig. 13). Then AE = x and FD — x “\^Z.
Since AD = AE + E F + F D , «= *+ »+ * yã . Hence, a~ b
V3 + 1
,( o - 6 ) ( V ã - l )

(q2^62) ( y 3 _ j )
Answer: S —
526. By hypothesis, AD = 44 cm and BC = 16 cm (Fig. 14). Hence, A E -f-
-f- FD = 28 cm. Denoting the length A E (in centimetres) by x, we have FD =

— 28 — x. By hypothesis, AB — 17 cra and CD — 25 cm. Consequently, BE2 —


= 172 — x2 and CF2 = 252 — (28 — x)2. And so, we get the equatioa
17a _ z%= 25a — (28 ~ x)z
whence x — 8 (cm). Hence, we find the altitude h:
k^zBE — 'y'172-—x2— 15 (cm)

527. Denote the sido of the inscribed square (Fig. 15) by x. From the
similarity of the triangles AKL ^wherein AK — ^ ^ 1 and LK —
Chapler V III. Plane Geometry 215

i V 3\ ... .. (
and 4E£^wherein AE 2 and ~~~~
BE = —^— ) we get the equation •

_a 1 /3 a V3
wherefrora we íind = «y 3 (2 -y ã ).
2 " " " ,,UM * " 2 + 1 /3 "
dnsuwf: 5 - 3a2 (2 - V 3)2 - 3 (7 - 4 V 3) a*.
528. By hypothesis, AD — 36 cm and DC = 14 cm (Fig. 16). The areas
.S'j and S2 oí the triangles ADB and CBD with a common altitude are in the
same ratio as the bases, i.e.
S , : S í = 36:14 = t?

Consequently, 5 1=*™-5\ where S — Si -f- S2 is the area of the triangle ABC.


By hypothesis, the straight iine EG divides the area S into two equal parts,
wiiich means that this Une intersects the base AC betweeu the points .4 and D

(but noí between D and C). We get the triangle AGE, whose area S3 is equal
to ~ S . Sinco the areas of the similar triangles AGE and ADB are in the same
ratio as the squares of the sides AG and AL), then
*a 4
g S :4 r S ^ 3 W :A G 2
25 2
whence we find
AG — 30 (cm).
Iience,
GC ~ AC ~~ AG (36 + 14) ~ 30 = 20 cm
Aruswer: 30 cm and 20 cm.
529. Sec the soiution of the preceding problem. From the comUtion
AD : DC — 1 : 8 we find that the area of the triangle BDC (Fig. 17) is -jj- of
the area S oí the triangle ABC. Since, by hypothesis, BD — 4. we have

e í * :l G = ~ S : .| .S
Ansuter: EF ~ 3 .
530. Since SE}iF —su £ Pa ~ SAj)Gc (Fig. 18), the area of the triangle EliF
h half the area of lhe triangle DBG and three times as smail as the area of
21(5 Answers and Solutions

the triangle ABC. Since the.se triangles are similar, EB2 :_DB2 : AB2 = í : 2 : 3.
By hvpothesis, /!/í-= a ; hence, £ /? = — = and B B —-l—^X .
y 3 '3
Answer: the sido AB is dividcd into the following parts: T
\ 3
^ (y ^ D a n d ^ U /ã -V I ).

531. By hypothesis, the area of the triangle ABC (Fig. 19) is cqual to S,
and lhal of the triangle KBM, to q. Three vértices of the quadrilateral coincide

with the points A\ B , and M: and the fourth vertex I. may he arbitrarily takeri
nu the sido Imteed, the area St of the quadrilateral LABA/ is the sum of
the area q <»f the triangle A'BA/ and the area of the triangle KLM . and the iatter
reinains muhanged as the vertex L moves along the straight. iine AC parallel
to the base KM. Let the altitude BE of the triangle ABC pass through the point
E of tlie base /H\ 1’ lacing the point L at the point \ve get the quadrilateral
KBME. whose diagonais are mutually perpendicular; consequentiy, Sx
,4 KM-HE. And íim f ? - i - K M -B D . ,V, : q BE : 111). But íincc tlie
triangle ABC is similar to the triangle K B M , we havo S : q BE- : BD-.
Consequent ly,
BE
Sr --q.
BI) 7 HL \/Sq
<i
Xote. If the point I. does not coincide with the point E. tlie solution is inodi-
fied: fiml
■S, 4- A M /-B I ) + 4 A M /.XI . ~ 4 A 'U <BD + 'V i > - 4 S M -BE

and then pmcoed in the same way as ahove.


.t//.u/vr: \ Sq.
532. Let the Jine-segment EF x (Fig. 20) divide the area of the trapezoid
ABCL) (AD «, BC - b) into two equa! parts. Then
(« -L- J-) EL (x b) FM
_ - •• 2 (a -\-x) E L-- (x-\-b) FM
C k a p ter V III. P la n e G ea rn etry 217

The altitudes FL and FM cannol be found separateiy (the length of one oí them
may be taken arbitrarily), but the ratio FL : FM is of a definite value. Namelyr
froin the sirnilarity of the triangles HFD and CFK (wherein IIL) = a — x
and CK — x — 6) \ve find
a —x x — b
TT^TÃT
Multiplying this equality by the preceding one, \ve gel
a- — j 2 — z2 — b2
xvhence

/
a2~\~b2

Alternate tnethod. Extending the nonparallel sides, ve get the similar triangles
BGC, EGF and AGÜ. Their areas Sx, E3 are proportional to the squares of

corresponding sides b, x, a so thal St ~~ qb': , S2 :: qx-> S3 =- </«-. vliere q is


a ccrtain coeííicient of proportionaiily, whose magnitude depends ou the alti­
tude of the trapezoid. By hypothesis, S» — Sx -V — F-: , i.e.
q (x* — b-) = q {a2 — xl)
and since q =/-- 0,
x- — b2 -- a2 — x‘

b
533. By hypothesis, B E -B F ~ (Fig. 21) and EF~~b. Ilence, EG and

BC
By the theorem on proportional line: in the right-angled triangle (BDE)
ve ímd, BB=
n r> BE* a*
■■■■■,.= ................... — Now vo find the side of the rhombus
BC _ f . t b
/ • M t f
{AD). The isosceles triangles ABD and BEF are similar, since their angles
(ali of them are acuto) are respectively equal {as the angles with mutually
perpendicular sides). Consequently,
AD : BD = BE : EF,
218 Answers and Solutions

AD : -a :6

/ a2- ( ! ) 2
wberefrom we find AD and then the area of the rhombus S = A D ‘ ã.
, 2a4
Answer: ■■■ ■ ... -.t—- .
b -|/4a2 — 62
534. Let AB = 27 cm and AC — 29 cm (Fig. 22); then the median AD ==
= 26 cm. Extend AD as iong as DE — AD . The quadri-
lateral ABEC is a parallelogram (prove it!> witk sídes of
27 and 29 cm.
The area of the triangie ABC constitutes haif the
area of the parallelogram ohtained, on the other hand,
the area of the triangie ABE is also equal to half the
area of the parallelogram ABEC. Consequently, the area
of the triangie ABC is equal to the area of the triangie
ABE, vhose sides are known {AB = 27 cm; BE = 29cm;
A E — 52 cm). Now the area of the triangie may bo
computed by using Heron’s formula:

*5= \ P ÍP ” 3 ) ( p — b) ( p — c )

i4«su?er: 270 cm2.


535. By the law of cosines a- 62-f-c2— 26c cos A, and
1 , . , . . . 25 4
since S : r-Tr-bcsio^, i.e. sm A = —— = — , we have
2 oc o

cos A = ± '\/i — sin2 A = ±


Fig. 22
We get two Solutions; both of them are suitable (in one case A is an acute
angle, in the other it is an obtuse one).
Answer: a = ~^/'624-c2 — ~ 6 c or = 62- f c2-J--|-6c.
536. From the triangie ABC (see
Fig. 23) we have:
m? — 6" ~j- c~ — 26c cos B
and since cos B = cos (180° — A) -
= —cos A , then
m~ = 62 - f c- 26c cos .4
From the triangie ADC we fínd
m3 — ar d’ — 2ad cos D

Equating this expression to the preceding one, we get


26c cos A -f- 2ad cos D — a2 — 6a + d2 — c2 d)
Chapter V III. Plane Geometry 219

In the same way, considering the triangles ABD and CBD, we get
2ac cos A + 2bd cos D = a2 — b2 — (d2 — c-1 (2)
Equations (í) and (2) yield cos A and cos D , and then we íind nr and Proceed
as follows: multiply (1) by 6, and (2), by a, and then subtract the first equation
írom the second. We get
2 {a2 _ b-) c cos A = (a2 - 6a) (fl — 6) - (d® - c2) (a + b)
Díviding both members of the equality by (a2 — ò2) [^ o ], we obtain
, <P— c2
2c cos i4 = a — 6--------- r-
a —b
j\Tow we íind
íd2 —c21/, a (c2 — fr2) 4~b (t?2 — d2)
m2= &24" c24*(2c cos A ) b — c2 ~V ab ---- -—g— a —b
Similarly, we íind
d2— ,
2d cos £) =x a — b 4--------j

and then
n2 = bi + d2+ (2d cos D ) b = a ■— - ]a (°~ —

Note. The line-segment AD — o is smaller than the broken !ine ABCD.


Therefore the problem is solvable only ií a < b -\- c -\- d. But this condition
aione is not suíficient, which is seen írom the following. Let a > 6 and c > d
(if these inequaíities are uot fulfillcd then we can always change the notation
to make the inequaíities vaiid). Draw a straight line BL parallel to the side CD
to complete the parallelogram DCBL. Now we íind: BL — CD — d and DL --
.= CB — b. In the triangle ALB the side LA -- DA — DL — a — b is larger
than the diííerence oí the sides AB --- c and BL = d. Therefore another condi­
tion should be satisíied, namely a — b > c — d. If either oí the conditions is
not íulíilled, then at least one of the expressions obtaincd for m1 and n- wíJJ
turn out to be negative.
The two conditions a < b 4* c 4~ d and a — b > c — d are quite sufíicicnt
for the problem to be solvable. Indeed, the first condition may be written in the
íorm a — b < c 4* d. Consequently, we can conslruct a triangle ABL with
the sides AL -- a — b, AB — c and BL — d. Extonding the side AL by LD — 6
and constructing a parallelogram DLBC, we get a quadrilateral ABCD, which
is a trapezoid with the bases AD — a, BC — b and nonparallel sides AB ~ c
and DC — d.
a (c2 —•h2) 4- b (a2 — d2)
Answer: m2
a —b
a fd2 — b2)-j-b («2 — c2)
lí. — ------------------------ ■-----------------

537. For the notation in Fíg. 24, whcro /_A üO1, we have
BD2 - AD2 - f AB* - 2-BA-AD -cos 60° - a2 4- !/- - ab,
A Ci ~ a2 4" b'1 4- ab
220 and Solutions

Sínce AC is longer than BD, lhe given ratio -y is equal to ^ but not
BD2 \
to -j ç rT J • From the equation

Í - Y + 1 4 --
gt + bZ+ ab 19 \t I T ^ b 19
— ab 7 0r / a \2 a ~~ 7
\ t ; + 1 - t

\ve find ^ ~ ~ 2 an^ y - = -y - Both of these values give one and the same
jjarallelogram (we may alter the notation in Fig. 24, denoting AB by a and A D ,

Answer: the sides are in the ratio 3 : 2.


538. Let O be an arbitrary point within the equilateral triangle ABC
(Fig. 25). Join the point O with the vertices. The sum of the areas of the iriang-

les AOB, BOC and COA is equal to the arca of the triangle ABC. Denoting the
side of this triangle by ay and the altitude, by h, we get

(OK + OL + OM) -í- = y

Hence,
h = OK - f OL - f OM
539. By hypothesis, BC =- 47 m and CA — 9m
(Fig. 26; the drawing is made not to scale); hence,
BA 56 m. Consequenlly, A D 'A E — 9-56 — 504.
Let AD — x\ then DE = x + 72 and, hence,
AE — 2x -1- 72. From the equation x (2r -f- 72) =
—504 wo find x •— 6.
Answer'. AE — 84 m.
540. The problem is rcduced to finding one of
the legs of the triangle OAB (Fig. 27), given,the
hypotenuse OA — m and altitude BD Let us denote the larger leg by x,
and the smaller one, by y. The area of the triangle OAB expressed in two
C h a p ter V III. P la n e G e o m elr y 221

difícrcnt ways (see solulion of Problem 512) gives the equation xy — a— , i.e.
2xy = am; besides, x2 -f- y- — m-, Adding and subtracting these equalions by
meinbers, \vc gel
x -f y^= m'2--\-am
and
x — y-~~\/rn2 — am
Both x and y can serve as the required radius.
Answeri — ( |/»<2-{-a,n ~\~~ f mí—üm)* 01‘ -y (' V -\~urn—~\/tri-—am)-

541. Since the radius oí the circle is equal U> 13 cm and MO - 5 cm, then
MU ~ 8 cm. MC -----18 cm (Fig. 28). Let us denote MB by x. Then AM -•-25 — x.
.Since A M -M B ^M D -M C , we have
222 Answers and Solutions

Sínce A D ^ A B -sin-™ (from the triangle ABD), we have

cot ( « • - - £ ■
R : r —---------- :-----------
sin a

R
Answer: — = ------- ;----------- -
r sm a
543. By hypothesis, a = BC — 13 cm, ft = CA — i4 cm, c — AB =
— 15 cm (Fig. 30). Denote OE = OF by R. Tbe area of the triangle ABC is

equai to the sum of the areas of the triaugles BOC and AOC. Since the areas of
these triangles are equai to and , respectively, then

27/?
Sabc —

On the other hand, by Heron’s formula


SAnc - V 21 (21 - 15) (21 ” 14) {21 — 13) = 84 cm’-
Equate the two expressions for the arca.
Answer: R -- 6-^- cm.
544. In the right-angled triangle OEB (Fig. 31) the angle EBO is equai
to 60°. Therefore
o 2//
1 /3 1 /3
Hencc,

V 1 /3 / 1 /3
From the triangle ABD wc find
, i / í , , H Í 3 l Í ± 2) an() 4 £ ,= / ? ( y 3 + 2)
1 /3
Chapter VIJ1. Plane Geometry 223

hence.
AC = 2 f í { Y $ + 2)

Answer: AB = BC = ? ã l Y ^ + 2'1 AC = 2R ( V ã + 2).


1 /3
545. From thc triangie ABD (Fig. 32) \ve have

B D**l/BAi — AD*=\% cm
Since
BC-BD = BA\
then
BA2
BC = =50 cm
Bl)
Consequently,
AC = yB C Z -~ B A t^ tâ cm

Answer: the scmicircumíerence is equal to 20n.


546. Since the angles B, D, and E ©f the quadrilaterai ODBE are the ngh
ones and DO = OE (Fig. 33), this quadrilaterai is a square, The sought-for

arc DE is equal to one fourth of the circumícrence of the circle. Let tis denote
its radius by /?. From the similarity of the triangles ADO and OEC \ve have
AD OE
AO ~ OC
Since
AD— y A O * -O ü * = V 1 5 2 -B 2 ,
then
V 1 5 2 -B 2 R
15 ~ 20
Hence, /? —12.
A nswer: ün.
224 Answers and Solutions

547. The area S oí the quadriiateral ADEB (Fig. 34) is


S = SA{)C — S DEC
W c liave
- B D — Í2 cra2

To find SDEC, let us notice that the triangles DEC and DBC have the
common vertex D and one and the same altitude (not shown in the draxving)
1
and that SDfíC = — S [iC — G cm2. Consequently, SEEC : Q= CE : CB. The

Fig. 35

unknown segmenl CE is found proceeding from the pmperty of sccants drawu


from one |K>int (O . We have CE-CB --- CD-CA, whcnce C E - - — — . Hence,
CB
CE .CD-CA 2-4
n -:Ü- - 1.2 cm2
CB CBi 22 -}- ü2
d/;s?rer: .S 10.8 em2.
548. The a N of the triangle ABC (Fig. 35) is cqual to lhe product of
its perimeter 2a — 2 ya~ /;2 and — (r is the radius of the inscribcd cirele):

S (a-f- V a 2 + /;2) r
<)n the other hand,
S = ~A C -B G = ah

Kquating the two exprossions, we find


ah
r~ f í

'l he segrnent DE is found from the proportion


DE : AC ;-= BF : BG
Chapter V I I I . Plane Geometry 225

where
AC=?2a, BF = h — 2r and BG ^h
Note. We may find r in a diífereut way: the straight line AO is the hisector
of the angle A . Hence, the Hne-segments GO = r and OB — h — r are propor*
tional to the sides AG and AB, i.e.
r_________ a
h—r y z r fjfl
4 h (l
Answer: r —— — — -
-f- ífi d- ®

ya Z + Ifl + a h2
549. Since OB-OA = Ü C-00 (Fig. 30) and OB == (9C, then 0/1 = 0/?.
The opposile sides AB and CD of the quadriiatcral ABCD are equal to each
O

olher; hence, the given lengths {fim and 2.4m) beiong to the sides Aü and BC
(A D — fim, BC ----- 2.4m). The lincs BC and A D cutting cqual segments off
the sides of the angle AOD are paraliel, which means that the quadriíateral
ABCD is a trapezoid {an isosceles one). Frorn the similarity of the trianglcs
BOC and AOD we find
BO : AO — B C : AD
whence
B O AD 2J3
A0 = m
BC 2.4 =
hence, AB ?> m. Now find the altitude of the trapezoid

h= riK == V aia - ak* = j 2^ 2a m


Answer'. S = 10.08 m2.
550. By hypothesís, AB — tim, AC = 7m, BC = 9m (Fig. 37). Let IlA,
Bfí and Bc be the required radii of the circles with their centres at A, B and C.
15—0133»
226 Answers and Solutions

Tben RA 4- Rb = Rc •— RA = 7, Rc — RB — 9, wlierefrom w e lind th e


radii Ra , Rjj and Rc .
Ansurer: ItA = 4m, RB = 2m, R c — Um.
551. Draw OzE paraílel to AB and OzP parallel to DC (Fig. 38). By hypo-
thesis, AB CD. Denote CD by x. Then 0 ZP = x, OzE = ~ -x . From the
triangles Oi E 0 2 and 0\P0Z we have

0,0| = 0 ,£2 + -|-i2 and 0, 05 = 0 , / * + * 2

Equate these two expressions and take into account that


0% E = OtA — E A = OiA — 0 ZB =
= 5 — 2 = 3 cm
and, similarly,
OtP = OiC - f OzD = 7 cm
Then we get
9
9 + -^ -x 2 = 4 9 -f^ 2

whence x- — 32. Therefore


OtOl - 49 - f 32 = 81
Answer: 0 j0 2 = 9 cm.
552.
the centres of the circles is less
than the sum of their radii, but exceeds the difference between them, the circles
intersect each other; bence, they have a common exterior tangent and (no

common interior tangent. Put OiC ~ x and OzC = y (Fig. 39). We have
x — y — 0\02 — 21 cm and x : y = OtA : OzB = 17 : 10
Ansirer, OiC = 51 cm, OzC = 30 cm.
553. Two tangents to tho circJe Of {MD and M A) pass tbrough the point M
(Fig. 40). Hence, MD = M A. In the same way we prove that MD ~ MB.
C h a p ie r V III. P la n e G eo m etry 227

Consequently,
MN = 2MD = A M - f MB = /IB
To fínd AB draw the straight line 0 2C paraliel to AB. From the triangle 0 %
0 2C,
wherein 0 2C = AB, 0 t0 2 — R - f r and OjC — R — r, we get
AB= y (R -\ -r )* -(fí_ r p
or
AB = 2 ~ [/ fír

A n sw er: M N = 2 ~ \ / R r.
554. Let MN be a common tangent to the two circles (Fig. 41). SinceAA/ =
— MP =- MB, MN is the median of the trapezoid ABCD. We have MN —

— AB = 2 ~\/Br (see solution of the proceding problcm). Let us now find the
altitude BG of the trapezoid. According to the theorem on proportiona! lines
in the right-angled triangle (EAB),
we have
AB2
BE
But
\BE- =0 x0 2 ^ n-\ -r
Hence,
4 Br
BG
R-j r
8 {Iir )3 /2
Answer: S =
fí + r *
555. Denote the radius of the required circle hy x. Draw lhe straight líno
MN |[ AB through its centre 0 3 (Fig. 42). Since AB is perpendicular to the
radii OsA, OzB and O-J), then AM BN x, and. hence, O,Aí ~= R — x
and OzN ■■■■ r — x. Furthermore, we have Oiú3 — R -f- x and 0-,03 — r-j- x.
Consequently,
m o 3- y < * + * > 2 — (/ *— * ) 2 = 2 ym
15*
228 Answers and Solutions

similarly,
NOz = 2 ~/rx
And since U N — 2 V i r (see Problem 553), we have
2 1 /2 5 + 2 1 / 5 = 2 y m
whence
yn?
y í=
V iJ + T /r
Rr
Answer: the radius of the circle is
(V Ã + V r)2 '
556. Since S — ab sin C, whore C is the angle between the chords, the
1 I 9ç
problem has no solution for 5 > - - ab. If 5 < — ah, thon we find sinC = - ^ ,
l 1 ab
and there exist two triangles with the sides a and b and arca S: in one tri-
angie C is an acute angle, in the other it is
452
obtuse. In the íirst case c o s C = j / /~:
a262 ’
i 452
in the second case cosC
j/ : a2&2 ‘
Iíence,
C2 = a2 -{_ 62__ 2ab COS C = a* _j_Jj2 Z+: 2 —4S2
{the minus for an acute C, the plus for an
obtuse one). At S - j ab we get a right-angled
triangle, so that c~ ~ a 2jrb-. The radius of
the circle circumscribed about the triangle is
found by the formula R -- -
sin C '
o—!- £)" -+- \"\/fl2fc2
Answer: R = ...... . For S > ~ ab there is no solu-
45
4
liou, for S < — oh — two Solutions (tho minus sign if the angle between the

chords ís acute, the plus one if it is obtuse). At S = ~ ab we have one solution


(the chords’ are mutually perpendicular).
557. By hypothesis (Fig. 43), AiBi ~ = /?, A^R* — 04 = R \ 2 and ^ 3/^3=
s=a3^ i? The altitudes of the triangles OAtBit OA2 B 2 and OA3 B3 are
OCt ... l{ j ; QCz=r. OC3= “ , respectively. Hence, we determine
the areas oí these triangles. Then we find the arca of the sector OAtDBt\ it
is equal to one sixth of the area of the circle; therefore

^QAiDBi = ‘g’
Chapler 1 7 //. Plane Geometry 229

1 1
S im ila r ly , S0AzDB2^ -^ - nR 2 and SOA3Dl{3 = ' j nR 2. S u b tr a e tin g th e area o f

e a ch tr ia n g le fr o m th e area o í th e r e s p e c tiv o s e cto r, we íin d th e area of th e


segm en ts:

in V 3

II
\ 0 4

s , ^ m i( ± -
\4
1 )
2 /

í 1 /3
4
The a rca of th e p o r tio n of th e c iv c le e o n ta in e d h c lw e e n th e ch ords ,1 j i i £ a n d

5 5 8 . F o r d e t e r m in ín g t h e r a d iu s OK = r (F ig . 4 4 ) o f th e in s c r ib e d c ir c le lo t
S -■ pr (p is t h e s c m i p e r i -
u s n ia k e u s e o í t h e fo r m u la fo r t h e a re a o f th e t r ia n g le :
n te te r o f th e t r ia n g le ). B y h y p o th e s is , A D
DC 1 4 .4 c m ,
2 5 . 0 c m , th e re fo re
AC - 40 cm. Hcnce. A li --■■■ \/A j) A C — 24 (cm), DC - \ ÜC -AC — 32 {cm).
Consequenlly, p 4 8 cm and S ■ ■■■■ 384 cm2.
Amnvi-r: the area of the circle is equai to 04n cm-’.

5 5 9 . T h e l i n e LN j o i n i n g t h e p o i n t s o f t a n g e u c y o f t h e U v « p a r a l l e i U n e s AB
and CD { F í g . 4 5 ) í s t h e d i a m e t e r o f t h e c i r c l e . T h e r e f o r e t h e i n s c r i h e d a n g l e s
230 Answers and Solutions

LEN and LMN (and, simüariy, the angies MLE and MNE) are the right ones.
Hence, the quadrilaterai LENM is actually a rectangle. ABD is an equilaterai
triangle (sinee AB — AD and Z.A — 60°); the line-segment LN (the altitude
of the rhombus) is equal to the altitude of the triangle ABD, i.e. LN — -.
The area S of the rectangle is
equal to

•“ j£JV2*sin /, LOE —

= i.iiV 2 .5 m C BAD

(the sides of the angies LOE and


BAD are mutually perpendicular).

Hence, 5 — ~ | j 2 sin 60°.

Fig. 46 Ansaer. . .
Io
560. It is required to determine the area Si of the figure MCNF (Fig. 46)
and the area Sz of the figure KDNE (the areas of the figures KALG and LBMN
are equal to Si and Sz, respectively). Since, by hypothesis, AC = AR, then

B C

OC = 2-0.il/; hence, z OCM ^ 30°. Then ve have / MON = 18CP— 2•30° = 120°
and z KON The area of the quadrilaterai CMON is equal to Rz 1/3»
and the area of the sector MONF, to —-n /í2. Hence, — R2 V 3 — ;

simiíarly, Sz —-3^.1. jL . .

f?2 (3 V 3 — ji) . i?2(2 V 3 - j t )


Answer: St
3 ’ “ 6
561. Since z A =30= (Fig. 47), the altitude BE = h of the trapezoid is equal
1
to y AB- ^ie Property of the circumscribed quadrilaterai, BC AD =
Chapter V I U . Plane Geometry 231

==AB-\~CD = AB. Therefore

s ^ A Ê + £ £ . h= ± A B ^

Answer: AB=~[/2S.
562. Given the area 5 = 20 cm2 and altitude BE = 2r — k cm (Fíg. 48), \ve
find the half-sum o£ the bases ~ —5 cm. Consequently, AB — 5 cm
<see the preceding problem). Now we find ÂE — 'V AB Z— BEZ= Z cm. But AE

B C

is the half-diffcrence betwcen the bases of the trapezoid. K now mg the half-sum
and the half-difíercnce, we find the bases themselves.
Answer: AD —8 cm
BC ^ 2 cm
AB — CD = 5 cm
5(53. The area Q of the trapezoid ABCD (Fig.49) is equal to
BC + AD
B M = (B C -{-A D ) R

{fí is the radius of the inscribed circle). Since this trapezoid is circumscribed
2fí 2R
about the circle, BC-\- AD ~AB-\- CD. But AB = and C D-
sin p
Therefore

ç = 2 f l * ( - 4 - H— ) = 2 a + 3m =
\ s m a 1 sinp / stnasm p
. a -fp a —-B
4/t2 sin •— cos —
sín a sin p

sin a sin p
Answer:
2 V Sin^±
564. Since lhe lateral side AB (Fig. 50), perpendicular to the bases, is equal
to 2r, the inclined side CD is greater ihan 2r. Consequently, the least side of
232 Answers and Solutions

3
the trapezoid, equal to y r, is the (sraaller) base BC. To find the larger base
AD draw straight lines OC and OD which are respectively the bisectors of the
angles MCD and NDC, whosesum amoimts to 180'. Hence, £M CO /_0DN ~
— 90’ . From the right-augled triangle ODN we find ^NOD - f i_ODN — 90°.
Cunsequently, ^NOD — / MCO, and the triangle ODN is similar to the triangle
OCíM. We obtain the proportion ND : ON = OM : MC, where ON — OM ~ r
and MC - <by hypothesis). Hence, ND = 2r, and AD = 4A r 4- ND ■
~ r ~}- 2r ;
9r2
Answer: S =
z
505. The triangle OMC is similar to the triangle OND {Fig. 50) fsee the
OD 4 ND ON
prcceding problem). Since y - 2, then - ^ = 2 and -7777— 2, i.e.
OC OM “ “ “ “ MC
ND 20M ----- 2r and MC =
ON 1
r. From the right-

angled triangle OND we find


r2 -j- (2r)2 = 42, whence,

T=N Í <Cm)
Now we find AD = AN ND =
12
=r -f-2r = 3r _ Ü _ cm and
1 /5
BC J L cm. The altitude
Vs
MN of the trapezoid is equal to
, 8
2 r~ — =- cm
1/5
Answer: S = 14.4 cm2.
Fig. 51 500. The centre O of the firsfc
circle (Fig. 51) divides the alti­
tude 13N - h in the ratio BC): ON = 2 :1 . Consequentlv, the diameter MN is equal
to ~ /* a n d hence, B M k . The socond circle is inscribed in the triangle
BDE, whose altitude is equal to one third of the altitude h of tlie triangle ABC.
Ilencc, the radius r^-^O^M is one. third of the radius r -~ON. Therefore,
if S is the area of the circle O j" S™ n ^ ----- j — ~ y J , then the area of the

c ir c le Oj w ill be St -- —^ S. And s in c e th e r e a ro th r e e s u c h c ir c l e s , th e ir to ta l

area (>t will be


Chapler V I I I . Plane Geometry 233

Reasoning in the same way, we find the total arca of the next three circies
1 i
^ 2“ 3 T ^ i== ant* 30 on-

Tlius, we obtain an infinite sequence of addends

S ~f- Q t + Q 2 t *?3 H~ •■ - — $ + - j S + 33" $ - f - p - S ~r *■ •

The terms of this sequence, beginning with the term (the addend S is
treated separately), form an infinitely dccreasing geometric pvogression
9 = -^ *) • The sum of this progression is equal to

l -< 7 _ _8 8
9

To get the required area the addend S should he added to the abo ve sums.
Answer: the required area is equal to
567. To find the area of the trapezoid
BMNC (Fig- 52) it is required to find
the base BM and altitude MN, since CN
is known. First determine CD — x. We have
x{BC-\-x)--=AD*
or
x (5 -|-1) = 150
Hence,
CD — x — iü (cm)
From the siinilarity tof the triauglcs
HMD and CND it follows that
BM CN BM 6
whence BM - 9 (cm). The altitude MN is found
BD ~1 CD 0r 15 ~ 1 0 '
MN ND
from the proportion HC " ~ “ ~CD~
CD *’ w*,ere N ü = yCÜZ — CN-. We get
MN —4 cm-
Answer: S —30 cm2.
568. Let O|, 0 2 and 0 3 be the centres of equal inscribed circies and let r
be lheir radius (Fig- 53). Since AO\ and C02 are the bisectors of the angles A
and C, each being equal to 60°, then £ O tAÜ-~ 30°; hence, AD ~ EC ~~ r ~\/3.
Furthermore, D E ~ 0 \ 0 2 -2r. Therefore 2 r(l f V 3 ) - - « -
a a ( V 3 —1)
Answer: r
2 (1 /3 + !)'
234 Answers and Solutions

569. The required area LMN (hatched in Fig. 53) is obtained bv subtracting
the total area oí the three sectors OiML, 0«LN and 0 3N M {which is equal to
fche area oí a semi-circle of rádios r) from tbe area oí the triangle 0 f0 20 3, whose
a ( 1 /3 — i)
side is equal to (see the precedíng problem); therefore
“ 2

SOio»oa-

The total area of the three sectors is equal to

_nr2
_ _ na2 ( l /—
3 —l)2 na2 (2—
— V-*)

570.
Solved in the same way as the preceding problem (Fig. 54).
a2 (4 — .n)
.<4nsu?er:
ò --- --------- 7 ^ ---------.
16
Alternate solution. The required figure KLMN is equal to the one which is
hatched iu Fig. 54. The latter is obtained bv subtracting two semi-circles from
the square B\CxMK.

571. Determine the radius R of the circular arc of the segment, whose peri-
meter is equal to the sum of the lengths of the arc ACB and chord AB (Fig. 55).

We get “ -n /? T fiy 3 = :j 5 , whence

2n-r 3 'l /3
Chapter VI1J. Plane Geomelry 235

The area S of the segment is equai to the area of the sector less the area
o f the triangie OAB, so that
1
JtH2-
myi
4
3a2(to- 3 1/3)
Answer: S =
~4 + 3 y w '
572. To find the sides AB and BC of the triangie ABC (Pig. 56), it issuífi-
cient to determine EB = BG — x, since AE = AD = 6 cm and CG = CD =

— 8 cm. For this purpose let us compare the foliowing two expressions for the
area of the triangie:
S = rp and S = y p {p ~ a ) (p — b){p — c)
■where p is the semiperiineter of the triangie, i.e.

j (EA + AD + DC + CG+ GD+ BE) = i - (28 + 2x) = 14 -j x

We get the equation ____________


4 (144- x) = y (14-r z)-jr*ü-8
llence, x = 7 (cm).
Answer: A B — 13 cm; BC —15 cm.
573. Let CD : DB=* m : n (Pig. 57). Then DD : BC - n : (m -j- n). Consequently,
■ s<«“ JJ=180“ - 2 C , co32C = cos(180’ -C ) =
AD DC m -f- n ' ’
= --------— . Hence
m -f- n ________
_ -./"1 + C 0 S 2 C í m
c o s C = í/ — j — = | /

-4n#u)«r: /? = arccos— ~—

i n
—-^-arccos
m -f- n ) ]
236 Answers and Solutions

574. The circle is divided into four pairwise equai ares: AB = BC and CD =
~ D A (Fig. 58). Let the arc BC he less than 90° \ve do not consider the simplest
case m : n = 1, when all the ares are equai to 90° each). Find the central angle

ct = £BOC measured by the arc BC. By hypothesis, DE : EB = m : n. Taking


the quantity — for the unit of length, we have DE = m and EB — n. Hence,
m
_DB_ _ m-{-n
_ _

and

OE = D E - DO = m — _m —
_n

Hence,
OE _ m~~n
cosa —
OC m -f- n
and
m— n
a = arccos---------
m+ n

The arc CD contains 180a— arecos—— — (degrees), i.e.


m-~n '
m— n ,
ji —arccos--------- (radians)
m -j- n

Attswer: the arc smaller than 47- is equai to arccos ~n ~ n (m > n); the arc
, n . , . m— n n—m
larger than is equai to n — arccos---------= arccos----------.
A m-f-n m -j- n
575. Let a (Fig. 59) be the angle of the parallelogram. Then
Ai —BM = <4i?-sin a
and
!í2 — BN — BC^sina
Chapler V III. Plane Geometry 237

Hence, ^h2 — [ABA- BC) sín a — p sina, whencc sin a — ■ If a is an

acute (or right) angíe, then o, ^arcsin «1.2 . Then the obtuso (or right} angle

of the paralielograra \viii be n — arcsin - * - .


Note. The problem has no solution i{ h{ P■ H /í i -H '2 < P< the probiem
is solvable (at h^-h^ — p we have a reetangle).
li. _i_ /j.,
Answer: one of the angles is equa! to arcsin -A-l— =, the other, to
• hi
n —arcsm —4*^2
—=.
P
576. By hypothesis, BD : BE = 40 : 41 (Fig. 60}. Let us take — part of BD
for the unii of length. Then BD — 40, BE — 41. Since the triangle ABC is

a right one and BE is the median of lhe right angle, AE = BE = 41. BDE is
a righl-augied triangle, therofore
D E -^ y ilE 2- ■BD2=^9
Consequently, AD AE —DE 32. From
AB AD 32 4
and ABC \ve find
BC BD 40 ' 5 '
AB 4
Answer:
BC
u i. ii
577. Sirue AO (Fig. 61) is the bisector of the angle a - / CADt /. BAO —
In tlic saino way we get Z ABO = -^ <90° — a) -^45° — —■, Frora the
triangles AOD and BOD we have

AI) ---- 0 0 cot ~ and D B ^ O D -m l(/ á °— |.)

Consequently,
c=:AB = AD + DII=OD £col i ) cot ( /i 5 ° - - | - ) J
238 Answers and Solutions

wherefrom we find r = . The denominator may be-


cot ~ ~ -j~cot |45°— ~ j

reduced to a form convenient for taking logarithms:

COS (45= - - 2 - )
c o t y + cot (45o — y ) = —

' ( 45° ~ f )

cos ~ sín (45o — y j -f-sin cos (4 5 °— j


sin 45°

sin y sin ^45° sin ~ sin (4 5 o — y ^

Answer: r — c ~\/t sin sin (4 5 ° — — j .

Note. By using the formula r — S : p {S is tbe area of the triangle; p ,


semiperimeter), we could get a solution in the equivalent form
c sín a cos a
r = - .------------ ;— :------
1-i-c o s a -f sm a
578. Let us denote the sides of the triangle by a, b and c, and let <2 = 7 cra,
b — 24 cm and c — 25 cm. Since a--\- b2 — c-, the given triangle is a right-

angled one. Couscquently, the radius R of the circumsoribed circle is equal to -5- .
5
The radius of the inscribed circle is found by the formula r = — , where 5 is
P
the area of the triangle and /?, semiperimeter.
Anstver: R — 12.5 cm, r 3 cm.

570. By hypothesis, Z i?.4£ = (p (Fig. 62). Consequently, l BAOx


It is re(]iiired to determine
ic R ~O
R ^ O xB
xB and r
r ~
~ O X ..
O X
We have
R ■ r = OtF \ F02 : OxO,-^d
Chapter V III. Plane Geometry 239

and
R -r ^ O tB -O t C ^ O iD
Froni the rightangled triangle 0 {D0Z, wherein

L 0\OzD = ^ BAOi = y
we find
OtD = OiOz-sm ~ , i.e. /? —.r = dsin •

From the two equations obtained we find

d ^1-f sin j
i? =
2
and
d (l-3 in | )
r = -----------2-----------

Substituting cos 190o — f orsi n- ^ we may transform these expressions

Ansu er: R = à cos2 ^45° — ~ j

r —dsin2 ^45°—y j
580. From Fig. 63 we have
DE MN 2r
sin A. BAD =
AD ' AD
By hypothesis, M N -DC~ Q, i.e. 2ra — Q and.
furthermore, nr2 —S. These equations enable
us to determine r and a separately, but since it
is sufficient to know the ratio — , it is better

to divide by terms the second equation by tho

whence ,
a nQ
Answer: /. ZMZ)~arcsin
nQ
180°
581. The area of the inscribcd regular 2n-gon is equal to nR2sin-

The area of the circumscribed regular n-gon is equal to nR2 tan . Bv


hypothesis,
180° ^ 180‘
240 Ansurers and Solutions

Heoce,

R= yp
T/n (tan a —sin a)
180°
where a —------ . The expression tana —sina may be transformed in the
foiiowing way
tan a —sina = tan a (1—cosa) = 2 tan a sin2 ~
Answer:
180®
Pcot
1
y
V7( tan---------
. 180° . 180°
sin
n )
90c

582. Regular polygons with equal number of sides are similar; thereíore
(Fig. 64) their areas is the area of the inscribed polvgon, S2, the area of the

D B

Fig. 65

cireuinscribed one) are in the «ame ratio as the squares of the radü
5, = ÜZ>2 : a 4 2
But from the triangle OAD vve have
OD , nm ISO2
-pr-r- " COS Z DOA ~ COS-------
ÜA n
.
Answer: C ; òc •>= cos--------
S, ■> 1800 .
i-í
583. Let AB — a (Fig. 65) be the side of the regular n-gon. Then

Z B O íV = a _ i ? 2 1 and /. NAM =■£■ = —


n l n
(as the inscribed angle subtended by the arc a). The area of the annulus is

(? = j i (0^2—OW\ = n .A N P --n ( - i j 2

The width d of the annulus may be íound from the triangle NAXI.
Chapter V III. Plane Geomeiry 241

Jiá2 , a . 90°
Answerl Q = =--7— ; d = —- tan----- .
4 2 n
584. Denote the required radius by x so tbat (Fig. 66) OoA — 0 2B = x. From
the right-angted triangie 0 i0 2A, %vhercin A .020 lA = ^ and 0 10 2= 0 1B ~

—0 2B ~ R -~ x , we have 0 2i4 = 0 (0 2sín - y - , ie. z = {ü-—x) s i n .

585. The arca St of the quadrifateral ABOC (Fig. 67) is equal to 2 - y 0 i ? x


X AB— R ico ta . It is necessary to subtract from it the area S2 of the sector
COBD, whose centra] angle is equal to
(180~2a)°. We have

=n/?2
180 —2a Xt/?2 9Ü—a
360 180
(a is measured in degrecs).
Answer: S — St — S2
=" H cota- T + í ® j
where the angle a is measured in degrecs, or
5 = 7Í2 J cot a' ■ a' where a" is measured
2 Fig. 68
in radians.
586. By hypothesis, the area of the triangie ABF (Fig. 68) is equal to ono
third of the area of the rhombus ABCD, i.e. two thirds of the area of the trian­
gie ABC. Since the triangie» ABC and ABF have the common altitude AG,
we have
BF = -BC:

16-01338
242 Answers anã Solutions

Therefore

A P ^ A B * ~ B P - 2AB. BF cos (180° - a) = P i . a2 j a2 cos á

Answer: .dF — d £ = y i 2 cos a .

587. Extend BAf (Fig. 69} to iutersect the sido OA of the angle AOB at t
point R. Froin the triangle AM R , wherein £ A M R — /_AOB — 60° (as the
angles with mutually perpendicular sides), we find M fí — 2AM = 2a. Conse-

quently, RB = RM 4- MB = 2a -f- b . Now, from the triangle ROB, wherein


Ofí -- 2OB, we find (2OB)- — OB- — (2a - f b ) ' - . Hencc,
2a- b
OB =
1 /3
The sought-for distance OM is determined from the triangle OBM.
Answer. OM ----- - ■/_ V a2*r®h4b2.
1 /3
588. The problem is reduced to íinding ^ A C B ~ 2a (Fig. 70). Extendin
AC and drawing BL||OC, let us prove (in the same way as in the theorem
on the bisector òf an interior angle of a triangle) that CL = BC = a. From the
similarity of the triangles ADC and ABL we get /?/,==ÍÍLl 5LÍ , and from the

isosceíes triangle BCL we have BL 7= 2a cos a. Consequently, 2a cos a — — -4 —■*


wherefrom we find cosa; then we find sin a and
1 i 1
S = — at sina :- j b l sin a~- — t ( a r b) sin a

Alternate solution. The area — af>sin2a of the triangle .4BC is the sum of

the arcas -^-bt sin a and — a /s in a of the triangles ADC and BCD, respeeti-
Chapter V I I I . Plane Geometry 243

vely. Consequently, ah sin a cos a — — fcí sin a + ~ at sin a whence we find


cosa.
Answer: 5 = /4a2b2-~(a-:-b)"i t'2.
4 ab
580. Let lhe rays CD and CE (Fig. 7Í) divide tlie angle ACD into three equal
parts: Z.BCD — /.DCE — LECA — a. J3y hypothesis. AC — CB = a and
CE = CD — t. In the same way as in lhe preceding problem we find from the

trianglc BCE: cosa — L— L.; then we find sin a. The required


area is the siini oí 1he areas of the triangies ACE\ DCE; BCD.
Answer: S -y— (2a-\ t) ~[/(lia • —
5ÍÍG. In lhe trianglo ABC (Fig. 72) CE is an altitude ami CO is a mediau.
Let us denote lhe required angle OCE by q. and the angles oí the trianglo hy
A, B and C. From the triangies A CE, BCE and OCE we find the following
exprcssions for the scgments of the base:
AE ^E C -cotA
BE EC-s oi B
and
O E -E C - tan <|
Since AO--OB, we have
AE — BE ----- (A()-\~OE} — (OB -O E ) - 20 E
Substituíing the expressions foimd for these scgments, we got
EC-cot A — /jC-cot B 2EC q
or
cot A — cot B —2 tan <j

Answer: tan <} ~ (cot d —cot B).


244 Answers and Solutions

591. The required area S (hatched in Fig. 73) Is equal to the three-fold
area of the figure EMFB. By hypothesis, 0 E — ~ A B ~ ~ . Ia the right-angled
triangíe OED the leg OD (the radius of the ínscribed cirele) is equal to

° ; cousequently, OD — O E ^ ~ . Hence, / , D E O ~60°. Similarly, / KFO^=


= 60°. Síace the angle EBF is also equal to 60°, OE\\BF and OF\[BE, and

the quadrilateral OEBF is a rhombus with the side —• and the angle 60® at

the vertes O. . Subtract the area of the sector EOF, equal to ~ n ,

obtained by 3.
Answer: S (3 V 3 — n ).

592. It is required to find S AB>DC (Fig. 74). The angle CFB is


a right one (as an ínscribed angle subtonded by the diameter). Consequently,
Chapler V III. Plane Geometry 245

DC — AF, and thus, S = •? But, by the property of the sccant, we have

AD-AF = A E l = ^ y

Answer: S — -g~.
593. Since /.DCA ~ / OBC (Fig. 75) and /.BCO — /OBC (for the median
OC is equai to balí the hypotenuse), we have /.DCA — /BCO. But, by hypothe-
sis, /ACE — / BCE. Subtracting the former equality from the latter one,
we get / DCE = /OCE, í.e. CE bisects the angle DCO.
594. The diameter 27? of the ctrcle circumscribed about the right-angled
triangie ABC (Fig. 70) is equai to the hypotenuse AB. The diameter 2r of
the inscribed circle is equai to
MC 4- CK (since MOKC is a
square). Hence,
AC - f BC - (AM 4- BK) 4
(MC 4 CK) — (AL ~r LB) 4-
4 (MC 4 CK) - 27? 4 2r.
595. In the same way as in the
preceding problem, prove that
<*46—2 {r4 7?), i.e.___a~{ b ~
= 2 (jR + R )= íc . Furtlier-
more, a24fc2 = c2.
Hence,
__Z_
~~ 5

b= T c (or “H h b= T c)
3 4
■Answer: sin A ~ -?•, sin 7i = .
596. Let us construct (Fig. 77) the triangles 0 £ 0 , and 0 /'0 2 (E and F are
the mídpoints of the sides of the parallelogram). They are congruent. Indeed,
OE — EC, and, by hypothesis, FC = 0*F. Hence, OE -- Ot F. Siuutarly, \vo
prove that OxE = OF. The angles OEOi and OFOz (both of them are obtuso) are
congruent, since their sides are mutuaily perpendicular. From the congruence
of the triangles OEO, and 0FO2 it follows that 0 0, ~~ 0 0 2 and /OQ\E ~
= / 0 2OP. And since 0,7? and OF íorm a right angle, the straight lines 0 0,
and 0 0 2 also form a right angle. Hence, the triangie 0 ,0 20 is an isosceles right*
angled one. The same refers to the triangles 0 20 30 , 0 30 40 and 0 40 ,0 , whicli
means that the quadrilateral 0 j0 20a04 is a square.
246 Ânswers and Solutions

CHAPTER IX
POLYHEDRONS
Notation (for this and next chapters):
V — volume
S or Sbaso — area ° f the base
Slot ~ area the lateral surface
Sioiai — total area
a — side of the base
r — radius of the inscribed circle
l i — radius of the circumscribed circle
H = altitude of a solid
h ~ altitude of the base
lf the above quantities are denotcd othcrwise, this fact is mentioned each
time. In the accompanying figures invisible lines are presented by broken lines
with short dashes, and auxiliary lines, by broken lines with longer dashes.

597. The projection of the diagonal AiC of the parallelcpiped (Fig. 78) on
the plane of the base ABCD is AC (the diagonal of the base). Therefore the angle
a between .-ÍjC and the plane ABCD is measured by the angle AtCA. From the
triangle A A tC we find
AAj — AC •tan a — \ a2---62 tan a

Substituto it into the formula S{at — (2a-p2fe)--4/lt.


.4usa'f?r: 5 íar ^ (fl-f h) V fl2-rh2 tan cc.
598. From each vertex of the prism, say from A t (Fig. 79), we can draw three
diagonais {AiE, A tD, A{C). They are projected on the plane ABCDEF as the
diagonais of the base (AE, A D , AC). Out of the inclined lines A%E, A iD , AiC
the greatest is the one having the longest projection. Consequently, the greatest
oí the three diagonais taken is A j/) (the prism has other diagonais equal to A\D.i
bul there is none longer than AiD).
Chapter IX - Polyhedrotis

From the triangle AtAD wherein L DA\A~<x and AíD = d, we find / / =


= i4i41= d cosa, AD —dsina. The area of the equilateral triangle AO& is
equal lo 1 .^ 0 2 .V 3 . Hencc Sbas^ 6 - j - O A ^ - V 3 - 6 - j V '3. The

volume Y ^ S - ll = 3-X^--AD2-AA..
8
3 Y'3
Answer: d3sin2 a cos a
.Xote. For graphical representation oí a regular hexagon (the base of the
prism) \ve may construct an arbitrary parallelogram
BCDO. Extending the lines DO, CO, BO and
inarking off the scgments OA — OD. OF ~ OC and
OE ----- OB, we obtain the hexagon ABCDEF. The
point 0 ropresents the centre.
599. (a) Drawing. The squave serving as the
base is represented by an arbitrary parallelogram
ABCD (Fig. 80). The point O of interseclion of the
diagonais represcnls the centre of the square.
Joining the midpoint /•’ of the side AB with the
vertex of the pyramid wc get the slant height EI'.
(b) Solution. We have

r ----i* 2"

where x is the side of the base (AB in Fig. 80) and


//, the altitude of the pyramid (OA). The angle a is Z.FBO (see solution oí
Prob!em597). From the triangle EBO we find II ^ m sin a; from the
iriangle OAB,
x — OB ■ 2 s= m j/2 - cos a
nO sin_ 2a-cos a
Answer. F = ™ mAcos2 a sin a
O •-*
600. Denoting the rcquired lateral edge by n in the saine way as in lhe
preceding problem, we find
m:i sin 2a cos a

whence we determine m.

Answer: m — \ / --- ------- .


V sin2acosa
601. Let us introduce the foHowing notation: AB -= x; EF . y (Fig. 80).
Then we have S — 2.ry. From lhe right-angled triarigle OEF, wherein OE II.
we find ;/2 -\ Il~. Eliminating y from the found equations, we get

j"* -{- 4 / / 2j' 2 — «V2 0

This equatíon has two real Solutions, but oniy ono of thern is positive.
Answer: x Y ~f Ml* i’2—2 //2 cm.
248 ylrtsioers and Solutions

602.* Joining the midpoints M and N (Fig. 81) of the sides BC and FE,
we obtain the graphical rcpresentation M N oi tbe diameter of the inscríbed circle
so that M N — d and OM = ~ . Since OM is the altitude of an equilateral

The altitude / / — OS is found from the triangle SCO:

n = V cs2~ o c z— j/ p - ~
The slant height m — SM oí the pyramid is found from the triangle SCM:

Answer: V ^ ^ - V 3 P - t P , Sia, = ~ Y l 2i2_d=.

603. (a) Drawing. Tho baso may be representei by any triangle ABC
(Fig. 82). The centre of the base is represented by the point O of intersection of
the medians**.
1 1 1 —
(b) Solution. We have V — -^-Sbase-H = •-g a2 ~[/3 II. The relationship
between a and H is found from the triangle AOD, wherein AD = a, and AO
is the radius R of the circle circumseribed about the base; thus a = 7? ~\/Z.

* For graphical representation of a regular hexagon see Note to Problem 598


on page 247.
** Then two of these medians, which are of no importance for solving the pro­
blem, may be erased, leaving only the point O on the median A E as is done in
Fig. 85 on page 250.
Chapter I X . Polyhedrons 249

We have i!2 = AD2— A02^ a z — -^ ~ = ^ a ~ . Substituting a2 = ^ -II- into íhe


1/3
expression of V, wo get r = -—r— II3.
o
3 / p
Ansieer: II — 2 1 / — — .
\ V3
604. (a) Drawing. As distinct from a rectangular parallelepiped, ail the laces
of which are rectangles, the base of a rigkt parallelepiped is a paralleiogram,
only the four lateral faces being the rectangles. But in drawing a rectangular
parallelepiped (see Fig. 78 on page 246) we are forced to represent the base also
in the form of a paralleiogram. Thcrefure the drawing oí a right parallelepiped

does not essentiaily difler írom thaí of a rectangular parallelepiped, which crea­
tes additional diíficultics for reading sudi drawings: it is uecessary to remember
that the acute angle of the paralleiogram shown in the drawing corrcspomls to
the actual acute angle of tlie figure represcnted. For the sake of clarity it is
recoinmended to make Ihis angle too acute, as in Fig. 83, and mark it obligatonj
with a letter (in the given case—witli the Greek letter a).
(b) Solution. In a right parallelepiped the diagonais (four in nurnberj are
equai pairwise: A tC AC% and BD 1 - B% D (in Fig. 83 AC, and DB, are not
shown). Let /_ DAIS a be an acute angle of the base ABCD; then ABC ~-
~= 180® — a is an obtuse one and A C > BD. Ilence, BDXis the smaller diagonal
of the parallelepiped (since BD\ ----- II2 -f BD2, whereas A XC2 — II2 -\- AC2;
hence, BI)\ < A tC2). From the condition BDX - AC we may find II. Namely,
from the triangle BDDXwe have
U2 - Bl)\ - BI)2 = A C2 ~ BI)2
From the triangle ABl) we find
BI)2 — a2 -{- b'1 — 2ab cos a
and írom the triangle ABC we find
AC2 — a2 -j- b2 — 2ab cos (Í80c — cc)
Consequently, //- = 4ab cos a.
Answer: V - - 2 sin a V í ^ c o s a.
605. Let us denote the larger side of the base {AB in Fig. 84) by a and the
smaller one {BC), by b. By hypothesis, a -j b --- 9 cm. To find a, b, and the
250 Answers and Solutions

acute angle a , iet us compute the diagonais of thejbase. As has been proved
in the solution of the preceding problem, the smailer diagonal (BDi = V 3 3 (cm)J
of the paralleiepiped is projected on the plane of the base as the diagonal BD.
Thereíore
BD2= B D \ - DBf = (V 3 3 )2- 42 = 17 (cm*).
In the same way we find .4(72 — 65 (cm2). And so \ve get the íoüowing two
equations:
a-~ b2 — 2 ab cos ct = 17; a2 r è2~r 2 ab cos a = 65

Adding them. we find a2 -t- 62 = 4 i, which, together with a-f& = 9» yields a = 5,


5 — 4 (we have denoted the largcr side bv a). Subtracting, we find 4aò cos a =
48
= 48, i.e. cos a = ^ ----- 0.6. Conscquently,

Sbasp ” sin a -- 4-5-0.8 = 16 cm2


Answer; V = G4 cm3. Sio!ai -104 cm2.
606. (a) Drau-ing. For constructing the point O soe Problem 603 (Fig. 82).
To construe* the plane angle of the dibedrai angle at the edge BC (Fig. 85), join

the midpoint E of the segrnent BC with the points D and A\ since CDB and
CA B are actually the isoscelos triangies. DE and AE are perpendicular to BC,
i.e. _ DEA q is the requirod plano angle. The altitude of the pyramid DO —
h lies in the plane DEA.
(In S»lution. W e have tan q £1^. , whore OD r h, and OE ~ ~ - A O (the
uiedians aro divided in the rafio t : 2). .40 is fmmd from the triangle AOD,
svherein
2h
.4/i.vj/v/-: ír aretan---------------.
I p -lfi
607. Flu* angle a is measured hy lhe angle ODE (Fig. 86), because OU is
lhe projeetion of the edge BE on the plane of the base. To construct the plane
Chapter I X . Polyhedrons 251

angle <p of the dihedral angle at the edge AB, join lhe midpQint F of the
side AB with 0 and E (see the explanation to Problem 6<M>). Sinee —
= a2_ _ _ _ t to compute V we have to find II —OE and d —DD. From the

triangle OBE \ve find i / ^ y t a n a , and hy hypothesis, ~ H ----S, Multiplying

tliese equations and lht»n dividing Ihem terimvise, \ve find


/ d \2
/ / '- S lan c S cot a

•S~ eot"a
• ■ 4 -*
The angle <p is determined from the triangle OFE, wherein
d
OF =
2 V 2

, 0K U 1 V $ tan « : \ S cot a \ 2 tan c

Anstver: 1 ■ S~ cot“a; tan q -; ^ 2 lan a.


tíÜH. (a) Drawing. The base of the pyramid is a regular pentagon (from the
equation 180° (n — 2) 540'" wo find n 5). And in the regular pentagon
ABCÜE (Fig. 87a) eaoh diagonal (say, AD) is divided hy each of the other diago-
1 /5 — 1
nals (for in?tanco#£') in the extreme and menn rntio.so thal/JM ---- -— AD s?
252 Answers and Solutions

& 0-6 A D . Furthermore, each diagonal is parallel to one of the sides (for instance
AD\\ BC). The centre O is the point of intersection of CM and EN. Therefore,
the drawing of the regular pentagon may be constructed in the íollowing way.
Construct an arbitrary triangle ABD (Fig. 876), Divide the sides AD and BD
by the points M and N in the extreme and mean ratio—approximately in tbe
ratio
A M : MD ^ 2 : 3
for this purpose it is sufíicient to divide one side and then to draw MN\\AB.
Draw .4£j| BD to intersect the extension of the line B M at the point E. Point C
is constructed Jikewise. The centre is
represented by the point O which is the
point of intersection of CM and EN.
(b) Solution. From the triangle COF,
wherein Z.OCF — a and CF — l, \ve find
H — OF = l sin a ; OC — l cosa . The area of
the base S =* 5- — -OC-OD x $inz.COD^

= -^--0(72-sin 72° = — Z2 cos2 a sin 72°.

--tnsu-cr: I '—A SM l3 sin 72°X


X cos2 a sin a.
Fig. SS 009.* The angle a is deterruined from
the triangle COF (Fig. 88), wherein
F C - CB a (by hypothesis, the triangle CBF is an equilateral one). And the
side OC (the radius of the circumscribed circle) is expressed in tenns of a
from tlie triangle COU, wherein the angle COU is equal to 36° and CU =r ~ .
OC l
We have OC - ■ o ..f , 1 lhence,
am v , cos
I.UJ a
u. - TTr, •— • o /. •
sin3b° CF 2 sin 3 b J
The angle tp is determíned from the triangle OUF, wherein F U ~ a y ^

(as the altitude of an equilateral triangle with the side a), and OU = — .y — ,
(from the triangle COU). We have
OU a cot 36' a~\/$ cot 30"
cos (p -
iu ^ r ~ : 2 ~“ y F
i cot 36°
Answcr: a ■-■■■arccos - S r=—
l sin 36° ^ V3
180°
C IO . We have (see Fig. 88): BC —a, OU~-~?rCo t The area of the base

na a 180° na2 180°


4 n

For graphical representation of a regular pentagon see the preceding pro-


blem.
Chapter I X . Polyhedrons 253

Denoting the required angle OCF by a , we have

where

Answer: a = arctan------------- ----------------- •


Preliminary Notes lo Problems 611 through 616
If all tbe lateral edges of a pyramid form equal angles with the base, then (1)
all the lateral edges are equal; (2) a circle can be circumscribed about the base;
(3) the altitude of the pyramid passes through the centre of this circle.

Fig. 89 Fig. 90

Proo/. Lct tho edges S/i, SB, SC and so cm (Fig. 80) form equal angles with
the plane ABCDE. Consider lhe right-angled triaugles AOS and BOS (OS is
the altitude of the pyramid). They have a comrnon altitude, and the acnte angles
OAS and OBS are equal to cach other (since they measure lhe angles of inclina-
tion of the edges SA and SB to the base, respecliveiy). Consequently, A S ~ BS.
Likewise, we provo that BS — CS and so on. Prom thesamc triangles A OS and BOS
we find AO ~ OB. Likewise, we prove that OB — OC and so on. Honco, the circle
of radíus OA and with O as tho centre will pass through the points B, C, and so on.
611. As has bcen proved, the altitude EO passos through the centre of the
circumscribed circle, i.e. through the point O of inlersection of the diagonais
(Fig. 90). The area of any parallelogram is equal to half the product of the
diagonais and the sine of the angle eontained between them. Therefore
254 Ansírers and Solutions

\
= -^b2 sin a. From the tríanglc AOE we finei:

H ~ A O -tm p = -^-tan p

Answer: F —^ 6 3sin a tan P-


612. (a) Drawing. According to the Preliminary Notes, the altitude oí the
pyrainid must pass through the centre of the circle circumscribed about the
isoseeles triangie ABC (Fig. 91). Since the angle a — A CAB at the vertex

remains arbitrary, the centre O may be represented by any point of the line-
segment AE (E is the inidpoint of BC) and even of its extension (in the latter
case the actual angle a is an obtuse one).
(b) Solution, The? altitude DO is determined from the triangie AOÜ, wherein
OA D - p, and AO fí is the radius of the circumscribed circle. Accordiug
to the Jaw oí sines the side BC is equal to the produet of the diameter 2R of
the circumscribed circle bv the sine of the opposile angle « , so that B -----.
2sina
Thequautity — BE is found from the triangie ABE ^ -^- — asín j . Hence

a sin -rj- tan P


II B tan p :

The area of the base


1 „ .
S — «‘-sin a

a2 sin ~ tan p
Answer: t’ ---------- ----------- .
(>
613. (a) Drawing. In tlie parallel projection a circle is represented asan ellipse.
The ellipse may be conslructed in the following way. Draw the diameter JI/-V
of the circle {Fig. 92) and from an arbitrary point P of the circle draw the straight
Chapier I X . Polyhedrons 2 55

line PP' perpendicular to ALX. Let R be tbe point oí inlersection of PP' and
MN. Shorten tbe line-segment RP in some ratio (say. to balí its length) and lay
off the sbortened segment RQ on tbe samc line PP' to both sides oí R (RQ ---
— RQ')* Proceed in the same way witb a number oí points on tbe circle to get
a number of points íor the ellipsc under conslruction.
The ellipse is symmetrica! about A/.Y (the major axis) and about tbe straight
line UU' drawn through the centre 0 perpendicular to MX (I T ’ is the minor
axis o/ the ellipsc.) The point 0 is called the centre oí the ellipse.
To depict a circle circuinscribed about a rectangle it is convenicnt íirst to
draw an ellipse ABCÜ representing the circumscribed circle (Fig. 93). It is
good practice to arrange the major axis of the ellipse in an inclíned position*.

Fig. 93 Fig. 94
One side oí the rectangle may he remesented by an arbitrary chord AB of the
ellipse. It is advisible to draw the chord homontally. Draw straight lines BD
and AC through the centre of the ellipse. The quadrilateral ABCÜ tlius obtained
is the graphical representation of lhe rectangle.
(b) Solution. The insrribed angle CAtí contains a*, since it is subtended
by the arc BC containing (2a)\ From the tríaugle BA C \ve have A B ^ 2R cosa ;
BC — 2R sin a, and so
S — 2 (AB BC) // 4/í fcos a sin a) II
Hence,

4R (cos a sin a)
We now find V - AB-BC-II. The condition that the arc (2a)' is subtended by
a smaller side of the rectangle is an unnecessary one.
. «S/fcos a sin « »S7?sin2a
Answer: F - --------------- :------ - —7=-------TZ---------•
cosa ; Sina y 8 c o s (4 5 ; — a)
6Í4. The arca of the base ,s\ — tfilana (Fig. 94). By hypolhesis,

Stut 2S y Cí2 lan a

• In Fig. 93 lhe major axis of the ellipse coincides with the diagonal AC of
the rectangle. This sinipliíies the drawing, but is not obligatory.
256 Answers and Solutions

On the other hand,

2.-J-)
2a cos2-
Slal =
Equating the two expressions íor Siat we find
a s in a a t
H=
:4 '
cos-2
a3 a
V = • -s- tan a tan
■4/i s u o t :
o 2
615.* Join the midpoint M of the side AB with O and S (Fig. 95).
£ The angie OMS is the plane angie of the
dihedral angie a (see explanation to Problem
606). Hence,
OM ~ SM cos a — m cos a

From the triangle ÂOM, wherein z AOM**


— 30°, we find

. V3 1 /3
AM * OM--

Then we find

Sbase == f ( i ) V a
and

s lat =

Substituting the íound expression for we get

stotal ~ Sbaae + Slot = 2 V ã m - cos a (1+cos a)


Answer. Stolai = i V 3 m2 cos a cos2-2..

616. By hypothesis, the incíined Unes AC and CB (Fig. 96) are equal to
oach other.' Hence, tiieir projections are also equal: A D — DB. The angie DEC
{E is the midpoint of AB) is the plane angie of the dihedral angie a.
* For graphical representation of a regular hexagon see Note to Problem 598.
Chapter I X . Polyhedrons 257

Since the triangle ACB at the vertex C is a right-angíed one, CE — AE —


Hence, ££)== — cos a. Finally,

AD^BD = yA E •£D - = ~ "|/t -f- cos2 a


„ c2 cosa
Answer: **ABD /

-r ,(j - | ' i C0S2 a )

Preliminar# Notes to Problems 617-704


If all the lateral faces of a pyramid are inclined to the base at one and the
same angle a, and the altitude passes through some point O on the base of the
pyramid, then:
(1) the slant heíghts of all the faces are
equal;
(2) a círcle cau be inscribed in the base of
the pyramid with point O as the centre;
(3) Sbase - s tat cos <*•
Proof. (1) Draw (Fig. 97) the siant height
PM of the lateral face BFC and join M with
O. The líne-segment OM is the projection of
PM on the plane ABCDE. Consequontly, it is
perpendicular to BC (“the thcorem on thre.e
perpendiculars”). llerice, the angle OMF is the
plane angle of the dihedral angle a. From the
triangle OMF \ve liave F M = S ^ — \ OM =
sm a
-O F -c o t cc. Ifwedraw PL, PN and the slant
heights of other lateral faces, we find likewise
that all of them are equal to —.
sm a
(2) The line-segmonts OL, OM, etc. are
perpendicular respeclively to tho sides i B,
BC, etc. and are equal to 0F*cot a. Thereíorc,
Fig. 97
if a circle of radius OM is (Jrawn from O as the
centre, itw ill be inscribed in the base ABCDE.
(3) As has been proved the point O, which is the foot of the altitude of the
pyramid is the centre of the inscribed circle.
(4) S0 nc = — BC ■OM = j BC (FM ■cos a) = ( - t BC ■FM ) cos a = SFKC cos a

Likewise we find that q S -Sr 50


ah'• ahcos a > an<l on. Adding theso equa-
lities, we get S!,„>;<>- - .S'/aí cos a.
617. The altitude PO of any pyramid (Fig. 97) is projected on lhe lateral
face BFC as a line-segmenl lying on the straight line PM. Therefore,
j£ 0 F M ~ y , Iíence. ct~-90°— <f. i-e. all the faces are inclined to the base at
one and the same angle. As has been proved
<>
òtat = sm <p
17-01338
258 Answers and Solutions

Answer: Siat -
sm
2Q c «s s ( 45°

if ; sm cp
618. From the triangle DOE (Fig. 98) * we find
- l , r r . tan
' 3
YVe have
1 o , „ -W .
^basc - 4- a- "}/
r 3 and
- 8,*u.
ia< = C0Sq,
(see íhe prel iminary note to the preceding problem).
a3 tan q>
Answer: 1'
24
fP
„ •»/~\ i ■
o~ ]/.] (1 — cos <p,) 0 “ V 3 cos2
*
Slota! 4 cos <\ 2 cos !{'
.Vote. The general expression for the total surfaee arca oí a pyraraid, whose
faces are inclined to the base at one and the «ame angle cp may be \vritten as
follows
=Sb,
28òns,cos2 .
I 1 + -COS (f / cos cp
619. Mako use oí the formula S(0(ai =
az \ 3 cos^ -i-
- , found irt the preceding problem.
2 cos tp
1 , ; r 'lS cos a
Answer: a - -
l 3

620. (a) Drawing. The straight line, joining the


points of tangencv L and N oí the opposite sides
Fig. of the rhombus (Fig. 99a) passes througn the centre
of the circle. Therefore, first draw an ellipse
(Fig. 995), representing the circle**, and then the straight 1'tnes NL and KM
passing tiirough the centre O. To complete the paralíelogram ABCD representing
the rhombus draw straight lines tangent to the ellipse at the points .V, L, K, M.
(h) Solution. To determine Sh(1Xt. find the altitude DF and the sido AB of
the rhombus. From Fig, 99a we find DF ~ 20K — 2r; from the triangle AFDt
wherein £.A ~~ a, we have
ÁD DF _ 2r
a ~~ ~~ sin a ~ sin a
* For representation and construction seo Fig. 82.
** For construction of an ellipse see Problem 613.
Ckapter I X . Polyhedrons 259

Then we find
4r2
*^6as< = AB-0F — a-2r=*
II is determincd írom the trianglc ONE (Fig. 99h), whcrein O N = r and
£ O N E — $. For determining Stotai niake uso of the Note to the preceding
problem.

(»)
Fig. 09
Sr- cos-o -J
Pj-
4 tan ft
Answer: V '
3 sin a sin a cos f>
621. Use the Note to Proldem 618.
Answer: (p = arccos — .
r a
622. (a) Drawing*. The section figure is the parailelogram A,D tCB
(Fig. 100). To depict the plane angle fonncd by the cutling plano AiDtCB and
the plane of the base draw lhe straight
line DM representing the altitude of the
rhombus A BCD. Since DM and />/>, are
actually perpendicular to the cdge AD,
the plane DD%NM is perpendicular to
AD, and, hence, to BC. This plane inter-
sects the cutting plane along the straight
line MDt, and thus /.IJxMÜ - p.
(b) Solution. The lateral suríace con­
sista of four oquai rectanglos (since the
base is a rhombus). The area of the late­
ral face AiDiDA is 5, =--= A J )X-DDU and
the area of the section figure is Q -~
~ A ,/>i -DiM. Frorn the trianglc DMD% wo have D D X : />jd/*sin p, tberc-
foro Si = Q sin p.
Answer: S[ai -- 4Q sin f>.
623. Tako into consideration the Prcliminary Notes to Problem 617. By hypo-
thesis, EO ~ d (Fig. 101). Point E (the midpoint of lhe hypotcnu.se ND of the
* For graphicai representation of a right parallelepiped see Problem 604.
17*
260 Answers and Solutions

triangie NOD) ia tbe centre of the circle circumscribed about the triangle NOD.
Therefore, ND — 2-ED = 2* EO — 2d. Fromthe triangieDON, wherein £ O N D =
— cp, find the radius ON — r of the circle inscribed in the base: r = 2d cos <p. To
find S^ase determine BN (haif the base of the isosceles triangie ABC) and AN
(its altitude). The centre O of the inscribed circle lies on the bisector of the angle
ABC equal to a, i.e. Z.OBN — — ■. From the triangie BON we find BN ~

Sbate = ^-BC -AN = BN-AN = BA'2- tan a = A cot2 — ■tra a =

=4d2cos2 cot2 — tan a


wherefrom (see Note to Problem 618) we find:
2Shasec m Z -í
s total z cos q>

Answer: Sf0tai ~ 8 & cos (p cos2 cot2 tan a.


624. Take into consideration the Preliminary Notes to Problem 617*. The
altitude of the pyramid is found írom the triangie ONP (Fig. 102): J7—r tan <p.
If flj, etc. are the sides of the base, then

Sta « = s A0B + SB0C + . . . = J AB. OM + i BC. ON + . . . =


1 1 1 1
=— y «8T+ •••— ~2 r (fli - r a2+ •••) = T r,2P“ r?
* F or con stru ction o f th e e llip se rep resen tia g th e c ir c le in scrib ed in th e base
see P rob lem 6 Í3 .
Chapter J X . Polyhedrons 261

Anmer. F = - Í £ | 2 i .
625. (a) Drawing. Having drawn the regular triangular pyramid DABC
(Fig. 103)*, let us construct the triangle ,4 whose sides arevparailel to the
respective sides oí the triangle ABC. The triangle A ,#,Cf depiets the upper base
of the frustum of the pyramid. The centre 0 , of the upper base is found at tbe
point of intersection oí DO and one of the

Fig. 103 Fig. 104

(b) Solution. The volume of the frustum of a pyramid

V = ^ - ( Q ~ q - V V Õ Í)
where Q and q are the areas of Ihe triangles ABC and A\BXC\ rcspeetively,
so that Q = az; The altitude H ~A\M is found from the
triangle AA\Mt wherein z. MAAt -a and AM — AO— AiOf. Rut AO and A fti
are the radii of tbe circles circumscribed about ABC and Therefore,
^ 0 = -^ -a n d A,0,= - ^ . Hobco,

„ l73
Consequently,
U = ?—-è- tan a
V 3
Answer: V — (<i3— b3) tan a .
626. (a) Drawing. The frustum of the pyramid is repre.-entcd as in the pre-
ceding problem. To depicl the plane angle of the required dihedral angle draw
A,E and #,/•' {Fig. 104) paraliei to OOt to intersect the diagonais AC and BD.
* For g ra phica l represen talion o f a regular triangular pyram id see Fig. 82.
262 Answers and Solutions

Then draw E F parallel to A B to intersect the edges A D and BC at points M


aad N . The plane is perpendicular to the edge A D , since it passes
through A tE and M N which are perpendicular to the edge. Conscquentíy,
/.E MAi = tp is a plane angle of the dihedral angle at the edge A D .

(b) Solution. From the trapezoid M A fliN we obtain M E — a . The

altitude of the truncated pyramid is found írom the triangle AEA^ where
AE — ~~~~ . We have
V2
a—b
tan a
V2

The volume is found hy the formula V = — fa2-f- a6-fb â). The requíred angle

tp= Z EM A { is found from the triangle A {ME. where d /ff ~ — (from. the
trapezoid MNBiA{). We have
AiE a —b
tan <p- — — tan a :
17F 1/2
(«3 — b3} tan a
Anstver: f = arctan ( V - tan a ).

627.'See tlie Preliminary Notes to Problem 611. The altitude of the pyra­
mid must pass through the centre of the circle cireumscribed about the base.
But in the right-angled triangle ABC (Fig. 105)
the centre lies in the midpoint of the hypotenuse
AB at the point E. Consequently, AE, BE and
C.E are vespective projections oí the lateral edges
AD, BD and CD on the plane of the base, and
thus l DAE ~ / D B E = / D C E ^p. The volume
of tbe pyramid is found by the formula V —
™ . DE. From A ABC we have: AC —
— c cos a, BC — c sin a; from A ADE we fínd
DE= tan (5. Lct us denote the plane angles at the
vertex: /, dZ>5 = 0j, /, BDC .•=02 and / ADC = 83.
Since these triangles are isosceles ones, theír alti­
tudes DE, DM and DN pass through the midpoints
of the corresponding sides of the base. From A ABD
we have E 0t — 180° — 2j3; from &.DBC we have
. 0., MB , , . 63
sin — -jj-jy and from A ADC we have sm - 5- =
AN
From A ADE we fínd AD~ DB:r= and from A ABC we fínd
: AD 2cosf3
BC c . . ... AC
MB-. --s in o. and Ai\ = - r - —
C h a p ter IX . Polyhedrons 263

. . T. c3 síd2a tan 6
24
8,=:1803~~2$
02= 2 arcsin (sin a cos p)
03 = 2 arcsin (cos a cos 0)
628. It is required to íind the volume oí the pyramid CtABC (Fig. 106).
Since its lateral edges are of the same length, they are inclinei! to the base at
one and the same angle (this theorem is converse to the theorem proved in the
Preliminary Notes to Probiem 611), and the altitude CxO passes through the
centre O of the circle circumscribed about the triangle ABC. Since this triangle

is right-angled ono. lhe poinl O lies at the midpoint oí lhe hypolemise Atí (see
the explanation lo lhe precedíng probiem). The angle V Ü C X (/> is the midpoint
of the leg AC) measures lhe incjination oi the lateral face ACCXA X lo the base.
The legs BC and AC are íoimd hum the íolloving two equations:
and BC —AC-tan a
we get

AC -- stn a - f cos a
1 - tan a ' sin a -) cos a
1
Then we íind Sbasi BC-AC. The altitude II is found from the triangle

&OCít whcre OD — -L-BC ias a midline of tlte triangle).


1 oí3 sin2 a cos a w<3?in2 a cos a
Answer: V : tan p - •tan p.
12 (sín a-j-cos a)3 r 24 V 2 cos® (a —45°)
629. Point O is the centre of the circle circumscribed about the base ABC
(Fig. 107) (see the Preliminary Notes to Probiem Gíl). OA — R is the radius of
264 Answers and Solutions

this circle. The volume of the pyramid


„ 1 BC>AE 1 AE-DO
■BC-- - Q-BC
V~ 3 * 2 ' VU ~ ‘ò ' 2
AE-DO
since Çj . The side BC is found by the law of sínes:
BC = 2R sin (ISO0 - 2a.) = 2R sin 2a
A ADO A ABE (since A ADO — A ABE = a)\ v e have the proportíon
AO
wherefrom AO-BE — AE-OD.
AE
Suhstituting
BC
--R, BE = AE-OD = 2Q

R-BC
■-Q

Eliminating R from the found formulas, ve


obtain
BC ^ Y S Q sin'lã
3 i
1 õ õ
Fig. 108 /Insuw; V (2Q}~ sin“ 2a.

630. If the faces ADE and CDE (Fig. Í08) are perpendicular to the plane of
the base, then the edge DE is the altitude of the pyramid. The angleD A E is
a plane angle of the dihedral angle EABC, since the plane DAE is perpendicular
to the edge AB (prove it!>. Consequently, i_DAE = a;
likewise, Z.DCE = f}. From the triangles ADE and
CDE, where DE = lí, \ve find AD and DC and sub­
stituto their values into the formula

F = j AD .D C-II
1
Answer: Y = — H3 cot a cot p.
631. From the triangle BDE (Fig. 109), where
A EBD — $ (prove it!) \ve find
Z>£ = lsinP and BD = ícos|3
Hence,
BD l cos P
AD =
yi~ y 2 _____
From the triangle ADE \ve find AE — \/AD~-~DE2. The angle cp of inclina-
tion of the edge AE to the plane of the base is a DAE (prove itl). From the
DE
triangle ADE v e find tan<p =
AD '
* As is obvious, Fig. 107 (where AO < AE) does not correspond to this rela-
tionship. But a drawing depicting the cundition of the prohlem (<p = 90° — a)
more accurately would be obscuro.
Chapter I X . Pohjhedrons 265

Answer: DE — i sin
< p =aretan(y2taiiP), AB = CE = r j / Í ± .| d2| .

632. The greatest area beiongs to the face ADB (Fig. 110), since its height
DE is larger than the heigbt DC of the other two lateral faces, the bases ofall

the faces being equal to a. From the triangle ACD we have

A D -■— — ■ and 7/--=atan(5


cos p
We then find from the triangle ADE

D E ~ V Ã m = m = - / 1£ ç - $ .

The angle CED is the angle cp of indination of the face ADB to the plane
of the base (prove it!). We have
II
Sc

where EC — ^ ~ L .

Answer: S ~~ , a „ V iT^eos- fj, a ~ arctan .


4 cos fi v H * y i

633. The area S of the section is equal to --A B -N M (Fig. 111). From the
right-augled triangle ACN, where /_ CAS 30°, we find

AN -■■■■~ -A B —.a and CN — ^-a


266 Answers and Solutions

From the triangle NCM we have

“ - / C T + (4 )’
where / / = a tan a may be obtained from the triangle ACD.

Answer: S =
" 4 cos a
634. (a) Drawing*. To depict a sectiun perpendicular to the base ABC
(Fig. 112) and bisecting the sides A B and A C of the base draw the raidline MN.
From the point /•’, where MN intersects the median A E , draw FK parallel to

the altitude OI). NMK is the required section. Indeed, tice plane NM K passes
through the straight liue FK perpendicular to the plane ABC (hence, the plane
NMK is perpendicular to the plane ABC). The dihedral angle a is measured by
the angle AED (prove it!).
The plane AED passes through À'F, since the points K and F lie iri the
plane AED.
(b) Solution. Let us take the triangle AMN as the base of the pyrauud
KANM. The area S constitutes one fourth of the area of the triangle ABC,
1 /-
i.e. S ~ a2 '\/o. Let us express the altitude KF through OD making use of

the fact that A AFK is similar to A AOD. Since AF is equal to -]-A0

|for A F ^ -~ A E , and A O ~ -~ AE j , K F ~ ~ OD. The line-segiuent OD is

found from the triangle DOE, where OE — ^ and ^ DEO = <x.


, .. a3 tan«
/I n s i a e r : V — .

635. The straight line MN (Fig. 113). along which the cutting plane
intersects the base. is parallel to BC. To construct the angle cp draw OF\\ AB
* For depicting a regular triangular pyramid see Problem 603.
Chapter I X . Polykedrons 267

and join the poínt K , at which OF intersects A/A', to E. Then /_OKE = y


(prove all this). The area of the seetioii figure S = ~ M X-KE, where MN = a

and K E - —----- . The altitude H is determined from the Iriangle EOF, where
sin <p °
OF and F E = ^ ^ c o í ~ (from the triangle EBF), We get

____ " sm T
a- "f/cos a
Answer: S -
4 sin ~ s i n <p
636.* The section figure is a triangle DK.X (Fig. 114). -\s in Problem 634,
let us prove that the plane AED is perpendicular to the sido BC. IJence, it is
perpendicular to the inidline K.X as well. Conse-
quently, / DME is a plane angle of the given 23
dihedra! angle a. From the triangle O M b . where

OM = ~ AE --- ~ — .. . we íin d
b 6 2
t Va
D M =-

The section area

Y -io 2
4Neos a

The area of the base of tho pyramid bA A A


is one fourth of the area of the base of the pyramid Fig. 114
DABC, the two pyramids having a common alti­
tude. Therefore the volume V( of the pyramid DAKN is equaí to ~ wltere
V ís the volume of the pyramid OA BC. Dmsequently, the volume of the pyramid
DKNBC V2 = 4- l . The volume 1 is equal to i 1 w ,..« i . i i .
4 H
X ..-1 Ü tan a.
12
y l a
Answer: S
48 cos a
„3
Vt- - tan a
192
V2
' 64
•For dep ictin g a regular triangular pyram id see F ig. 82.
268 Answers and Solutions

637. By hypothesis BE : EA = 2 : i (Fig. 115). Tbesection figure is ADEC.


Find its area 5. The triangle DEC is an isosceles one, since £C = ED as cor-
responding sides of congruent íriangles AEC and AED (AC — AD ; A E is
a common side and Z.CAE — /.DAE = 60c). Draw its aititude EN; then

íaw of cosínes);
£ ( ’- _ AC* + AE1 - 2-AE -AC -cos 60° = — a*
Now írom SESC we find

EN = y È C i - N C * =
V TTa 4 6 V »
Denote the seclion angles i'.ECD : I.EDC by ct. Then Í.C-ED n — 2a.
Frora the triangle CEN we have

1/19 o-
Answer: S = b --- n — 2 arccos
12
- I' 2 1 /7
638.* lh e lateral face BCLiBt (l ig. 116) is an isosceles trapezoid with the
bases BC - a and BtC, =■- b (a > b) and anglu a at the base a. The line-segment
B t* its altitude. We find tfj.V ^ tan a. From the triangle BlN F l where

ES , we find

II = B,F = y.\ B l - F X i. . . y t^ y zri


The volume
,, J! , 2 .v a3-5 3 -----------
\ = — (tf-yb -^ a b ) = — - — l / t a n - a — i

* For depicting a truncated pyraraid see Frublems 625 and 626.


Chapter I X . Pohjhedrons 269

Note i. If the acute angle a is Jess than 45°, the radicand is negative. But
the angle a cannot be iess than 45°, Indeed, the sum oí the plane angles BCCi —
= a and DCCl = a of the trihedral angle C always esceeds the third plane angle
BCD\ but £BCD ~ 90°, therefore 2a > 9ÜC, i.e. a > 45\
Note 2. The expression 1/ian2 a ~ l can be transformed to the íorm
I f s‘ ft2 ct —cos2 « __ V —cos 2«
r cos- o. ~~ cos a
Since 2a is more than 90: (but less than 180", since a is an acute angle), cos 2a
is always negative. Hcnce, the radicand (—cos 2a) is always positive.
. a3~ b 3 a3— b3 ,------r-r-r----- -—
Answer: \ —-r.-------- - V ~-eos 2a — -r.-------- 1 cos (180- — 2a>.
b cos a 6 cos a
639. The projection oí the diagonal BDX (Fig. 117) onto the lateral lace
BCCtBi is BC}. Therefore £.CsBDs—a. From tlie tríangíe BCXDX. where

£>,C, ò, we íind BC\ — b cot a. From the triangle BxCiB we have

ll=\/B C\ — B fii =
Then
1/cos 2a
v = fc2// = b3
sin a
Note. The radicand cos 2a is always positive kere (see Note 2 to Pro-
blem 638), since a -<45°. lndeed,
tan a -
s BC\ ~ BCi
But BXC\ is a leg, and BC\ is the bypotenuso oí the triangle ££,<?,. There­
fore t a n a < l , i.e. a < 45°.
V cos 2a
Answer: V — b3 -----:--------.
sin a
640. lf CD (Fig. 118) is lhe altitudo oí lhe triangle ABC dropped onto lhe
hypolenuse AB — c {CD may be drawn inside the angle ACB arbttrarily), then
270 Answers and Solutions

Z.CDCi — p (prove it!). We bave


CD=zAB*sm a cos a = - j c sin 2a
and
H = CC, = CD-tan p
Substitute these expressions into the formula

r = jS B -± .^ '.C D .B
1
Answer: V — ^ c3 sin2 2a tan p.
641. One of tbe portions of the prisra is a triangular pvramid BtABC
(Fig. 119). Its volume 1', where V is the volume of the prism. Hence,

the volume V2 of the other portion (of the quadrangular pyramid BtAiCiCA)
is equal to ~ V. Find V.
By hypothesis BC - f .415 = m, and from the triangle ABC we find BC =
— /lB -co s a . Consequently,
m cos a _ m cos a
BC--
1 -r cos a
2 cos2
e area 5 of the prism base i

S = 4 - 4c-BC= 4 - ec2'tan ct
The altitude ll = BB\ is determined from the &BCB\, where Z.BCBt — 6
(prove itl). We get H = i?6’ *tan p.
, .. m3cos3a tan a tan P .. w3cos3a tan a tan 6
Answer' V$~ ..........-..................... . ■ ; V»— ................. .
48 cos8-—- 24 cos8 ™
642. According to the Preliminary Notes to Problem 617, Sbase — S cos 9 =
- - S s in a. On the other hand, S&fl8C = K • Equating these two expressions,
Chapter I X . Pott/hedrons 27 í

we get a — 2 V i ’ cos a. Point O (the centre of the circle inscribed in the triangle
ABC, Fig. 120) lies at the point of intersection of the bisectors of the angles
of the triangle, hence,
Z0C £ = -
and
OE — EC-tan tan -

from A EOE we find


H ..0£-tan (p

1 ~ CL
Answer: 1' = — (S cosa)" tan - y

Stotai — s (1 +COS <fj = 2SC0f- ( 45° - 9


643. In Fig. 121 OA OC — fí are radii of the circle circumscribed about
the isosceics triangle ABC (AB -- AC -■ a), fíy virtue of the coudition a > 45°
the centre 0 lies inside the triangle ABC {ai
a < 45° the anglc A — 180° — 2a would be
obtuse, the centre of the circumscribed circle
would lie outside the triangle ABC, and
then lhe plane drawn through the altitude of
the pyramid and vertex C would yield no sec-
tion). The altitude of tho pyramid passes
through the centre O (seo the Preíiminary Notes
to Problem 611).
From the triangle AOD we liave / / — H tan fí.
Since by the law of sinos AC ~ a ■= 2/f sin a,
II = — tan fí.
2sina 1
Let us find the base CE of the sectiou figure
frorn the triangle A CE, in wliich Z.CAE ~
180c — 2a and Z.ACE at the base of the
Isosceics triangle A OC {AO-= OC « li) is cqnal to
L.CAO = j-Z C A E - 90'- — a. Hence, ±AE C =-
, CE
— 3a — 90®. By lhe law of sines • whence
sin (18ü° — 2a) sin (3a- -90-')
«sin (180°—2a) sin 2a
CE-.
sin (3a — 90°) sin (3a — 90°)
Note. We may write (—cos 3a) in the dcnominator; but the angie 3a is
contained between 135° and 270°, since 4.V < a < 90°. Thus, (—cos 3a) is
a positivo number. Therofore, when performing computations with the aid of
tabies, it is more convenient to deal with lhe angie 3a — 90c contained between
45® and 180®.
a2 cos a tan fí
Answer:
2 sin (3a — 90°) *
272 Answers and Solutions

644. (1) Find the area Q of the base of the prisin (Fig. 122).We have: Q =
= S\ - f Sz, where Si is the area of the right-angled triangle ABC, and S2
is the area of the right-angled triangle ADC,
c AB*BC 1 sin a>l cosa /2 sin 2a
-- 1---
and
c Z2sin2P
52= |------

Hence
/-> lz
, • o , • no, /2sin ( a ---13) eos
Q ~ ~ (sm 2 a -f sm 2p) = ---------i—
(y.—
Bi
J------ —

(2) Find the altitude H of the prism from the condition S — BD-H. Since
in the quadrilateral ABCD the suin of the angles at the vertices B and D is

equal to 180c, it can be inscribed in a cirele uf diameler oquai to the diago­


nal ACbccause the latter snbtcnds the inscribed right angles. From the triangle
B C D , inscribed in the circle, we find (by the law oí sines)

BD — AC - sin S.DCB ~ l sin (oc -f f>)


Ilence,
h _ _ £ _ = _____ « _____
BD i s in (c i-p )

Answcr: F = S 4 cos (a — [}).


645. The faces ADE and B C E {Fig. 123) are isosceles trianglos. The plane
E M N ( M and .V are the midpoints oi tiic edges A D and B C ) is perpendicular
to B C and AD and passes through the altitude E F oí the pyramid (prove it!).
By hypothesis the exterior angle a - _ E d //. of the triangle E M A is an acute
one. Therefore, the altitudo EF intersects the extension oí M N .
To determine F find the side A B of the square A B C D . We have
A B = M N » NF - M F = 11 (cot f> - col a)
Chapter I X . Polyheàrons 273

Hence,
-AB *'H = y H 3 (cot p —cot a)2

Let us now construct the plane angle qj of the dihedrai angle, at which the
face ABE is inclined to the base. To this end intersect the dihedrai angle by
a plane EFK, which is perpendicular to the edge AB. To depict it draw Ftf I! AD
to intersect the extension of the edge AB (prove it!). From the triangle EFK
we find
JJ 2H_
tan c
FK ~ AB cot P—cot a
1
Answer: V —~ y //3(cot P—cota )2=:

P)
3 sin2 asin2 p

<p=arctan
cot P — cot a '
2sin «s in p
=arctan
sin(a — p)
646» The altitude EF oí the pyra-
mid (Fig. 124) lies in the face CED
which is perpendicular to the base. The plane, drawn through EF and perpen­
dicular to the edge A B , intersects the base of the pvramid along MF\\BC and the
lateral face AE B —along ME perpendicular to AB (Z.EMF = P). Since AD
and BC are perpendicular to the plane DEC, LBCE — 90° and 2.ADE — 90c
(all this should bc proved).
Let us find the altitude II = EF. By hypothesis E F E M ~ m \ furthermore
EM , Thereíore EF (1 -j— 4 - q- \ = m, whonce
sm p V 1 sm p /

= £F = m : ( 1H— / H — )
V sm (i / \ cosa /
2 cos2 -

Then írom the right-angled triangle DEC we find


EC II
= DC =
cos a ~ sin a cos
Finally, we find
b — BC — M F ~ I I cot p — / / tan a
Hence
tan a H*
F= i / » = i / / 3 - sjn a cosa
’ 3 cos2a
The sura 5, - f of the areas of the lateral faces BEC and AED is equal to

i B C .E C + 1 A D -E D -^ H B C + B D ) - ^ ( - ^ + -J L )
cos a /
18-0133$
274 Answers and Solutions

. „ m3 cosa
Answer: V -- ----------------
24 cos8 —-

„ c m2 (sm a -f-cosa) m2 cos(45° — a)


—— — = ................ ■,
8cos4-^- 4 V 2 cos4~

647, (a) Drawing. Construct the altitude EF (Fig. 125), joining E to the
midpoint F of the side DC. Join the vertex E to the midpoint M of the side AB.
Then cp = £F E M represeots the angle between the faces ABE and Z)C£|(pro­
ve it!).
<7

(b) Sohition. The triangle BCE is a right-angled one, in it /_ BECl= a


(prove it!). Hence, BC — b sin a. Frora the triangle ABE we have AB =
= 26 sin a and ME = 6 cos a. From the triangle MFE, where MF — BC =
= 6 sin a, we íind
FE = V ME2— MF2 — b V ^os2 a — sin2 a — 6 "|/cos 2a
Note. The radicand cos 2a is ahvays positive here, since 2a < 9 0 °. Iiuleed.
the sura of two face angles of the trihedral angle at the vertex B ^ / ABE =
180o — 2a
2 and Z.CBE — 90o —a j exceeds the third ono ( /, ,4ZÍC = 9Ü0),
180°
..v. - -_p(90° — a )> 9 0 ° , since 2a < 9 0 °.
It is best of all to fínd the angle q> by its sino.

Answer: V — - j 63sin3 a V co9 2a; <P= arcsin (tan a).


618. The plano BCE (Fig. 126) is drawn through the side BC perpendicular
to the edge AS. The dihedral angles between the lateral faces (all of tnem being
Chapter I X . Pohjhedrons 275

of the same value) are measured by the angle BEC = cf. The triangíe BEC is a»
isosceles one.
To determine the area S of the section figure aDd the angle <p it is sufíicient
to find DE (D is the midpoint of BC). For this purpose \ve consecutively find BS
(from the triangíe BSD, where BD — and BSD = ~ ) , then BE (íruin
the triangíe BSE, where /_BSE = a), and, íinaliy, DE = ~\/BE1 — BD1.
We get
E

and
■cr

Note i. The sum of face angles at the


vertex S is always less then 360°, Therefore
0 < a < Í20°. At this condition 2 cos ~ > i,

i.e.--------— <; i and, hencc, the equadon

• 9 i
---------- always has a solulion. Fig. 127

Note 2. If a > 90°, i.e. the angle ASB at lhe vertex of the lateral face is
aa obtuse one, then the altitude BE of the triangíe ASB intersects the extension
of the base, and the plane BEC gíves no section of the pyramid. Nevertheless
the formula

yields a definito value of S even with a» obtuse angle a {Iess lhan 120°, see
Note 1).

$49. Al! eight faces of the octahedron are equílateral triangles, thus

contained ín divides the octahedron into two equal regular pyramids so that
276 Answers and Solutions

V - 2 —^a^’ OE where

3o2 a y i
oe - y E m -o m = y .
4 = 2
Ail the dihedral angles o? the octahedron are equal. The angle a = Z BMD
[M is the midpoint of CE) measures the dihedral angle at the edge CE
(prove itl). From the triangle OMB we find
OB a 1/2 t a 1 /3 /T
‘ BM ' : 2 : 2 ~V 3

Answer: y = J ~ £ _ ; a==2arcsin
650.* The isosceies triangles BMA and FMA
(Fig. 128) are congruent. Therefore, their alti­
tudes dropped from the vertices B and F pass
through one and the same point N on their com-
mon side and are equal to each other: BN —
‘ = FN. The angle BNF is equal to tp (prove
it!). The angle p = A B A M is expressed through
the required angle a — a BMA by the formula

0 = 90“ — í -

First we find the trigonometric function of


the angle p. From the right-angled triangle
BN
ABN \ve have sin P = (a is the side of the base). From the iso-
BK a 1 /3
sceles triangle BNF we find BN = . But BK = ~ (as the altitude
<P

of the equilateral triangle ABO). Consequently,

■ R_ V3

sin (9 0 » — 2 - ) = V3
2 sin

Note. The dihedral angle at the edge of a regular hexagonal pyramid always
exceeds AFAB (compare the triangles BNF and BAF)y i.e. it is more than 120°.
1 /3
Therefore the quantity — ------- is always less than unity.
2 sin

1 For drawing a regular hexagon see the Note to Problem 508.


Chapter I X . Polyhedrons 277

Answer. a = 2 arccos------------ .
2 s in T
651. The faces AM F and A MB (Fig. 129a) passing through the edge AM
(perpendicular to the plane ABCDEF) form right angles with the plane of the
base. Find the total sum of the angles formed by the faces EMF and CMB with
the plane of the base. Drop a perpendicular AG from A to CB (íhis line should
M

Fig. 129

be parallel to CE, sce Fig. 1296J. Then P — Z ACM (prove ití). We have tan P ™
— A G = C K ~ e^ ~ (Fig. 1296). But from the triangle AMD we

have tana = ~ ; hence,


2a
4 tan a
%Y 3 V5
Since AC ±D C (prove itl), v — z ACM is the plane angle oí the dihedrai angle
at which the face DCM (as also DEM) is inclincd to the plane of the base. From
the triangle ACM we have tan , where AC ~ a ']/3 (Fig. 1296).
. Q , 4 tan a ' 2 tan a
Answer: P = arctan------ -— ; v —arctan-----r=— .
Y 'i Y 3
652. Through a straight line we can draw a plane perpendicular to another
straight line only if tbese Unes are perpendicular to each other. Let us prove
that BC X AS (Fig. 130). Draw a piano ASO through the edge AS and altitu­
de SO. Since A and O beiong to plane A SO and at the same time to the plane oí
the base ABC, tbese planes intersect along a straight line AO, i.o. along the al­
titude AD of the isosceles triangle ABC. The triangies OCD and OBD are
congruent (provo iti), therefore OB — OC, consequently the inclincd lines SC
and SB are also equal to each other and, hence, SD being the median of the
278 Answers and Solutions

iiosceles triangle BSC also serves as its altitude. Since, as has been proved, AD
and SD are perpendicular to the edge BC, then the edge BC is perpendicular to
lhe plane ADS and, hence, to AS lying in this plane, which completes the proof.
To draw through BC a plane perpendicular to >15 it is sulficient to drop
a perpendicular DE to >15. The plane BEC is perpendicular to the edge >15,
since two straight lines lying on it {DE and BC) are perpendicular to A 5. Cutting
the dihedral angle the plane ADS which is perpendicular to the edge BC yields
an angle ADE (the plane angle of
S this dihedral angle).
The triangle ASD is an isosceles
one (since the altitude SO passes
through the midpoint of the base
AD). Consequently,
LÁSD - 2 lA S O = 2a
(/.ASO = Z ADE = a as angtes
with perpendicular sides). The ratio

Fig. 130 Fig. 131

of the volume F, of the pyramid SBCE to the volume V of the pyramid ABCE
(these pyramids have a comraon base BCE) is equal to the ratio of their
altitudes, i.e. F, : V = SE : AE. Frora the triangle DSE we have
S E ~ D E ’ Cot Z ESD = DE-cot2a
from the triangle AED we find
AE = DE‘ tan a
Ilence,
F j : F = cot 2a : tan a
Answer: Fj = V cot a cot 2a.
653.* To draw a sectioa bisecting the dihedral angle at the edge AD (Fig. 131)
it is necessary to have the plane angle of this dihedral angle. Such is the angle
BDC, since the plane BDC is perpendicular to the edge AD. Indeed, in any regu­
lar pyramid the lateral edge AD is perpendicular to the opposite side BC of the
base (proved as in the preceding problem); furthermore, in the given case the
edge AD is perpendicular to FD. Indeed, by iiypothcsis the triangle AFD is a
right-angled ono, and since its angles at the vertices >1 and F are necessariiy

For drawing a regular triangular pyramid see Problem 603.


Chapter I X . Polyhedrons 279

acute, Z.ADF is a rigbt angle. Since OF — -^OA t R,

R
OD = yO F -O A =
T/2

^where R — The angle 9 = £AFD measures the angle of inclination


« í the face BCD to the plane of the base. We havc
00 R t R_
tan 9 = =V 2
: OF “ -\/2 ' 2 =
Note. The lateral edge AD forms a right angle with the edge BD (and the
«dge CD)\ since the pyramíd is a regular one, the edges BD and DC also form
•a right angle.

Answer: V = 1V 2 ;. (p= arctan ~\/2.


24
654.* The only quantity which remains
unknown and is necessary to compute the total
surface area of the pyramid is the slant
height ND. It is determined in the following
way: first íind the line-segraents A M and MD
(Fig. 132) into which the edge A D is divided
by the perpendicular NM (N is the midpoint of
BC). Tuen_ from the triangle A NM, wbere
AN —1 ^ 5 , find MN, and finally, from the
triangle NMD find ND.
From the given condition it is not clear Fig. 132
which ratio — A M :M D or MD :A M - -
«qual to........
m :n, n, tueruiuiu
thereíorc we
we muj
raay put
put mMD
u = m x , MA = nx,
nx, so
so mai
tbat A D ■
(m 4" n) x. From similarity of the triangles AM N and ADO we have
AM AN
~ Ã Õ ^ lD
where

AN
«nd
2
A0 ~ T
We get the equation
nx (m -)- n) x v i_ ± y ±

where

V 2 «("■-!-») ’
280 Answers and Solutions

so that
MD = and A M = -
~\/2n (m -f n) 1/2 n(m-j-n)
Furthennore

and
ç2 {n_~ 2m)
ND* = M D *+ M m -. :

Now we find
q * V 3 , 3q-ND
$ total'- 4 ^ 2

w = ü V Í .[i+ / i í í ± ^ ] .

655. We have {Fig. 133): / BDtA = a and jLBD í C — a (prove itl). The
triangles BD\A and BD^C are congruent (prove itl). Consequently, the base

ABCD is a square with side a = d sin a. Then we find


ADt — d cos a
and
H = 1/ADl - AD2 = cos2 a - d 2 sin2 a = djVcos 2a
The plane ACDt forms with the plane of the base the angle
DDi
9 = / DOD^ tan 9 =
: OD : 1/2 '

Armuer: V*— d3 sin2 a V cos 2a; 9 = arctan ( \.


T \ sin a /
656. The angle EOC — a {Fig. 134). To construct the angle p formed bjr
the line-segment OE with the lateral face B B i C i C draw OF jl B C . Then FE ie
Ckapter I X . Polykedrons 281

the projection of OE on this face, and so zO E F = {5. Let us introduce the follow-
ing notation: AB — a, BC = b and CCt — c, tben V = abc and 5 ÍQt =
88 2(a - f c) b. From A OEF we have
= OF = m sin P = m sin 2a
FE — m cos p = m cos 2a
from A OEC we have
4 r= E C = m sin a
from A FEC we have
6
—FC = 1 / FEí — ECz = m~\/cos2 2a —sin3 a
Reduce the radicand to the form convenient íor taking logarithms:
l + c°s4 g 1- c o s 2 a cos t o + cos2g
cos 3a cos a
Hencc, 2 2_________ 2
b = 2m ~\fcos 3a cos a
Note. The angle P — zO E F is less than ZOEC ~ 90° - a (compare their
sinesí). And since by hypothesis P = 2a, then 2a < 90° - a. Hence, ít must
be o < 30°.
Answer: V = 8m3 sin 2a sin a ~\/cos 3a cos a

Slat 10 m2 sin -~ -c o s ~ ~\/cos 3a cos a


657. (a) Drawing. The seraicirclc is reprosented by a seraiellipse (AB a
diameter of the ellipse; Fig. 135*), DC is drawn parallel to AB. Straight lincs
perpendicular to AB. are represcnted by
straight lines parallel to the tangent
imes AM and BL.
(b) Solution. Let us introduce the
following notation: AB = a; DC = b;
DP = CE = k\ then
a -fb
hll

By hypothesís a — 2R\ the side b is


K>und by the iaw of sines from the triangle
DCD, in which ZDBC is measured
by half the arc DC = 2a; we have b —
— 2R sina. From the triangle ODF, where OD — R and ZAOD is measured
í>y the arc A ü ■- 180° —2* ■ 90° — a, we find
2j
h = PD = R sin (90° — a) = R cos a
. Tbe altitude / / is found from the triangle A tAD, where Z A XDA — a (prove
>tl) and AD can bedeterrained from the right-angied triangle ADB, where ZABD
* For constructing an ellipse see Problem 613.
282 Answers and Solutions

subtended by the are AD is equal to (45° — . We get

H ~ 2 R sin ^45° — j tan a


Consequently,
V = 27í3 (1 - f sin a) sin ^45° — ~ j tan a cos a

After a nuraber of simplifications we get

1-f sin ct — 2 cos2


and so on.
Answer: V — sin 2a cos ^45°— .
658. The projcction of the diagonal D\B on the lateral face A A fiJ ) (Fig. 136)
is ADú thereíore /_ADiB = (5. The angle a between the cutting plane DBB^Dx

and the face ADD^Ax is measurcd by the angle ADB (prove it!). From the triang-
le AI)iB we find AB and AD ú from the triangle ABD we find AD. DDi = H
is determinod from the triangle ADiD
d
U — "\/AD\ ~AD'~ y d»sin2 a —d2 cos2 a cot2 a = Vãín* a — cos4 a =
sin a
d
V — cos 2a
sin a
Xote. The angle p is ahvays less than the angle a (compare their tangente!)
Since by hypothesis fí •- 90° — a, we have 90° — a < a, hence, a > 45c.
From the inequality
45° < a < 90°
it íollows that the angle 2a belongs to the second quadrant, and so cos 2a < 0,
and - cos 2a > 0. For computation purposes it is convenient to substitute the
Chapter I X . Polyhedron; 283

«xpression cos (180° ~ 2a) for —cos 2a, since lhe angle 180c — 2a belongs to
the first quadrant.
Answer: V — d3 cos a cot2 a V co s (180° — 2a)
659. The drawn lines areAjjV and B XM (Fig. 137). The quadrilateral A ,
is an isosceles Irapezoid (prove it!). From the isosceles triangle MK.X. where
/LMKN = a and MN we have

,, ~ b t a
KD — — cot —
4 2

From lhe triangle A{KB k we find

KD^ - T c o t T

Adding thcse equalities, we obtain


nn 3b a
DD, ——— cot -7T
4 2

From lhe triangle DEDV where D E ^ -CE b 1 /3 , we find

II = ED. col2- <'ot20ü°

_ 36
” ~4~ / (c *}('

® i 1 ( 0 0 « ^ ~ ) s i n (<Í0’

4 sin - y sin b03

Aítíurer: V — — ---- sin |t>0° - j - y j sin 11)0° -

660. To construcl the angle formcd by the diagonal ABX and the lateral
face BBiCtC we have to find the nrojection of ABt on this face (Fig. 138). The
Ppint A is projected ínto the miapoint D of BC (prove it!). The projectioa is
B\D, hence /_ABXD — a. From A B\BD we find

II ^ BBi =r -/ B ilJ t-IW *


BiD is found from the trianglo A B tD. The expression ohtaincd for // is trans-
formed in the samo way as in the preceding problem.
3a2 V sin (00°H-«) (60a— a)
Answer: S,
sín a
284 Answers and Solutions

661. The projection of the diagonal AB± on the face AAXX is AC*
(Fig. 139), hence, Z BiACi=$. The altitude of the prism
CC, = yA C $ -A C Z
where ACi is determined from A B^AC^; we have
CC\ — Yè2 tai*2& cot2 p—i>2 = b cot PV tan2 a — tan2 p=

i;usg^sio(i V si°(a + P)3in(a-P)

Answer: F = 2cosasin(S Vsin(a + ü)sin(a-P).

662. By hypothesis a~-\-2a-ME = S (Fig. 140). But írom the triangle BME
we have ~ co t~ ; hence, S=a2 f 1 -fc o t~ ) ; whence a —

From the triangle OME we now find


1 — cot -

/ / = ] / A/£2 - ( | . ) 2= - i j / a2 - ( c o t 2 - | - - l ) =

The expression cot-2 — 1 can be transformed as follows

sin (4 5 °— 2_) V 2 s m (4 5 °-i)


cot - -1 — cot - -c o t 45°
sin 45°sin -
2
Chapter I X . Polyhedrons 285

S sin
( 45° - t )
Answer: II =
/ 2 V ^ s in

663. From the triangle AOM (Fig. 141), where


180° 180°
L AOM = we have OM — -^- cot

hence,
180°
Sbase—~
From the triangle EOM we find
ISO*
H = yM E *'--O M 2 = Y y cot* 4 - - ~ cot2

The radicand is transformed as in Problem 039.


, . 180° . ( 180° a \ . / 180c
3 c o t_ J / sm ( — --------- T ) s > n ( —
Answer: V =
180°
24 sin — sin

664. Denoting (Fig. 142) OD=OA by x t we get

ODi — AO-c ot — = x c o t “
and

H = flfl, = l / o i l f - 0/J2 = I j / 'c o t * - — 1


286 Answers and Solutions

The total area S of the pyramid D^ADC is equal to

S = DO■AO + AD ■B + AO ■OZ>i = x2+ * l /2 x J /^ co t2 -2. — 1 - f * .* cot ,


whence
„ . a
S sm T
z * ~ -----------------------------------------
. a , t / õ------- ,
sin -j- + V 2 cos a + c o s - y
a

The total surface area of the prism ______


SMai = 4xt + 4.zV 2-/ I = 4x2 ( 1+ V ^ c o s a \
V si“ f /
45 ^sin -——j- "l/2 cos a j
Answer: 5 Í0Íai = ----- - -------- ^ ....................... ■
síd —h cos "2—h V 2 cos a
665. The altitude DO passes through the centre 0 (Fig. 143a) of the circle
circumscribed about the triangle ABC, where AB = AC —21 sin “ and [5C =

*8 *
= 2 /s in —- . The point 0 lies on the perpendicular KO to the side AB
dravvn through the midpoint of AB. Therefore, from similarity of the triang-
les AOK and ABL we get proportion AO: ~ AB — A B : AL, whence

2i2sin2~
M = - Z j -----= .......? = = = ,v;::.......
AL ty f 41-
tf* sin2
• <» a— l2 sin2 8

* See the Preliminary Notes to Problem 611.


Chapler I X . Polyhedrotus 287

Then from. the triangie AOD we fiud

u = v v —aoz^ i |/

and
1
BC -AL-II - F P s i „ T sin2a - ■sin2 -
3-2
The radicand may be transformed in the same way as in Problcm C56.
Alternate melhod. Let the face BDC (Fig. 1436) be the base of the pyrarnid.
Its area is S ii a i e ~ ~ l 2sin The face BDC is perpendicular to the] plane
ADL (prove it!) and, consequently, the altitude of the pyramid AOx lies in'this
plane. Draw OiE perpendicular to BD. From similarity of the triangles 0 XÜE
and BDL we have = , where from the
Lu DL
triangie ADE
ED = l cosa , BD = l and OL — l cos-
hence
l cos a

From the triangie ADOi, we find


U = AO{ = y A W ~ DO\ -
l
] / coSí - | -

Answer: V = T !3 s il 2 / »in ( « "T — j SÍíl ^Q


666. Since the triangie ABC {Fig. 144) is the projectíon of the triangie
DBC, DA is perpendicular to the base. The area of the triangie ABC is

The area of the triangie BCD is


£»2—y a2cot£
By hypothesis
a2 {cot p —cot a) — .5

whcnce a . -2S
cot (5—cot a
288 Answers and Solutions

The area o! the face DAC is and that of the face DAB, S4

= - y c / / . Consequeatly,

1 í
S4— S3 Y H (c — b) = — aH (esc a — cot a)

The altitude H is determined from the triangle ACD:

/ f= yZ )C 2 -4 C 2 = y fl2 cot2 P - fl 2 COt2 a


Hence,

5 4— S3 = a2 y c o t 2 p —cot2 a (esc a — cot a) =

2S
- y c o t 2 p —cot2 a (esc a —cot a ) =
2 cotp —cota

_ 5(1-—cosa) y/~ cot2 p — cot2 q — ç + 1 c t , / cotp -r-cota


sin a f (co tp —cota )2 811 2 V c o t p —cota

The lateral faces ADC and ADB form right angles with the base. The face
BBC forms with the base an angle which is measured by the plane angle
DCA — qp
^ AC cot a
C°s<p- DC

sin (a-j-P)
Answer: S4— S3 = 5 tan
t2 / sin (a — P)
/ cota \
q?= arccos
\ cotp /
667. All the lateral edges of the pyramid
areequal assides of isosceles right-angled triang-
les (Fig. 145), therefore the altitude DO of the
pyramid passes through the centre O of the circle circumscríbed about the base;
1
■ W e = -2 -b2sina
From the triangle DOC we find
//= V bc2 - oc2
b
1/2
the triangle ABC. Since the triangle ABC is an isosceles one, / B / l C = 90o-
—“ and, hence, by the law of sines
Chapter I X . Polyhedrons 289

whence
OC — R ~ --------- - ---------
» a
2cos-^-

Answer: V ~ — ■"]/cos a.

668. The altitude passes through the centre of the circle circumseribed
nboutthe base* (Fig. 146). The bisectors of the anglcs A El.) and BEC are abo

medians of the isosceles triangles AED and BEC. The area of the section
MEN is equal to .OE and = AK ~ l sin . From the trinngle
Eq k \ve find
O E = y E K * -O ÍC *

where EK — l cos-^- and OK = BN —•l sin , thus

OE = l j /" c ó s 2—— sin2-|-

. . a * ./" ctr P a — fi
Answer: Ssec — i2sm — y co s — ^— co s — g— .
669. Through the vertcx A\ (Fig. 147) draw planes AiEO perpendicular to
AB and A xFO perpendicular to AD. These planes are perpendicular to the base
(prove *t!>, and the linc A s0 along which they intersect is the altitude of the
paralleiepiped. The right-angled triangles A\AE and A XAF thus formed are
congruent (since they have a cominon hypotenuse A A , = c and equal angles
L A KAE = £A\AE — a). Consequently, A tE — A tF and therefore, the trian­
gles 4 , and A xOF are congruent; and hence, OE — OF and AO is the bisector

* See the Preliminary Notes to Problem 611.


10-01338
290 Answers and Solutions

of the angle RAÍ). We havei? — T /A í-C2 — OE3. SinceAE O F isasquare, OE —


= AE. A E and AjE are found from the triangle AA\E\ we get H ==
= c V s ia 2 a — cos8 a = c V —cos 2a.
Note. In the trihedral angle at the vertes A either of the two equal face
angles is equai to a, the third being a right one; consequently, the sum of two
face angles 2a must be more than the third one ( 9 0 ° ) , i.e. 2a > 9 0 ° or a > 45°.
At this condition —cos 2a > 0 , and, hence, H has a real value. The lateral edge
A A i forms an angle Z A t A O ~ q) with the base, since A O is the projection of
the edge on the base
AO
cos <p=
AAt
«V2c
Answer: V ~ a b c *|/cos (180o— 2a); S (at — 2c (a-f-b) sina
q>— aiccos (V 2 cos a)
670. The construction here is the same as in the preceding problera. The
bisector of the angle B A D is the diagonal AC of the rhombus (Fig. 148)
5 base ~ Sin a
From the triangle AA^E we find
H ^ V A A f — AE*
where A A, = a; to determine AE first
fínd AF from AA^F, and then AE from
the right-angicd triangle AEF. We get
A E ^ a cos a
a
ws —
whence

H ™ ----- - ^ / rcos2 — cos2 a


cos - 5-

. rr o i - a
Anstcer: F = 2a3sm — y
i /s.i n -3ay - s m. -ay .
671. The problem is solved analogously to the preceding one. We can use
the same figure (148), introducing the notation: L BAD = a and / A j AD — <p
instead of a — /.AiAB.

Answer: F ~ 2a26 sin - j / s i n |cp— sin ^ q>-i— *


672. The base ABCD is a rectangle (Fig. 149). To construct a plane angle
of the dihedral angle £>jAC£> draw a plane through the edge DD{ and perpen­
dicular to AC. The lines along which this plane intersects the faces of the
dihedral angle DiACD form the plane angle D XED — q>. We have
DE _ hj
cosç> =
DiE h
Chapter I X . Polyhedrons 291

Let us introduce the following notation:


AB = DC = a
BC = AD = b (a > b), DDt = II
D tE = h, DE *= ht
In the isosceles triangle AOB thc sum of interior angles at the base A B
is equal to the exterior angle 2a, hence, l BAC = a. From the triangle ABC
we find
a — 2R cos a; 6 — 2li sín a
From A DEC, where CD = a, we find
/tj = a sín a = 2R cos a siu a and EC a cos a -- 2R cos2 a
From A D tEC we find
k =- £C*tan P = 2R cos2 a tan P
From A D iDE we find
H = Y Í ) tE 2 - D E 2 - y Ã2TTJ cos» a tan2 p - -'./^ in 2 a cos2 a =
-- 2R cos2 a y t a n 2p — tan- a .
Transíorra the cxpressio» tan2 P — tan2 a as in Problem <>59.

Antwer: S[a( — BR2 cos a cos (tB*— a) scc p * y 2 sin (P-f- a) sin (P --a )
5 «ec “ 2/?2 cos2 a. tan p; atccos ^ ^ j
673. If the leg AC (Fig. 150) subtends the are equal to 2p, then £ ABC i»
®QuaI to p as an inscribed angle having the «ame arc. The plane passing througl*
the diagonal B%C perpendicular to tne face BIiyC\C must nass throiigh AC,
AC is perpendicular to this face; the plane angle of the dihedral angle
BtACB is a. B\CB ~ p. The hypotenuse AB is the diameter of the circumscribed
circle and, hence, AB — 2li. Let us denote: BC — a, AC b and AB ~ c.
A quadrangular pyramid B\AA\CSC is cut off the prism hy the plane A CR,,
Sínce the volume of the pyramid B^ABC is equal to one third oi lhe volume
292 Answers and Solutions

of the prism, the volume of the remaining portion, i.e. of the quadrangular
2
pyramid BiAAiCtC is equal to -y of the volume of the prism. If we denote the
volume of the pyramid B\AA%CiC by Vt, and the volume of the prism by V,
then
abfí
3
from the triangle ABC we find a and b, and from £ B i B C , H. For the lateral
-surface area we get the following expression:
■Stat — ( 2 R c o s p - f 2 / í s i n p - f 2 B) •2 ü f c o s p t a n p — 4 B 2 s i n p { c o s p - f s i n P - f 1 )

The expression in parentkeses can be reduced to the form convenient for taking
‘iogariíhms:
cos p 4- sin P- f 1 = (1 4- cos P) - f sin P =

— 2 cos2 -— - f 2 sin cos -|~ = 2cos |cos -j~sin J=

= 2cos-~-|^sin ^90°“ — J 4 -sin -~ J = =

= 2 c o s -~ -2 sin 45° cos ^45°— j = 2 ~\f% cos — cos ^45°—

Answer: S[at~ S ~\/2Rz sin Pcos -~-cos ^45° — j

r ^ - ^ - ^ s i n f l s i n 2p.
674. The altitude EO (Fig. 151a) passes through the centre O of the circl
cimimscribed about the trapezoid A B C D *. The ares AD, DC and CB (Fig. lnlft)

are equal {since by hypothesis the sides AD, DC and CB are equal),
and J.B — 180° — a is measured by half the arc ADC . Hence each oí the ares
* See the Preliminary Notes to Problem 611.
Chapter I X . Polyhedrons 29.?

AD, DC and CB contains 180° — a; consequently, the arc AmB is equal to


360° — 3 (180° — a) — 3a — 180°. From the triangle AOB, where AB — a,
we find:
AO = R =
3 a -1 8 0 ° 3a
2 sin 2 cos

|the quantity cos is negative, since a is an obtuse angle and so 135°<


3a
< < 2 7 0 ° j . From the triangle ODC we find

180o —a
DC = b=2 Rsi n
3a

From the triangle ADF, where AD — b and Z.A — ISO0 — a, we find the alti­
tude of the trapezoid
a
a sm a cos - y
DF = h = b sin a = — ........ ,,---------

From the triangle BOE (see Fig. 151a) where OB = R and /_OBE = we find
/ / — R tan (5. The arca of the base
3a
a2 |c
S= y (« + &)/! = 3a
2 cos2 2 cos2 -

. a3 sin3 a tan 6 a3 sin3 a tan 8


Answer: V ----------------- ------- —------------------------- n— -
12 COS3- y - 12cos3 í í 8 0 ° - y - )
675. The altitude EO passes through the centre O of the circle circmnscribed
about the trapezoid ABCD* (Fig. 152). The angle ACB — 90° must be subtend-
ed by the diameter as one inscribed ir> this circle. In other words, the centre O
lies on the side AB. The trapezoid ABCD, being inscribed in the circle, is an
isosceles one, and thus /_DAB = Z.CBA.
Let us introduce the following notation: AB — a; DC-~b\ Z AEB — ^,-- 2a.
By hypothesis, —5'and from the isosceles triangle AEB we have a —

~- 2U tan ~ = 2II tan a.


From the two equations we find
H rs S cot a and a — 2 "\/S tan a

Soe the Preliminary Notes to Problem 611.


294 Answers and Solutions

The side b = DC is determined írom the triangle ADC inscribed in the circle
o£ the diameter a. In this triangle
JLDAC = £ D A B - £ C Â B = £CBA - Z CAB
Sínce the triangle ACB is a right-angled one, /.CBA ~ 90° — Z.CAB. Hence,
Z.DAC - 90° - 2 I C A B « 90° — 2a
and \ve have
b = a sin (90° — 2a) = a cos 2a
Finally,
CN = h ~ A C -sin a — a cos a sin a
Now \ve get

V —- L , a + t = + 2a)cosasin a // —

- i - 4 5 tan a2 cos2 a cos a sin a ~]/S c o t a s S' S,^ g V ^ c o l a

The face ABE forras a right angle with the plane ABCD. To determine the
angle q>t formed by the face ADE and the plane ABCD drop a perpendicular

from O onto AD (it is depicted by a straight line OK parallel to the diagonal BD


so that the latter is perpendicular to AD; the diagonal BD is not shown in the
drawing; £ E K 0 — <pt). In the triangle AOK the angle OAK is equal to
Z.ABC = 90° — ACAB = 90° — a. Therefore
OK = ZO-sin (90° — a) — ~ cos a
and
tan // _ 2H 2II________ 1
a <^í — OK a cos a 2H tan a cos a ” sin a
To determine the angle cp2 formed by the face DCE and the plane ABCDt draw
OL _L DC; AELO = tp2. Since Oh — NC — h, \ve have
_ // _ lí____________ 1
an<*’2“ cos a sin a ~~ 2sin2a
Chapter I X . Polyhedrons 295

sin22a
Answeri V = l / £ 3 cot a
[<Pi —arctan (esc a)
— arctan ^ -y c s c 2 a j
676. It is required to determine (Fig. 153) the sum of the areas of thetríang-
les ABC, ABD and ACD. The area of the triangle ABC is equal to
S, = - J AB-C£ = - r * 1 y3
The area of the triangle ABD is equai to
1 1 CF
~ A B - D £ = AB-~----------
2 2 cos q cos q
the area of the triangle ACD is equal to
s 3= l - A C - C D ~ ~ Aü-CD = y AB-CE-lan q —
Consequently,

SM = Si + S, + S3 = ■ ( 1 -r cos <p+ sín rf )

The expression in parentheses is transformed as in Problein 673 to be equai


to 2 1 /2 co s ™ cos ^45c — ^ j . If in the denominator of the formula for S!at we

subslíuitc sin (90- ■q) for cos ((., then the expression for S/a< can be reiluced by

cos ^45° — |

a2 1 /6 cos 9
Answer: Si.

677. Since the plane of the base ABC (Fig. 154) passes through AC, and the
cutting plane A tBC\ through A& i parallel to AC, lhe edge flfN of the dihedral
296 Answers and Solutions

angle (3 is parallel to AC and AiCi, Therefore to construct the plane angle draw
BD ± AC and BDi X A tC\ (D and are the midpoints of AC and A tCi). We
have
Síal = {2AB + AC)-DDi « (2AB + A C ) - B D - tm $ =
= 2a2 (1 -f cos a) sin a tan (3
2
The volume Vx of the quadrangular pyramid BACC íA í is equal to y of the volu­
me V of the prism (see Problem 673) and, hence,

-S -DD i

where

- a2 sin (180p— 2a) — -*-a2sin 2a


2

Answer: $ j af = 4a2 c o s 2 - sin a tan j


a3
Vv sin 2a sin a tan (3.
678. As in Problem 630 let us prove that the
face DCE (Fig. 155) is inclined to the base ABCD at
an angle a = Z.ADE and the face BCE at an equal
anglea = / AB E ; both faces are right-angled triangles ( , / £ £ £ = £C B E — § 0o).
The area of the triangle ADE (as also the area of the triangle ABE) is equal
to Sj = i^AB *AE . From the triangle AB E , where BE = 2R, we find
AB = 2R cos a; A E = 2/? sin a
and thus S\ — 2R* sin a cos a.
The area of the triangle CDE (as also of the triangle CBE) is equal to

S, = ^ -B C .B Ê = 2ff=cosa

We have
St0tai — S 4- 2Si - f 2S2 = 4fí2 (cos2a ~f cos a sin a - f cos a) —
= AR2cos a (cos a -f- sin a -f 1>
The expression in parentheses is transformed as in Problem 673.
Answer: Síota[ = S ~\/2R2 cosa cos ~ -c o s ^45° — .

679. The cutting plane ECD (Fig. 156) parallel to the hypotenuse AB inter-
sects the face ABB\A\ aíong a straight line ED which is parallel to AB. Drop
perpendiculars CM and CF to AB and ED to get a right-angled triaDgle CMF
in which A.CFM = j3 (prove itl). Consequently,
A CMF = AC MB
(they have a common leg MC and LCBM — 90° — a and by bypotbesis 0 —
= 90° — a).
Chapter I X - Pohjhedrons 297

It is required to find thc volume V oí lhe pyrannd CA DDE, whose base


A DDE is a rectangle, and the altitude is equal to CM = a sin (5 = a cosa .
Wo have

V ~ - j - A B - M F * C M = ~ - A B - M B - C M = 4~ BC2. CM = n3 cos a
ô à
(tiie leg BC is a raean proportional between AB and MB)
Theii we have
$lat ~ (BC -j~ AB + AC) H = all ^1 ~ n"a ' -f-cot cc ^

here a/I is the area of tbc face CBBiCt, which by hypothesis is equal to the area
SatsC of the triangle CDE. Consequently,

aII = S,sec = Y AB■C f = -t- AB-CB =


Ilence,
-----(1 -i— r—------‘r cot a 1 , f-A— (sin cc —
f—1—
t- cos a
2 sin2a \ \4 -1r sin
sin a / 2 sin2
sm2 a
The expression in parentheses is transfouncd as jn Problem 673.

Fig. 156

For the plane CDE to intcrscet the face ABB^A^ it is necessary 'that 'the
líne-segment M P ^ M B ~ - a sin a he less than lhe line-segment A/Af = //= .
a2 a . . a
From the inequality a s i n a < - r we find
2 sin a 2 sin a 2 sin c
1 y ã
sin2 a < — , i.e. sin a < - i ~ - . Henco the angle a must he less than 45c.

V 2 « 2c o s ~ c o s ^45°—
a3 cos a
Answer: V - a < 45°.
3 ’I *^/aí
,aI - sin2 a
680. (Fig. 157). The lateral surfaco of thc pyramid is
„ l í 2 co t a s / / 2cotfl II2 cot p / / 2cota
2 sin a ‘
298 Answers and Solutions

Hence,
BS
Slot — 2 sin a sin p- (cos a s in P -f sin a c o s ^ -f cos p 4 - cos a)

The expression in parentheses can be reduced to the form convenient for


taking logarithms, taking into account that cosa sin P 4 sin a cosP =
«s s m (a -fp ) and
cos a 1
p -f- cos a = o2 cos —
Gt-fP a—
~-Z- cos — ^f
We get
sin(a ~r P)-f 2cos ■- ■y - cos - ^ ^ —2sin - y ^ -cos-
> 0 a-t-p / . a -fP a —P \
4- 2 cos -i— JLcos -2- -=2cos— —- ^sin — ^ -- 4. cos —— •J

Substituting sin ^90°“ -^— -^ for cos a - P and transforming the ex­
pression in parentheses, we get

4 cos - ^ cos |45°— j cos ^45a —

2 fí2cos — cos ^45°— cos ^45°— | -j


^nsircr: Siat = -
sin a sin p
681. Let r = ChV be the radius of the circie inscribed in the base of the pyra-
mid*. From the triangle DON (Fig. 158) we have DO = H = r tan a. Since

the centre O oí the inscribed circie lies at the point of intersection of the bisectors
of the angles A and B, /.OAM = ~ and

i OBN = ..= 4 5 ° - -

* See the Prelímmary Notes to Problem 617.


Chapter I X . Polyhedrons 299

Since the angle C is a ríght one, the quadrilateral MCNO is a square and MC =
= CN = r. Hence,

AC = b = AM + MC = r ( c o t - | - + l j
■and

CiJ = <j = r £cot (45o — j - ) + l ]

The bracketed e.xpression is transformed as in Problem 662 and we get

j j V 2 r c o s -2 - V 2 r s in +-J-)
$ bai •ab —
sin ^45c —

“ r%cot cot ^45°

Consequently

1 — 3 **base‘■Í/
' = ™ r3 tan a c o t c o t ^45° — — j

This expression can be simplificd if we lake into consideration that

sin a'
tan c
cosa sin (9°“ — a) 2 sin ( 45a ---- 1 cos ( 45° — ~~ )

*Tbe lateral and total surface arcas can be found by the formulas
, a
c ^$base C^S2 -tj~
...... cosa ...... " ..

r3 cos2 - r2 cot -
Answer: F = Slat =
3 sin2 ^45 -
2 )

r2 cot - - COS2
$total =
sin2 f 45g
f )
682. The plane cuts from the prism a pyrainid BtABC (Fig. 159), whose
altitude passes through the centro O of the circlo inscribed in the base of the
pyramid; therefore all the lateral faces are inclined to the base at ono and the

* See the N otes to P roblem a 617 and 618.


300 Amwers and Solutions

same angle a, consequentiy,

base COS" -
$ total=

We find
BC-AD
$bai --DC-AD.

From &OCD, where OD — rt and /O C£> = — , find D C = r cot — From

A ADC find J4Z>= .OC*tan a = r cot - tan a . Hence

2r2Co t 2 y tanacosZ-^-
Sba se~ r2 c o t 2 t a n a and Stotat =

The obtained expressions may be simpiified by representing tan a in the form

Jf 2 sin - y cos - y
sina
cos a cosa

The volume of the prism

V — Sbase H
where
H = r tan a

4r2 cos4 - y cot - y


4nwer: Stotal~
cot2 a

V = r3 cot2 - - tan2 a.

Fíg. 160 683. From A BMC (Fig. 160), where


/ MCB = 45°, and L MBC — 180o—
— (45° -f-a) — 45° = 90° - a , according to the iaw of sines, we have
BC m
sin (45®- f a) sin (90° — a)
Uence,

g ç = fl= - S . ?.Ín(45° + t t ).

* See the Notes to Problems C17 and 618.


C h a p ter I X . P o ly h ed r o n s 301

From A ABC we find


m sin (45°-t-ct)
AC = b = a cot a -

From A DCM we find


II ■
- m tan a
The angles DNM and D KM are plane angles oí the dihedral angles OACB
and DBCA\ they are equal to each other, since the following triangles are eon-
gruent pairwise: MKC and M.XC (by hypotemise and an acute angle), DMK
and DNM (by hypotenuse and a legj. Lct us denote thein by cp; tlien tan n —
H , m
= W where MA = y 5 '
1 ,s in 2 (45c -cc)
Answer: V = q>—arctan ( y 2 tan n.).
6 cos2 a
684. Let ABE (Fig. 1(>lo) be the first, and ADE the second lateral far
By hypothesis they are inciined to the base at one and the same angle cs. Co

sequently, tbe point O, through which the altitude passes, lies on the diagonal
AC. Indeed, if we drop perpendiculars OM and O.X* from O (Fig. lGlb) to the
sides AB and AD, then /OME — a and /.ONE> - a (prove ití); hence,
OM = / / > t a
and
ON =f!//;cot a
*•0. OM as ON. Hence, the point O lies ort the biseetor of the angle BAD, i.e.
0Q the diagonal AC of the rhombus ABCD.
But then we also have OMt — ONt {OMj and O.X{ aro extensions of OM
and ON), whence it follows that the trianglcs 0\f,E and ON,E are congruent
and, consequently, / QN% E -= / OMxE, which completes the proof.

* On the drawing (Fig. ICla) one of lhese perpendiculars, say OM, may he
shown by an arbitrary straight line, but the second ono is then constructed m
a quite definite manner, sínce MN must he paraliol to the diagonal BD. It is
easuy proved in Fig. 161b.
302 Answers and Solutions

From the triangle OME we íind OM = II cot a and from the triaogie OMtE
we have OM\ = II cot p. Consequently, the altitude of the^hombus is equal
to h — MAU = / / (cot a -f- cot p).
Hcnce,

V = -|- = 4 " ahB = T 0íí2 (COt “ + COt W


—<7(fe-f-MiT + ATt£)
where
H II
ME =
sin a sin 6
Then
/ i 1 \ / 1 - f cos a 1-f cos P|\
S to t al ~ QH ( co ta -j— :------- bcot B -f • q ) — aH
\ sin a r ' sin p / V sina sin p /

Expressing the numerators and denominators through and — and redu-


cing the fractions, we get

St ot al^ afI ( cot-f- + COtT )

sin (a-t-p) .
Answer: afí2 (cot a - f cot P) —
sin a sin P ’
a -f P
aH sin
2
Stotal ” cot -T + r a t - j ) a
sin P sin
2
683. Let Z.A (Fig. 162) be the acute angie of the rhombus, so that AC is the
greater diagonal and £OA D — Draw M K 1 AC and MN }_BD*. Let <?•
be the angie at which the plane EAC is inclined to the base. Then ^ M K E — 9
and £.MNE = 9 . To detennine H express MK and MN through / / ; we obtain
MK — II cot 9 and MN — H cot 9 ; substituto these expressions iuto the rela-
tionship
a = AD = AM + MD = - Í ^ L +
. a a
sin — cos

YVe get
cot 9 \
a^II
a I
cos T J

* In Fig. 162 MK should be drawn parallel to BD, and MN parallel to AC r


since the diagonais of a rhombus are mutually perpendicular (see the footnoto
overleaf).
Chapter I X . Polyhedrons 303

a3 sin a
Answer: V=
^ / cot tp cot 9 \
I . a ^ a
\ m 2 C0S 2 /
tvhere the larger diagonal of the rhorahus serves as lhe edge of lhe dihedral angle
9 and the smaller one, of the dihedral angle 9 .
686. The line-segment AB (see Fig. 163) depicts the hypotenuse of the base.
To construct the plane angle a we have to intersect the edge BBXby a plane per­
pendicular to this edge. In this case such a plane can be drawn through the leg
AC. To prove this, we have to prove that AC X BB

By hypothesis the vertex B t is projected into the point D (the inidpoint


of BC) which lies on the leg BC. Consequently, if a straight line KLi> drawn
through B and perpendicular to BC, then KL is also perpendicular to BBK(the
theorem on three perpendicular*}. And hence AC 11 KL, AC ± B B t, which comp­
letes the proof.
Through AC draw a plane AEC perpendicular to B B The lateral suríace
A
area of the prism is equal to the perimeter CIC -f- C - f A E of the perpendicular
section figure multiplied by lhe edge BBt. From the right-angled triangle BCE,
where Z.CBE — p (prove it!) and BC «, \vc find CE — a sin p. The straight
«no KL, and hence, the line AC which is parallel to it are perpendicular to the
face JSBjCjC. Thereíore the triariglc ACE is a right one at the vertex C. Hence,
AC = CE tan a and AE = - — , thus
cosa
C E + A C + A E - a s m f t ( l + tón d f g )

From the triangle BDBit where B/) = “ we find tho edge BBt. We get

hcncc'

S,a, = [CE + AC + AE)-BBl = f - ^ - ( l + tan a í -----— )


2 \ cos a /
304 Answers and Solutions

Transforra the expression in parentheses in the same way as in Problem 673,


and co s a in the same way as in Problem 68i.

a2 tan p cos ■-
Answer: Siai — ------------------------- — - .
V2sin —y )
687. As in the preceding problem, let us prove that the edge A A X ± BC
(Fig. 164), and kence, B B t j_ BC and the face BB iC xC is a rectangle. £ A XA C =
= /.AiAB = 2a {for the proof see Problem 669) and, consequently, the face
AA tCiC = A A i B tB. Point E is the midpoint of tne side A B and EO i A B (O is

the centre of the circle circurascribed about the triangle ABC); then A XE A.AB
(by the theorem on three perpendiculars). According to the law of sines we have
AB = 2R sin (90° — a) = 2R cos a;
then
s ba$e — - y 2a — 2R2 cos2 a sin 2a.
From the triangle A A tE we have
AE AB R co s a
A í4í = J=
cos 2a ~ 2 cos 2a cos 2a
From A AAxO we find
//== "),/p — /?2—— Ü-— 1 / cos2 a — cos2 2a
cos La
(Transform the radicand in the same way as in Problem 656). The side BC. =
= 2fiD = 2 -/lF -sin a . Hence
R
“ Sba$e‘ / / = 2i?2 cos2 a sin 2a "V cos2 a — cos2 2a =
cos 2a
— 2J?3 cos2 a tan 2a "]/cos2 a — cos2 2a
C h a p le r I X . P o h jh e d r o n s 305

and
•S|oi = 2S/lllBiB + SBBlClC = 2l-/lli-sin2a4-2! AB.sin a »
21 -AB {sin 2a -j-sin a)
Answer: V ~ 2 R 3 cos2 a tan 2a V 's*n 3a *iu a:
0 . 3a a
8i(- cos2 a sm —— cos—

688. Draw the altitude 0 3/ in th<? trianqle OCE (Fig. H>5); thon /_ BMD ~f>
(prove it!). Denote OC — OB hy x and fiud x frotn the formula OC- — CE-CM,
where C E ~ l and CM = '\’' x^ — OM-■ From the Iriangle OMB we find

<3.1/ = OBcot |- = x cot


hencc
_

CM = X b J l - o o l 2 i

8uUsÜtuling into the formula O í2 - CE ■CM, \\e gel the equatinn

X 2 -ÍX j / t - C O ^ - f

lhe root x —0 does ool obviously meet the given comiitiou and wc have

X : = O C ^ t y ' 1-COt2-^
Consequently,
// = '\/CE2 —OC2 V í 2 -- .c2 I cot -
Now we find
V ■--- ~ 2x~ !l

Note. The quantity of cos ff is negatíve, since -&•> 45° tan ---
OC
' ~ W ’ bul lbe *ncl*,ie^ OC ^ longer lhan the perpendicular OM,

hence tan -| -> 1 j .


R
«> o ft . cot -íj-costf
F = 4 p cot 4f ( í - cot2 j - - T P -------— r,---- •
d 2 V - / .i sín2 J1

669* From the triangle Axl'E (Fig. í 00), where £ AXFE~ a. we find
F E ^ H c o t a and from the triangle AXCE, where AtC ~ d, we fjiid EC ---
20—01338
306 Answers and Solutions

— '\/d2 — II2 and, consequently,


H2

Now we find the sides of the bases


AB = a = EK + EF
and
A xBt = EG = b = E K - G K = E K - E F
So that for the quantity
a2-f"aM~&2
entering the formula for the volume of a truncated pyramid we get the fol-
íowing expression:
fEK + £F)2 + (EK - f EF) (EK — EF) -f (EK - E F f = 3£Â'2 +

ylnsirer: V - 11- (3 .£ Í 2 + £ f 2) = 4 r l3 (d2- //2 / r 2//= cot= a).


3 : ' 6
690. We can use the same drawing (Fig. 166) as in the preceding problem,
introducing the following notation: AÂi — l and /. AiAC =>$. From the right-

From

the trianglo AA^E we find II ~ l sin {3 and ^ lA ^ /co s B , hence FE = — ;


1/2
consequently,
I .. „ I cos
b ^ E G = F K — 2FE = (1 — 2 cos- P) =
l/2 cos (3 1 /2 cos ti
C h a p ter I X . P o ly h ed r o n s 307

Now \ve get

~ {a2 4 a b - r t / l ) —- í ~ i i í L (1 — cos 2fj f cos2 2{1)

lí both lhe numerator and deiioininator are imiltiplied bv (1 t cos2p) (appiyíng


the formula for a sum of cubes), we get a somewliat siropler expression.
Note. The angle p must exceed 45°, since / A' > 2-I'E. Therefore, cos 2 fl< 0 .
. f3sinP . „ -n I3 si» p (1 -ç-cos3 2p)
6cos2 ^ 12cos-»p
691. From the Iriangles AAtE and EAXC (Fig. 167) * we have
A E / / c o t a and EC ~ II cot|5
The lateral surface area is equal lo

Slal = A . l ^ L . A lX ^ 2 { a + l,).AsN

The slant height AiN is found from the triangle AtEN, wliere
rv
EA AE—~ —
..... 11 — cot
f a
V 2 y ■>
We get

/ ■ 4
The sum
a-í b ^ A B f AiBt - 2AiBi -t 2AN = 2•NB - EC■V 2 - H ■V 2 cul p
Consequently, ___________

Si0t 2 // V 2 <<>l // M — cot2 a

Answer: Siat = 2 //2cot fi V 2 ^ cot2a.


692. In the triangle A^EN (Fig. 107), where

E.X AN ; ~ ( ‘^/3— l)
we find
//-/!,£ ~~ ( 1 /3 — 1} tan y

A,N
g ( V ã ..D
2 cos y
We obtain now

y _ -§ .(3 o í-t “2-i ‘ 2 V5) = 4 ( V 3 - 0 U I V 3) lai> T

• For depictíng a truncatcd pyramid see page 261.


20 «
308 Answers and Solutions

and
a (V 3 -0 2a2
S,at = 2 ( A B + A tB t) AtN = 2a 0 / 3 -j- l).
2 cos v cos 7
Consequently,
2d2 (14-2 cos y)
•^íoíoi“ ^/aí 4~3a2-í-a 2 =
cos 7
The expression in parenthcses can be reduced to a form conveoient for taking
logarithms

- ' - “3 ( 3 V ã “ 1 ),!,n 2 = ,0 .7 a 3 t a „ T;
6

^ 8d2 cos ^ ~ -f 30° j cos ^“ —■30° j


. 2a2 (i 4-2 cos 7)
ÍoÍqÍ CÕS'V — — COgy
693. Let us denote the side of the cube by x (Flg. 10S). From the simi-

larity of the triangles EO\K\ and EOC \ve have


EOi OtKi
Etí " OC
Here

E O i ^ E O - O O ^ r I I - . r, EO / / , 0,A*.
V*
Consequently.
H -~x ____ _ j r _____
u "'y iy ir z -r ã
u y - m — ir-)
Ansu er:
I I 4 y'2 ( r - i r - 1
C h a p ter I X . P o ltjh e d r o n s 309

694. From lhe triangle EOF (Fig. 169), where OF ^ and _ OEF —a . we

have H — — co ta . Consequentiy, tíie volume oí the pynimid

K = 4-a2//=-i<iScota
ò b
Let us exprc-ss the side a througti the edge of the cube i - M M t We have
a = 20F — 20M -f 2MF ~ KM 4 2.1/d/j - tan a = -r \/Tl -r tuna
ConsequeDtly,
-r3 (V 2 -i-2 tan a )3 cota

Here x3= V i is the volume oí lhe cube.

__
/tnsu>er: •
V _
( ‘|/2 -f- 2 tan a )3 cot a .
695. (a) Drawing. Let us first depict the section A , M J i x (Fig. 170) con-
taining the “upper” face oi lhe cube (this is ;t right-angled triangle
with the ríght angle at the vertex MD. Since
the vertices K\, Lt, M\, Ah lie on the lateral ”
faces, they are found on the sides oí the triangle
AiMiBi (Mt coincides with the vertex oí the
ríght angle; MiK\ represents the hiseclor of the
r*ght angle, since ==• MXL\). Now const-
ruet the cube K L M X , A í n s í d e the
quadrilaleral ÀhLjA/iAi take an arbitrary point
Oi depicting the point oi intersection ot the
altitude DO and the íace K)L,Mu\!i and join
•t with the point O situated likewise in the
quadrilateral KLMN. Draw 0 , / l „ OxBu OlMl
and then OA, OB, OM paraliel to thern, re*-
pectively. The points A. B, C oí intersection
ot D Au DBít D M t and OA, OB, OM (respecti-
ve*y) aro the vertices of the base of the pyramid.
(b) Solution. By hypothesis, AC - d;
BC = 8; DO = 24*. Denote the edge oí the
cube by x, Then OOx = x and DO\ 24 — x. By the propertv oí sections
paraliel to lhe base oí the pyramid we have BtM t : BC DO : DO,
BxM\ : 8 — (24 — x) : 24, whencc
»S(24 — x)
BXM j
24

6ince the trlanglcs KiBtLi and ABC are similar, we liavt-


A ',L,: B{Li ^ 6 : 8

* The figure is drawn not to scale.


310 Answers and Solutions

hcrc
24 - 4 x
KiLi = x and BtLi

24 —4x
Hence, x : 6 : 8, whencc x -3.
Answer: 3.
696. The scction BCCtBi (Fig. 171) is a trapezoid (prove it!). Draw tlie plane
MNE (A/ and N are thc midpoints of the sides AD and BC) to intersect the plane
BCCiBi along N K (K is the midpoint of BiCi), We have i_NME /.MNE —
— a and Z.MNK = p (prove it!). The altitude KN of the trapezoid BCCtBt

Fig. 172

is found from the triangle KNM, where MN a and Z.MKN — 180°


asin a
- ( a + P). By the law of sines — a , -57 i.e. KN =
s in a s m (a
. -fP
. ..) si» (a + pj
Now we find the upper base of the trapezoid (SjCi); since the triangle ADE is
similar to B 1 C1 E, we have
_ _ a-KE a-KE
lí it 1
ME NE
KE
The ratio - tttt is found from the triangle KNE, where . KNE = -P and
NE ’
/_ N K E ~ a -\ -$ (as an exterior one for A KNM). We got
KE sin (a — ft)
NE sin(a-r-P)
hence.
a sin ( a —P)
B,C
sin (a 4 P)
The seetion area
a sin (a - p)
sin (a-;-P)
sin (a + P)
C h a p ter I X . P o h jh e d r o n s 311

a2 sin2a cos p
Answer: SKÍ
sin 2(a-fp)
697. (a). Drawing. On constructíng tlie pyramid EHPGQ (Fig. 172) draw
the line MN of intersection oi' the planes. It is paraliei to the side HP and inter-
sects the axis OE at the point B. The end points M and N of the iine-segment
MN lie on the slant heignts EF and Ei). Draw PN and GN, UM and QM to
depict the planes intersecting along MN. Mark the points A t and C't of intersec-
tion of AB and CB with the slant heights EA and EC, respectively (.4 and C
are the midpoints of HP and QG). The angle ABC is the plane angle of the ob-
taíned dihearal angle. By hypothesis, /.ABC — 90'. i.e. the triangle ABC is
an isosceles right-angled one and

(b) Solution. From the similarity of the triangles EMN and EDF, where-
in DF — a, wc have MN ----- a • . The angle OAE is the plane angle of the
EO
dihedral angle a, hence EO — AO tan a —— tan a. Furtherraore, EB — EO —

•BO -- -(tan a — 1). Consequently,

tan a — 1
: a (1 —cot a)

V2asin(ct—45')
Answer: M N —a (1—co ta )—
698. (a) Drawing. Draw the straight line
CM (Fig. 173) depicting the perpendicular
dropped from C to A E. Through the point 0\
« f intersection of CM and EO draw KN paraliei
to BD. The quadrilateral KCNM renresents lhe
saçlion. The proof follows from tne solution
below.
(b) Solution. Since the plano KCNM is
perpendicular to the edge A E , the sidos MK
MN, as well as the diagonal CM of the section KCNM, are perpendicular
lo AE. Since the diagonal CM lies in the piano of the isosceles triangle
£EC, it intersccts EO which is the altitude of this triangle. On the other
hand, the diagonal KN contained in the plane of the triangle BED (and, as we
®re just going to prove, is paraliei to the base BD of this triangle) also intersccts
EO which is the altitude of the triangle BED. And since the plane KCNM and
the line OE have only one common point Oit the diagonais KN and MC interscct
«t this point.
The plane KCNM is perpendicular to the edge AE; therefore the angles
EMK and EMN are the right unes. The right-angled triangles EMK and EMN
congruent (prove it!); consequently, MK MN and EK = EN. It follows
ífom the last equalíty that A All BD and thal KOf — 0 {N . Hence, the diagonais
MC and KN are mutualíy perpendicular and Sw . MC-KN.
312 Answers and Solutions

The diagonal MC ís found írom the right-angled triangle A MC, whereíu


i,C A M = rp and AC — a V ^ . Wo get MC — a ~S/l sin <p.
The diagonal KN is íound írom the isosceles triangle KEN, wherein
Z.EKN — <p. We have K N — 2-0%E cot ç , where 0%E — OE — 00%. The
iine-segment OE is determined írom the triangle AOE (or BOEy, we find OE —

^-■a ^ 2 tan tp. The line-segment 00% is determined írom the triangle OCO,,
wherein LOCO% — 90° — Z.MAC — 90c — fp. We íind
aVI
00% — 0 C 'tan (90° — q) = —^— cot <p

Now we get

A'-V -- 2-0%Ecot (p ~ 2 | tan <p— - .X .^ cot <p>j cot tp— a ~\/2(1 — cot2 q).
ílence,
Ss„c — 4- M C ‘ KN ~ a 2 (1.— cot- tf ) sin q=^ — ~ 5
s' c 2 v -t smtp
Note. For the plane KCNM, which is perpendicular to AE, to yield a section
oí the pyrainid it is uecessary tbat the point M oí its intersection w ith d C lie

on the line-segment AE itselí (but not on its extension), íor which purpose the
augle AEC must be acute. i.e. l AEC — t8n4 — 2q> < 90c. Consequenlly,
(p > 45', aml thoreiore cos 12cp is a negative quantity.
. fl2 cos2cp a2cos(lS0° — 2(f)
.•Insiccr: A,,,..— -------- :------ - —---------\------------ .
suífp sinq)
099. The quadrilateral AM K N (Fig. 174), yielded by the section oí the late­
ral surface oí the prism, is always a parallelogram (prove it3). For the section
íigure to be a rhoinhus it is uecessary tbat A M -- A N. Since the íriangles ADN
aml A EM are congruent (prove it!), DN — EM. ílence, MN is parallel to IW
and to lhe plane AHCÜ as wcü. Consequenlly, the line EF of intersection o?
C h n p ter I X . P o h jh e a n t::-

the planes AM K N and ABCD is paraliel to lhe diagonal MA (as weii as to lhe
diagonal BD) and, hence, perpendicular Io the ollier diagonal AK of the rhombus
{and aiso to the diagonal AC). Thereírom it lollows that <p ~ /.CAK is the
plane angle oí the required dihedral angle. The line OOi. joining lhe centre oí
the rhomDus 0\ with the centre oí the base oí the prism is perpendicular to the
base (prove it!).

From the triangle AOO\ wc íind


AO OB OxM , a
COS ç = : -^ 0 7 ' T

Mote. The plane drawn through the slraight lines AM and /t;Y inter^ects
the edge CC%only ií CC\ > CK. i.c. if lhe altitude oí the prism is not less than

« Vã j 1 — tan2 -
a V 2 cos a

Otherwise the required section can be drawn neither llirough lhe point A , uor
through any other point on lhe edge AA\. ____
ílnsuier: cp5=aiccostan-y- . The problem is solvable only ií II > —kJ:cosa

700*. (See the solution oí the preceding prohiem.) Since MA = AC (Fig. 175#
and B K > B D , and by hypolhcsís, Z í A 'MA, \ve liavc A C > B D , i.e. AC is
the greater diagonal of the rhombus, hence, ,/ is an ohtu.se angle, and
L BÀD, an acute one.
The angle ty=z/_OBO{ is the plane angle of the required dihedral angle.
Prom the triangle 0 0 XB we have cos ^ ~~~ , where OB-- O. I -tan -2- .

For drawing a right prism see Fig. 83.


314 Answers and Solutions

OjB-tan ~
And since O A ~ O i M ~ 0 { B , then cos© = ----- ----------- = t a n -^ -. Here tan -s- <
UjD Z ' z
•<1, since a is an acute angle.
Answer: q>= arccos tan ~ ; the problem is solvable only if

nn BD l / c o s a
DDi>------
sm T
---
701. Cf. the preceding problem. The area Ssec of the rhombus BNKM

From the triangle MO\Bi wherein we find:

líOi — MOfCot
Hence,
Sm = 2-MO; cot — = 2A02 cot -2-
v 1 4 4
AO is found from the triangle AOS, wherein A B ~ a and L ABO — We

get AO — a sin-^ -.

Answer: Ssec ~~ 2a2 sin2 cot ~~ .


702.* Let the cutting plane be drawn through the midpoint Aí (Fig. 177)
o f the edge AB and parallei to the edges AC and BD. The edge AC is contained

* For drawing a regular triangular pyramid see Fig. 82.


C h a p ter I X . P o lt/ h ed ro n s 315

in lhe plane ABC. Therefore, lhe plane drawn through M and parallel lo AC
intersects lhe face ABC along MN parallel to AC. Hence, MN is a midline of
the triangle .4J5C ^AfAr = y AC ~ y j , i.e. A' is the inidpoint of lhe edge BC.
The edge BD lies in the plane BCD, and the cutting plane is parallel lo this
edge.Therefore, NL j|BD ^ y BD = ~ j and L is the inidpoint of the

«dge CD. Similarly, we prove that MK — y , JD

■and that K is the raidpoint of the edge AD.


Consequently,
KL\\AC and A 7.= |-

Hence, the section MNLK is a rhombus-


Furthermore, the angle A'MK is a right one-
Indeed, the edge BD is contained in the plane
BDE (E is the inidpoint of AC), which is per­
pendicular to the edge AC. Consequently,
BD±_AC. But, as has been proved, MK ||BD
and M N ||AC, hence. MKJ_MN, whercírom it
follows that MNLK is a square with the
aido

Answer: Ü
4 '
703. Let CD (Fig. 178) be the lateral edge perpendicular to the base. Sínce.
l)y hypothesis, Z.DAC — / DBC ~ a. we have AC — CB, i.e. the triangle
ABC is an isosceles one at the vcrtex C of the pyramid and. hence, i>v hypothe-
ais, £ c = 90°.
Any section of the pyramid perpendicular to the base ABC is a quadrilateral
NKLM
-• with
rriiw two
m u right angles ({/_NKL
i, and /_KLM). For this quadrilateral
to become a square the following condition
co m n u u u should
>uuum be
u c satisíied:
sau su u u ; KN
n .v —- KL
i \.l -
~ LM — x. From the congruence of the triangies A KN and BLM (prove it!)
KL x t,
it follows that AK — BL, hence, KC CL, and KC — —__ t-rum the
y-2 v 2
triangle AKN we finei AK KN cot a —■ x cot a. Since KC -j- A K --- A C =
~ a, we get the equatiou
x
——-p arcot a ~ a
V2
whence
aV 2
1 -|- y 2 co la

Answer: S.,
(1 1/2 cot a )2
704. The section yields the trapezoid A//ljtf,A? (Fig. 179) equal to the lateral
tace DD íCíC (prove it!). In the cut-off portion A\BiCxD\MNCD we have A xü x —
•-M6 Answers and Solutions

= B\Ci = NC = jt/D (as the segments oí parallel lines contained between


parallel planes). The obtained solid is an oblique prism with the base CCtDiD.
Oraw the plane FGQQí through the slant height FG of tbe frustum o! a pyramid
and the apothem OG of the base; we get LFGL = a (prove it!). The perpendicu­
lar LK dropped from L to GF is the altitude of the prism (prove it!). From the
triangle LKG, wherein LG — QiF = a»
we have LK = a sin a. From the triangle
FLG we find F G = ^ L = _ £ _ . The
cos a cos a
volume of the prism is computed by
the formula
DjC i^D C
V-- FG-LK

Now we find the total surface area S


of the solid AMA\tí^NB cut off by the
plane AtBv\TM. The face AA^B^ is equaí
to the section (prove it!). Each
of these faces has an area Si = . ÇQt,

where QQ* — FG« -------. Either of the


cosa
faces AA\M and BNBt has an area
A M - A tP
, where AM — A D - = 3a — a = 2a and <4,P = FG =—— . The
trea o f the face ABNAt is S$ -= AM •AB — Za-Za. We have
5: = 25j -t-2S2T -53.

lía^cos--^-
Answer: F — 2a3 tana; S —---------------
cos a
Preliminanj JVotes to Problems 705 to 708
When solving Problems 705 to 708 use should be made of the following the-
orem.
If a polygon ABCDE... contained in a plane P is orthogonally projected on
a plane P, as a polygon A iB íC iD i then the area S of the polygon ABCDE...
and the area St of the polygon A iB í CíD i E í ..... are reíated in the following way
Si — S cos a,
where a is lhe anglc between the planes P and Pj.
Proof. First consider the case when the projected figure is the triangle ABC
(Fig. 180a). whose side AB is parallei to the projoction plane P {. Draw the
plane Q through AB and parallel to the plane P, {E is the point of intersection
with the projecting line CCj). We get the triangle ABE congruent to the triangle
AiB\Ci. Draw the altitude CD of the triangle ABC\ ED is then the altitude oí
the triangle AEB, and the angle a ~ jiEDC is the plane angle of the dihedral
angle CABE equal to the angle between the planes P and Pt. From the triangle
DCE we find DE — CD cos a. Consequentfy,

Si AB* DE = AB-DC cos a — S cosa


C h a p ter I X . P o lt/ h ed ro n s 317

Then consider the case when the nrojected figure is the triangle LMN
<Fig. 1805), whose sídes are not paraileí to the plane l\. Sueli a triangle can be
divided into two triangles oí the type considcred above. For this purpose it is
sufficient to draw the plane Q parallel to P, through one of its vórtices M wbicb
should be neither the closest to, nor the remotest írom the plano Pu This plane
iutersects the triangle LMN along the straight liue KM parallel to Pj. (f s'

Fig. 180
and 5 ' are the respective arcas of the triangles KMX and LMK. and S[ and 5J,
the areas of their projections (i.e, of the triangles KtMiNx and L,MiK,)s then,
has been proved,
cosa and N',' cos a
And since S = S'-\- S" and St ~ 51 -j- S'i. we liave
Si = 5j = S' cos a 4 - 5" cos a = (S‘ -J- 6'") cos a — S cos a
If the polygon has more than three sides, then we divide it into triangles and,
feasoning in the same way as ín the above case, prove the general lheorem.
Let us draw our altention to the facl that this theorem liolds trui* for the
freas of curvilinear figures as wcli. To prove it we have to inscrihe a polygon
}n tho givcn curvilinear figure and pass to the
limit.
«2V 3 .
705. We have (Fíg. 181) Sbase ^ ud
tí BBi ~ BD -f- DB%. From the triangles BF.l>
a,ld BxEyD (E and Ei are the midpoints of A C and
A jCj) we have
-V ã .
B D = BE tan a = “1 T t a n a
and
■y * tan p

Honce,
v-*s.base‘ H '■ 3a3 _ 3«3 sin (a. -j- P)
(tan cc - tan p);
8 cos a cos P
. The section ADC is projerled on the plane of the lower base as lhe triangle
ABC. As bas been proved (see the Preliminary Notes) the arca .Vof the section ADC
318 A nswers and Solutions

is related to the area of the triangle A B C , i.e. to the S&ase, by the formula

Sbase — S cos a» hence, S ■ ccs a ' ^ rocee<^’ DS *n fche same way {i.e. pro-
jecting the scction A tDCi on the upper base), we find that the area of the section
AiDCj is S' = f ^ * e . Consequently,
cos p

S + S'
(co s cos p /
3a3 sin (a + ft)
Answer: V =
8 cos a cos P
a2 l / 3 c -P
S -fS ' =
y 3 cos a + cos P_
4 * cos a cos p 2 cos a cos p
706. (a) Drawing. Join the midpoints K and L (Fig. 182) of the sides A B
and A D . Through the point E of intersection of K L and A C draw the straight
line E N (the angie N E C depicts the plane angleof
the dihedral angie a). Through the point O2 of
intersection of E N and the axis OOi draw P M
paraílei to BD . The pentagon K L M N P represents
the section. The proof is obvious from tne solu-
tion below.
(b) Solutíon. Since K L || B D , the plane K L M N P
(passing through KL) intersects the diagonal plane
D BB iD i (passing through the diagonal BD ) along
the straight line P M paraliel to K L and BD. The
axis OOi of the prism lies in the diagonal plane
D BBi Di and, hence, intersects P M . The plane
K L M N P intersects the diagonal plane A C C t A t
(passing through the diagonal A C) along the straight
line N E (E is the midpoint of K L) , which also
intersects the axis OOt. But since the plano
K L M N P , containing P M and EN , intersects the
Fig. 182 axis OOi only at the point Oz, both E N and M P
pass through this point, i.e. the point of intersec­
tion of P M and E N lies on the axis OOt. The íines EC and E N are perpendi­
cular to K L (the theorem on three perpendiculars); hence, Z.CEN — a.
The area S of the pentagon KL BCD is equal to the area of the square A B C D
less the area of lhe triangle A K L , thus, S — b-— — 62. The area Ssec
O O
of the pentagon K L M N P is determined accordiDg to the theorem proved in
the Preliimnary Notes to Problera 705. We have -—-ô3 = Sscc co s a , i.e.
„ 7ò2
^sec — 8cõ s a
Comparing the triangles M 0 2N and BOC (BO ~ MOz and MN > BC)*
we make sure that MNOz < Z.BCO; and since í_BCO ~ 45°, £. MNO% <
< 45° and, consequently, the angie 9 = Z.MNP is acute. All the rest of the
angles of lhe pentagon are obtuse (the acute angie z.NMOz — 90° — £ M N 0 7
Chapter I X . Polyhedrons 319'

exceeds 45°; the anglo MLK is equa! to 180a — / LM0 2 -- 180 — / X \fO-,)
From the triangle AI02N \ve have
jp MO»
ta n
2 NO»
but
OC OB MO,
A'Oi -
co s a cosa cos a
H en ce,
cp
tan -~ = cosa

Answer: Ssec — ~------- ; q>= 2 arctan (cosa).


sec 8 cos a T '
707. (a) Drawing. First draw separately the base oí the prism (Fig. 183a).
Theii construct an eílipse (Fig. 1836) depicting the circle about which the base
ts circumscribed*. Draw a diameter (MN) oí the eílipse and through its ends-

F ig . 183

tangent lincs CD and AB representing the straight iines on which the bases
j isosceles trapezoíd lie. Draw some iine KL paraüel to CD and AB to
«ntersect tho eílipse at points K and L. Through these points draw tangent lines
ÜA an(i BC to the eílipse. The quadrilateral A BCI) depicts the isosceles trapezoid
®**cumscribed about the circle. Then complete the urawing oí the right prism
fBCDAtBiCiDt. The cutting plane passing through the side A D and verte* B ,
jQlersects the face A A XBXB along the straight line A B t and the íace DD^C^C
which is parallel to AAtBtB) along ÜG parallel to ABt. The section yields the
Quadrilateral ABiGI). From the point B draw the straight line BE parallel to
tno radius OK joíning the centre O with the point of tangency K. The line repre-
vp*8 lbe perpendicular dropped from B to Aü . Consequently, the anglo BEBh
«epicts the plane angle a.

* For constructing an eílipse see Fig. 92.


320 Ansivers atui Solutions

(b) Solution. From the triangle DFA (Fig. 1836), wherein D F = MN ~


2r and Z.DAF = a, v e ímd BC — /U? == . Denote d B by a; CD,
by b; A D BC, by c. By the propcrty of the circumscribed quadrilateral

a + b=AB->rCD*aAD + BC =
sm a
U'e ha ve
2r 4r2
Sb* - s in a sm a
Consequently (see the Preliminarv Notes to Problera 705),

S ^ 4r3 _ 8r2
s' c cos a sin a cos a ~~sin 2a
The altitude II -- BBt is found from the triangle BB^E, BE being determin-

od beforehand from the triangle B E A , wherein

.4// - a _ 2.1.1/ , 2 0 .]/-cot Z OAM 2rrot -


We have
BE --- a sin a and !I — BE lan a
Hence,
H — 2 r c o t -~ sin a tan a
Now we fínd
Slat^ II 2c) ^ UI c
$r~
Answer: Siat =- 16r2 tan a cot ; Ssec — —— .
2 sin 2 a
708. (a) Dranúng. The cutting plane P may be drawn through either o!
the two diagonais of the face BCC,#, (Fig. 184). Lct us draw it through the
diagonal B C ,. By hypothesis. PSj.lD. Consequently, the plane P intersects the
plane containing the hase ABC along the straight line BK parallel to AD (the
entire line-segment BK lies outside the triangle ABC). Since tho face BCC\Bt
is perpendicular to AD, it is also perpendicular to BK; hence, /_CBC\ is the
plane angle of the dihedra! angle (5 at the edge BK.
C h a p le r I X . P o ly h ed r o n s 32

Depict now the triangle yielded by the cutting plane P. One side of íhis
triangle {BCx) is known, \ve have only to find the opposite vertex, i.e. the point
of intersection of the plane P and the edge AA For this purposc it is suífident
to join the point E , at which B K intersccts the extension of the edge AC. with
the point Ct. The point F at which CtF intersccts the edge A A %is the required
vertex.
Let us prove it. Since the point E lies on tlie line BE of intersection of the
planes P and A B C , this point belongs to the plane P. On the olher hand. the

oint E lies on the Une AC of intersection of the planes ACC 1 A 1 and ABC.
Eence, it belongs to the plane ACC\Â\ (it is actually situated on the extension
oí the face ACC\Ai). Consequently, the point E must belong to the line of inter­
section of the planes P ana ÁCC^A By hypothesis, the point C\ also belongs
to the line of intersection oí these planes. Consequently, the planes P and
ACC\A\ intersect along the straíght line C\E. i.e. on this line is positioned the
side {C\F) oí the section íound on the face CC-.AiA. Hence, the point f of inter­
section of C\E and the edge AA\ is the required vertex.
(b) Solution. Since the triangle ABC is the projection oí the triangle FBCt,
contained in the plane P, on the plane of the base, llien

* ec cosP cosp
'vhere a = AC is the side of the isosceles triangle ABC. Express a2 through the
lateral surface area S. \Yc have
S (2AC ~r BC)‘ CCt
where AC — a, BC - 2a cos a and CC\ — BC-tan p — 2a cos a tan p. Hence,
S =— 4a* cos a (1 + cos a) tan p — 8a2 cos a. cos2 ■— tan p.

5 tan
S siri 2a cot p
Answer-, S„
ItT _ 4 sin p
cos p c o s a cos2 ■—
709. (a) Drawing. Extending lhe líne-segment BC (Fig. 185) depicting
• *eg of the base by a length CD — BC, we get the point D, which is actually
2 1 -0 1 3 3 *
322 Answers and Solutions

symmetricai to B about the leg AC. Let us tako the point M in tbe middle of tbe
edge A A i and draw the section oí the prism by the plane P passing through the
points B u M and D . To this end join B j with D by a Une to intersect the edge
CCi at the point N. The triangle B iN M is the required section. Indeed, the point
D lies on the line BC and, hence, belongs to the plane CBBiC\ (D is situated
on the extension of the face CBBiCi). But the point D also iies in the plane P,
therefore it is positioned on the line of intersection of the planes P and CBB\Ci.
The point Bi is also found on this line. Hence, the planes P and BCCiBi inter­
sect along the straight line BiD. The point N at which B XD intersects the edge
CCi is one of the vertices of the section, thus, the section of the prism is the
triangle BiN M .
Since BC = CD and CN \\BBi, CN is the midline of the triangle BB\D%
i.e. N is the raidpoint of the edge CC j. Consequently, M N is paraliel to A C
contained in the plane of the base. Therefore the lin e/)£ , along which the plane
P intersects the plane of the base, is paraliel to .4 £ and, hence, perpendicular
to the face BCCtB x. Therefore /_BDBi is the
Cf fi* plane angle of the dihedral anele <p at the
—■ edge DE.
(b) Soluiion. We have (see solution of the
preceding prohlem)
_ Sbase_ Qb
sec~ coscp ~2cos<p
(where a = £C, b~A C ), and since b —a tan p,
we get
a a2 tan p
^ «"T cosç
Fig. 186
Find a2. By hypothesis, p is the smallest one
of the acute angles of the triangle A B C , thus,
6 < a and the area bH of the face A CC\A % is smaller than the area a ü of the
face BCCiB\. Therefore, the difference S of these areas (we assume that it is
positive) is equal to (a—b) H. From the triangle D BBu wherein BD — 2BC — 2a,
we find H — 2a tan «p. Consequently,
S = 2a2 (1 — tan P) tan (
whence we find a2.
.4 nswer:
tan P S sin P
(1 — tanpisinç ‘ s |/'2 ’ sin (45° — Ç) sin q?

710. The angle between the non-intersecting diagonais B A t and .4Z?j


{Fig. 186} is equal to the angle <p — í . A xBCi between B A X and the line BC\
paraliel ioADi. Wehave z.CBC{ — /_DADi — a and L A B A i = P •To determine
the angle <p find A xC\ first from the triangle A iBCx (by the law of cosines) and
then from the right-angled triangle A\B\Cx, and equate the obtained expressions.
We get
BA14- BC\ - 2 ■BA*. BCt •cos q>=* BiAf £,Cf
Hence
2 ■BA i •BCt ■cos q>« {BÂ\ ~ BtAf) + (BC\ - BjC2) = 2 . BB\
C h a p ter I X . P o ly h ed r o n s 323

Substituting
BBt
BA<
sin 0
BBj
{from the triangle BAA{) and BC{ into the last equality, we get
s»u a
cos tp — sin a sin 0
Alternate method. Through the edge B,Cj draw the plane fí^C^C-^Bi perpendi­
cular to BAi (it is possible, since B\C\ ± BA t). Let E be the point of intersection
oí BAi and From the right-angled triangle BCtE we find BE ~ BCi cos «p
and from the right-angled triangle BB tE, whe-
rein Z.BtBE — 90° — 0, we have Ü
BE = BBXcos (90° - 0) ^ BBt sin 0
New we express the line-segrncnt BB, through
BC\ from the triangle BB,Ci, wherein ^ByBCi —
= 9 0 ° — a. We get BBt — BCt sin a and, hence,
BE = BCi -sin a sin 0.
Equating the two expressions ÍotBE, we obtain
BCfCOS (p = BCj-sin a sin 0
Answer: cos cp — sin a sin 0-
711. Let us denote the dihedral angles at Fig. 187
the edges SA, SB, SC (Fig. 187) by cp^,
<Pc- Through a point (F) on the edge SC draw a plane (DFE) perpendicular
to SF. Then I.DFE <p-c . Determine ED- first from the triangle EFD and
then from the triangle ESÚ, and then equate the obtained expressions. We wjnd
FE2 + FDZ — 2FE-FD -cos tpc ^ SE- + SD* — 2 -SE-SD cos y
Hence
2‘FE-FD-cos q>c « 2-SE-SD‘Cosy - (SE*- FE*) - (SD*— FDf,
2-FE-FD -cos <pc = 2-SE-SD -cos y - - 2SF*
Substituting into this equality
FE — SF tan a
FD = SF tan 0
SF
$£ =
cosa
and

cos 0
we obtain
cos y
tan a tan 0 cos q>c = —1
cosa cos0
21*
324 Answers and Solutions

whence
cos y — cos a cos P
COS =
sin a sinj3
Similarly, we find cos and cos cps .
c o s a —cosficosv
Answer: cos 9a =
sin p sin v
cos P— cos y cosa
COS Çj5 =
sin y sin a
cosy — cos a cos p
COS <Pç =
sin a sin ã
712. Solved in the sarae way as the preceding problem.
A nswer: cos y = cos a cos p - f sin a sin p cos A .
713. See Problem 711.
Ansmer. the required angle contains 90°.
714. Let the point M lie on the face Q (Fig. 188). By hypothesis, A M forms
an angle a with A B , and MB is perpendicular to A B . TÜrough BM draw the

plane MBN perpendicular to the cdge, and drop the perpendicular MN from M
to BN. The line MN is also perpendicular to N Ay and £M A N — p (prove it!).
We also have q> = Z.NBM. The angle <p is found from the triangie N BM,
wherein MN = A M *sin a (found frora lhe triangie A NM) and B M = AM sin a
(found from the triangie AMB), We get
_ MN __ AM sin P sin P
* BM ~ AM sma~sma

Answer: sin ® — .
y sma
715. Fig. 189 shovvs the common perpendicular PQ to skew lines LL' and
M M '. To obtain the angle at which the line-segment PQ is seen from the point A
we have to draw the ray AP; then Z.PAQ = a. Similarly, Z.PBQ — p. Through
the point P draw lhe straight line PE paralle! to M M '. Then the angle between
MM' and LL' is (by definition) the angle <p = LEPB. Drop the perpendicular
AE from A to PE and draw AB (the rest of the Unes depicting aparallele-
Chapter I X . Polyhedrons 325

piped with the edges PQ, QA and PB are constructed to make the drawing more
vivid). From the right-angled triangle BPQ we find
PB = PQ cot p = h cot (1
Simiiarly,
PE — QA — h cot a
Then,
BE* = PB2 + P £ 2 — 2-PB-PE cos <p =
(cot2 a -f- cot2 P — 2cot a cot $ cos <p)
AE is perpendicular to the plane EPB, since it is parallel to PQ whuh is the
common perpendicular to PB and PE. From the right-angled triangle A EB
we find
AB* = AE* + BE2 - K1 -f BE*
Answer: A B 2 — h2 (1 - f cot2 a - f cot2 P — 2 cot a cot p cos tp).
716. See the drawing to the preceding problem (in the present problem
<p = 90°).
We have J3
BE = ~[/ PEi - f PB1 = h y cot“ a + cot*p
The anele between .AP and PQ is equal to the
angle between A B and AE parallel to PQ.
Denoting it by y, we have
4___ BE h V c o t 2 a -fc o t 2 p
i n x y - AE- h

Answer'. ta n y = "\ /co t2 a~)-cot2 p.


717. Let (Fig. 190)
D M _m j D N __ m2 DP rn3
MA nj * NB «2 ’ BC «3
Let us first find the ratio of the volume \\
of the pyramid DM NP to the volume V
of the
pyramid D ABC. Let the face BDC be the base of the pyramid DA BC and the
face NPD, the base of the pyramid DMNP. Let the edge DA be projected on
the plane DBC as a line-segment lying on DE. Then the points A and M are
projected into some points K and L lying on DE. Consequently, the altitudes
AK = h and ML = hx are contained in the plane ADE and the triangles
DML and D A K are similar. Hence,
A, DM DM mi
h ~~ DA * D M + M A +

The area S, of the base NDP is to the arcaS of the baseBDC as DN 'DP
to DB-DC (since the triangles NDP andBDChavethecommon angle D).
Hence,
Sf DN DP m2 «3
326 Answers and Solutions

Hence,
h li.
V '* 5 ' (^ 2+ «a ) (m3 + n3)

Now ' íiod the ratio Vi


, in which the volume o! the pyramid DABC
V — Vi
is divided.
Answer —J-l_______________
‘ V— (m2+ n 2) (m3+ n3) — ^ ^ ^ 3 '
718. The plan of solution: from the similarity of the triangíes OEL and
MEK (Fig. 191) let us express OL in terms of MK — b and ME — ~ ; from the
similarity of the triangíes OCE and MEN let us express OC in terms of MN = h
and ME —-y . Substituting the expressions found into the relationship OC2 =
; 2-0Lz, we get the equation for finding H.
Solution. We have
OL : H = MK : EK,

OL: H = b
var
wlience
m m
OI*=
m -m
4h*H*
similaríy, OC- , Hence,
m - m
m -m
•=2
mm
w -w m -m
Dividing by H2 and transforming, we obtain
2bk
//=
y'262I a*

Now we find
462^2 2W
0 & = i/2 —4fe2 _ A2—

and
-(2 Oi)2- //

16W
Answer: V- 3 (í,2 _ j,2 ) y 2 i ,2 _ f t 2 '
Cltapler X. Solids of fíevolution

CHAPTER X
SOLIDS OF REVOLUTION
719. Answer: 1* = -^
» y 3
720. /Insw.r: r = 4 ^ r l/4n*12— c2
24n~ 1
721. >lnsu;er: V = -~ -
4rt
722. .W < r : r = ‘i W / g s i n a
4jt
723. The radius of the base R = l s in a (Fig. 192)*, the altitude of lhe cone.
a

H — l cosa. The volume


nRWl nl3 sin2 a cos a
V
3 3
Thesurface
5 — ji/? (í-f-/?) = ;Tí2 sin a ( i - f sin a)
By>hypothesis, l-'r II — m; hence,
l:
1 -f-cos a n «a
1 2cos2 y

Answer. V - ” ff|3 s i n 2 g c o s g

24 cos® ~

nm2 sin a cos2 |45°-— y j

2 cos4 ~
724. (Fig. 193). The planes AiB\ and A 2B2 cut off the cone A C B the cones
* For drawing nn ollipse (depicting the circie in the base of a cone) see page 254.
328 Answers and Solutions

AíCBí and v42CJS2 which are similar to the given cone. And the volumes (V, Vj
and F2) are in the same ratios as the cubes of the altitudes:

The volume Fm,-d of the mid-portion AiA^B^Bt is equal to Ft—F2. Suh-


tracting the second proportion from the first"one, we find Vmid'
7
Answer: Fmíd= ^ F .

725. From the triangle AOE (Fig. 194) we find


AB a
OA - B =

From the triangle OBD we have H — R cot p.


na3cot p
Answer'. V ■
24sin3 -2-

726. By hypothesis, OC-~Ot\ =■ h (Fig. 195). We have


OC ~ /í cot p
and
OCi —fí cota
Ilence,
h
R
cot P— cot'^
Chapter X . Solids of Revolution 329

The required volume V is equal to the difference between the volumes of


the cones ACB and AC^B. Hence,

V = ~ i x W (0 C -O C ,) = y

ji/i3 nk3 sin2 a sin2 p


Answer: V -
~3 (cot P— c o ta )2 3 sin2 (a~~P)
727. By hypothesis,
nfíl = S
The area of the base
Sbase —
is equal to P — S. Dividing (by terms) the equality
nW ^P~S
by the equality
n R l—S,
p p _£
we get — ~ —-ç— . Let us denote the required angie by p; from the triangle
OBD (see Fig. 194) we have
sin P ~ -j-
p _£
Answer: P —arcsín —^— .
728. From the isosceles triangle ADAj
(Fig. 196) we find AD —--- -— . I fa is
0 . a
2sin y
tlio radian measure of the angie ADAt.
then
ABC A, - AD ■a - -...

Prior to developing the curved surface the !ine-segment AD was an eiement


oí the cone, thus
l = AD = -
2 ®*n ~2
the wcABCAi was the base circumference, hence

2nfí = ã a -~
2sin -
The altitude of the cone is
//= = - V 4 n 2- a
4n sin -
330 Answers and Solutions

g 2a 3 V 4 n 2 — a 2
Answer: V — , where a is the radian measure of the given angle.
192ji*sin»-y
729. The angle DOM (Fig. 197) is equai to the angle <ç— £DEO. From
A ODM and A OEM we find
OD = H -
COS q>
and
OE
sinq>
From A OCE we find
OE
OC~R

na3
Answer: V =
3sin2<pcos (p cos2 y

730. The radius of the base circle of the cone is R - (Fig. 198). From
V2

H= V a EZ~AO'^
a a V cos a
2sm2-s- = — ---------
2 sin - 2 „ . a

The surface S is equai to

nR (l + R )
V 2 siu - V2 \\ y sm 2>
Chapter X . Solids of Revolution 331

The expression in parentheses may be transformed as foilows:


1 . a . , . a . 90s + cí 90’
=4-sin —sin 45°-f sin -rr = 2 s in ----------- c o s ------
V 22 2 2 4 l

Answer: V
12sin •

„ / _ . a \ „ . 9 0 °-fa 90 — a
na2 f 1 + y 2 sin y j n a2 sin — ^— cos — ^—

2 y '2 s i n y sin-—

731, From the triangie AAtC (Fig. 199) we have 4C = fc o s a . From the

triangie AAtB we find A 8 = 2/í ==—í— ; thus AO


cos a

A<Oi — rz=AO— A C ~ l l — --------c o s a )


1 1 \2 cos a /
Now we find
ScuTvtâ^nl r) Ji/2 ^ — 'Cos a j

Answer: Scurved — - - .. sln a _ n /2 tan gsin « ,

732. From the expressions V = y n/?2/ / and /?= > // c o t a wo get

/ / = Y E E E Li and « = y r E .^ ta

Let it be required to halve the curved surface arca. Since the cones ABC
and AiBtC (Fig. 200) are similar, their curved surface areas S and Sj are in
332 Answers and Solutions

the same ratío as H 2= OC2 to / / f —OjC2. Consequently,

„ i:B ^ y s ~ s = y r ^ ie .

Let it now be required to balve the total surface area. Then

SubstituÜng /?i = / í i c o ta ; L = = - ~ - and l — we get


* sma sin a 6

, j r f £“ “ « + £ 2 2 ia \
1 sm a 2 \ 1 sm a /

whence / / j = H cos
2 *
1 3 / 3F tan2 a.
Answer: ií the curved surface area is balved, then Ht ~ ——1 /
1/2 *
SV tan2 a
if the total one, then H ^ c os
j V1
733. Let us denote (Fig. 201) the radius of the sphere by R, the altitude of
the segment ACB, by h and the iine-segment DA, by r. The volume V of the
2
ç sector is equal to V = nR*k. From the

triangle .4CD, wherem /.CAD = |as an

inscribed angle subtended by the arc BC —


— tt j , we fiud k ~ r tan •— . From the

triauglc ADO we have r ~ R sin-~-. C011-


sequently,

V = -I n& h = \ nRi. R sin -2. tan 2.


3 3 2 4
The surface oí the spherical sector is made up oí the curved surface of the
sphericaí segment ACB equal to 2nRh and the curved surface of the cone .405
equal to nrR. Consequently, S — 2nRh 4 nrR = nR (2h 4 r).
Answer: V= ™ sin2 ~ ; 5 = nR* sin~ ^2tan ~ 4 1j .
734. (See Fig. 201.) With the notation of the preceding problem we have
5 = 2jiRh 4- «r2. From the triangle ADO we have AOz = A D2 4 OD ; since
OD ~ R - h, then fí2 = r2 4 {R — h)2 and r2 = 2Rh ~ A3. Hence, S =
=■ 'inRh ~ n/r. Hence

k .íR ± y ^ = ’ is
JX
Since A < i ? , the plus sign is not suitable.
Chapter X . Solids of Revolution

Answer. h — 2R- 4/?2_


735. Figure 202 shows an axial section oí the solid obtained by rotating the
tríangie ABC about the side A B . This solid is made up oí two cones. Its volume

; ~ n •DC* ■AD + ~ a ■DC2 ■Dfí = r DC2 (A D ~ - DB) = ± D C 2-AB

Remember that DC-AB — 2S and DC — b sina.


. .. 2«5ôsína
Answer: V = -------------- .

736. (Fig. 203) The volume oí the solid of revolution (see the preceding
problem) ia
V = j n - D A * ( B D + DC) = y aa ■DA‘

To determine DA procecd in the íollowing way: írom the tríangie BAD \ve find
BD = DA -cot B, and írom the tríangie DAC we íiud
DC - DA -cot C
Consequently,
a ~ BD -f DC = DA (cot B -}- cot C)
Hence we find D A.
wer:. tr
v— 1 _ _ na3
_ _ _ _ na3 sin2 físin2— .

737. The volume of the solid oí revolution (whose section is shown in


Ftg. 204) is equal to the sum of the volumes of two equal írustums of cones, ob­
tained by rotating the trapezoids A MBC, and A NDC less the sum of the volumes
oí two equal cones obtained by rotating the triangles A MB and A XD. The
radius of one base of the frustum is A C d, that of the other is MB ----- ,
334 Answers and Solutions

We have

From & AOB we find


BO = ~ - t a n - l

Jid2 tan y
Answer: V —------z------ .

738. The volume V (Fig. 205) of the soiid of revolution is equal to the volu­
me of the frustum of a cone obtained by rotating the trapezoid OOxBC less the
volume of tvo cones generated by rotating the triangles AOxB and AOC.

thus,

We have (see Fig. 205): / / = èsin *~ , i? = 6 c o s ~ (found from the triang-

le AOC) and h = c s i n y ; r^ e c o s ™ (found from the triangle AOxB). Hence,

V = i (II - h) (J>~+ ltr + - i Ar2 =


o o o

sin ~ c o s 2 ™ c) {b ~ -\ -b c ~ $ - 63— c3j

.nfcc(&-f-c)sinacos-^-
/Ín sírcr: V—
3
Chapter X. Solids of Revolution 335

739. The suríace S of the solid oí revolution (Fig. 206) consists of the suro
of the curved suríaces oí two equal cones with the axial sections DADi and CBC%
and the curved suríace of the cylinder with the axial section CDÒXC\. With the
notation adopted in Fig. 206 \ve have
r = è s 'n a . h = M N = A B — 2AM = — ------2ècosa
cosa
Hence,
„ „ .... 2nò2 s in a , . . » 0 ,
S — 2nr (b 4 -h )~ ----------- (cos a + i ~ 2 cos2 a)
cos a

Answer: S = 2nb- tan a (cos cH~1— 2 cos2 a ) ~ 4 jx62 tan a sin ~ sín .
740. By rotating the given planes about the altitude of the cone without
changing the angles a and {5, we can bring them to a posítion (shown in

Fig. 207) in which thcy intersect along the common element BD oí the cone.
From the triangles OBM and OBN we find

= = OM- = -^ - + OA'2

here OM = H c o t a and O N ~ H c o t$ . Consequently,


a2 f l2 J>2
B.‘í — - 7-A - / / 2cot2a and - 7- - f //2 cot2 a f / / 2cot28
4 4 4 r
The equations yield / / and li.
Answeri Y 71(62 COÍl a ~~ tttcoia ^ ~ «2
24 (cot2 a — cot2 p)‘i/2
741,, Shown in Fig. 208 is an axial section oí the cone. Intersecting the sphere
of radius r it yieids a circle of radius OD = r, inscribed in the triangle ABC.
336 A nswers and Solutions

We have
r ~ R tan y — l cosa tan y

4nÍ3coss (x tan3 —
Answer: V = ------------^------------- ■.
742. Through the point Af (Fig. 209) on the curved surface o! the cone the
tangent line MB is drawn, forming the angle 0 = I.BMA with the element
CMA. Another angle a — OAM is also known; it is required to find the angle
qp formed by MB with the plane P of the base of the cone.

The line MB, tangent to the cone, iníersects the plane at a point B lying
on the tangent AB to the base circíe *. Dropping from the point M the per­
pendicular MN to the radius OA, we get the projection BN of BM on the
plane P. Hence, q> = / NBM. From the triangle AM N we have
A M -* ™
sina
from A MAB we have

cosO sina cos 0


from A MNB we have
MN .
sm ^ ~ ~m W ~ sm a cos ®
Answer: q> = arcsin (sin a cos 8).
743. The surface S of the solid of revolution is equal to the sum of the curved
surfaces of two cones with the axial sections BABi and BCB\, With the notation
* This can be proved only on the basis of the definition of the tangent to the
curved surface of a cone. But such a definition is not included in the tertbooks
on elementary geometry.
Chapter X. Solids of fíerolution 337

adopted in Fig. 210 we have i> nBc -f nfía. From the triangle CBE we have
h
sin P
h.V the law of sines we have
c a
sin [18U — (a -T-P)J — sin a
hence,
a sin (a -f p)
sin a
from A BCD, wherein /_BCD v. • (1, we have /?-=a sin Hence,
^ nh- sin (a - ft) fsin (a ft) sin a|
sin2 sina
The expression in hrackets may he transformed according to the formula for
the sum of sines.

2nh2 sin (a • fV) sin ( a -f -§-) cos -jj-


Answer: S ---------------------------------------- — --------- .
sin cc sin2 p
744. Figure 211 shows an axial sectioji oí the conic. vessel; ADB is the water
«evel. The triangle ABC is an equilaterai one; the circle DEL (the great circle

°1 the sphere) is inscríbed iu it. With lhe nolaticm adopted iu Fig. 214 B
OD-tan 60° ~ r \ZTi and // * CD - 'Ar*. The volume V of water in the
vessel is equal to the volume of the cone ABC less lhe volume of the sphere, i.e.
V -- {Jt~D 1r:i) nr3

. * The radíus of a circle inscrihed in an equilaterai triangle is equal to one


°f the altitude of lliis triangle; it íollows from the fact that tlie poinl of
Qtoraection «f medians in any triangle divides each rnedian in th e r a lio l:2 .
22'-0!33S
338 Answers and Solutions

When the sphere is removed, the water drops to a levei MN and filis tho
cone MNC. Let C E ^ h , then M E = C E -ta n 3 0 °= -4 _ tilus
1 /3 ’

v= j-a/£2.C£=2í-
We get the equation
.nft3__5
9 “ 3 nr^
Arisiaer: h = r y 15.
745. If the radius CMi (Fig. 212) is denoted by r, then the altitude A\M
oí the prism is also equal to r, and from the triangíe wherein AiBj =
= 2r, we have

Now we fiml the lateral area of the prbm:


2li*
Sirtt (2r 2r eos a 2r sin a) •r — ----- í ( 1 t c o s a i-s in a)
1 r cot -£■

OM„ , . . . 1 2/?2sin -s-sina


2/1-( I —-cos a - f sm a) 2
.‘bi.vinr: Sjaf-
Chapter X . Solids of Revolulíon 839

746. The radius R — OF (Fig/213) of the cylimler is equal to ~ B F * . But

BF = BE — FE — BE — FE{ -cot a «V 3 (cot 3(JÇ— cot


a sin (a — 30°) a s in ía — 3 0 )
2 sin a sin 30' sin a.
Therefore the volume of the cyJinder is
r a sin (a — 3ü:n 2 <*
n [ 3sj[|a j j

The volume of the pyrnmid R AtRiCi is

The problem is possibfe if BE^> FE, i.e. * v ü > -7j- cot a or cot 30* >

> cot a; hence, a > 3 0 '.


na3 sin2 (a — 3ü°)
Answer: V}
18 sin2 a
a3 sin3 (a — 30 ) tan a
V2 — ---------- —rr:-------------------
3 V 3 sin3 a
P relim inary iYo/c.c Io P roble ms 7 4 7 -7 8 0
The* methods oi correct graphical representaiion of a sphere and i(s srctions,
as also of various solids inserihed in and circumserihed aiio\it a sphere aiv ralher
involved. Therefore, the problems helow are stippiird with schematic piano
drawings which are niuch simpler to construct and stili present a ctear picture
quite sufficient for underslanding and solving the giver» prohlems. When a plane
drawing faiis to serve these purposes it is accompanied hy a three-diinensional
drawing.
747. Jntersecting the spliere, the planes containing the Fases <>í the prism
(triaugles BAC and in Fig. 214) yicld Uvo circlcs in u h i d i the right-
engled triangles ABC and A tBtCi are inseribed. Therefore, the h\ [lolennses
Atí and are the diameters of the obtained circlcs. The plane AUBXA\

See footnote on page 336.


340 Answers and Solutions

passes through the centre of the spherc. Since, by hypothesis, ABBxAi is a squa-
re, \ve ha ve / / — A A, — R ~/2 and AB — R V 2 .
y _ i?3 sin 2a
Answer:

748. intersccting tlie sphere, the plane coníaining the base of the pyramid
yields the eircle ABCD (Fig. 2Í5) circumscribed about this base. The altitude
of the pyramid passes through the centre Oiof this circlc (since all the cdges are
inclined to the base at equai angles) and also through the centre O of the sphere.
A eultíng plane drawn through the diagonal AC of the base and vertex E yíelds

a great circk* circumscribed ahoul the diagonal section of tlie pyramid AEC.
From the triangle A E C . wherein the angle A E C is equai to !80: 2q, we find
. ! ( ’ - 2/f sin (180' - 2q) - 2fí siii 2(p (by the íaw of sines); hence. A O j -
a -iu 2i(. From the triangle AEO\ we find the altitude of tlie pyramid
EO<. - U - AOftan cp - li sin 2q tan q

Answsr: V -- R* sin3 2q sin a tan cp.


740. Since the radius OE (Fig. 216) of the cirde inscribed in the base is
equai to R> AB -- ‘-R V 3. From A l)OE we find DO - I! ^ R cot ™ .

.-Iníirer; V - \'A /i3cot-^- .


Fig. 217 shows an axial section. We fiave
SCurBcd nl (ri + r2) = n •AD-(AM + DN)
But A M ■ DX AEA-DF^ Thcrefore 5c«r^rf = n-AD*, From tJ)e triangle
AED. wherein DE AlAr 2r, we find AD =-. —.
4nr3
Antuvr: >S'curt>n/ = sill2 a
Chapter X . Solids oj JRevolution J

751. See the preceding probiem. We have S — Scuri:e<j-^ n (rj-j-rs). From


the triangle AOM (see Fig. 217) we find

AM = r\ = OM -cot -y- = r cot™

From the triangle DON, wherein /. ODN —- , we have

D X — r2 r cot | 90" — ~~ j - - r lan - ~

Calculations become sirnpler ií the expressio» rj-4-r.] is transíorrned as follows


rf r r| = (rj-f r2)2 — 2rirz. Since rt -r■r2---- /, Scur,,e(j nlz (see the preceding
Z?

Probiem) and, from the right-anglcd triangle Aül), Ai' l í> O/'2 or rtr2 - r - ,
we have
S 7ilz-j-j\l2—2nr2T.,:2Ji{l2—r2)
‘Substituto l .. inlo this exprossion.
sin a
^4osujer: 5 • :2.nr2 ^—— ------- 1 ^ .
V siü -a /
752. See the precedíng probiem. We have

Substituto l J í ..
sin a
A n sw er: V ~ nr ( — ---------1 ) .
d \ s m -c c /
753. Let us denote lhe length oí the equa! chords DA, Üli, DC (Fig. 218)
ky l. From lhe isosceles triangle D BC we fiml BC ~ 2 1 sin-—- . Similar!)-, AB -
342 Answers and Solutions

= AC = 2ísin — . Consequently, the triangíe ABC is an equílateral une.

Dropping the perpendicular DOi to the plane ABC and revealing congruence
oí the triangles DüiA, DOxB, DOxC, lct us prove that AOi Büx = COt, i.e.
that is the centre of the base (thus, the pyraraid DABC is a regular one».
Since Stie points .4, B, C lie on the surface of the sphere, OA — OB — OC
(i0 is the centre of the sphere). Dropping a perpendicular from O to the plane
ABC, let us prove that the foot of the perpendicular is the centre of the triangíe
ABC, i.e. coincides with the point 0 j. Consequently, Oüx (and, hence, Uüi)
lies <m a diameter of the sphere {DF in Fig. 218). From the right-angled triangíe
DAI’, wherein DF — 2R, we íind l- ~ ÜA“ = 2R-DOi. The line-segment DOx
may he related to l by anotlier formula. Namely,

D O ^ y A I P —M l
wliere

2 /sin -2~
BC
AD-^l and AO{ ■■■=
V'3 1 '3 "
H en ce,

/
4 sin2 -
ÜO, = l V 1-

Substituto Ihis cxprrssion into the equality /2 .~2R*DOt. We find

l.'-2 R \ 1--------- 5-21


3

Redm e this expression to a form convenient for taking logarithms. We have

-r 2 (1 --cosa) 2R - ,/ r ~ q -----
l 2R ]- 4 ----------------- ;;----------- ----- -zm r V 1+ - a «■
1/3

2R
7 =- l ' 2 t a > s t i U ' - ; - c o s a ) j / cos c o s ^ 3 0" — 4 j- j

Aiisicer: l 2R 1

h V - K ) » ( » - } ) .
7.'i4. The isosceles trapivoid ABC D (Fig. 210) represents an. axial section
of the írnstura of a cone. F3y hypothesis. _ AOD a. and /_ DOC ~ p. Therefore

/?! — AE — AO sin tf sin — and R2 DF --- R sin


C h a p ie r X . S o lid s of Iíe v u lu tio n

The angle AOD = ...i — L fi = 180° - . Thcrcforo l = AD =

= 2fl cos — y-P . We liavc

s cuned = n t(tt, « 2) « . 2rc/t~ CCS (sin-|- : sin-|-)

Answer: Scurved = 2nli- sin cos a .

755. From the triangle OME (l-ig. 220) ivc lave


OM — r m cos a ; £ O ------ / / -•=w sin a
By the formula S = n r (r-yl), whero ( m, we íimi
S -:r nm- cos « (1 ; cos a)
or

S - 2mnz cos a cos- —

Jhe angle (p ~ / EDO, at which the lateral edge DE is im linod to the base,
*s determined from lhe iriangle EDO, wherein OD OM '\/2 — m '\/2 cos a.
We have tan <p
OD ,„ y 2 c o s c t l x2
Answer. S = 2rtm2 cos a cos- ; <p- - arctan • -m- .

736. From A ASB (Fig. 221) we íincl AD = 2/ siri ~ ; henoe, 11 OA AD

- 2 is í n ~ - , From A ASO we íimi

S Q ^ II . ly l~ 4 s m 2~
344 Answers and Solutions

We gct

F = i n i P / / = y.4i2sin2-| - i y

The radicand may be reduced to a form convenient for taking logarithms in


the same way as in Problem 753: we get

1 —4sin2 ~ = 4sin sin

.4hsuw: V ~ ~~íi/3sin2-^- y sin | 3 0 --f-~ j sin ^30c — .

r - OM = AM ■tan 30° = l s \ n - í . —
2 1 /3
and

,43/
R ----- AO =.
cos 30: 1/3
From A AO D we íind

ou u =.-1 'p r i p | / 3 _ ,S f i n 2 .

The volume of the cone is

/2 sin2
V - -4- r2// -- 4 - ------r,— ™ • - [/ 3 — 4 sin2 -
3 3 á
3 y 3 \
C h a p le r X . S o lid s o f R e io lu tii

The radicand may be transformed in the samc way as in Problem 753.


JiÍ3sin2 -“ ,-------------------
Answer: V = ----- ——— y 3 — 4sin2 -—- —
0V3
2nl3 2 s in y cos 1 30° + — j cos 1 30" — ~ j

9V3
758. The volume of the sphere (see Fig. 223) is equal to - .i/P , and the

volume of the cone ACB, to y nP-COf = y nr'~If. By hypoíhesis,

o
4 - JirV/ = 4- . 4
4 3
nR3
i.e. r2/ / — IP. Another relationship butween r
aad R we obtain Ironi the right-angled triangk*
CAD\ nameíy, AO\ — CÜi -DOi, i.e. r-
— / / (2/? — //) . Substituting this expression
•nto the preceding equality, we get IP — 2H-R~r
+ IP ~ 0. Though this equation in the un-
hitown fí is of the third degrec. its one soiution
R 11 is quite obvious (it cmild he guessed
tmmediately by the givon conditions, sinco the
volume of a cone, in which both the radíus oi
the base and the altitude are equal to the radíus
of the sphcre, is equal to a quarier of the volurne
°1 the sphcre). Consequently (according to the
remainder theorem), the leít member may be Fig. 223
factorized, one of the factors being R — li.
cor this purnose it is sufficient to divide li'1 211-R -•[• IP by R ~ 11 or
accomplish the following transíonnation:
n* - 2ii2fí + j p = y p — ip R ) _ u i-R _ ip ) =
= li {11 ~ 11) {R -f 11) - IP (R ~ //) - Ui - 11) UP-r RH —IP) o
// ( V ã — i)
The equation R- + RH — 11- 0 has one positive root R

^ th e n e g a t iv o r o o t H= - — ís n o t s u it a b lc j ■ G e o m e t r ie a ily , t h is m e a n s

that lhe radius of the sphcre is equal to the larger portion of the altitude of the
cone divíded in extreme and rnean ratios.
Answer: the problem has two Solutions:

V ^ -jn / r * and V ^ -ix (y '5 — 2 ) / / 3

759. The altitude of the prism is equal to the diameter 211 of the inseribed
sphere. If a plane is drawn íhrougli lhe centre of the sphcre and paralief to the
«ases of the prism, then the section of the prism by this plane yiehis an equilate-
ral Irtangle {KLAl in Figs 224, 224a) equal to the base of lhe prism, while the
scclion of the sphcre is a greal cirele {RXQ) inseribed in the triangle {KLM).
346 linswers and Solutions

From the triangle LOA\ wherein ON = R and / NLO = 30", \ve find LN —
« R V 3 . Consequcntly, LM = a - 2R V ã . The lateral arca ef lhe prism is

SIa: = 3alf = 12/i2 1/3 . The area oí the base S,mK= ' ^ ~ - = ZR'i V ã. Hence,

St0(ai = 12fl2 V 3 + 6Í 2 V ã = 181i2 y3


The surface of the sphere is equal to 4nR2.
2a
Answcr: the required ratio is —^ -= -.

Fig. 224 Fig. 224a


760. (a) Drawing. The centre Ot of the sphere inscribed in the pyraraid (if
it is possible to inscribe a sphere in this pyramid) must be equidistant from the
lateral face BEC and the base ABCD
(Fig. 225). Therefore. it must lie in the
bisedor plane of the dihedral angle q; at the
edge BC. Sinülarly, Ot lies in the bisector
planes of the dihedral angles q. at the edges
A B , A l). I)C. Hence. aíl the lateral faces of
the pvramid iRAfíCl) (it is not shown in
the drawing) are inclined tn the baseatone
and the same angle ~ . Consequcntly, the
altitude OtO oí the pyramid OxABCD passes
tlirough the centre Òof theeirde inscribed
in the rhombus d /?(./) (see lhe Preiiminary
Notes to Problem 617). The altitude EO
of lhe giveu pyramid passes througli the
same centre. Hence, the centre Ox of the
sphere lies 011 the altitude EO.
The nointof tangency of the sphere and the face BEC is the íoot L oí the per­
pendicular dropped from the centre Oj of the sphere to the plane BEC. Hence,
the plane OxEL is perpendicular to the face BEC (prove it!). At the same time
the plane OxEL is perpendicular to the base ABCD (since it passes throiigh the
altitude EO). Consequently. the plane OxEL is perpendicular to the edge BC.
Hence, the straight lim* A/.Y, aiong which the planes OxEL and ABCD intcrsect,
is the altitude oí the rhornbns (drawn tlirough íts centre O). The same thing is
C h a p ter X . S o lid s of fíe v o lu tio n

with the remaining threc points (A', Q and P), at which the lateral faces touch
the sphere.
Hence, the foliowing constructions: draw the altitude SOM of the rhombus
ABCD (it is desirable to make it horizontal), construct tlie section SE M (an
isosceles triangle) and depict the cirele inscribed in the triangie SE M . The
points L and Q at which this circle touches the sides ME and SE are the points
o f tangency of the sphere and the faces BEC and A El). To find the poínt A draw
MS\\AC. Then OS (not slunvn in the drawing) represents the olher altitude of
the rhombus (prove it!). Draw ES and through the point L draw LK ]|MS (not

Fig. 22t> Fig. 220a


shownjin the drawing). The imirlh point P is foimd in a similar way. As ioliows
from this construction, the >phere with Os as the centre and radius B O^L
ls actually inscribed in the pyraniid.
(b) Soluíion. From the triangle M ()0% we find
OU = 0 0 , cot -Sj- = li cot i
so that
11 OH =OM ■lati <| H rol 4 " 1:1,1 <1
Then from lhe triangle fíVA (uhore « c find
Ui cot “
nu
Kl 2s.t/lt
•OM
AB
sin a
Hence,
■MU vot* -

4/f3 tan <pcot3


Answer: V-
3 sin «
.. 761. (a) Drau/lnu. The centre O of the equator o) the hcmisphere (Fig. 220)
*ies on the altitude Süi of the pyramid. Since
OM - OOt r,
Answers and Solutions

the point M lies on the bisector 0\M of the angie OOyM. Marking M as the point
nf intersection of OtM and SF, draw the section KLMN parailel to the base.
The midpoinls K, L, M%A; of the sides of the section are the points of tangeney
of the equator and the lateral faces. The simicircle KOtM is the section of the
hemisphere by the plane ESF.
(b) Solution. The side of the base is
a = EF - 2 = 2 {0,M , + MtF)
But O iM } = OM — r, and M tF — M M j-cot a -- r cot Hence,
a 2r (1 ~ cot a)
We have
2i'6„.,e cos2 4

(see the Note to Problem G19). Iiere Sf)asc - rfl2—4r2 ( í - r cota )2.
8r2 (1 f cot a)2 cos2 ~
4r2sin2(45: — a)
Amwer. Slotal = ------------ ------ --------- --- =

762. íntersecting the hemisphere, the plane ESF (Fig. 227) vields the semi-
circle NPM touching the slant heights of the pyramid at the points Q and G.

If we denote the side nf the base of the pyramid by a and the radius of the hc-
misphcrc. by r, thcn the surface of the hemisphere is
S\ 2nr- — .nr- o.nr-
and the total surface area of the pyramid
o o , cc
2a- cos2 —
ò\ = ------------~ .
cosa
C h a p ter X . S o lid s oj fíe r o lu fio n 349

(see the Note to Froblem Gi9); their ratio is


3;*r2 cos ol
Za2 eos-—

From A OGF we liave Otí - O F > * in a, i.e. r - T si» a. Substituto tltis expres-
sioii intü the preceding oqualiíy.
To determine the volume V uf the hemisphere íind r procecding from the
condition a — 2r =• m and from the above equality r ^ — sin a. We get

m sin a /» sin a
2 (1 — sin a)
4 sin- ( 4 5 '— -rr- )

'òn
Answer: q sin 2a lan —
8
V’ _ n/»:i sin3 a
90 sin11 ^ 4 5 ---- —•|
763. Figure 228 shows an axial seetion of the cone
aj»d the sphere inscribed in it. The required volmne V h
obtaincd by suhtracting the volume of lhe sphorirai seg-
níeiil MEN froin lhe volume ol the cone M t'X. llencet f í . K

wherc r is the radias ol the sphere. ITom the frinngle .1 Ol) we find
D AC
r - 0£> : w l/M an -
■ (« - t )
Now, írom the triangle O.MK. wlierein -_OM K a (lhe sides ol the angies
OMK and MCK are mutually perpendicular), we have
MK O M -cosa /-c o s a and OK / sina.

Herice, XE - OE — O A r (I — sin a). Finally. KC • M K -cot a


r cos a cot a. Consoquently,
V - 71 -a ... { r 11 “ SÍÜ |
— 3- rJ cos3a cot a — nr- <1 - s m a)- r ------------ 3------ -

= T r3í i S íiy L ' - <í — sin':t>2,:í ‘ « “ * > ]


Fhis expression rnay be simplified. Factor oul (t — sin a)- on huving transform-
ed C0s< cc heforehund; namely.
cos4 a — (1 —- .-in- a)- -■ (1 — si» a)- (t -f- >'» a)*
Aiiswers and Solutions

Now we have
rtr3(1 —sina)3 . . ,
V—----- -----------— Í(1 ~fsma2- (2-i-sina sina

The expression in hraekets is equal to unity. We get


y _ Jtr^íl — sina)2
” 3 sin a
Substituto into it the found expression

r~ .fi tan ^45° —

We may also use the formula

1—-sin a= :2sm 2 |453—

4.Tfí3taiví [ í ã sin4 Í45' —


Answer: V ------------------ ----------------------------------— .
3 sm a
764. Using the notation adopted in Fig. 229, the given condition is expressei!
by the equality n fiil — /}) --n-4nr2. From the triangle OBOi we find r =

ç '- /? ta n ^ -, and from lhe triangle BOC we have-

BC ^ l -— —— . Wiien rcduoed by xfí2, the abovo


eos a J
equality takes the form
t ■ 1 . . „ a
i - --------- —An tan- —
cos a 2
Apply the formula
1— tan2

4 -j- tan2 -~
We have the equation

=4n tan2 -

Putting tan^ - = 2, we get*

=0
2n

* Getting rid of the denominator, we could introduce an extraneous solutio»


(tan-— ~ i)i but we do not get sueh a solution, since it does not satisfy the
original equation.
Chapler X. Solids of fíevolution 351

whence

2 - V 4 i
Hence it is clear that for n < 2 lhe problem lias no solution (since the rradi-
cand is negative). For n > 2 both values of the quantity c- are positive

(siuce j / 2 - - < j / ^ - , i-e. ] / ^ — ^ - < T ) ' Sinct’ the -[quantity

tan ~- must be positive, only two Solutions are possibíe:

2 = tan-|— r ] / y + j /
and

]/f Z 4--
Since tlie angle ~ is íess than 45°, tan ™~ must be !ess than unity: hence,
there must be s2 < 1. But this inequality is ahvays satísfied, because

+ / f í < ! + / T =1
and

- / | - Í < T
i4«íícer: lhe problem is solvable only if « > 2. For n > 2 lhere are two
Solutions:

at n ~ 2 both Solutions coincide ^ tan ■


765. Using the notation adopted in Fig- 229, we liave
1
T n /í2// " " 3 :
Suhstiluting
r = li tan ~ and II fí tan cc
We the cquation
ta n rc - tn t a n - '—

Appiying the formula


352 Answers and Solutions

and denoting tan-^- by z, we get the equation

« (■ p h r- ^ H
It decomposes into two equations, but one of them (z ~ 0) disagrees with two
given conditions (the angle ct must be non-zero). The other equation is reduced
1
to the forin z4 — z- -f ^ -- 0, i.e. it coincides with the equation in the pre-
ceding prohieni. We obtain the íollowing two Solutions:

,anT ’
At n - 4 one soiution is

. ct i / t , , 1 -i/i 1 t . /" 1 cos 45-


tanT - : M r V t t t K — i— =
— cos 2 2 c3 0 ' * 0 .9 2 3 9 ;

the other is
ta n = * in 2 2 = 3 0 ' ^ 0 .3 8 2 7

(henee ctj * <S5:28' and a2 ^ 4 i;53').


Answer'. the same as in lhe preceding p?>-blem.
At n 4we have ot| 2 arclan (cos 22 30') (5- 8.V28')
cío 2 arclan (sin 22 30') ( s 4153')
7G G . T h e area «*i‘ t h e a x ia i s e c tio n is HH. The s u rfa ce i s nfíl - j- nll'-. By

h y p o th c s is , ■■■■■ j j — - :== n. I f [5 is t h e a n g l e h e t w e e n t h e a x i s a n d g e n e r a t o r . th e n

li - l s in f) a n d ll - / c o s (i. S u b s t i t u t i n g tb o se e x p r e s sio n s, w e g el

1— s i n f i n
c o s f» n

This equation may be solved in soveral wavs; the sbortest one is to appiy
. . , 1 - cosa . ct ,
the formula — :------ ^ eot —•; we get
sin ct 2
1 - s i n (i I c o s (9 0 -■ f))
c o s fi " s i n (90=' — p ) = V ‘ ir ~ T /

Consequcntly,

w b e r e fn m i w e d e t e r m in e th e a n g le 4 5 o — and th e n th e a n g le

But th e p r o b le m is s o l v a b l e not at any In deed , th e a n g le f) is w ith in

th e ra n g e h etw een 0 a n d 9 0 °; h e n c c , th e a n g le 4 5 "— ~ is b etw e e n 0 a n d 45 °.


Chapter X . Solids of Revolulion 353

i.e. the quantity ~ — cot must exceed unity by all means, i.e. n
must be greater than n. At n — 1, 2, 3 the problem has no solution.
Note. The equation
n

COS(5 ~
may be solved in a different way. Let us rewrite it in the form — cos p — 1 =
==sinp, square both members and replace sin2 {5 by 1— cos2 (L We obtain two

F»g. 230

Solutions: one of them, cosfi —O, turns out to be an extraneous one ^ it is the

solution of the equation ™-cos fl — 1 ~ —sin p j ; the other, cos ,


coincides with the preceding one.
But now one may easily arrive at an erroneous conclusion that the problem
is solvable at « - - 1 , 2, 3 as well. Indeed, at any positive value of n the
quantity ranges from 0 to 1 ^we have 1
Therefore wíthin tlie range bctween 0 and 90° one can always fiíid an angíe,
whose cosine is equal to ■
The error of this reasoning consisls in the following. From the relationship
cos p =a nn and from the given equation it follows that sin 6 = A ;— ,
n * jt-
whence it is obvious that n must be greater than n (otherwise the angie
will be negative, which is impossible).
Answer: if n < n , the problem has no solution. If n > jt, thcn
fi —GO"1— 2 arccot —
1 ji
or
_ 2nn . n2 — n2
» ~a‘ n2i «2 ” «2 Ji2
767. Using the notaüon adopted in Fig. 230, we have
R (l + R) 18
2r2 5
23-01338
354 Answers and Solutions

We find (from the triangle AOD)

r —R cos z. AOD -- R cos z ACO = R eos-


and (from the triangle ÁOC)
R
l =

The preceding equality takes the form

i-s m T lg _ 1-j-sin-
18
' 5 ’ 5
2sin-y- cos2 2sin-|- ( i - s i n s - f - )

Reduce the fraction by 1 sin-y- (this quantity is non-zero). The equation i3


reduced to the form
. „ a . a , 5 n
sm2T - SmT + 36 = 0
5 1
Answer: aj —2arcsin-y ( « Íf2°53‘) and ct2= 2 a rcsin y ( « 19*11').

768. Using the notation adopted in the preceding problem, we have —■RZH ~

— —••yitr3. Denote the required angle by p |in Fig. 230 p = . Tkenr=


— R cos p and H — R cot p. From the preceding relationship we get 3 cot p —
— 8 cos3 P = 0. Multiplying both members of this equation by tan P (which,
by the sense of the problem, cannot be zero), we get the equation
3 — 8 sin p cos2 p = 0
whence
8 sin3 p — 8 sin P *f 3 = 0
To solve this cubic equation we have to apply an artificial method. Thus, the
left member may be factorized in the following way:
8 sin3 p — 8 sin p - f 3 ~ (8 sin3 p — i) — (8 sin p —■4) =
- 1(2 sin p )3 - 1] - 4 (2 sin p - 1) =
- (2 sin p - 1) [(2 sin P)2 - f 2 sin P 1 - 41
Consequently, the found equation decomposes into two equations. From th8
. V l3 -Í
first one we find sin P = —■, and from the second, sin p . (The other
solution of the quadratic equation is not suitable.) A check shows that both of
the found Solutions are suitable.
Answer: Pi==30a; _
. y i3 -i
p2= arcsm ——j------ •
Chapter X . Solids 0/ Revolution 355

769. By hypothesis, the curved surface of the cone MCN (see Fig, 231)
must be equal to one half of the curved surface of the cone ACB. But the curved
surfaces o f these cones are in the same ratio as the squares of their eiements, i.e.
cm i .
^^2 ““"2' * since
CN = CO,
we have
CO \2 1
CB ) 2 ’
i.e.

cos2 a ~ y
Ánswer: a = 45°.
770. By hypothesis, the volume V of the spherical sector CMKN (Fig. 232)
must be equal to one half of the volume of the cone A CB. Let us denote the line-
G

segmcnt KL by h, and the altitude of the cone CO, by H. Then V = — rtr-h.

We get the equality ™ nr2h =~ n/?2// , i.e. 4r-h — fi2JI or 4r-h ~


= U9 taní a. Expressing h in terms of r, we have
h = LK ~ C K ~ C L — r — r cos a ~~ 2r sín2

We get the equation

8r3 sin2 ~ ~ / / 3 tan2 a

ü ím n t : r "
1 V sin2
771. In Fig. 233 the axiai section of the portion of the snhere, whose volume
must be determined, is hatched. This volume V is obtained by subtracting the
volume Vj of the cone MCN froru the volume V2 of the spherical segment
23*
356 Answers and Solutions

CEMKNF. Let us introduce the followíng notation: M K — r and KC — h.


Since the radius of the sphere is OC — CD — , we have

Substitute the expressions h — MC *cos a — H cos2 a and r — AfC*sin a =


— Ii cos a sin a {the computation is simpÜfied if r2 = M K 2 is replaced by
CK-KD = h{H -h )Y , then
a
6
. _r n //3c os* a
Answer: l ---------- -------- .
b
772. With the notation adopted ín
Fig. 234 we have: SCUraed~ n ( r + rj)Z.
Draw radii OM == R and 0\Mi — R %to
the points of tangency and the straight
line OiK perpendicular to OM. We
gel the triangles OiMiEif OME and
0\K0, which are similar to one ano-
ther (as the right-angled triangles with
an equal angle a). In the triangle
OtKO we have
O fi - R - f Ru OK = i? — i?,;
OtK - MMi == l
Hence,

/ = - 2 -\/Wi

From the similar triangles OME and O^KO we have , whence


/? - /? !
IR 2R V W i
r " R~ Rx " ii-, /?,
From the triangles Ojd/jEj and OiKO we have n Rj
; hence rj =
i n -, Ri
y «si
/l/um-r: 6(.(xrDfd ~ 4ít/?7?j.
773. Four balis of the radius r lie on the plane P (Fig. 235), touchingit at
the points M, N, K and L. Their centres 0 5. 0 2, 0 3, 0 ; are equidistant from
the plane: OuM — 0*N — 0 3/v -= -- r. The distance hetween the centres
of two contacting balis is equal to 2r, i.e. ()t0 2 = 0«03 ~ 0 304 = 040| — 2r.
The íiíth bali is in eontact with each of the four balis; conscquently, its centre
0 5 is situated also at a distance of 2r frora the centres Ou 02, 03f 0 ., i.e. O1O5—
= OzO5 -- O3O5 = O4O5 — 2r. Therefore, the figure 050i020304 is a regular
quadrangular pyramid with equal edges, The distance between the centre of
the fifth bali and the plane P is equal to 0 0 5 -f OA\ — qq 5 - f r. The topmost
Chapter X. Solids of Revolution 357

point A of the fiftb. bali is found on tbe extension of the perpendicular A tO$
at a distance of 0$A » r from tbe centre 0 5. Thus, tbe distance A A t between tbe
topmost point of the fifth bali and tbe plane P is equal to 2r 0 0 5. The line-

segment 0 0 5 is found from the right-anglcd triaugle OtOO$, wherein


0 {0 2 2r
OiO'J— 2r and 0 0 1= -yj=" «=

Answer: A A t — r (2 -}- ~\/2).


774. The centres Ou <93, 0 4 of tbe four balis must be at a distance of
2r from one another (see the preceding problcm). Hence, the ligure 0 i0 20 30*
is a regular tetrahedron with the edge equal to 2r. The cone ACB (Fig. 236),

ciKumBcribed about the four balis, contacts one of them (Ok) aloug the circle
**7 and each ono of the remaining three balis {for instante 6M at hvo points:
one of which, A', lies on the base, the othcr, M, on the curved surface. The axis
358 Answers anã Solutions

of the cone coincides with the altitude 0 40 o! the tetrahedron. Tbe centre 0,
lies in the plane of the axial section A CD passing through the point of tangency
M (since 0\M is perpendicular to the common tangent plane to the cone and the
balí, and the plane of the axial section ACD is perpendicular to this tangent
plane). Hence, the plane ACD intersects the balis 0 , and 0 4 along their great
circles, the element AC being the common tangent to these circles. Consequently,
AC ||O,04 and ^ 0 ,0 t0 — i _ACD — ™ (a is the required angle at the vertex

C of the axial section). Hence, sin _- °°i But 0,04 = 2r, and the line-
2 " 0Í04
segment 0 0 , (the radius of the circle circumscribed about the triangle 0,0203)
i * 0j02 2r t . a i .. „ a
is equal t o W e get sm . Hence, co s a — cos2 y —

. „ a 1
-S “ 2 = T •
Answer: a = 2 arcsin — = arccos 4-.
V3 3
775. The plane bisecting the dihedral angle at the edge A iA z (Fig. 237)of
the frustura of a pyramid passes through the altitude 0 ,0 2 and is perpendicular

to the face B íCí C«Bz (prove it!). The same thing is with the other two lateral
edges. Therefore, the centre of the sphere touching the faces of the pyramid is
situated on the altitude (namely, at its midpoint, since the sphere is also in
contact with the bases) and the point K of tangency of the sphere and the face
BiC\CzBz lies on the slant height 0 , 0 2 of this face. The same is true for other
lateral faces. We have
•a») l
i + “i) + 3 —

(a, = B\C\ and <z2 = B«Cz are the sides of the bases and l — 0 ,0 2 is the slant
height of the lateral face). If r, = 0 ,0 , and r2 = 0 20 2 are the radii of the cír-
cles inscribed in the bases, then a, — 2r, 1 /3 and az = 2r2 V S . Therefore
Spyr = 3 Vã (r; + r’ ) + 3 Vã (r, + r.) I
Ckapter X . Solids of Revolution 359

In the same way as in Problem 751 we find that rr -f r2 = Z and rf-frS f2— 2r2.
Then we get
Spyr = 6 V 3 (l« - A) = 6 V 3 ( _ r2 )
2n sin2 a
Xnsieer: Ssph:S pyr^
31/3 (4—sin2 a)
776. Denote the radius OL of the cylinder (Fig. 238) by x , and the radius OB
of the base of the cone by R. Since, by hypothesis. ML = /?, the surface of the
cylinder S = 2jtz2 -f- 2nxR. By hypothesis, 2 n z- -f- 2 nxR ■ ■nR- or x2 -f-

3 /? 3
+ B x -----r R* = 0, whence x —-tt (lhe negative solution x — — -?r R is not
4 2 *
suitable). From the triangle LMR we find
LB R -x 1
tanff — ~JJKT Ti 2

Answer: 9 = arctan — ,
777. The centre O of the inscribed sphere (Fig. 239) lies on the altitude of the
Pyramid and the points K, L, M, N, at which the sphere touches the lateral
«ces, are found on the slant heights EKi, ELt, EM ,, ENX (cf. Problem 775).
The quadriiateral KLMN is a square which is lhe base of the pyratnid, whose
volurae is to be dctermincd,
Through the radii OM and ON draw a plane SOM which turns out to be
Perpendicular to the face BEC (since it passes through the fine OM perpendicular
to the plane BEC) and also to the face UEC (since it passes through ON). Conse-
queatly, the plane NOM is perpendicular to the edge EC.
Let P be the point of intersection of the plane NOM and the edge EC. Then
the angle NPM is a plane angle of lhe dihedral angle a. In tho quadriiateral
360 Answers and Solutions

OMPN two angles (namely, at the vertices M and iV) are right ones. Conse-
quently, z NOM = 180° — a. Hence

a = NM = 2-OM.sm - ^ ~ í = 2rcos -2-

From the triangle OOtM t where O iM ~ we línd

h^OOi = y O M * -O iilfz = r 1—2 cos2 -^-

Answer: V = ™ r3 cos2 ~ y 1 — 2 cos2 r3 cos2 ~ V —cos a.

778. We can draw two planes perpendicular to the given element of the cone
(CA in Fig. 240) and tangent to the inscribed sphere, the points of tangency (N
and A'j) lying on the diameter parallel to CA. Let us first take the plane
ND touching the sphere at the poínt N. The quadrilateral ONDK (K is the point
of tangency of the element CA and the sphere) is
^ a square, hence DK — ON = r. By hypothesis
A CD = d. Consequently, CK = d - f r. From the
yfel\ triangle KOC we find

ui ! \ CO — y (d— r)2^ r2
Hence,
Pm
Kjf ' //
1 X\ // = CF = OF + OC= r + V (d-j- r)2_
'Á ° J\ From similarity of the triangles AFC and KOC
we find
m ). k l\ J \
(s A F : H — O K : KC
a £,F b whence
Fig. 240 0K-H r \ r + y { d + rf.~r!-\
KC d-~r
If we take the plane N{Dif then d = C\Dt, and we gct in thesame vvay
H = r -f V (d — í")2-t r2

and R = ± ± . V ( í - n » + > * ]
d— r
Answer:
T/ nr2 [ r + V {d -r r)2- r 2J3 _ T, _ nr2 [r -f V (d —r)24~r2]3
3 (d + r)2 3 (d— r)2 *
779. The centre O of the sphere (Fig. 241) lies on the diagonal AB. Indecd,
the point O is equidisíant from the faces AAiN\N and AAiQ^Q. Hence, it lies
on the plane bisecting the díliedral angle at the edge A A Similarly, the point O
rnust lie on the plane bisecting the dihedral angle at the edge AiY. And the two
planes intersect along the diagonal AB.
Let C and D be the points of tangency of the sphere and the faces ANUQ
and 4/iiA riA\ and r the radius of the sphere. Then OC — OD — r, and the plane
ODGC is perpendicular to the edge AN, and also to the edge BQ%.
Chapter X . Solids of Revolution 361

Since, by hypothesis, lhe edge BQ\ is tangent to the sphere, the plane ODGC
intersects the edge at the point E oí its tangency with the sphere: consequentiy,
OE ~ r, On the other hand, the point £ is a vertex of the square FGKE obtained

in the section 0/ the cube by the plane ODGC: hence the quadrílateral MOLE
\OL and OM are the extensions of OC and OU) is a square. Consequentiy, OM =
OD ^ MD ^ a, whence r --{2 — ] /7í)
~ l 7 I ’ Sínce "

J)

The porlion of the surface area of the sphere fouml mitside the cube is made
of three equai setrinent^ l'7/i'L heine one of them. The area oí this segment
,s equai to
2nr-LZ - 2nr (CZ —CL) r: 2/tr (2/- -- //)

/„?
ÍLTer\» ■ « ( * - Vã) <•; s “ w ( w - 7 Vã).
The centre of the sphere contacting the edges ot the tetrahedron A BCD
' r , &* 2 4 2 ) coincides with tne centre oí the tetrahedron (i.e. with the point O
362 Ansteers and Solutions

which is equidistant írom the vertices A , B, C, D), and the points of tangency
of the sphere and edges are the midpoints of the edges. For instance, the point
of tangency N is the midpoint of the edge AD. Indeed, all six isosceles triangles
AOB, BOC, COA, BOD< COD and AOD (only the triangle AOD is shown in
the accompanying drawing) are congruent {having three equal sides). Conse-
quently, their altitudes OM, ON, etc. are equal. Therefore, if a sphere of radius
ON — r is described, it passes through the midpoints L, M , N, O, K, R
of the edges and is tangent to them at these points (since ON j . AD , ana so on).
Through the altitude of the tetrahedron DG and edge AD draw a plane ADG
which is perpendicular to the edge BC (the proof is given in Problem 652) and
intersects this edge at its midpoint M. The section yields an isosceles triangle
AMD {AM — MD). Draw the altitude MN of this triangle (N is the midpoint
of AD). The centre O lies on MN (since it is equidistant from A and D). Conse-
MN
quently, MO = NO. Hence, r — — . The altitude MN is determined from the

triangle .4 NM, where AN = ~ and A M = (as the apotbem of an equilate-


ral triangle ABC). We have

MN =

Hence,
MN a a 1/2
r“ 2 ” 2Vã ~ 4
The portion of the sphere situated outside tetrahedron is made up of four
equal segments cut off the sphere by tbe faces of the tetrahedron. Consider one
of the faces—#Z)C. The circle LMK serving as the base of the spherical segment
is inscribed in the equüateral triangle BDC (since the sides o f the triangle are
tangent to the sphere; hence, they are also tangent to the small circle LMK
ccmtained in the plane BDC). The radius of this circle FM —
6
Gmsequently,

Hence, the altitude of the segment

K - t E .O E -O F = ^ - - ^ ± £ Í (3 _ V ã )

The volume of one segment

; ^ ( 3 - v ã ) ] '. [ -
V2 “V '2 -i /v\I na3 |/2 (9 —4 V 3)
38 V J 432
The required volume
1 — segm
Chapter X I . T rig o n o tn eiric T r a n s fo r m a lio n s 363

Note. The circle LK M inscribed in the triangle BCD is depicted as an eiiipse,


whieh is readily constructed witbout any French curve, if in addition to the
points L t M , another three poinh are markeú respectively symmclrical
to them about F whieh is the point of intersection o í the medians in the triangle
BDC.

Ansver: K . ...
4 ’ 108

C H A P T E R XI
TRIGONOMETRIC TRANSFORMATIONS
781. Express secants in terms of cosines to gct in the left member
1
cos ^^ + a ) cos | — a j
Since
cos ( £ + a ) = sin [ £ - ( £ + « ) ] = sin ( - £ - - « ) ,

the left member is equal to

2SÚ, ( » _ „ ) cos ( » -a ) Si n ( i - 2 « )

782. Reduce the left member to a common denorainator and briag


2sin a cos (a-f-P) to the forin
sin [a + (a + ^)j í-sin [a —(a-f-P)! " £**n + P) + s*n (-P )
783. The left member is equal to
2 d + c o s 2a)^ 2; 2 c o g « _ ^ , cot a
siu 2a 2 sin a cos a
To pass over to the angle , use the formula cot 2<y~~~2'cin q>"’ (^ ie an^°
a t
*2" is denoted by q j . We obtain

co t2 - | - _ l
2 cot a 5= 2 ---- ---------- -- cot —
j----tan

. 784. Dividing ,bolh tho nurnerator and denominator oí tho fraclion in tho
teft member of the equality by cosa, wo get
cos a ‘ sin a 1 tan a
cos a —-siiTa 1 — tan a
364 Answers and Solutions

Since 1 = tan 45°, let us represent the obtained expressioa in the form
1-f-tana tan 45°-ftana . , ,
1 — tan a~ ” i~-tana;*tan 450~~ m ~!~a
which completes the proof.
785. Muitiply both the numerator and denominator of the leít member by
cosa 4-sin a. After simpHfications we'get ....or
•® cos 2a
1 sin 2a
-sec 2a -f tan 2a
cos 2a- +
^ cos 2a
1 _ ços 2(0
786. Since sin*2 q>=------^— - , let us represent the leít member as

i~ c o s ( x + 2a ) — i-r-cos — 2a j Cos — 2a j — cos - f 2a j


~ - - “ õ

Usiug the formula for a difference of cosines (or representing the expressions
cos |~ — 2a j and cos | -j- 2a j by the formulas for the cosine of the sura
and the difference, we get

2 sin —r~sin 2a .
______4________ sm 2a
2 y T
787. The numerator is equal to cos 2a; the denominator is transíormed to
the form

2 tan ( - 2 - - a ) sin2 a ) ] = 2 tan ( - 5 - — a ) cos2 (-|- — a )

Wilh the aid of the formula

we get

2sin — a j cos — a j ~ sin — 2a ^

this expression is equal to cos 2a, hence the leít member is equal to i.
788. We have
Chapter X I . Trigonometric Transformations 365

Considering the angie - J - a as half the angie y — 2a and using lhe half-
angle formulas, \ve obtain

,/» \ l-c o s (f-2 a )


tan2 j
1 -• sin 2a
■( t - H
789. Expressing tangent and cotangent in terms of sincs and cosines, we get
. „ cos 2a
cot- a — tan- a = —r-s--------^—
sin2 a cos2 a
Substituto the obtained expression into the denomiuator of the ieft member,
then the leít member yields
. ^ » 4 sin2 a cos2 a 1 . „ ,,
sin2 a cos2 a —--------- ;--------- — ~r siu2 2a

790. Heplace s in a by cos ^ y - ~ a j and co s a by sin ^ y — a j . and use


the formulas for a sum of cosines and a difference of sines.
/9 1 . Replace (in the numerator) unily by sin-2 a -b cos2 a, and sin 2a by
^ stn a cos a. We get in the numerator (sin a -b cos a)2; the denomiuator being
equal to
cos2 a — sin2 a -- (cos a sin a) (cos a — sin a)

Reducing the fraction by cos a -t-sin a , we obtain cos a ■ ~m a ^ Dividing


^ cos a —sin a n
both the numerator and denomiuator by cosa , we find — . As is

shown in Problem 784, this expression is transformed to tan -t a| .


792. In the same way as in Problem 790, transform the Ieft member to
the form c o t ^ y — Now apply the formula cot ~ ^putting
* a \
4 y — ^ j • We obtain

’ c-(Ç , . i.?in2y
. / n .. \ COS 2 IJ
8111 ( t
793. Expressing the Ieft member of the given identitv in terms of sino and
f ■lno* performing subtraction of lhe obtained íraclions and using the formula
or a difference of squares, we get the leít member in the form
(sin a cos p — sin ft cos a) (sin a cos ~f•sin ft cos a)
cos2 a cos2 b
this expression yields immediately the rigbt-hand member.
366 Answers and Solutions

794. ÜS8 the formula

(putting j — y = y ) • We get

cos a
1-f-siaa

and then the left roember is transformed into the right one.
795. Solved in the same way as the preceding problem.
796. Replace 2 cos2 a by 1-f-cos 2a; tnen the numerator takes the íorm:
2 (sin 2a-f-cos 2a). Group the terms oí the denominator in the following way:
(cos a — cos 3a) 4 (sin 3a — sin a) and use the formula for a differen-
ce of cosines and sines. Taking 2 sin a outside the brackets, we obtain
2 sin a (sin 2a 4 cos 2a). On reducing by 2 (sin 2a 4 cos 2a) we get the right
member.
797. Transform the numerator of the fraction in the left member of the iden-
tity:
sin a + sin 5a — sin 3a = 2 sin 3a cos 2a — sin 3a — sin 3a (2 cos 2a—i)
Carrying out similar transforraations in the denominator we get
cos 3a (2 cos 2a — 1).
798. Transform the sum of the first two terms in the left member of the
identity using the formula for a sum of sines, and consider the third addend
sin (è — c) as a double-angle sine. We get

Now apply the formula for a sum of sines to the bracketed expression.
799. Considering the expression sin8 x 4 cos6 x as a sum of cubes, factorize
it and take into account that sin2* 4 coss x = l. Then the left member of the
equality is brought to the form
—sin4 x — 2 sin2 x cos2 x — cos4 x 4 1 = 1 — (sin3 x 4 cos2 x)3 = 0
800. Transform the sum of the last two terms as a sum of sines. We get

sin ( a 4 - y - ) + sin 4 - y - ) = 2 s i n ( J t 4 a ) c o s —2 -i- sin a = —sina

Honce, the left member is equal to zero.


801. Taking into account that
1 —cos 2a 1 — cos 2f}
sin2 a 2 2
cos 2^ —cos 2a
—sin (a 4 b )s in (a — 0)
2
Chapter X I . Trigonometric Transformations 367

the left member of the equation can be representei! in the form


sin (4 5 ° + a 4 30° — a) sín (45° 4- a — 30° 4 a) — si n 15° cos (I5: 4 2a)
and since sin 75® — cos 15®, the expression takes the form
cos 15° sin (15° 4
2a) — sin 15° cos (Í 5 ° 4 2a) =
— sin (15c 4 2a — 15c) = sin 2a
which completes the proof.
802. Transform the nuraerator of the left member as follows
(sin2 <p 4 cos2 ç ) — 2 cos2 tp = sin2 <p — cos2 ip
803. Replace sin 2a by 2 sin a cos a in the right member. Reducing the
— co s a ,
fraction by 2 sin a, \ve get in the right member the expression - —-------- ecjual

to tan2— .
804. Grouping the second and third terms, take outside the brackets
cos (a 4 ip) = cos a cos ç — sin a sin <p. The left member takes the form
cos2 <p — (cos a cos <p — sin a sin q) (cos a cos q> 4 sin a sin q)
Transforming the produet of sum by difference, \ve find:
cos3<p — cos2 a cos2 ç 4 sin2 a sín2 ip ~
— cos2 <p (i ~ cos2 a) 4 sin2 a sin2 cp =
cos2 ip sin2 a 4 sin2 <p,
«Hl thi. expression yields sin2 a.
805. Expanding the expression cos (a -j- P). 've get:
sin2 a 4 sin2 [5 4 2 sin a sin p cos a cos P 2 sin2 a sin2 p
keave the third term unchanged, group all the rest of the terms and carry out
the following transformations:
(sin* a — sin3 a sin2 p) 4 (sin2 p — sin2 a sin2 p) ~
— sin2 a cos2 p 4 cos2 a sin2 P
Now the given expression takes the form
feín a cos P)2 4 (cos a sin p)2 4 2 sin a sin p cos « cos p
== (sin a cos p 4 cos a sin P)2 sin2 (a 4 p)
A n m „: sin2 (a + p>.
OUÜi iransfonn the sum of the first three terins in the following way:
. « , . . „ 1— cos 2a , 1— cos 28 . „
sm^ a - f 8»n2 P f s i n 2 y ^ ------- ----------1---------sin- y

Sínce by hypothesis y = ?i — (a4P )> we havo

«In* a + sin» p + sin* y = 1 — y (cos 2a -i cos 2P) -f sin2 (a i (i) =~

zí.1 —cos(a ( P )c o s (a -P ) ! |1— cos2 (a f P)|


368 Answers and Solutions

or
sin2 cc -f- sin2 f5 - f sin2 y = 2 — cos (a - f (J) [cos (a — 0) -f* cos (a - f P)1
But the expression in square brackets is equai to 2cos a cos (5, and since a - f 0=
= 7i — y, \ve obtain
sin2 a - f sin2 + sin2 y = 2 + 2cos y cos a cos p
whcnce the required relation follows immediately.
807, Represent the left member in the form
cot A cot B + (cot A - f cot B) cot Ç

The expression in parenthoses is equai to , and the factor cot C,


sm A sm B
on replacing C by an equai expression n ~ { A - j - B ) , takes the form—cot (A-f-B).
Hence, the given expression is equai to
COS (A -fi?)
cot A cot B —
sin .4 sin B
üsing the formula for eosine of a sum, transform it as follows:
* n / cos /I cos B sm
sÍnJsinB
J sm B \ , , , „ , .
cot A cot B — -t— 7—.— n —s :---------—
l S;l f f i T ?») = c o t -4 coí ^ - ( c o t A cot B - l ) = l
Vsm Asm B sin
4jt
0 sin —s—
808. Replace the factors cos-z- and cos-^- by -------- and
0 . 2íi ’
2 sin - 2sm T
4ji
respectively. Then the left member takes the form sin ~ : 4 sin . And since

sin ~ = sin | — y j vn ™ , the left member bocomes .

809. Transform the left member using the formula for a sum of cosiaes.
We get 2 cos 2 Í cos — . Then procced as in the preceding problem.

810. Since 1 : cos « ..2 r...- ~ , the given expression takes the form

2 cos2 -f cos or 2 cos — | c o s j • Write cos 60: instead of y i

we get 2 cos |cos -r cos 60° j .

Answer. 4 cos - y cos ^ y - + 3 0 ° j cos |-2— 30° j .

811. Transforming the given expression as in the preceding probiera, vo get

2 cos a ^ cos a — J ■ Instead of ~ ~ "'rito cos 45°.


, . 45° a- a . 45° — a
Answer: 4 cos « s i n -----r,----- sin------------ .
Chapter X I . Trigonometric Transfortnations 369

812. Rewrile lhe giveri expression in lhe form cos2 (a -j- |5) ~ sin2 {a — P),
lhe latter expression is reduced to a form convenicnt for taking logarithms
in lhe saine way as in Problem 656.
Answer: cos 2a eos 2p.
813. Group lhe terms as follows:
(í - f cos « ) -f- (lan a 4- sin a)
and tako tan a outside the brackels in the second group. Wc gel (1 - f cos a) x
X (1 -j- lan a). Instead of 1 -f- tan a \vritc
sin (45° 4- a)
tan 45! -r tan a —
cos 45° cos a

2 1/2 cos2 sin (45° -f a)


Answer: ---------------------------------------
cos a
814. l.'sing the formula 1— cos a — 2 sin2 - and sin a --2 sin - - cos .
. . . . , „ . , a , „ . a a , t . a / . a a\
vve get m the numerator l sin* ~ 4 - sm - y cos — 2 sin — 1 sin cos — 1-

The expression in parentheses is equal to s i n ~ 4~sin ^903— . Making

use of the formula for a sum of sines. reduce ít to the form V 2 cos ( 45- — \.

Answer: 2 V - cm ^ 45° — ) .
.... ■ ■ 1 . cosa — s in a ;--1
81o. íhe given expression is equal to -------- coso-------- ’ ^ lc numerator is

transformed to 2 1 /2 cos y sin ^45° — ~ j (see lhe preceding problem). The


fraction can be still simpüfied hy representing lhe denominator in lhe form

sin {90^— a) 2sin ^45D— cos ^45° —

\ /r,l cos ya
Answer: ------------------------ .
cos (45° — 4 )
816. Since cos a — eos 3a 2 sin 2a sin a , \ve have

2 sin 2a sin a H- sin 2a — 2 sin 2 « / sin a -j ~ ) 2 sin 2a (sin a sin 30")

Answer: 4 sin 2a sin |y 4 lh5- j cos ^ y —15° ^ .


817. The given expression is equal to
t a n a 4 -1 la n a — 1 4 tan a . 2 tan a
1—l a n a " 1” 14- tan a 1 — tan2 a ’ ’ ' 1 — tan2 a
Answer1 2 tan 2a.
2 4 — U1 3 3 8
370 yl/isujers and Solutions

818. Replacing sin 2(5 by 2 sin (5 cos (5 and reducing by 2 sin (5, we gefc
t — cos p appiying the formula
14- cos (5 J
1 —cosft
ia° i " 1 -r cos p

we obtain tan2—■.

Answer: tan2 .

819. Transforming the sum cos a-j-sin a in the numerator 1 /2 —


—(cosa-j-sina) and the difference s in a — cosa in the denominator in the
same way as in Probiem 814, we get
a —45°
2sin
~]/2 [ 1 — cos (a —45°)]
V 2 s in (a —45°) 0 . a —45° a —45°
2 sin — ;r— cos — t;— .

Answer: tan — g-----.


820. Transform the sum of the last two terms:
cos 2 a -f 1 2 cos2 a
cot 2a -f esc 2a = — cot a
sin 2a 2 sin a cos a
Answer: 2 cot a.
821. Repiace cos 2a by cos2 a — sin2 a, and sin 2a by 2 sin a cos a.
Answer: 1.
822. Repiace 2 sin2 a — 1 by — cos2a and represent the given expression

in the form 2 ^-^“ -sin 2a— |-cos2a j . Write cos303 instead of - ™ - and
“l
sin 30° instead of .
Answer: 2 sin(2a—30°).
823. The numerator is equal to
cos 2a cos a -f sin 2a sin a __ cos (2a — a) _ 1
cos 2a cos a — cos 2a cos a ~ cos 2a
The denominator is equal to
sin2 q ---cos2 a J ____
sin a cos a
sin 2a

Answer: tan 2a.


824. The given expression is equal to
o, a _ 2 (1 -f sin 4a)
‘ sin 4a sin 4a
Chapter X I . Trigonometric Transformalions 371

(see the preceding problem). The expression íd parentheses is equal to

1 ~f- cos (90° — 4a) = 2 cos2 ^ ^ ^

. 4 cos2 (45° — 2a)


Answer: ---------------------- - .
sm 4a
825. The last addend is equal to cos2 x, thus the given expression can be writ-
ten in tbe íorm (tan x — 1) (1 — sin x) -f- cos2 x. Repiacing cos2 x by 1 — sin2 x
and taking 1 — sin x outside the brackets, we obtain
(1 — sin x) [(tan x — 1) - f 1 -j- sin x) =
— (1 — sin x) (tan x - f sin x) —
— (i — sin x) tan x (1 -f- cos x)
The íirst íactor is transíormed in the same va y as in the preceding problem.
Answer: 4 tan x cos2 — siir ^45“ — ~ j .
82G. The numerator and denominator oí the fraction are equal to
(1 ~f- cos 2a) ■+ (cos a + cos 3a) = 2 cos2 a 4* 2 cos 2a cos a
and
cos a - f cos 2a, respectively.

Answer: 2 cos a.
827. The given expression is equal to
(1 — sin2 p) — sin2 a cos2 a — cos4 a = cos2 p — cos2 a (sin2 a - f cos2 a)
We get the expression cos2 p — cos2 a, wbich is transíormed as in the solution
of Problem 650.
Answer: sin (a -f P) sin (a — P)-
828. Reduce the given expression to a common denominator cos x cos y cos r.
The numerator will be
sin x cos y cos 2 - f sin y cos z cos x -r
4- sin 2 cos x cos y — sin [(x -f- y) + z)
The last term is equal to —sin (x -{■- y) cos 2 — cos (x + y) sin 2. The sum
of the íirst two tcnns and the term —sin (x - f y) cos 2 are mutually anmlnlated,
and the numerator takes the íorm:
sin 2 cos x cos y — cos (x - f y) sin 2 = sin 2 (cos x cos y — cos (x </)J
Expanding the expression cos (x -{- y), we get in the numerator sin 2 sin x sin y.
Answer: tan x tan y tan 2.
a —P
829. The given expression is equal to 2 sin — j - -f-[sin y. Rut by
hypothcsis y = 180o — (a 4 P); hence, we get
t — P , 0 . a -j p a -i - p
2 sin K cos - 2 sin — cos —

24*
372 Ansurers and Solutions

Take 2 sin ■ outside the brackets |or, which is the same, 2sin i í —— - =

~ 2 cos j . The bracketed expression becomes cos a ^ ^.-f-cos , which


is transformed according to the formula for a sum of cosines.
. » a 6 Y
Answer: 4 cos — cos —■cos .

C H A P T E R XII

TRIGONOMETRIC EQUATIONS
830. After simplificatíons we get sin 5x — sin 3x = 0. Using the formula
for a difference of sines, we have 2sin x cos 4x = 0, and the equation is reduced
to the two equations: sin x = 0 and cos 4x = ü. From the íirst one we have
x = jt« (/i is any integer), from the second 4x — 2nn ± —y (4n ± i), i.e.

x ^ | .(4 n ± 1)

The expression An ± I coinprises all odd numbers (the nurabers —3, I, 3, 9, 13,
etc. are yielded by the expression 4 m + 1; the numbers —1, 3, 7, 11, 15,etc.,
by tiie expression An — f). Therefore, instead of An ± 1 we may write 2« -f
- f 1 (or 2n — 1), where n is any integer.

Answer: x = í\n\ x = -~-(2«-: i), where n is any integer.


o
831. Transform the left member of the equation in the following way:
sin x - sin 2x-( sin 3x-f sin 4.r = (sin x~- sin 3xH- (sin 2x---sin4xt =
= 2 sin 2xcosx : 2sin3x cosx --2 cos x (sin 2x-f-sin 3x) =
, . 5x x
~ A sin cos -ç- cos x

The equation takes the form


. 5x x
sm -rp cos -p cos x ~ 0

and reduces to three equations:

cos~ ^0; cosx= 0

.-lnsuvr: x~-72°n; x ---180'- (2o -f 1); .r --1HV {2/r-f 1).


832. Períorm the following transformaíions:
cos (x H- 60: ) - cos (90° - (3( f - x)} - sin (30° - r)

1 -f- cos 2x — 2 cos2 x


Chapter X I I . Trigonometric Equations 373

The equation takes the form


sin (x - f 30°) -j- sin (30° — x) = 2 cos2 x
Apply the formula for lhe sum of sínes; this is the result:

sin 30° cos x — cos- x = 0 or cos x — cos x j = 0

Answer: x = 90c (2o - f 1); x -■= (iüc (G« + 1).


833. Transpuse all the terms of the equation to the left side and group them
in the following way:
(sin x + sin 3x) — (cos x -f- cos 3x) ~f (sin 2x — cos 2x) = 0
Transforming the first two groups, we get
2 sin 2x cos x — 2 cos 2x cos x (sin 2x — cos 2x) = 0
or
(2 cos x -j~ 1) (sin 2x — cos 2x) = 0
This equation is reduced to the following two:
2 cos x -\- 1 — 0 and sin 2x — cos 2x = 0

The first one yields: cosx-r- — ™ ; -fc ™ ít. Divíding the second equa­

tion by cos 2-r, we get tan 2-r 1, whence 2 x ■. n n .

Answer: x (3« ± 1); x=~ — (4n-i i).


3 o
834. Ferform the following grouping:
(cos 2x - f cos Gx) —• (i -j~ cos 8x) = 0
Using the formula 2 cos2y *■- 1 - f cos a and transforming the sum of cosines,
we obtain
2 cos 4x cos 2x — 2 cos2 4x = 0
Take 2 cos 4x outside the brackets and transform the difference of cosines
cos 2x — cos 4x. We obtain the equation
cos 4x sin 3x sin x — 0
It is reduced lo the following tbree:
(1) cos 4x 0; (2) sin 3x -- U; (3) sin i - O
No consideration may be given to the third equation, since all its solutiuns are
covered by the Solutions of the equation sin 3x ---■ 0. Indeed, if sin x = 0,
then also sin 3x 3 siri x — 4 sin3 x — 0.
Answer: x (2n ■] i ); x - - - ^ - .
o «>
3x 3x
835. Hepresent the right mernher in the form 2 sin ~ cos — (instead of
sin3x). The equation takes the forrn
„ . 3x . x . 3x 3x
2 sm — sin y 2 sin cos - - -
374 Answers and Solutions

or
. 3* / . x 3x \
sm - y I sm ——cos ~ j = 0
Write the bracketed expression in the form

Hence, the given equation is reduced to the foilowing three:

(1) s i n - y —0; (2) s i a ( ~ 4 ' y ) = 0 ; (3) sin ( * — ■y ) = 0

Answer: x = x = - y (4b — i); x ~ y (4 n -f-1 }-

836. The right member is equal to


sin [90° — (x -j- 30°)1 = sin (60a — x) — —sín (x — 60°)
The equation takes the form
sin (x — 60°) = —sin (x — 8Ü=) or sin (x — 60°) — 0
whence x — 60° — iS0°/2.
Answer: x — 60° (3n - f 1).
837. Replacing 2 sin2 x by i — cos 2x, reduce the equation to the form
2 sin 3x cos 2x — cos 2x = 0. Thís equation is reduced to the foilowing two:
i 1
(i) cos 2x = 0; (2) sin 3x = y • Since is sin 30°, the second equation yields

3x - 180°n - f ( - i } i 30°
-4rsuw: x — 45° (2n + 1 } ; x — 60°n -f" ( —l)'1 10J.
838. Rewrite the right member: 3 (sin x cos x — sin2 x + 1) —
— 3 (sin x cos x - f cos2 x) — 3 cos2 x (tan x 1). The given equation is reduced
to two ones: (1) tan x -J- 1 — 0; (2) sin2 x — 3 cos2 x = Ü. From the second
one we get tan x = ± V ^-
Answer: jx —y ( 4 n — 1); x = ~ (3/í ± i).

839 We have the equation


cos 4x ~f - cos2 x = 0
Since 2 cos2 x = 1 - f cos 2x, the left member is equal to
(í - f cos 4x) - f cos 2x — 2 cos2 2x -j-[cos 2x
We get the equation
cos 2x (2 cos 2x + 1) = 0
which is reduced to:
(í) cos 2x — 0 and (2) 2 cos 2x - f 1 » 0
The second one yields 2x = 360°n ± 120°.
Answer: x = 180an d; 45a; x = 180'n + 60°.
Chapter X I I . Trigonometric Equations 375

840. Multiplying both members of the equation by sin x and rcplacing


unity in the right member by sin2 x -f- cos2 x, we get the equation sin x cos x =
== cos2 x.
Note. Multiplying both members of the equation by sin *, we introduce no
extraneous Solutions, sínce sin x never vanisnes at either of the found vaiues
of x.
Answer: X( = ~ (2n + 1): x2= (4« + 1).
841. Rewríte the equation in the following way:
sin 3x— sin — 2x^=^0
It is reduced to the following two equations:

(1) c o s ( | + ^ ) - 0 and (2) sin ( - y — - j - ) = °

The first one yields ~ = (2n - f 1), wlience x — The second

gives x s = ~ ( 4 n + l).

Answer: x — - {4n 4-1); x = (An ~r 1).

842. Add 2 sin2 y cos2 y to both members of the equation; then in the
left member we ha ve

sin4 ~ ~ r 2 sin2 ~ c o s 2-~~|" cos4- y = ^ sin2 cos2 — ^ — 1

and the equation takes the forra


, 5 . x „ x „ . „ x » x 3
1^ >sm2 cos2 -y or 2 sin* -g- cos2 — — -g-

Multiply both members of the equations by 2 and apply the double-angle


. , , , . . , 2x 3 . . 2x , VZ
formula for srae. We obtain s i n - - j - = , whence s i n - ^ ~ = ± — .

Answer: x = -~ -(3 «± ; i).


843. Ropresent the equation in the form 3 tan2 x~~(í -j tan2 x) = 1, whcnce
tan x -v: ^ t .
Answer: x = 45® (2h 1).
844. Replac© l-j-cos4 x by 2 cos2 2x.
4
Answer: nn 31 ; x = nrl -f
~t —- i ,( — l)n
4\n -^r
71 .

845. Add 2 sin2 x cos2 x lo both members of the equation. We get (sin2x-f-
- f cos2 x}2 ~~cos 4x j 2 sin2 x cos2 x or 1 — cos 4 x— sin2 2x.

Answer: x = 4Jj- n.
376 Answers and Solutions

846. Replace sin 2x by 2 sin x cos x and divide all the tenns by cos2x-
It is obvious that no roots are lost. Indeed, ií cos x = 0, tben sin x — ±1»
but these values do not satisfy tbe given equation. We get
3 ~ tan2 x — 2 lan x — 0
whence
tan x — 4 and tan x ~ ~ 3

Answer: x = nn x ~ nn — arctan 3.
847. Write sin2 x - f cos3 x instead of unity and, dividing both members
by cos2 x (seo the soiution oí the preceding problem), we get
tan2 x 4- 1 /3 tan x — 0
whence
tan x = 0 and tan x — — ~\/s

Answer; x —rtn; x ~ ™ ( 3 n —i).


848. Replace 2 by 2 sin2 x - f 2 cos2x , and then the equation is solved as
the preceding one.
Answer: x — n n -f -r- ; x =- nn — arctan - r .
4 4

849. ^nsu-rr: x ~ (4n-+• 1); x = xn -f- arctan— .

850. Replace V 3 by cot 30° (we introduce an “auxiliary augle1' 30c). Then
the given equation becomes
cos 30°
sin x -r- ——— cos x = 1
sm 30"
or
sin x sin 30° - cos xcos 30° - sin 30°
or
_t_
cos (x —30°)
2
Hence, x — 30° = 360° n ± 60*.
Answer: x --- 36Üc/i-b 9UV 90° (4n-f* I)» x ~ 360cn — 30c = 30c(i2 «—i).
851. The leít roember can be represented in the form of a product:
V I cos (x — 45'). Then we get the equation cos (x — 45c) ™ — ; it yieids

x — 45° — 360o» — 45c and .r — 45c — 3C0'n - f 4 5 / i.e. x = 360'» and


x = 360c« -f 90% or x =- 90'*4» and .r ~ 90' (4« - f i).
Alternate metkod. Squaring both members of the equation, we get
sin2 x -j- 2 sin x cos x -f cos2 x -- 1
or sin 2x = 0. This equation lias the Solutions x 90 /?, but among tliein there
are extraneous ones (compare with the preceding result).
Extraneous Solutions liave resulted írom squaring both members, whereby
we introduced one moro equation (in addition to the given one): sin x - f cos x —
.. 1 (which also yieids sin 2.r -- 0). To reject extraneous roots we have to
Chapter X I I . Trigonomelric Equations 377

accompiish a chcck. At n — O we have x — 0C and the given equation is satis-


fied. It is also satisíied at n = 4, 8, 12 and, in general, at n = 4A- (i.e. at x —
= = 360°A*). At n — i we liave x — 904 the given equation is satisíied
once again. It is also satisíied at n —■ 5, 9, 13 and, in general, at 4À- - f 1 {i.e. at
ar = 90° (4A- - f 1) = 90° -J- 3G0°A). But at n - 2. 6, lü (in general, at n «
= 4k 2), the same as at « — 3, 7, 11 (in general at n — Ak 4- 3) the given
equation is not satisíied (instead, the equation sin x 4- cos x — — 1 is
satisíied).
Answer: x — 90°*4n; x — 90c (4n - f 1).
852. Transíorm the right. meniber:
1 + sin 2x == sin- x - f cos- x ~f- 2 sin x cus x =•• (sin x 4- cos x)2.
now the equation takes the íomí
sin x + cos x = (sin x - f cos x)2
or
(sin x - f cos x) (sin x 4* cos x — 1) - 0
The latter is reduced to two equations:
( 1) sin x 4- cos x — 0
(2) sin x 4" cos x — i =- 0

Solving the first one, we íind x — (4« — 1). The second one is solved in
the jireceding problem.
ti n n
Answer: x = - 4 -(4 n — 1); x - -rr ( 4 n 1); x — — .An.
4 *
853. Solved in the sarne way as Problem 851.
Answer: x — 15c (8/i -J- 1).
854. Using the formula
1
sin a sin P --g- [cos (a — P) — cü$ (a : p)j
we get
~ (cos (x — 7x) — cos (x 4* 7x)| [cos (3x — 5x) — cos (3x 4 - 5x)|
or, after sirnplifications, cos Cx — cos 2x O. This equation is reduced to the
following two: sin 4x — 0; sin 2x - O, all the roots of the second equation
hoing among tliu roots of lhe first one.
. Jln
Answer: x - —7— .
4
855. Apply to botli members oí the equation lhe formula
sin a cos P — -J- |sin {a j •p)4*sin'(a — P)|

Answer: x = ——
859. We have
4 sin x sin 2x sin 3x siri 2 (2x)
or
sin 2x (2 sin x sin 3x — cos 2x) = 0
378 Answers and Solutions

Replace 2 sin x sin 3x by cos 2.r — cos 4x {see Probiem 854) to get the following
equation
sin2x(cos2x —cos4x— co s2 x )= 0 or sin2xcos4x = 0,

Answer: z — ^ í - ; x = 4 r (2n-f 1).


2 o
857. Replace sin2 x by 1 — cos2 x; we get
5cos2x-|-4cosx —3 = 0

t/ 1
9 _2 T /Í9 4-2
whence c o s x = —— =------ . The other root cosx = — - — - is not suitable,
5 5
since its absolute value is more than unity.

Answer: x = 2nn ± arccos |— —.

858. Using the formula cos 2a and expressing cosine through sine, we get
10 sin2 x -r- 4 sin x —5 = 0.
, , , . -2 ± V 5 4
Answer: x = JW-f { — t)«arcsin------- ---------- .

859. Applying the formula for tangent of a sum, we get


1 tan x
tan
tan x
and reduce the given equation* to tan2 x —4 t a n x - f i = 0 .
Answer: x = nn-\- arctan (2 ± ~\/3).
\_cos X 1
860. Since tan2 T X _ _ - and sec* = _ , we have the equation

8 (1 —cos x) , 1
l~ -cosx ~ "^cosx
which is reduced to the form
9cos2x —6cosx-j-1 = 0 or (3cosx —1)2 = 0
1
Answer: x = 2nn ± arccos — •
861. The left member is equal to

cos — x\ 2 sin y c o s -~
\2 / __ sin x x
= tan
1-f-cosx ~ l-r c o s x n o—* 2f
2 cos2

* When getting rid oí the denominator one should be careful not to introduce
extraneous Solutions, but we do not conduct analysis in the next three problems
(since they have no extraneous Solutions). Beginning with Probiem 865 much
attention is paid to such an analysis. See also Probiem 867.
Chapter X I I . Trigonometríc Equations 379

The right member is equal tosec2 -^— i — tan2 . \Ve obtain

x _ x
tan y = tan2 y

Answer: x=2nn\ a: = ~ ( 4 n - r l ) .
862. Since
, x i . * -r x x
cos (ji —•x) = — cos x and ssn— ^— —c o s y
we have
1 + co s x -f-co s y - = 0 or 2 cos2*—--f-cos =0
4ji
Answer: x — n (2n-f i); x = -y -(3 n 4; 1).
863. Applying the reduction formulas

-cos ar and tan


sin( - i L- I ) = (t - t )-
- get the equation
2 (1 -f-cos x)— '\/3 cot —0

Let us make use of the formula


X i 4 COS X
cot -rr ~ — ^--------;
2 sm x
then the obtained equation is reduced to the following two equations:

(1) l-f-cosx = 0 and (2) sin x = V 3

Answer: x = Ji(2n-}~l); x~ n n -'r ( — l)n y .


864. Repiacing cos2 x by 1— sin2x, after simpiifications vve get 3 s in x -f
-f- cos x = 0 or t a n x = —y ,

Anmer: x = jtn — arctan-y.


865. The left membor is equal to
cosa? , sin x cosx-j-cos2x - f sin2x _ f-f-cosx
"sin x ^ 1-j cosx ~ sin x (l-j-cos x) sin x (1 -[-cosx)
Reduco the fractíon by i + c o s x , assuming that l-f-cosx 4 -- 0. We get the
1 1
equation ~ 2. i.e. sinx - - 5- (at th is valuo of sin x the quantity cos x is
not equal to — 1).
Answer: x = nrt + { — l)n- y .
380 Answers and Solutions

866. According to lhe reduction formulas


cot (x — ji) = — cot (n — x) = cot x
The given equation may be rewritten as

2 cotx — (cosxH sinx) f —4 ----------í—


V sin x cos x /
On reducing the left member to a comrnon denominator this equation takes
the form
1
sin x cosx

wlience sin xcosx = 4- or sin 2x


: 2 '
Jl
Answer: x = Y » + ( - t ) n
líT*
867. The right member is equa! to
1
----------- cos x . „
cosx __ 1 — cos-x sin- x
2 sin x ~ 2 sin x cos x ~ 2 sin x cos x
Reduce the íraction by sin x. It is assumed that .sin í í O, should \ve obtain
such a solution, for which sin x ----- 0, it svouid not be suitabíe. The given equa­
tion (after appiying some reduction formulas to its left member) takes the form

sin x-; tan x ~ ~ tan x or sin x -f tan x = 0

This equation may be vepresented in the form

sin x J— W t )
\ ' 2 cosx /
and it is reduced to two equations
sin x — U and 1 -r -i— — 0
2 cos x
But the first equation vields extraneous Solutions, since v e reduced the íraction
by sin x beíore. To get a bctter understanding substitute sin x ™ O into the
right member; then instead of cosx \ve have to substitute 1 or —t. in both
cases we get the indeterniinnte form — .
2n
Answer: X” — ± i).
868. The left member is eijual to
Chapter X I I . Trigonometric Eguations 381

Reducíng by 1 — ian-^ (we assume that í — tan ~ = 0, see the solution of

the precediug probiem}, we gel —tan -í-, and the equation takes the form

— tan — ~ 2 s in ~ or sin ~ ^sec~--~2^ =Ü

It is reduced to two equations:


X X I
(1) sin-y = 0 and (2) cos — = — j

From the second equation we find —- — 3C0'/t ± 120; and get the solution
x = 720'n ± 240°. The first equation yields only extraneous Solutions
(x = 360e«), though for another reason than in the preceding probiem. Namely,
the quanüty cot — , cntering the given equation, loses sense (“becomcs equal
to infimly,‘) at x — 360°n; bence, the whole left member of the equation bas
no (direct) mcaning.
Answer: x 240° (3« ± 1).
869, Using reduction formulas, we ohlain the equation sin x — tan x --
= sec x — cos x or sin x — ; —í------- cos x. Muftiply both memhers
cos x cos x
oí the equation by cos x (or. wliich is the same, reduce it to a comimm donomi-
nator and then reject it). It is assumed that cos x 0, since if cos x 0,
then the exprcssions and —-— lose their meaning ("become infínitelvgreat”).
1 cos x cosx
We get the equation
cos x sin x — sin x sin2 x
which is equivalent to the following two:
(1) sin x = 0; (2) cos x — sin x -- 1
The second one raay be rewritten as ^ 2-cos(45' -f- x) -= i (cf. Probiem 8ói),
whence x - ■ 360 « and x -- 360 h — 90°. The solution x ~ 360 n is íound arnong
the Solutions of the first equation (x - 180 «), and the solution x - 360 o — 90''
is an extraneous one, since we have cos (36UTi — 90') - ü.
Answer: x --- 180«.
870. Uso the formulas: sec2 x —- tan2 x - 1 and cos 2x = cos2 x — sin2 x.
We get the equation
cos2 x —sin2 x
í — tan x
cos2 x
which is reduced lo the form tan2 x — tan x .--O.
Answer: . - ( 4 n T-1).
871. Rewrite the equation in the form
sin3 x (sin 2 COS X ) ^ COS:i X (sin x f COS X)
scos2 x —sin2 x
sin x cos x
382 Answers and Solutions

Assuming that sin x =£ 0 and cos x # 0, reduce the íractions, transpose all
the terms to the leít side and take sin x -f- cos x outside the brackets. Weobtain
(sin x 4- cos x) (sin2 x -f- cos2 x — cos x + sin x) = 0
Replace sin2 x - f cos2 x by i. The equation is reduced to
(1) sin x -f- cos x — 0 and (2) cos x — sin x = 1

The first equation yields x ~ ^ - ( 4 n — 1); the second (see Problem 869) has

two Solutions x = 2,-in and x = (4/t — 1). Both of them are extraneous, since

at x — 2ítn we have sin x = 0, and at x — — (4n — 1) we have cos x — 0.

Answer: x = ~~(bn~-1).

872. Use the triple-angle formulas:


sin 3x — 3 sin x —4sin3x, cos3x = 4 cos3x-—3cosx *
The left member is transformed to the form
3 3
3 sin x cosx (cos2 x —sin2 x) = y sin 2x cos 2x = — sin 4x

i
and the given equation takes the form sin4x —— .

. nn , , ... n
Answer: x - —^--*- ( — l)n ,

873. Rewrite the given equation:


tan 2x = tan 3x — tan x
and divide both members of the equality by 1 -f- tan x tan 3x to apply to the
right member the formula for tangent of a difference of two angles. We get
tan 2x
= tan (3x —x),
1 tan x tan 3x
whence
tan 2.r = tan 2x (1 + tan x tan 3x)
or
tan x tan 2x tan 3x — 0
Consider the following three cquations separately:
(1) tan 3x = ü; (2) tan 2x - 0; (3) tan x = 0

The solution of the first is x = ^ . The tbird equation yields notbing new
since all its Solutions (x = nm) are found among the Solutions of the first equa-

* If they are not familiar to the reader, it is easy to reduce them using the
formulas for sine and cosine of a sum of two angles: 2a and a and then the formu­
las for .sine and cosine of 2a.
Chapter X I I . Trigonometric Equations 383

tion (at n — 3m we have y — nm). The second equation yiclds x ~ ~ . At

even n these Solutions again yieid notbing new {at n 2k we have ~ = nk);
with odd n (n = 2n' 4 1) they are not the Solutions of the given equation.
Indeed, the quantities tan x and tan 3x, entering the equation, lose their ineaning
{“become infinitely great’’) at x ~ y (2n' 4- 1)- Therefore, the second equation
should be rejected.
. jm
Answer: x ——y .
874. Applying tho formula for cosine of a díffercncc, reduce the right
member to the form \ '2 ^cos y 4 sin —*j . Thereíore, express the left memher

through tlie a rgu m en ty. We have

(1-j- cos x )-f sin x ~ 2 cos2y 4 2 sin y cos y --- 2 cos y ^ cos sin ■
—j

Transposing all the terms to the left side we get the equation

^cos y 4 sin y j ^ 2 cos y — ~[/2 j = 0

equivalent to the following two equations: one yiclds x — 36üs« — 90\ the
otner x = 720°n ±. 90°. in the latter expression the double sign ruay be replaced
by the plus sign, since all the quantities 720 « — 90c are aiuong the quantities
360°n — 90° (if in the expression 360 n — 9Ô'J we take oniy even n i.e. if we
put n = 2n', we get 720on' — 90c).
Answer: x = 360°n — 90c; x 72(fn 4 90°.
875. Rcwrite the given equation: sin2 2x — sin 3x 4 sin x; hence, sin- 2x =
~ 2 sin 2x cos x. Transposing all the terms to the left side we get
sin 2x (sin 2x — 2 cos x) — 0 or 2 siri 2x cos x (sin x — 1) ~~ 0
The equation is reduced to:
(1) sin 2x = 0; (2) cos x ^ 0; (3) sin x = 1
Equations (2) and (3) are of no interest, since all their Solutions are among the
Solutions of the first onc. (We have sin 2x ~ 2 sin x cos x — 2 sin x~ f 1 — sinV,
so that if cos x = 0, or if sin x 1, then sin 2x - 0.)
Answer: x — 90°«.
876. The left memher is equai to 2 cos2 x — 3 cos x. Tho right one loses
its meaning at 2 = ~ n , since cot 2x “becomes infinitely great". Therefore*

we consider that x y n. The denominator of the right memher is equai to

cos2x cosx (2 cos2 x — 1) — 2 cos2 x _ —t


sin 2x sin x 2sinx-cosx 2 s in x -c o s x ’
384 Answers and Solutions

thus, the right member is equal to


•—esc (n —x)-2sinx*cosx = —2 esc x-sin x*cosx

The product cscx-sin x -5™ -j can be replaced by unity, since the

values of x, at which the fraction iÍL f. would turn into the indeterminate form ^ ,
sm x 0
are rejectetí. We get the equation
2cos2x—3cosx——2cosx or cosx ^cosx ——j —0

whence cosx —0 or cosx — In the first case we obtain the values

x— 1), which were rejected above.

Answer: x — (6« ± 1).


877. The left member is equal to
(cos x -r sin x)2 -}-1 — 2 -f 2 cos x sin x,

the right member is equal to cos2 x, assuming that sinx ^ 0 .

The equation takes the form


2 {I — cos2 x) -r 2 cos x sin x — 0 or sin2 x - f sin x cos x — 0
It is equivalent to the two equations: sin x - f cos x -- 0 and sin x = 0, but
at sin x — O the right member has no (direct) meaning.
Answer; x ~ —
878. The right member is equal to

2 sin|-^- j x j cos | -~ -{-x j —sin ^ — 2xJ —cos2x —cos2 x —sin2 x

Then proceed as in the preceding problem.


Anmer; x — (4/i — i); x —
879. The left member is equal ío 2 — sin3x, the right one to
. / ji 3x \ . / ji 3x\ . / ?t _ \
1 — 2 cos ( -£ -— 2” ] sin ^ ------ j -J = i —sm ( ^ - - S x j ^ l _ Cos3x

The equation takes the form


cos3x — sin3x-j-1 —0
Solve it using the (first) method of Problem 851, transforming cos3x — sin 3x

lo y i s i u | -J ~ 3 x j . We obtain

_1_
-i!ií f :iJ) ' ~ ~ f ? sin ( 3 l - - J - )
V2
Chapter X I I . Trigonometric Equations 385

Consequently,
3x— ~ — ( — 1)» ■— -fjin , i.e. 3 x ~ ™ [í -f-( —

At even n the expression in square brackets is equal to 2, and at odd n


it is equal to zero. Therefore, if we put n — 2n' («' ís an intcger), we get
3x=;-^—f-2nn', and íf we put n = 2n'*i-l, we get 3ar = ji (2n' -r- i).
Altemate solution. Besides the alteniate method indicated in Problem 851
(which introduces extraneous roots), we can use here (as aiso in Probíem 851)
the following method. Getting, as above, tbe equation cos 3x — siri 3x + i — 0,
use the formulas
1 -f cos 3x = 2 cos^ and sin 3 x = 2sín-^- cos

We get an equation which is reduced to the following two: one =

y i e l d s = ~ (2u-f 1), i.e. 3x — n (2«-{-1). The other |cos^p —s i n —0 j

yields J l - f j t * , i.e. 3x = “ -f-2nn,

Answer: x = 1); x ~ (in + 1).


880. Reprcsent 1 - f sin 2x in the form
(cos2 x -f- sin2 x) -j~ 2 sin x cos x -- (cos x sín x)2

and renlace tan x bv . Heduce all the terins to a common denominator


* J cos x
(cosx) and then get rid oí it, assuming that cos x =£ U. We get the equation
(cos x — sin x) (cos x -f- sin x)2 - - (cos x -j- sin x) -- O
which yields two equations: the first one
cos x - f sin x — 0
has the solution
“-(4— 1).
the second
cos2 x — sin2 x - 1 = O, or cos 2x — i
has the solution

Answer. x ~ ~ -(4 /i — 1); x = n


881. Reprcsent 1 — sin 2x in the form (cos x — sin x)2, and cos 2x in the
form (cos x + sin x) (cos x — sin x). Reduce the fraction by cos x — sin x,
assuming that this quantity is non-zero. We get the equation
cosx-f-sm x
cos x-psin x =
cos x —sin x
l (z 2 5 ~ 0 í 3 : íh
386 Answers and Solutions

Getting rid of the denominator (under the same assumption), we obtain


(cos x - f sin x) (cos x — sin x) — (cos x -f- sin x) = 0
or
(cos x — sin x — í) (cos x -f- sin x) = 0

Solving the equation cos x - f sin x = 0, we fínd x — n n ~ The equation

cosa: — sin x — 1 = 0 can be solved in the íoüowing way (see Problom 879)-

Represent it in the íorm 1/2-sin —- x j — i , i.e., sin *


V 2
Hence,, x — j ~ ~ ( — l)n | W i t h even n — 2m we have x — nn^

= 2nm. With odd n ( = 2m — 1) we have (A m -l).


Apply the other method from Problem 879.
Ansteer: z = ~ (4/i--~i); x = 2jw ; x ^ ~ {4 n ~ ~ l),

882. The right member is equal to cos2x, and the left one to
(cos x-r sin x)2 (cos x —sin x )~
= (cos x -j- sin x) (cos2 x — sin2 x) — (cos x -f sin x) cos 2x

Ansuwr: x ■(2n -fl)', x = 2nn\ x = - —■


883. The left member is equal to
í 4 sin2 x cos2 x 1 , .
4 cos2
(assuming that cos x 0). To the right member apply the formula
1 1
sin a sin p = y [cos (a — P) — cos (a-fP)]* We get y (cos 60a — cos 2x)

= s-i —(t — 2 sin2x) j — — I x . . N o w the equation takes lhe íorm

- i —sin2i = — ( - i —sin2i )

1 . . 1 . 1
whence sin2x = = y » i.e. sinx = -^- or smx — — The two Solutions x =

= 180°/i --- { — l)n 30° and x = 180cn — ( — l)a 30° can be reprcsented by one
formula: x — 180°n ± 30°.
Answer: x —30° (ün ± 1).
884. The left member is equal to sin 60° cos x; the right one to
tan x cos4 x -f- cot x sin4 x = sin x cos3 x 4* cos x sin3 x
(assuming that cos x ^ 0 and sin x ^ 0). This expression is equal to
sin x cos x (cos2 x + sin2 x) — sin x cos x
The equation is reduced to the forra cos x (sin 60° — sin x) = 0.
Chapter X I I . Trigonometric Equations 387

It is reduced to two equations, one of which (cos j - 0) yields an exlraneous


root.
Answer: x = 180°n - f (— l)n 80°.
885, Using the formula tan -^-==--jC0S1 , we get the left meml>er ?ec2x —
2 sin -r
— l= ta n 2 a; (reducing by sins we assume that s in x ^ O ). The left member
is equal to
sin (z — 30c) - f siri (z 4-30°) _ 2sinxcos30°
- V ^ tan x.

The equation is reduced to the íorm tan x (tan x — =■- 0 and is equivalent
to two equations, one of which, namciy tanz —0, yicids exlraneous Solutions
(sinee if tanx = 0, then also s in x = 0 ).
Answer: x = 60° (3«4~1); x~2stn.
886. The expression tan-g--fcot ~ is transformed to
i 2
. x x sin x

We get the equation

V 2s
1^2 sin x

Answer: x = — (2n-\-i).

887. The left member is equal to 2 - (sin x -]■ cos x); the right one
2 cos2 x 2(1 — sin2x)
:2 (1 — sinx). We get the equation
t 4-sin x 1-f-sinx
2 (sin x + cos x) — 2 (1 — sin x) or (1 — cos x )~ 2 sin x ----- 0

X
2 sin2 — — <5sin
, . J- -t
y coS ~ z'

Answer: x ~ 2 nn; x = 2 (nn-f arctan 2).


888. The fraction in the left member is equal to
2
~.y.- (sin 2x — cos 2x tan x) cos2 x --

: (sin 2x cos x — cos 2x sin x) cos x — — ~ sin x cos x


1 /3 1/3 •
The right member is equal to
(cos2 x 4- sin2 x) (cos2 . •sm2 x) — cos2
388 Answers and Solutions

(double angles are of no use here). Write the equation in tbe íorm:
2
{ i —cos2 x)-j-sin2 x ---------- sin x cos x = 0
1 /3

2sin2x -------=*sinx cos x —0


1/3
Answer: x — 180°n; x — 180°n -f- 30°.
889. The left meraber is equal to 3 sin x — 4 sin3 x, and the ríght one to
4 sin x (í — 2 sin2 x). We get the equation
sin x (4 sin2 x — 1) = 0
Answer: x = 180°n; x = 180°n ± 30°.
890. The right member is equal to

The numerator of this expression is equal to cos ^x— —j — cos , thus, '

get cot ~ in the right member. The left member is equal to

, . 2 cos2 ~r
1 -f cos x 2

The equation takes the form

-cot -rr — 0

Taking cot ~ outside the brackets, we obtain

=0, i.e., cot — {tanx-—1 )= 0

Answer: x —rtn ~f- -™-; x —2nri-j-n.

891. The denominator of the fraction is equal to


sin 2x cos x — cos 2x sin x sinx tanx
cos x cos 2x ~ cos x cos 2x — cof2x
The whole fraction is equal to sin 2x. The equation takes the forra
sin 2x — 2 sin (45° - f x) cos (45° - f ,r) = 0
Chapler X I I . Trigonometric Equatíons 389

sin 2x — cos 2x = 0
whence tan 2x — 1.
Answer: x — 90cn -f- 22°30\
892. We have
tan (x — 45°) tan (x + 45°) = tan (x — 45°) cot (45® — x) ~ — i
it is assumed that x # 45° (2n -f- 1), since otherwise one oí the factors vanishes,
and the other becoraes infinitely grcat. The denominator oí the right member
is transformed to
2 cos x
— 2 cot x
sin*
si

ít is assumed that x ^ 380®», since then eithertan y , or cot y loses its meaning
(becoraes infinitely great). We gel the equation
4 cos2 x

which (assuming that x ^ 180°n) is reduced to the íorm tan x cos 2x = 0.


The last equation has the solution x — 18G°/t and x — 45w(2« -f- 1), hut they
disagree with the above assuraptions.
Answer: The equation has no Solutions.
893. The right member is equal to — tan x (see the preceding problem).
Let us represent the left member in the íorm:

y ftan (x + 45°) —cot (x -f 45°)| —

■ „ _ col (2 x + m
2 sin {x 45") cos ( x 45°)
We get the equation
tan 2x — —tan x
U can be written in the form
tan 2x — tan (—x)
whereírora we conciude that the anglcs 2x and —x díffor by 180'n, and frora the
equation 2x = —r - f 180:V< we find x — 00'n.
Answer: x ~ 60®«.
894. The left member is equal to
sin 2x sin 2x
cos (X -r a) cos (x — a) (C()S 2 a c o s 2x)

We get the equation


2 sin 2x
o s 2a - f cos 2x
390 Answers and Solutions

Using the formulas sin 2x = 2sin x c o sx and cotx = ~ ^ í * . reduce the eaua-
6 sm x *
tion to the form
cos x (2 sin2 x — cos 2x — cos 2a) — 0
The equation
2 sin2 x — cos 2x — cos 2a = 0
with 2 sin2 x replaced by 1 — cos 2x yields
2 cos 2x — 1 — cos 2a
whence
cos 2x = sin2 a

Answer: x = ~ {2 n -^ \ ); x~an 4; ™ arccos (sin2a).

895. The left member is equal to 1 — sinx; the denominator of the right
member, to
x . ,/ a , x \ . x , .x
tan -g- — tan { - y -í ~ 1“ tan y - f cot

This expression is reduced to the form . We get the equation


1 — sin x = sin x.
Answer: x — 180o» -f- ( —l ) n 30°.
896. The left member is equal to tan x. The right one (cf, Problera 894),
to 1 4- 2 tan x.
Answer: x = 45" (4« — 1).
897. We have
©
©
H

sin* x "• (sm- xr — y------ 2“


analogousiy,
‘ 1 —cos |2x-i--^-j * 2
|/ « 1-f sin 2x j 2
sin* ^x <- -
L l2
The equation takcs the form 1 — cos 2.r -f- sin 2x = 0, or
2 sin2x ~r 2 sin x cos x = U

x = a »; x -- a» — r - .
A n su w :
4
897a. Represent the equation (?ee the preceding problem) in the form
| l~ -~ cos2 x j 2 ^ t ----- sin 2.r j 2 ^ t — s i n 2 x j 2 __ 9

After algebraic transformations we get


9
3 — 2 cos 2x -f- cos2 2x 4- 2 sin2 2x ™—

Roplacing sin22x by 1— cos22x, we obtain the equation

cos2 2x - f 2 cos 2x — “ —0
Ckapter X I I . Trigonomeíric Equalinns 391

It yields cos2 x ~ — 1 - f ^eos 2x ~ — i — ™ ^ i s rejected, since jts abso-

lute value exceeds unity j .

Anwer: x = nn ± ~ arccos ^ — 1 + j .

898. Represent lhe left meinber oí the equatíon in the form ~ (cos x 4- cos y).

Solving the System, ve find c o s x ^ ~ \ co s y = ~ * .

Answer: x — 2nk ± ; y = 2nl ± ~ ~ .


899. Since
COS(x—»/)—COS{X-}-y)

the second equatíon can he wrilten in the following way:


cos (x — y) — cos (x -{- y) 2m
But x -}- y = a; conscquently,
cos (x — y) — 2m -j- cos a
whence
x — 1/ = 2nn ± arccos (2m -f cos a!
and the given system is reduced to the following two sy.steins:
( r-y -^ a
\ x — y —2nn ; arccos (2m--r-cos a)
and
' * -fy = a
x — y = 2/t«— arccos (2/n-{"Cos a )
■dnsujer:
f Xj = jtíí -] —arccos (2m-r cos a)

——arccos(2a«4"cosa)
f x2 nn 4- -r— 1arccos (2/« cos a)
a

y2^ _ n ; í 4 " '2 —h*2" arccos (2//? {-cosa)

900. Using the formula


sin (a4-p)
tan a4“ tan fi ----
cos a cos 8
392 Answers and Solutions

write the second equation in íhe following way: ^ xcos^ f' =m' ^eP*ac*D£
, cosfx4-ü)-f cos (x—y) j , i
cosx cos y by---- -— 1----- — and x-\-y by a , we get

2 sina
=m
cos a-j-cos {x—y)
or
, , 2 sina
cos {x—y) —-------------- cosa

Hence, we have either


2sin a
x — y — 2jí/i 4~arccos ^
ia )

- , / 2 sin a \
y — x = 2nn 4- arccos l — --------cos a i

Either oí these equations should be solved together with the equation x 4~ y — «>
By the way, of the two systems obtained one differs from the other only in that
the unknowns change their roles, thereíore it is sufficient to solve one o! the
systems.
a , t / 2sin a \
Answer: x { ( — y2) = -+■-j - 4" — arccos I — —-------- cos a 1
, a 1 / sin a \
y i ( s=x5) = — nn-r-g— y arccos í — ~------- cos a i

901. Solved as the preceding problem.


Answer:

x , = - 5 - (4 n + l) x2 — — nn

Sl= - m í2= 4 (4n+,)

902. Since 1 = 2° and 4 = i62, the given system may be rewritten as


f sino:4 -cos y = 0
i
sin2 x -fcos 2 y =
2
whence
l i 1 1
<i) sin x = » cos y — — ^ and (2 )s in x = — T , co$y = -$

Answer: :rj=180on 4 ( — i}n 30% y\—360°n ± 12071


x2= 1 8 0 ° n - ( - l ) n 30°, y2 = 3G0s7i ± 60°
903. The second equation can be written in the form
sin xsin y 1
cos x cos y ~ 3*
Chapter X I I . Trigonometric Equations 393

wherc, by virtue of the first equation, sin zsin y ~ —-- ^ r . We get the
system of equations:
3 1
cos x cos y = ^ yT£ sm xsin y =
4V2
Adding and subtracting them, we get

C 0 S (I -J t ) = ^ f and cos( * 4 -í ) = y ^ j

whcnce
\ Jl
x~ -y = 2nm ± arccos-----— , x — y ~ 2nk ±
2 V*2 4
where m and k are arbitrary integers. In each of thcse equations we can take
any sign.
Note. The numbers m -j- k and m — k are also integers, but not completely
arbitrary (if one of them is even, the other is also even; and if one of tnem is
odd, the other is also odd).
Answer: (1) x = j i n + 4 'arccos—
2 21/2 8
1 1 ji
í = « + T a r c c o s ^ 7=— T

(2) * = nn + y arccos

i '= nt + T 310003i 7 f + T
1 f Tl
(3) I = n » —y arccos ~ p y f + T
i i n
V= *'~2 ‘" “« - J y f- T
... 1 1 JI
(4) x = n n - T a r c c o s -p ^ = --T

, = arccos ^ -^ = - + -5-

where n — m 4- fc; t — m — k (m and k are any integers).


904. Square both members of each of the given equations and then add them
by terms, We get
1 — 4 sin2y ~f ^ cos2y or 1 ==4 ( i —cos2 y) + c o s 2y

whence cos2 y = •— an<* sin2 y = -— . In each of the expressions cosy — ± —^=r


b 5 1/5
and sin y =# ± ~~~ either sign may be taken (thus, in the interval between 0

26-01338
394 Answers and Solutions

and 360° thc angle y can have four values). Substituting these values into
the given equations, we find ibat the angles x and y satisfy one o! the fol-
lowing four relationships:
i 2
(1 ) CQSX- cosy = sm y=.
' V5 ’ yr : Vã' 1 /5
1 2 2
(2) cos x = — —, cos y —— - r , sin y = —
:ys ’ V 5 1/5 V5
1 . 2
(3) cos x - — — > smi = —— ■ COSy =
1 /5 1 /5 'y i siny 1/5
2 1
(4) cosz = cos y = sm y =
1 /5 ' “ Vã 'V ã * ' Vã
1
Consider the first of them. If we take separately the equality cosx =
8 Vã
then it yields x —2jtm ± arccos —~ . B«t (by the definition of the principal
V5

value of arccos) the angle 9 —arccos belongs to the first or second

quadrants, where the function of sine is always positive. Hence, the plus
sign shouid be retained. Indeed, from the equality x —2;rr/?±(p it follows
that siox = ± sin<p = ± — =■• But in the first relationship sinx = — -=r
>5 F 1/5
2
^but not— -^ | -j . The same with the angle y, thus in case of relationship

(1) we get
1 2
x~ -2nn 4- arccos—— , r/ — 2n«j 4- arccos ——
vo y 5

where n and are any integers. Reasoning in the same way, we find that
for the second relationship

o —arccos—1— , y —o2nn, — arccos—2—•


x —2ítn
V5 1/5

The third and fourth relationships are considered analogously.

Answer: x = 2nn ± arccos |


)

y = 2jxnj ± arccos
-)
where the signs in parentheses are the same for x and for y and the signs before
arccos are also the same.
C h a p ter X I I I . In verse T r ig o n c m e tr ic F u n c tio n s 395

C H A P T E R XHi
INVERSE TRIGONOMETRIC FUNCTIONS
905. We bave
ít
, arccot( — 1)~
3
1
arccos (
1/2 :
5n
Answer:
ü
906. The angle q) — arccos x is found in the interval between 0 and 180°
(by the definítion of the principal value of arccos). Ilence, sin <p is positive
(or equal to zero). AVe bave cos <p — x, whence sin 9 = -f- ~\/1 — z2 (the radicai
is taken only with the pius sign). Consequentiy,
Y iz n 2
tan cp= —------------,

V i-
tan (arccos x)
which completes the proof.
907. See the soiution of the preccding problem.
908. Let us put arccot ^ — -L-j = <p, thus cot <p = —- ~ , The angle <p is in
lhe interval between 90° and 180° (since the principal value of arccot is conlai-
ned between 0 and 180c). It is required to íind s i n ~ . Let us use the formula

1 — COS Cp
± J/ :
where out of the two signa only the plus sign is taken (since the angle ~ belongs
'to the first quadrant). First we have to íind cos <p using the formula
cot a
=fc V l + cot2a
we get
_ 3_
4

í + 16
(the radical is taken with the plus sign only, since cp belongs to the second quad­
rant). Now we íind _____
3
coscp
Vs
396 Answers and Solutions

Ânswer: sin arccot ^ —■


- - j J —~ ~ .

2 1/2 \ 2 1 /2
( ------ 1 — j ^ 9 , hence sin 9 = ------ . The angle 9 is in

the interval between —90° and 0* {since the principal vaiue of arcsin is con-

i required to find sin -|-. This vaiue


tained between —90* and +90°). It is

is negative. Therefore in the formula

“ “ 2 “ -1- V 2
only the minus sign is to be retained. We get
•COS9

where
2 l
3
we take the radical only with the plus signl).
, . ri . / — 2 V 2 \n 1
Anmtr. sin j^yarcsm ^ ^ — ^71"

41
910. The angle <p=arccos { —y ) is contained between 90° and 180° (see

the Solutions oi the two previous problems). Hence cot y - is positive, and

€* ± , - / ‘ ± £ “ £
2 V 1 — cos <p
/
{the radical is taken only with the plus sign). Substitute coscp = —y into

this expression.
V 33
Answer: cot arccos | )] =
: ii ’
911. Since
arctan H -
3

. V3
arcam-—- —

we have
3n
=tan

Answer: — 1.
Chapter X I I I . Inverse Trigonometrie Functions 397

912. We have
arctan 1 /3 = -—*
and

aiw » 2 3

Then proceed as in tbe preceding problem.

Answer, —

913. /irtsteer. .
914. Let us put
arctan (3 + 2 1/2) = a (1 )

( 2)

It is required to prove that

a —p = (3)
Find tan(a--jS):
tan a — tan p
tan (a — p) =
1 + tan a tan $
with the aid of (1) and (2) we get
V2
(3 + 2 1 /2 )-
tan (a—{3) = (4 )

1 + (3 + 2 1 /2 )

Gn the otber baud, it is obvious írom (1) and (2) tbat either of tbe angies a
and p lies between 0 and “ , and a > p ^sínce 3 + 2 ~\/2 > ^; cousequen-

tly, theangie a — p a priori Jíes bctween 0 and , heuce, from (4) we get a — (1--;

x • whích completes tbe proof.

Note. To provo that the angle a — 0 is just equal to , i.e. to 45° (but not
to 225° or to —135° and so on) we can make uso of tbe correspondiug tablcs tofind
directly the angies a and fi. Herewemay confine ourselves to rough approxima-
tions (for instauce, taking ínto account only degrees). Thus, putting 1/2 x 1.4,
We f*üd a 5k arctan 5.8» which corresponda to about 80° ( the error a priori does
398 Answers and Solutions

not exceed - y j . In the same way we find P « arctan0.7, which corresponds to

about 35° |the error is a priori less than-yj. Consequently, a—P does not
differ from 45° by more than I o, and, hence, is exactly equal to 45°.
915. Put
,/ 2
arccos J / y = a

V e+1
2 1 /3

so that cosa —j / " y and cosfl— °f an8^e8 a anà P

belongs to the first qnadraut ***. It is required to prove, that a —p = ~ .

Find sin (a--p ), but first compute


sina = V f —cos2a
and
s ÍnP ~ l/l—cos2P
(eacb radical is taken only with the plus sign, since a and p beiong to the
first quadrant). We find

1 ;V 6
Sjn a ™ _ _ _ and sinp = " | /-

heuce,

1 Vb + l
sin (a —P) =
1 /3 ‘ 2 1 /3

Let us prove that the found irrationai cxpression is equal to . To this end

transform the “ double irrationality” \ 5 — 2 V ê — K õ — 1/24. It can be per-


forined with the aid of the formula

K a -V ã = y
* This problem may be solved without introducing auxiliary quantities a
and p using the method mentioned in the note to the preceding problem.
** The principal value of arccos lies between 0 and n.
Chapter X I I I . Jnverse Trigonometric Functions 399

<at A== 5, B = 24); we obtain

j/ tz l^ y í-V í

But it is simpler to represent the radicand 5 —2 V 6 in the form 3 - f 2 —


— 2 V 2 - V 3 = = ( V S — V 2 ) 2! and then we have

K 5 —2 1 / 6 = V w i - y w = y ã - 1 / 2 *
Since either of the angles a and 0 lies witliin the intervaí between 0 and ~

the angle a - - P undoubtedly lies within the intervaí from — ~ to ,

then from the equality s ía (a — 0) —~ it Collows that a — whi ch


completes the proof **.
916. Let (see the two previous probíems)
. 4 16
arcsin-p-:

Then
4 3
sin a = - = - , cos a
5 5
. „ 5 12
COS P
s m íi = T 3 -' " 13
16 63
cos y
s m Y = «?■ " 65
Hence

siri (a -fP ) = 11
13 ‘
_63
: 65
Both angles a and p beloug to the first quadrant; therefore the angle a-f-6
lies between 0o and 180°, and sinco cos(a--j-p) is positive, a -rP belongs to
the first quadrant. Furthermore, cos (a -j- P) ~ sin y and sin (a-j--P) =

“ 3i“ ( t - v)-

* The number 1/3—]/2 is positive.


* * I f c o s ( a — p) is computed instead of sin (a ~ p), we would find
cos (a ~ p) — between — ~ and -f- ™ we would have two values of cc — p,

namely, — .5. and -f-~ ; therefore we would have first to establish that a > p,
6 0
••e. that cos a < cos p.
400 Answers and Solutions

Therefore, a+P and - y —y may differ only by 2m, and since y — y also

belongs to the firsfc quadrant, we have n—0. Consequently, a + p “ — —Yt

i.e. a-f-p-f-Y —y which completes the proof.


1 n
917. We have arccos y ~ y ; let us denote arccos | —y j by p, so tbat
1 jt
cos p= —y . The angle p is contained between — and n (see the three pre-
vious problems). Therefore

sin j^but not —] / " l —( y ) J .

?= | V 3
We find
COS -Tn - COSafl
■(t + 0 O
V3 11
- t -(-t ) 2 ' t V*- ‘ 14
To prove tbe validity of the given identity we have to make sure that the
angle y + p belongs to the second quadrant j^since the angle arccos ^ ” '54')

in the right member lies in the second quadrantJ . The angle P^arccos |—y|

is contained between and it; consequeutly, the angle y + p lies between

4r- and But it does not, however, follow from this estimate that the
b 0

angle y - f p belongs to the second quadrant |since the angle -y~ is already

found ín the third quadrant J . But taken intoaccount that —y > —y and

that, consequently, arccos |~ y j <arccos ^—y| , i.e- arccos^—y j <

< y - , it follows that y -j- arccos |—y j < jí. And since this angle is more

than - y , it lies in the second quadrant. Hence, the given identity is proved.

Note. The fact that the angle y - f P belongs to the second (and not to
Ckapter X I I I . Inverse Trigonometric Funcíions 401

the third) quadrant can aiso be demonstrated in a difíerent way: we have

,f+íi)= - cos - j- sin p =

JúL
2 ( 4) í
+ — • - 1 /3 = -V 3

Since this number is positive, the angle -~ -4-p beíongs to tbe second quadrant.
918. Put
1 1
arctan-=- = a and arctan-7- = 6
5 4
whence
tan a — 4- and tan 8 = 4-
5 r 4
Compute
tan 2a + tan £
tan (2a 4- p) =
1— tan 2a tan (5
First find
±
5 JL
tan 2a ------- -
12
1-

and then
_L 4 ±
12 4 3°
taM 2a + P )= t -4 3
12 ' 4

The angles a ~ arctan ~ and p = arctan ~ belong to the first quadrant, but
it does not yet íollow from this íact that the angle 2a 4- P belongsto the first
(and not to tbe third) quadrant. But if we take into consideration that either
oí the angles a and p is less than ~ (since their tangents are less than unity),

it proves that 2a - f p is less than and since, furthermore, tan (2a ~f~ p) = ~

is positive, 2a - f p lies in tho first quadrant, i.e. 2a 4 - P ” arctan ||, which


completes the proof.
Note. Instead of proving that the angle 2a 4- P remains within the limits
of the first quadrant, we can fiud this angle anproximately with the aid of the
corresponding tables (see the note to Problem 914). We get: a~arctan
- g -a s ll ', P = arctan-j-% 14», hcncc 2a H- P % 36°.
402 Answers and Solutions

919. Put

arctan ~ ~ a , arctan— ^P» arctan 4 -= y » arctan 4 - = ô


O O l o
and first find
4 .+ 1
tan (a+P) = ■ 3 l ^ = j .
í _ _ . _

thea
i +i
tan f (a -f P) + V3—------Jj— ="g
T
and, finally,
I+ I
tan [(o -fP + Y) + ô] = — L - l - = 1
i — L .-i
9 8
As in the preceding problem, prove that the angle a + 0 - f y - f ô lies in the
first quadrant. Consequently, cc-f-jJ-j-Y + ô — ~ .

920. Wc have arctan (x2 — 3x —* 3) = ~ , whence x2 — 3x —•3 = tan-í-»


4 4
i.e. x2 — 3x — 3 = 1. Hence, xj = 4; x2 = —1.
Ansuser: xt = 4; x2 = —1.
Acíe. If instead of the equation arctan (x2 — 3x — 3) — — we would have

the equation arctan (xs — 3x — 3) = — , then the latter would have no solu-

tion,since the principal value of arctan cannot be equal to — — . If no atten-


4
tion is paid to this circumstance, \ve can obtain the same equation x2 — 3x — 3 =
= 1, but the roots of the iast equation are not suitabie.
921. We have
arcsin (x2 — 6x---8.5)=-?-
o
whence x2 — 6x - f 8.5 — 0.5.
Answer: xj — 4; x2 = 2.
922. Taking t&ngents of both members of the equation and remembering
that tan (arctan a) = a, we get
(x-f-2) — (x -f 1) _
l-f(í+ 2 )(i:+ !)
whence x\~ —1; x2= —2. Check tliese roots. If x = —í, then

arctan (x ~ 2) = arctan 1 —-r-


4
Chapter X I I I . Inverse Trigonometric Functíons 403

and
arctan (x + 1) = arctan 0 — 0
thus tbe given equation is not satisfied. We prove in the same way that the
second root is also not suitabie.
Answer: Xj = —1; x2 — —2.
Note. Why sucb a check is necessary is clear from the foüowing exampie.
Consider the equation
arctan (x 2) — arctan (x -j-1) = -----y
which diífers from the given one only by the value of the constant terrn. It is
impossible to staie beforehand that it has no Solutions (cf. Problem 920). If,
say, arctan (x + 2) is equal to — ~ , and arctan (x + 1) to ~ (thesevalucscan

be principal values of arctangent), then the left meraber would be equal to —


Taking tangents of both members of the equation under consideration, we again
get the equation
(z + 2 ) - ( g - f r l ) ..
l + (x -f2 ) (x -f 1)
but now neither of the roots xj = —1 ,x 2 — —2 is suitabie. See also the Note
to Problem 925.
923. Take tangent of both members of the equation. First find (see the preced-
«og problem):
i_
2
12 arctan y j =

and then we obtaín


4
T~* i
4x
1+ 3

The root of tbis equation is x — ; it should be checked (see the Note to the

preceding problem). Substituling x — y inlo the left meniher of the equation,

we get 2 arctan ~ -----arctan —•. The angíe a —arctan lies between 0 and ™

(sinco t a n a = ~ < l ) . The angle p ^ arctan y lies within lhe same range.
Tbe angle 2a belongs to the fiest quadrant, and the angle 2a — P lies between
and But tan (2a — p) = 1, hence, 2a -- p — jm j- y . But only
at n = 0 the angle 2a — p turns out to lie wilhin the íound bouuduries. Conse-
quently, lhe given equation is satisfied.
Answer: x ~ ~ - .
404 Answers and Solutions

924, Let us take sices of both. members o£ the equation, We can put

arcsin—^ r = a and arcsinYl —x —fi


3 yx
(see the solution o! Problem 915), but we can do without tbem ntaking use o!
the formulas sin (arcsin x) = x (wbich immediately follows from the definition
of arcsine) and cos (arcsin x) = ~\/l — x2*. Consequently, sine of the left
member is equal to

y r r ;

and the given equation takes the form


2 y -
y r r i.
3 Vx 3Vx

Solving it, we find x = y . Thís root shoulá be checked (see the Note to

Problem 922), i.e. we bave to prove the identity


. ,/ T
arcsin y y~
Yr
lt is proved in the same way as in Problem 917.
Answer: x = y .
925. Taking tangents of both merabers of the equation, we get
a a—b
b a -fò
i J L a a~ b
1+T * h T
whcnce x = 1. Tbis value should be checked (see the Note to Problem 922).
Substituting x — 1 into the given equation, we get
. a . a —b __
arctan - — arctan------r = 45° (1
b a~\-b
Introducc the following notntion

arctan (2)
Here the angle 9 (the principal value of arctangent) lies beiween —90° and 90a
—90° < <p < 90°. (3)

* Thia formula is deduced in the following way. Put arcsin x — a. Tben


sin a = x and cos a = V 1 ~ x3. The radical is taken only with the plus sign
since the angie a — arcsin x lies between — 90° and - f 90a (the principal value
of arcsinel). Substituting arcsin 2 fora, we get the required formula.
Chapter X I I I . Inverse Trigonometric Functions 405

Using this notation, we have


. a—b x 6tan<p~6
arctan —--T-=arctan -7-— = arctan lan (íp — 45°) (4)
a-^b b tan cp-f-6 VT ' ' f
and so we have to check the equality
<p — arctan tan {<p — 45°) = 45° (5)
This equality is true if and only if
arctan tan (<j> — 45°) = tp — 45° (6)
And the equality (6) holds true when the angie íp — 45° (the principal vaiue of
arctangent) lies within the following intervai:
—90° < 9 — 45° < 90°, (7)
i.e. when
-4 5 ° < q> < 135° (8}
Taking into consideration (3), we get a more narrow intervai for the angie
—45° < <p < 90° (9)
From (2) and (9) we find

1. = tan q> > tan ( —45°),


i.e.

T > ~ 1 (10)

Conversely, fo r -| -> — 1 the anglo 9 satisfies inequality (9). Consequently,

the given equation has a solution (x —1) for — 1. For 1 there is


do solution.
For example, at a = — V 3, 6= 1 we have

arctan = arctan ( — ~[/3) = — Cܰ

arctan =: arctan « arctan 3.732 = 75*


a+ 6 -V 3 + 1
thus, the leít memker of the given equation is equal to —135°, and the right
one at z s s l is equal to 45°.
Answer: x «* 1 for ~ > —1; the equation has no solution for ~ < —1.

926. Let us take cosines of both merabers of the equation. We get 1—9x2=
(see Problem 924). This equation has only one root x — -g-. Let us check

ft. The angie a = arcsin 3z = arcsin ~ belongs to the íirst quadrant; the
406 4nsu?grs and Solutions

4
angle p = arccos kx = arccos j - also belongs to the íirst quadrant. Here
3 4 4
sin a — y ; hence, cos a = — . On the other hand, cos P = y ; hence, a —

Answer: x = ~ .

927. Let os take the sines oí both members of the equation. We get {see

Problem 924) 2x ^ í — x2^ -— - , whence x j= 0 , x3 — — .

Check tliese roots.


/tnsuier: x = 0 .
928. From the íirst equation we find
2a
tan (x -f y) — *

tan x-ftan y 2a
1 — tan x tan y — 1— a2
Taking into consideration the second equation, we get
tanx-rtany 2a
TTp T -a 2
or
tan x -f tan y — 2a
From the system of equations
tan x -j- tan y = 2a
tan x tan y ■= a-
\ve íind tan x — a\ tan y ----- a. Hence, it follows that x — i80°n 4 arctan a,
y 180oro 4- arctan a, wherc n and m are integers. But oniy one of them may
be íaken arbitrarily, since according to the íirst equation the quantity x -j- i'
rnust be coníaíned between —90° and +90° (as the principal value of arctangent).
To identify suitable values of n and m substitute the found expressions into
the íirst equation. We get
2a
180° (rc-f m)-i 2 arctan a = arctan 1 —a2 (A)
Since by hypothesis |a j < 1, the angle arctan a lies between —45° and
4-45°, i.e. 2 arctan a lies between —90° and 490°. Tbe angle arctan
(the principal value of arctangent) lies within the same range. Conseouently,
these two angles diífer by less than 180°. Therefore, the equality (A) holds true
oniy at n + m = 0.
Answer: x = 180°« 4 arctan a, y — — 180°n 4 arctan a.
Mir Publishers welcome your comments on the content, transla-
tion, and desigu of the book.
VVe would aiso be pleased to receive any suggestions you care
to make about our future publications.
Our address is:
USSR, 429820,
Moscow, í-110, GSP.
Pervy Rizhsky Pereuiok, 2,
Mir Publishers
UDC 51f+512+513+5ií(075.4)=20

0222-233
A 041(01)—74

You might also like